*NURSING > NCLEX > Actual tests NCLEX-RN - P1 (800 Questions with answers and rationale) Latest 2020\2021, Graded A (All)

Actual tests NCLEX-RN - P1 (800 Questions with answers and rationale) Latest 2020\2021, Graded A

Document Content and Description Below

NCLEX-RN 828q Number: NCLEX Passing Score: 800 Time Limit: 120 min File Version: 12.5 NCLEX-RN .NCLEX-RN QUESTION 1 A 25-year-old client believes she may be pregnant with her first child. She ... schedules an obstetric examination with the nurse practitioner to determine the status of her possible pregnancy. Her last menstrual period began May 20, and her estimated date of confinement using Nägele's rule is: A. March 27 B. February 1 C. February 27 D. January 3 Correct Answer: C Section: (none) Explanation Explanation/Reference: Explanation: (A)March 27 is a miscalculation. (B) February 1 is a miscalculation. (C) February 27 is the correct answer. To calculate the estimated date of confinement using Nagele's rule, subtract 3 months from the date that the last menstrual cycle began and then add 7 days to the result. (D) January 3 is a miscalculation. QUESTION 2 The nurse practitioner determines that a client is approximately 9 weeks' gestation. During the visit, the practitioner informs the client about symptoms of physical changes that she will experience during her first trimester, such as: A. Nausea and vomiting B. Quickening C. A 68 lb weight gain D. Abdominal enlargement Correct Answer: A Section: (none) Explanation Explanation/Reference: Explanation: (A) Nausea and vomiting are experienced by almost half of all pregnant women during the first 3 months of pregnancy as a result of elevated human chorionic gonadotropin levels and changed carbohydrate metabolism. (B) Quickening is the mother's perception of fetal movement and generally does not occur until 1820weeks after the last menstrual period in primigravidas, but it may occur as early as 16 weeks in multigravidas. (C) During the first trimester there should be only a modest weight gain of 24 lb. It is not uncommon for women to lose weight during the first trimester owing to nausea and/or vomiting. (D) Physical changes are not apparent until the second trimester, when the uterus rises out of the pelvis. QUESTION 3 A 38-year-old pregnant woman visits her nurse practitioner for her regular prenatal checkup. She is 30 weeks' gestation. The nurse should be alert to which condition related to her age? A. Iron-deficiency anemia B. Sexually transmitted disease (STD) C. Intrauterine growth retardation D. Pregnancy-induced hypertension (PIH) Correct Answer: D Section: (none) Explanation Explanation/Reference: Explanation: (A) Iron-deficiency anemia can occur throughout pregnancy and is not age related. (B) STDs can occur prior to or during pregnancy and are not age related. (C) Intrauterine growth retardation is an abnormal process where fetal development and maturation are delayed. It is not age related. (D) Physical risks for the pregnant client older than 35 include increased risk for PIH, cesarean delivery, fetal and neonatal mortality, and trisomy. QUESTION 4 A client returns for her 6-month prenatal checkup and has gained 10 lb in 2 months. The results of her physical examination are normal. How does the nurse interpret the effectiveness of the instruction about diet and weight control? http://www.gratisexam.com/ A. She is compliant with her diet as previously taught. B. She needs further instruction and reinforcement. C. She needs to increase her caloric intake.D. She needs to be placed on a restrictive diet immediately. Correct Answer: B Section: (none) Explanation Explanation/Reference: Explanation: (A) She is probably not compliant with her diet and exercise program. Recommended weight gain during second and third trimesters is approximately 12 lb. (B) Because of her excessive weight gain of 10 lb in 2 months, she needs re-evaluation of her eating habits and reinforcement of proper dietary habits for pregnancy. A 2200-calorie diet is recommended for most pregnant women with a weight gain of 2730 lb over the 9-month period. With rapid and excessive weightgain, PIH should also be suspected. (C) She does not need to increase her caloric intake, but she does need to re-evaluate dietary habits. Ten pounds in 2 months is excessive weight gain during pregnancy, and health teaching is warranted. (D) Restrictive dieting is not recommended during pregnancy. QUESTION 5 Diabetes during pregnancy requires tight metabolic control of glucose levels to prevent perinatal mortality. When evaluating the pregnant client, the nurse knows the recommended serum glucose range during pregnancy is: A. 70 mg/dL and 120 mg/dL B. 100 mg/dL and 200 mg/dL C. 40 mg/dL and 130 mg/dL D. 90 mg/dL and 200 mg/dL Correct Answer: A Section: (none) Explanation Explanation/Reference: Explanation: (A) The recommended range is 70120 mg/dL to reduce the risk of perinatal mortality. (B, C, D) These levels are not recommended. The higher the blood glucose, the worse the prognosis for the fetus. Hypoglycemia can also have detrimental effects on the fetus. QUESTION 6 When assessing fetal heart rate status during labor, the monitor displays late decelerations with tachycardia and decreasing variability. What action should the nurse take? A. Continue monitoring because this is a normal occurrence. B. Turn client on right side.C. Decrease IV fluids. D. Report to physician or midwife. Correct Answer: D Section: (none) Explanation Explanation/Reference: Explanation: (A) This is not a normal occurrence. Late decelerations need prompt intervention for immediate infant recovery. (B) To increase O2 perfusion to the unborn infant, the mother should be placed on her left side. (C) IV fluids should be increased, not decreased. (D) Immediate action is warranted, such as reporting findings, turning mother on left side, administering O2, discontinuing oxytocin (Pitocin), assessing maternal blood pressure and the labor process, preparing for immediatecesarean delivery, and explaining plan of action to client. QUESTION 7 The predominant purpose of the first Apgar scoring of a newborn is to: A. Determine gross abnormal motor function B. Obtain a baseline for comparison with the infant's future adaptation to the environment C. Evaluate the infant's vital functions D. Determine the extent of congenital malformations Correct Answer: C Section: (none) Explanation Explanation/Reference: Explanation: (A) Apgar scores are not related to the infant's care, but to the infant's physical condition. (B) Apgar scores assess the current physical condition of the infant and are not related to future environmental adaptation. (C) The purpose of the Apgar system is to evaluate the physical condition of the newborn at birth and to determine if there is an immediate need for resuscitation. (D) Congenital malformations are not one of the areas assessed with Apgar scores. QUESTION 8 A pregnant woman at 36 weeks' gestation is followed for PIH and develops proteinuria. To increase protein in her diet, which of the following foods will provide the greatest amount of protein when added to her intake of 100 mL of milk? A. Fifty milliliters light cream and 2 tbsp corn syrup B. Thirty grams powdered skim milk and 1 eggC. One small scoop (90 g) vanilla ice cream and 1 tbsp chocolate syrup D. One package vitamin-fortified gelatin drink Correct Answer: B Section: (none) Explanation Explanation/Reference: Explanation: (A) This choice would provide more unwanted fat and sugar than protein. (B) Skim milk would add protein. Eggs are good sources of protein while low in fat andcalories. (C) The benefit of protein from ice cream would be outweighed by the fat content. Chocolate syrup has caffeine, which is contraindicated or limited in pregnancy. (D) Although most animal proteins are higher in protein than plant proteins, gelatin is not. It loses protein during the processing for food consumption. QUESTION 9 Which of the following findings would be abnormal in a postpartal woman? A. Chills shortly after delivery B. Pulse rate of 60 bpm in morning on first postdelivery day C. Urinary output of 3000 mL on the second day after delivery D. An oral temperature of 101F (38.3C) on the third day after delivery Correct Answer: D Section: (none) Explanation Explanation/Reference: Explanation: (A) Frequently the mother experiences a shaking chill immediately after delivery, which is related to a nervous response or to vasomotor changes. If not followed by a fever, it is clinically innocuous. (B) The pulse rate during the immediate postpartal period may be low but presents no cause for alarm. The body attempts to adapt to the decreased pressures intra-abdominally as well as from the reduction of blood flow to the vascular bed. (C) Urinary output increases during the early postpartal period (1224 hours) owing to diuresis. The kidneys must eliminate an estimated 20003000 mL of extracellular fluid associated with a normal pregnancy. (D) A temperature of 100.4F (38C) may occur after delivery as a result of exertion and dehydration of labor. However, any temperature greater than 100.4F needs further investigation to identify any infectious process. QUESTION 10 What is the most effective method to identify early breast cancer lumps? A. Mammograms every 3 yearsB. Yearly checkups performed by physician C. Ultrasounds every 3 years D. Monthly breast self-examination Correct Answer: D Section: (none) Explanation Explanation/Reference: Explanation: (A) Mammograms are less effective than breast self-examination for the diagnosis of abnormalities in younger women, who have denser breast tissue. They are more effective forwomen older than 40. (B) Up to 15% of early-stage breast cancers are detected by physical examination; however, 95% are detected by women doing breast self-examination. (C) Ultrasound is used primarily to determine the location of cysts and to distinguish cysts from solid masses. (D) Monthly breast self-examination has been shown to be the most effective method for early detection of breast cancer. Approximately 95% of lumps are detected by women themselves. QUESTION 11 Which of the following risk factors associated with breast cancer would a nurse consider most significant in a client's history? A. Menarche after age 13 B. Nulliparity C. Maternal family history of breast cancer D. Earlymenopause Correct Answer: C Section: (none) Explanation Explanation/Reference: Explanation: (A) Women who begin menarche late (after 13 years old) have a lower risk of developing breast cancer than women who have begun earlier. Average age for menarche is 12.5 years. (B) Women who have never been pregnant have an increased risk for breast cancer, but a positive family history poses an even greater risk. (C) A positive family history puts a woman at an increased risk of developing breast cancer. It is recommended that mammography screening begin 5 years before the age at which an immediate female relative was diagnosed with breast cancer. (D) Early menopause decreases the risk of developing breast cancer. QUESTION 12 The nurse should know that according to current thinking, the most important prognostic factor for a client with breast cancer is:A. Tumor size B. Axillary node status C. Client's previous history of disease D. Client's level of estrogen-progesterone receptor assays Correct Answer: B Section: (none) Explanation Explanation/Reference: Explanation: (A) Although tumor size is a factor in classification of cancer growth, it is not an indicator of lymph node spread. (B) Axillary node status is the most important indicator for predicting how far the cancer has spread. If the lymph nodes are positive for cancer cells, the prognosis is poorer. (C) The client's previous history of cancer puts her at an increased risk for breast cancer recurrence, especially if the cancer occurred in the other breast. It does not predict prognosis, however. (D)The estrogen-progesterone assay test is used to identify present tumors being fedfrom an estrogen site within the body. Some breast cancers grow rapidly as long as there is an estrogen supply such as from the ovaries. The estrogen-progesterone assay test does not indicate the prognosis. QUESTION 13 A 30-year-old male client is admitted to the psychiatric unit with a diagnosis of bipolar disorder. For the last 2 months, his family describes him as being "on the move," sleeping 34 hours nightly, spending lots of money, and losing approximately 10 lb. During the initial assessment with the client, the nurse would expect him to exhibit which of the following? A. Short, polite responses to interview questions B. Introspection related to his present situation C. Exaggerated self-importance D. Feelings of helplessness and hopelessness Correct Answer: C Section: (none) Explanation Explanation/Reference: Explanation: (A) During the manic phase of bipolar disorder, clients have short attention spans and may be abusive toward authority figures. (B) Introspection requires focusingand concentration; clients with mania experience flight of ideas, which prevents concentration. (C) Grandiosity and an inflated sense of self-worth are characteristic of this disorder. (D) Feelings of helplessness and hopelessness are symptoms of the depressive stage of bipolar disorder. QUESTION 14A client with bipolar disorder taking lithium tells the nurse that he has ringing in his ears, blurred vision, and diarrhea. The nurse notices a slight tremor in his left hand and a slurring pattern to his speech. Which of the following actions by the nurse is appropriate? A. Administer a stat dose of lithium as necessary. B. Recognize this as an expected response to lithium. C. Request an order for a stat blood lithium level. D. Give an oral dose of lithium antidote. Correct Answer: C Section: (none) Explanation Explanation/Reference: Explanation: (A) These symptoms are indicative of lithium toxicity. A stat dose of lithium could be fatal. (B) These are toxic effects of lithium therapy. (C) The client is exhibiting symptoms of lithium toxicity, which may be validated by lab studies. (D) There is no known lithium antidote. QUESTION 15 Which of the following activities would be most appropriate during occupational therapy for a client with bipolar disorder? A. Playing cards with other clients B. Working crossword puzzles C. Playing tennis with a staff member D. Sewing beads on a leather belt Correct Answer: C Section: (none) Explanation Explanation/Reference: Explanation: (A) This activity is too competitive, and the manic client might become abusive toward the other clients. (B) During mania, the client's attention span is too short to accomplish this task. (C) This activity uses gross motor skills, eases tension, and expands excess energy. A staff member is better equipped to interact therapeutically with clients. (D) This activity requires the use of fine motor skills and is very tedious. QUESTION 16 Three weeks following discharge, a male client is readmitted to the psychiatric unit for depression. His wife stated that he had threatened to kill himself with a handgun. As the nurse admits him to the unit, he says, "I wish I were dead because I am worthless to everyone; I guess I am just no good." Which response by the nurse is most appropriate at this time?A. "I don't think you are worthless. I'm glad to see you, and we will help you." B. "Don't you think this is a sign of your illness?" C. "I know with your wife and new baby that you do have a lot to live for." D. "You've been feeling sad and alone for some time now?" Correct Answer: D Section: (none) Explanation Explanation/Reference: Explanation: (A) This response does not acknowledge the client's feelings. (B) This is a closed question and does not encourage communication. (C) This response negates the client's feelings and does not require a response from the client. (D) This acknowledges the client's implied thoughts and feelings and encourages a response. QUESTION 17 Which of the following statements relevant to a suicidal client is correct? A. The more specific a client's plan, the more likely he or she is to attempt suicide. B. A client who is unsuccessful at a first suicide attempt is not likely to make future attempts. C. A client who threatens suicide is just seeking attention and is not likely to attempt suicide. D. Nurses who care for a client who has attempted suicide should not make any reference to the word "suicide" in order to protect the client's ego. Correct Answer: A Section: (none) Explanation Explanation/Reference: Explanation: (A) This is a high-risk factor for potential suicide. (B) A previous suicide attempt is a definite risk factor for subsequent attempts. (C) Every threat of suicide should be taken seriously. (D) The client should be asked directly about his or her intent to do bodily harm. The client is never hurt by direct, respectful questions. QUESTION 18 The day following his admission, the nurse sits down by a male client on the sofa in the dayroom. He was admitted for depression and thoughts of suicide. He looks at the nurse and says, "My life is so bad no one can do anything to help me." The most helpful initial response by the nurse would be: A. "It concerns me that you feel so badly when you have so many positive things in your life." B. "It will take a few weeks for you to feel better, so you need to be patient."C. "You are telling me that you are feeling hopeless at this point?" D. "Let's play cards with some of the other clients to get your mind off your problems for now." Correct Answer: C Section: (none) Explanation Explanation/Reference: Explanation: (A) This response does not acknowledge the client's feelings and may increase his feelings of guilt. (B) This response denotes false reassurance. (C) This response acknowledges the client's feelings and invites a response. (D) This response changes the subject and does not allow the client to talk about his feelings. QUESTION 19 A long-term goal for the nurse in planning care for a depressed, suicidal client would be to: A. Provide him with a safe and structured environment. B. Assist him to develop more effective coping mechanisms. C. Have him sign a "no-suicide" contract. D. Isolate him from stressful situations that may precipitate a depressive episode. Correct Answer: B Section: (none) Explanation Explanation/Reference: Explanation: (A) This statement represents a short-term goal. (B) Long-term therapy should be directed toward assisting the client to cope effectively with stress. (C) Suicide contracts represent short- term interventions. (D) This statement represents an unrealistic goal. Stressful situations cannot be avoided in reality. QUESTION 20 After 3 weeks of treatment, a severely depressed client suddenly begins to feel better and starts interacting appropriately with other clients and staff. The nurse knows that this client has an increased risk for: A. Suicide B. Exacerbation of depressive symptoms C. Violence toward others D. Psychotic behaviorCorrect Answer: A Section: (none) Explanation Explanation/Reference: Explanation: (A) When the severely depressed client suddenly begins to feel better, it often indicates that the client has made the decision to kill himself or herself and has developed a plan to do so. (B) Improvement in behavior is not indicative of an exacerbation of depressive symptoms. (C) Thedepressed client has a tendency for self-violence, not violence toward others. (D) Depressive behavior is not always accompanied by psychotic behavior. QUESTION 21 Nursing care for the substance abuse client experiencing alcohol withdrawal delirium includes: A. Maintaining seizure precautions B. Restricting fluid intake C. Increasing sensory stimuli D. Applying ankle and wrist restraints Correct Answer: A Section: (none) Explanation Explanation/Reference: Explanation: (A) These clients are at high risk for seizures during the 1st week after cessation of alcohol intake. (B) Fluid intake should be increased to prevent dehydration. (C) Environmental stimuli should be decreased to prevent precipitation of seizures. (D) Application of restraints may cause the client to increase his or her physical activity and may eventually lead to exhaustion. QUESTION 22 A psychotic client who believes that he is God and rules all the universe is experiencing which type of delusion? A. Somatic B. Grandiose C. Persecutory D. Nihilistic Correct Answer: B Section: (none)Explanation Explanation/Reference: Explanation: (A) These delusions are related to the belief that an individual has an incurable illness. (B) These delusions are related to feelings of self-importance and uniqueness. (C) These delusions are related to feelings of being conspired against. (D) These delusions are related to denial of self- existence. QUESTION 23 A client confides to the nurse that he tasted poison in his evening meal. This would be an example of what type of hallucination? A. Auditory B. Gustatory C. Olfactory D. Visceral Correct Answer: B Section: (none) Explanation Explanation/Reference: Explanation: (A) Auditory hallucinations involve sensory perceptions of hearing. (B) Gustatory hallucinations involve sensory perceptions of taste. (C) Olfactory hallucinations involve sensory perceptions of smell. (D) Visceral hallucinations involve sensory perceptions of sensation. QUESTION 24 A schizophrenic client has made sexual overtures toward her physician on numerous occasions. During lunch, the client tells the nurse, "My doctor is in love with me and wants to marry me." This client is using which of the following defense mechanisms? A. Displacement B. Projection C. Reaction formation D. Suppression Correct Answer: B Section: (none) Explanation Explanation/Reference: Explanation:(A) Displacement involves transferring feelings to a more acceptable object. (B) Projection involves attributing one's thoughts or feelings to another person. (C) Reaction formation involves transforming an unacceptable impulse into the opposite behavior. (D) Suppression involves the intentional exclusion of unpleasant thoughts or experiences. QUESTION 25 Hypoxia is the primary problem related to near-drowning victims. The first organ that sustains irreversible damage after submersion in water is the: A. Kidney (urinary system) B. Brain (nervous system) C. Heart (circulatory system) D. Lungs (respiratory system) Correct Answer: B Section: (none) Explanation Explanation/Reference: Explanation: (A) The kidney can survive after 30 minutes of water submersion. (B) The cerebral neurons sustain irreversible damage after 46 minutes of water submersion. (C) The heart can survive up to 30 minutes of water submersion. (D) The lungs can survive up to 30 minutes of water submersion. QUESTION 26 One of the most dramatic and serious complications associated with bacterial meningitis is Waterhouse- Friderichsen syndrome, which is: A. Peripheral circulatory collapse B. Syndrome of inappropriate antiduretic hormone C. Cerebral edema resulting in hydrocephalus D. Auditory nerve damage resulting in permanent hearing loss Correct Answer: A Section: (none) Explanation Explanation/Reference: Explanation: (A) Waterhouse-Friderichsen syndrome is peripheral circulatory collapse, which may result in extensive and diffuse intravascular coagulation and thrombocytopenia resulting in death. (B) Syndrome of inappropriate antidiuretic hormone is a complication of meningitis, but it is not Waterhouse-Friderichsen syndrome. (C) Cerebral edema resulting in hydrocephalus is a complication of meningitis, but it is not Waterhouse-Friderichsen syndrome. (D) Auditory nerve damage resulting inpermanent hearing loss is a complication of meningitis, but it is not Waterhouse- Friderichsen syndrome. QUESTION 27 An 8-year-old child comes to the physician's office complaining of swelling and pain in the knees. His mother says, "The swelling occurred for no reason, and it keeps getting worse." The initial diagnosis is Lyme disease. When talking to the mother and child, questions related to which of the following would be important to include in the initial history? A. A decreased urinary output and flank pain B. A fever of over 103F occurring over the last 23 weeks C. Rashes covering the palms of the hands and the soles of the feet D. Headaches, malaise, or sore throat Correct Answer: D Section: (none) Explanation Explanation/Reference: Explanation: (A) Urinary tract symptoms are not commonly associated with Lyme disease. (B) A fever of 103F is not characteristic of Lyme disease. (C) The rash that is associated with Lyme diseasedoes not appear on the palms of the hands and the soles of the feet. (D) Classic symptoms of Lyme disease include headache, malaise, fatigue, anorexia, stiff neck, generalized lymphadenopathy, splenomegaly, conjunctivitis, sore throat, abdominal pain, and cough. QUESTION 28 The most commonly known vectors of Lyme disease are: A. Mites B. Fleas C. Ticks D. Mosquitoes Correct Answer: C Section: (none) Explanation Explanation/Reference: Explanation: (A) Mites are not the common vector of Lyme disease. (B) Fleas are not the common vector of Lyme disease. (C) Ticks are the common vector of Lyme disease. (D) Mosquitoes are not the common vector of Lyme disease.QUESTION 29 A laboratory technique specific for diagnosing Lyme disease is: A. Polymerase chain reaction B. Heterophil antibody test C. Decreased serum calcium level D. Increased serum potassium level Correct Answer: A Section: (none) Explanation Explanation/Reference: Explanation: (A) Polymerase chain reaction is the laboratory technique specific for Lyme disease. (B) Heterophil antibody test is used to diagnose mononucleosis. (C) Lyme disease does not decrease the serum calcium level. (D) Lyme disease does not increase the serum potassium level. QUESTION 30 The nurse would expect to include which of the following when planning the management of the client with Lyme disease? A. Complete bed rest for 68 weeks B. Tetracycline treatment C. IV amphotericin B D. High-protein diet with limited fluids Correct Answer: B Section: (none) Explanation Explanation/Reference: Explanation: (A) The client is not placed on complete bed rest for 6 weeks. (B) Tetracycline is the treatment of choice for children with Lyme disease who are over the age of 9. (C) IV amphotericin B is the treatment for histoplasmosis. (D) The client is not restricted to a high-protein diet with limited fluids. QUESTION 31 A 3-year-old child is hospitalized with burns covering her trunk and lower extremities. Which of the following would the nurse use to assess adequacy of fluid resuscitation in the burned child?A. Blood pressure B. Serum potassium level C. Urine output D. Pulse rate Correct Answer: C Section: (none) Explanation Explanation/Reference: Explanation: (A) Blood pressure can remain normotensive even in a state of hypovolemia. (B) Serum potassium is not reliable for determining adequacy of fluid resuscitation. (C) Urine output, alteration in sensorium, and capillary refill are the most reliable indicators for assessing adequacy of fluid resuscitation. (D) Pulse rate may vary for many reasons and is not a reliable indicator for assessing adequacy of fluid resuscitation. QUESTION 32 Proper positioning for the child who is in Bryant's traction is: A. Both hips flexed at a 90-degree angle with the knees extended and the buttocks elevated off the bed B. Both legs extended, and the hips are not flexed C. The affected leg extended with slight hip flexion D. Both hips and knees maintained at a 90-degree flexion angle, and the back flat on the bed Correct Answer: A Section: (none) Explanation Explanation/Reference: Explanation: (A) The child's weight supplies the countertraction for Bryant's traction; the buttocks are slightly elevated off the bed, and the hips are flexed at a 90-degree angle. Both legs are suspended by skin traction. (B) The child in Buck's extension traction maintains the legs extended and parallel to the bed. (C) The child in Russell traction maintains hip flexion of the affected leg at the prescribed angle with the leg extended. (D) The child in "9090" traction maintains both hips and knees at a90-degree flexion angle and the back is flat on the bed. QUESTION 33 A child sustains a supracondylar fracture of the femur. When assessing for vascular injury, the nurse should be alert for the signs of ischemia, which include:A. Bleeding, bruising, and hemorrhage B. Increase in serum levels of creatinine, alkaline phosphatase, and aspartate transaminase C. Pain, pallor, pulselessness, paresthesia, and paralysis D. Generalized swelling, pain, and diminished functional use with muscle rigidity and crepitus Correct Answer: C Section: (none) Explanation Explanation/Reference: Explanation: (A) Bleeding, bruising, and hemorrhage may occur due to injury but are not classic signs of ischemia. (B) An increase in serum levels of creatinine, alkaline phosphatase, and aspartate transaminase is related to the disruption of muscle integrity. (C) Classic signs of ischemia related to vascular injury secondary to long bone fractures include the five "P's": pain, pallor, pulselessness, paresthesia, and paralysis. (D) Generalized swelling, pain, and diminished functional use with muscle rigidity and crepitus are common clinical manifestations of a fracture but not ischemia. QUESTION 34 When administering phenytoin (Dilantin) to a child, the nurse should be aware that a toxic effect of phenytoin therapy is: A. Stephens-Johnson syndrome B. Folate deficiency C. Leukopenic aplastic anemia D. Granulocytosis and nephrosis Correct Answer: A Section: (none) Explanation Explanation/Reference: Explanation: (A) Stephens-Johnson syndrome is a toxic effect of phenytoin. (B) Folate deficiency is a side effect of phenytoin, but not a toxic effect. (C) Leukopenic aplastic anemia is a toxic effect of carbamazepine (Tegretol). (D) Granulocytosis and nephrosis are toxic effects of trimethadione (Tridione). QUESTION 35 A six-month-old infant has been admitted to the emergency room with febrile seizures. In the teaching of the parents, the nurse states that: A. Sustained temperature elevation over 103F is generally related to febrile seizures B. Febrile seizures do not usually recurC. There is little risk of neurological deficit and mental retardation as sequelae to febrile seizures D. Febrile seizures are associated with diseases of the central nervous system Correct Answer: C Section: (none) Explanation Explanation/Reference: Explanation: (A) The temperature elevation related to febrile seizures generally exceeds 101F, and seizures occur during the temperature rise rather than after a prolonged elevation. (B) Febrile seizures may recur and are more likely to do so when the first seizure occurs in the 1st year of life. (C) There is little risk of neurologicaldeficit, mental retardation, or altered behavior secondary to febrile seizures. (D) Febrile seizures are associated with disease of the central nervous system. QUESTION 36 A child is admitted to the emergency room with her mother. Her mother states that she has been exposed to chickenpox. During the assessment, the nurse would note a characteristic rash: A. That is covered with vesicular scabs all in the macular stage B. That appears profusely on the trunk and sparsely on the extremities C. That first appears on the neck and spreads downward D. That appears especially on the cheeks, which gives a"slapped-cheek" appearance Correct Answer: B Section: (none) Explanation Explanation/Reference: Explanation: (A) A rash with vesicular scabs in all stages (macule, papule, vesicle, and crusts). (B) A rash that appears profusely on the trunk and sparsely on the extremities. (C) A rash that first appears on the neck and spreads downward is characteristic of rubeola and rubella. (D) A rash, especially on the cheeks, that gives a "slapped- cheek" appearance is characteristic of roseola. QUESTION 37 The priority nursing goal when working with an autistic child is: A. To establish trust with the child B. To maintain communication with the family C. To promote involvement in school activitiesD. To maintain nutritional requirements Correct Answer: A Section: (none) Explanation Explanation/Reference: Explanation: (A) The priority nursing goal when working with an autistic child is establishing a trusting relationship. (B) Maintaining a relationship with the family is important but having the trust of the child is a priority. (C) To promote involvement in school activities is inappropriate for a child who is autistic. (D) Maintaining nutritional requirements is not the primary problem of the autistic child. QUESTION 38 The child with iron poisoning is given IV deforoxamine mesylate (Desferal). Following administration, the child suffers hypotension, facial flushing, and urticaria. The initial nursing intervention would be to: A. Discontinue the IV B. Stop the medication, and begin a normal saline infusion C. Take all vital signs, and report to the physician D. Assess urinary output, and if it is 30 mL an hour, maintain current treatment Correct Answer: B Section: (none) Explanation Explanation/Reference: Explanation: (A) The IV line should not be discontinued because other IV medications will be needed. (B) Stop the medication and begin a normal saline infusion. The child is exhibiting signs of an allergic reaction and could go into shock if the medication is not stopped. The line should be kept opened for other medication. (C) Takingvital signs and reporting to the physician is not an adequate intervention because the IV medication continues to flow. (D) Assessing urinary output and, if it is 30 mL an hour, maintaining current treatment is an inappropriate intervention owing to the child's obvious allergic reaction. QUESTION 39 When teaching a mother of a 4-month-old with diarrhea about the importance of preventing dehydration, the nurse would inform the mother about the importance of feeding her child: A. Fruit juices B. Diluted carbonated drinksC. Soy-based, lactose-free formula D. Regular formulas mixed with electrolyte solutions Correct Answer: C Section: (none) Explanation Explanation/Reference: Explanation: (A) Diluted fruit juices are not recommended for rehydration because they tend to aggravate the diarrhea. (B) Diluted soft drinks have a high-carbohydrate content, which aggravates the diarrhea. (C) Soy-based, lactose-free formula reduces stool output and duration of diarrhea in most infants. (D) Regular formulas contain lactose, which can increase diarrhea. QUESTION 40 To appropriatelymonitor therapy and client progress, the nurse should be aware that increased myocardial work and O2 demand will occur with which of the following? A. Positive inotropic therapy B. Negative chronotropic therapy C. Increase in balance of myocardial O2 supply and demand D. Afterload reduction therapy Correct Answer: A Section: (none) Explanation Explanation/Reference: Explanation: (A) Inotropic therapy will increase contractility, which will increase myocardial O2 demand. (B) Decreased heart rate to the point of bradycardia will increase coronary artery filling time. This should be used cautiously because tachycardia may be a compensatory mechanism to increase cardiac output. (C) The goal in the care of the MI client with angina is to maintain a balance between myocardial O2 supply and demand. (D) Decrease in systemic vascular resistance by drug therapy, such as IV nitroglycerin or nitroprusside, or intra-aortic balloon pump therapy, would decrease myocardial work and O2 demand. QUESTION 41 Which of the following medicationsrequires close observation for bronchospasm in the client with chronic obstructive pulmonary disease or asthma? A. Verapamil (Isoptin) B. Amrinone (Inocor)C. Epinephrine (Adrenalin) D. Propranolol (Inderal) Correct Answer: D Section: (none) Explanation Explanation/Reference: Explanation: (A) Verapamil has the respiratory side effect of nasal or chest congestion, dyspnea, shortness of breath (SOB), and wheezing. (B) Amrinone has the effect of increased contractility and dilation of the vascular smooth muscle. It has no noted respiratory side effects. (C) Epinephrine has the effect of bronchodilation through stimulation. (D) Propranolol, esmolol, and labetalol are all - blocking agents, which can increase airway resistance and cause bronchospasms. QUESTION 42 In the client with a diagnosis of coronary artery disease, the nurse would anticipate the complication of bradycardia with occlusion of which coronary artery? A. Right coronary artery B. Left main coronary artery C. Circumflex coronary artery D. Left anterior descending coronary artery Correct Answer: A Section: (none) Explanation Explanation/Reference: Explanation: (A) Sinus bradycardia and atrioventricular (AV) heart block are usually a result of right coronary artery occlusion. The right coronary artery perfuses the sinoatrial and AV nodes in mostindividuals. (B) Occlusion of the left main coronary artery causes bundle branch blocks and premature ventricular contractions. (C) Occlusion of the circumflex artery does not cause bradycardia. (D) Sinus tachycardia occurs primarily with left anterior descending coronary artery occlusion because this form of occlusion impairs left ventricular function. QUESTION 43 When a client questions the nurse as to the purpose of exercise electrocardiography (ECG) in the diagnosis of cardiovascular disorders, the nurse's response should be based on the fact that: A. The test provides a baseline for further tests B. The procedure simulates usual daily activity and myocardial performanceC. The client can be monitored while cardiac conditioning and heart toning are done D. Ischemia can be diagnosed because exercise increasesO2 consumption and demand Correct Answer: D Section: (none) Explanation Explanation/Reference: Explanation: (A) The purpose of the study is not to provide a baseline for further tests. (B) The test causes an increase in O2 demand beyond that required to perform usual daily activities. (C) Monitoring does occur, but the test is not for the purpose of cardiac toning and conditioning. (D) Exercise ECG, or stress testing, is designed to elevate the peripheral and myocardial needs for O2 to evaluate the ability of the myocardium and coronary arteries to meet the additional demands. QUESTION 44 The cardiac client who exhibits the symptoms of disorientation, lethargy, and seizures may be exhibiting a toxic reaction to: A. Digoxin (Lanoxin) B. Lidocaine (Xylocaine) C. Quinidine gluconate or sulfate (Quinaglute,Quinidex) D. Nitroglycerin IV (Tridil) Correct Answer: B Section: (none) Explanation Explanation/Reference: Explanation: (A) Side effects of digoxin include headache, hypotension, AV block, blurred vision, and yellow-green halos. (B) Side effects of lidocaine include heart block, headache, dizziness, confusion, tremor, lethargy, and convulsions. (C) Side effects of quinidine include heart block, hepatotoxicity, thrombocytopenia, and respiratory depression. (D) Side effects of nitroglycerin include postural hypotension, headache, dizziness, and flushing. QUESTION 45 Which of the following ECG changes would be seen as a positive myocardial stress test response? A. Hyperacute T wave B. Prolongation of the PR interval C. ST-segment depressionD. Pathological Q wave Correct Answer: C Section: (none) Explanation Explanation/Reference: Explanation: (A) Hyperacute T waves occur with hyperkalemia. (B) Prolongation of the P R interval occurs with first-degree AV block. (C) Horizontal ST-segment depression of>1 mm during exercise isdefinitely a positive criterion on the exercise ECG test. (D) Patho-logical Q waves occur with MI. QUESTION 46 Clinical manifestations seen in left-sided rather than in right-sided heart failure are: A. Elevated central venous pressure and peripheral edema B. Dyspnea and jaundice C. Hypotension and hepatomegaly D. Decreased peripheral perfusion and rales Correct Answer: D Section: (none) Explanation Explanation/Reference: Explanation: (A, B, C) Clinical manifestations of right-sided heart failure are weakness, peripheral edema, jugular venous distention, hepatomegaly, jaundice, and elevated central venous pressure. (D) Clinical manifestations of left-sided heart failure are left ventricular dysfunction, decreased cardiac output, hypotension, and thebackward failure as a result of increased left atrium and pulmonary artery pressures, pulmonary edema, and rales. QUESTION 47 To ensure proper client education, the nurse should teach the client taking SL nitroglycerin to expect which of the following responses with administration? A. Stinging, burning when placed under the tongue B. Temporary blurring of vision C. Generalized urticaria with prolonged use D. Urinary frequencyCorrect Answer: A Section: (none) Explanation Explanation/Reference: Explanation: (A) Stinging or burning when nitroglycerin is placed under the tongue is to be expected. This effect indicates that the medication is potent and effective for use. Failure to have this response means that the client needs to get a new bottle of nitroglycerin. (B, C, D) The other responses are not expected in this situation and are not even side effects. QUESTION 48 Which of the following would differentiate acute from chronic respiratory acidosis in the assessment of the trauma client? A. Increased PaCO2 B. Decreased PaO2 C. Increased HCO3 D. Decreased base excess Correct Answer: C Section: (none) Explanation Explanation/Reference: Explanation: (A) Increased CO2 will occur in both acute and chronic respiratory acidosis. (B) Hypoxia does not determine acid-base status. (C) Elevation of HCO3 is a compensatory mechanism in acidosis that occurs almost immediately, but it takes hours to show any effect and days to reach maximum compensation. Renal disease and diuretic therapymay impair the ability of the kidneys to compensate. (D) Base excess is a nonrespiratory contributor to acid-base balance. It would increase to compensate for acidosis. QUESTION 49 Hematotympanum and otorrhea are associated with which of the following head injuries? A. Basilar skull fracture B. Subdural hematoma C. Epidural hematoma D. Frontal lobe fracture Correct Answer: ASection: (none) Explanation Explanation/Reference: Explanation: (A) Basilar skull fractures are fractures of the base of the skull. Blood behind the eardrum or blood or cerebrospinal fluid (CSF) leaking from the ear are indicative of a dural laceration. Basilar skull fractures are the only type with these symptoms. (B, C, D) These do not typically cause dural lacerations and CSF leakage. QUESTION 50 A client with a C-34 fracture has just arrived in the emergency room. The primary nursing intervention is: A. Stabilization of the cervical spine B. Airway assessment and stabilization C. Confirmation of spinal cord injury D. Normalization of intravascular volume Correct Answer: B Section: (none) Explanation Explanation/Reference: Explanation: (A) If cervical spine injury is suspected, the airway should be maintained using the jaw thrust method that also protects the cervical spine. (B) Primary intervention is protection of the airway and adequate ventilation. (C, D) All other interventions are secondary to adequate ventilation. QUESTION 51 In a client with chest trauma, the nurse needs to evaluate mediastinal position. This can best be done by: A. Auscultating bilateral breath sounds B. Palpating for presence of crepitus C. Palpating for trachial deviation D. Auscultating heart sounds Correct Answer: C Section: (none) Explanation Explanation/Reference:Explanation: (A) No change in the breath sounds occurs as a direct result of the mediastinal shift. (B) Crepitus can occur owing to the primary disorder, not to the mediastinal shift. (C) Mediastinal shift occurs primarily with tension pneumothorax, but it can occur with very large hemothorax or pneumothorax. Mediastinal shift causes trachial deviation and deviation of the heart's point of maximum impulse. (D) No change in the heart sounds occurs as a result of the mediastinal shift. QUESTION 52 Priapism may be a sign of: A. Altered neurological function B. Imminent death C. Urinary incontinence D. Reproductive dysfunction Correct Answer: A Section: (none) Explanation Explanation/Reference: Explanation: (A) Priapism in the trauma client is due to the neurological dysfunction seen in spinal cord injury. Priapism is an abnormal erection of the penis; it may be accompanied by pain and tenderness. This may disappear as spinal cord edema isrelieved. (B) Priapism is not associated with death. (C) Urinary retention, rather than incontinence, may occur. (D) Reproductive dysfunction may be a secondary problem. QUESTION 53 When evaluating a client with symptoms of shock, it is important for the nurse to differentiate between neurogenic and hypovolemic shock. The symptoms of neurogenic shock differ from hypovolemic shock in that: A. In neurogenic shock, the skin is warm and dry B. In hypovolemic shock, there is a bradycardia C. In hypovolemic shock, capillary refill is less than 2 seconds D. In neurogenic shock, there is delayed capillary refill Correct Answer: A Section: (none) Explanation Explanation/Reference: Explanation:(A) Neurogenic shock is caused by injury to the cervical region, which leads to loss of sympathetic control. This loss leads to vasodilation of the vascular beds, bradycardia resulting from the lack of sympathetic balance to parasympathetic stimuli from the vagus nerve, and the loss of the ability to sweat below the level of injury. In neurogenic shock, the client is hypotensive but bradycardiac with warm, dry skin. (B) In hypovolemic shock, the client ishypotensive and tachycardiac with cool skin. (C) In hypovolemic shock, the capillary refill would be>5 seconds. (D) In neurogenic shock, there is no capillary delay, the vascular beds are dilated, and peripheral flow is good. QUESTION 54 Which of the following would have the physiological effect of decreasing intracranial pressure (ICP)? A. Increased core body temperature B. Decreased serum osmolality C. Administration of hypo-osmolar fluids D. Decreased PaCO2 Correct Answer: D Section: (none) Explanation Explanation/Reference: Explanation: (A) An increase in core body temperature increases metabolism and results in an increase in ICP. (B) Decreased serum osmolality indicates a fluid overload andmay result in an increase in ICP. (C) Hypo-osmolar fluids are generally voided in the neurologically compromised. Using IV fluids such as D5W results in the dextrose being metabolized, releasing free water that is absorbed by the brain cells, leading to cerebral edema. (D) Hypercapnia and hypoventilation, which cause retention of CO2 and lead to respiratory acidosis, both increase ICP. CO2 is the most potent vasodilator known. QUESTION 55 A client who has sustained a basilar skull fracture exhibits blood-tinged drainage from his nose. After establishing a clear airway, administering supplemental O2, and establishing IV access, the next nursing intervention would be to: A. Pass a nasogastric tube through the left nostril B. Place a 4 X 4 gauze in the nares to impede the flow C. Gently suction the nasal drainage to protect the airway D. Perform a halo test and glucose level on the drainage Correct Answer: D Section: (none) ExplanationExplanation/Reference: Explanation: (A) Basilar skull fracture may cause dural lacerations, which result in CSF leaking from the ears or nose. Insertion of a tube could lead to CSF going into the brain tissue or sinuses. (B) Tamponading flow could worsen the problem and increase ICP. (C) Suction could increase brain damage and dislocate tissue. (D) Testing the fluid from the nares would determine the presence of CSF. Elevation of the head, notification of the medical staff, and prophylactic antibiotics are appropriatetherapy. QUESTION 56 A client with a diagnosis of C-4 injury has been stabilized and isready for discharge. Because this client is at risk for autonomic dysreflexia, he and his family should be instructed to assess for and report: A. Dizziness and tachypnea B. Circumoral pallor and lightheadedness C. Headache and facial flushing D. Pallor and itching of the face and neck Correct Answer: C Section: (none) Explanation Explanation/Reference: Explanation: (A) Tachypnea is not a symptom. (B) Circumoral pallor is not a symptom. (C) Autonomic dysreflexia is an uninhibited and exaggerated reflex of the autonomic nervous system to stimulation, which results in vasoconstriction and elevated blood pressure. (D) Pallor and itching are not symptoms. QUESTION 57 The initial treatment for a client with a liquid chemical burn injury is to: A. Irrigate the area with neutralizing solutions B. Flush the exposed area with large amounts of water C. Inject calcium chloride into the burned area D. Apply lanolin ointment to the area Correct Answer: B Section: (none) Explanation Explanation/Reference:Explanation: (A) In the past, neutralizing solutions were recommended, but presently there is concern that these solutions extend the depth of burn area. (B) The use of large amounts of water to flush the area is recommended for chemical burns. (C) Calcium chloride is not recommended therapy and would likely worsen the problem. (D) Lanolin is of no benefit in the initial treatment of a chemical injury and may actually extend a thermal injury. QUESTION 58 The most important reason to closely assess circumferential burns at least every hour is that they may result in: A. Hypovolemia B. Renal damage C. Ventricular arrhythmias D. Loss of peripheral pulses Correct Answer: D Section: (none) Explanation Explanation/Reference: Explanation: (A) Hypovolemia could be a result of fluid loss from thermal injury, but not as a result of the circumferential injury. (B) Renal damage is typically seen because of prolonged hypovolemia or myoglobinuria. (C) Electrical injuries and electrolyte changes typically cause arrhythmias in the burn client. (D) Full-thickness circumferential burns are nonelastic and result in an internal tourniquet effect that compromises distal blood flow when the area involved is an extremity.Circumferential full-thickness torso burns compromise respiratory motion and, when extreme, cardiac return. QUESTION 59 During burn therapy, morphine is primarily administered IV for pain management because this route: A. Delays absorption to provide continuous pain relief B. Facilitates absorption because absorption from muscles is not dependable C. Allows for discontinuance of the medication if respiratory depression develops D. Avoids causing additional pain from IM injections Correct Answer: B Section: (none) Explanation Explanation/Reference: Explanation:(A) Absorption would be increased, not decreased. (B) IM injections should not be used until the client is hemodynamically stable and has adequate tissue perfusion. Medications will remain in the subcutaneous tissue with the fluid that is present in the interstitial spaces in the acute phase of the thermal injury. The client will have a poor response to the medication administered, and a "dumping" of the medication can occur when the medication and fluid are shifted back into the intravascular spaces in the next phase of healing. (C) IV administration of the medication would hasten respiratory compromise, if present. (D) The desire toavoid causing the client additional pain is not a primary reason for this route of administration. QUESTION 60 The medication that best penetrates eschar is: A. Mafenide acetate (Sulfamylon) B. Silver sulfadiazine (Silvadene) C. Neomycin sulfate (Neosporin) D. Povidone-iodine (Betadine) Correct Answer: A Section: (none) Explanation Explanation/Reference: Explanation: (A) Mafenide acetate is bacteriostatic against gram-positive and gram-negative organisms and is the agent that best penetrates eschar. (B) Silver sulfadiazine poorly penetrates eschar. (C) Neomycin sulfate does not penetrate eschar. (D) Povidoneiodine does not penetrate eschar. QUESTION 61 When the nurse is evaluating lab data for a client 1824 hours after a major thermal burn, the expected physiological changes would include which of the following? A. Elevated serum sodium B. Elevated serum calcium C. Elevated serum protein D. Elevated hematocrit Correct Answer: D Section: (none) Explanation Explanation/Reference: Explanation:(A) Sodium enters the edema fluid in the burned area, lowering the sodium content of the vascular fluid. Hyponatremia may continue for days to several weeks because of sodium loss to edema, sodium shifting into the cells, and later, diuresis. (B) Hypocalcemia occurs because of calcium loss to edema fluid at the burned site (third space fluid). (C) Protein loss occurs at the burn site owing to increased capillary permeability. Serum protein levels remain low until healing occurs. (D) Hematocrit level is elevated owing to hemoconcentration from hypovolemia. Anemia is present in the postburn stage owing to blood loss and hemolysis, but it cannot be assessed until the client is adequately hydrated. QUESTION 62 The nurse notes hyperventilation in a client with a thermal injury. She recognizes that this may be a reaction to which of the following medications if applied in large amounts? A. Neosporin sulfate B. Mafenide acetate C. Silver sulfadiazine D. Povidone-iodine Correct Answer: B Section: (none) Explanation Explanation/Reference: Explanation: (A) The side effects of neomycin sulfate include rash, urticaria, nephrotoxicity, and ototoxicity. (B) The side effects of mafenide acetate include bone marrow suppression, hemolytic anemia, eosinophilia, and metabolic acidosis. The hyperventilation is a compensatory response to the metabolic acidosis. (C) The side effects of silver sulfadiazine include rash, itching, leukopenia, and decreased renal function. (D) The primary side effect of povidone- iodine is decreased renal function. QUESTION 63 The primary reason for sending a burn client home with a pressure garment, such as a Jobst garment, is that the garment: A. Decreases hypertrophic scar formation B. Assists with ambulation C. Covers burn scars and decreases the psychological impact during recovery D. Increases venous return and cardiac output by normalizing fluid status Correct Answer: A Section: (none) Explanation Explanation/Reference: Explanation:(A) Tubular support, such as that received with a Jobst garment, applies tension of 1020 mm Hg. This amount of uniform pressure is necessary to prevent or reduce hypertrophic scarring. Clients typically wear a pressure garment for 612 months during the recovery phase of their care. (B) Pressure garments have no ambulatory assistive properties. (C) Pressure garments can worsen the psychological impact of burn injury, especially if worn on the face. (D) Pressure garments do not normalize fluid status. QUESTION 64 A client with emphysema is placed on diuretics. In order to avoid potassium depletion as a side effect of the drug therapy, which of the following foods should be included in his diet? A. Celery B. Potatoes C. Tomatoes D. Liver Correct Answer: B Section: (none) Explanation Explanation/Reference: Explanation: (A) Celery is high in sodium. (B) Potatoes are high in potassium. (C) Tomatoes are high in sodium. (D) Liver is high in iron. QUESTION 65 Which of the following would the nurse expect to find following respiratory assessment of a client with advanced emphysema? A. Distant breath sounds B. Increased heart sounds C. Decreased anteroposterior chest diameter D. Collapsed neck veins Correct Answer: A Section: (none) Explanation Explanation/Reference: Explanation: (A) Distant breath sounds are found in clients with emphysema owing to increased anteroposterior chest diameter, overdistention, and air trapping. (B) Deceased heart sounds arepresent because of the increased anteroposterior chest diameter. (C) A barrel- shaped chest is characteristic of emphysema. (D) Increaseddistention of neck veins is found owing to right- sided heart failure, which may be present in advanced emphysema. QUESTION 66 The nurse assists a client with advanced emphysema to the bathroom. The client becomes extremely short of breath while returning to bed. The nurse should: A. Increase his nasal O2 to 6 L/min B. Place him in a lateral Sims' position C. Encourage pursed-lip breathing D. Have him breathe into a paper bag Correct Answer: C Section: (none) Explanation Explanation/Reference: Explanation: (A) Giving too high a concentration of O2 to a client with em-physema may remove his stimulus to breathe. (B) The client should sit forward with his hands on his knees or an overbed table and with shoulders elevated. (C) Pursed-lip breathing helps the client to blow off CO2 and to keep air passages open. (D) Covering the face of a client extremely short of breath may cause anxiety and further increase dyspnea. QUESTION 67 Signs and symptoms of an allergy attack include which of the following? A. Wheezing on inspiration B. Increased respiratory rate C. Circumoral cyanosis D. Prolonged expiration Correct Answer: D Section: (none) Explanation Explanation/Reference: Explanation: (A) Wheezing occurs during expiration when air movement is impaired because of constricted edematous bronchial lumina. (B) Respirations are difficult, but the rate is frequently normal. (C) The circumoral area is usually pale. Cyanosis is not an early sign of hypoxia. (D) Expiration is prolonged because the alveoli are greatly distended and air trapping occurs. QUESTION 68A 55-year-old man is admitted to the hospital with complaints of fatigue, jaundice, anorexia, and clay-colored stools. His admitting diagnosis is "rule out hepatitis." Laboratory studies reveal elevated liver enzymes and bilirubin. In obtaining his health history, the nurse should assess his potential for exposure to hepatitis. Which of the following represents a high-risk group for contracting this disease? A. Heterosexual males B. Oncology nurses C. American Indians D. Jehovah's Witnesses Correct Answer: B Section: (none) Explanation Explanation/Reference: Explanation: (A) Homosexual males, not heterosexual males, are at high risk for contracting hepatitis. (B) Oncology nurses are employed in high-risk areas and perform invasive procedures that expose them to potential sources of infection. (C) The literature does not support the idea that any ethnic groups are at higher risk. (D) There is no evidence that any religious groups are at higher risk. QUESTION 69 A diagnosis of hepatitis C is confirmed by a male client's physician. The nurse should be knowledgeable of the differences between hepatitis A, B, and C. Which of the following are characteristics of hepatitis C? A. The potential for chronic liver disease is minimal. B. The onset of symptoms is abrupt. C. The incubation period is 226 weeks. D. There is an effective vaccine for hepatitis B, but not for hepatitis C. Correct Answer: C Section: (none) Explanation Explanation/Reference: Explanation: (A) Hepatitis C and B may result in chronic liver disease. Hepatitis A has a low potential for chronic liver disease. (B) Hepatitis C and B have insidious onsets. Hepatitis A has an abrupt onset. (C) Incubation periods are as follows: hepatitis C is 226 weeks, hepatitis B is 620 weeks, and hepatitis A is 26 weeks. (D) Only hepatitis B has an effective vaccine. QUESTION 70The nurse is aware that nutrition is an important aspect of care for a client with hepatitis. Which of the following diets would be most therapeutic? A. High protein and low carbohydrate B. Low calorie and low protein C. High carbohydrate and high calorie D. Low carbohydrate and high calorie Correct Answer: C Section: (none) Explanation Explanation/Reference: Explanation: (A) Protein increases the workload of the liver. Increased carbohydrates provide needed calories and promote palatability. (B) Dietary intake should be adequate to ensure wound healing. (C) Increased carbohydrates provide needed calories. (D) A highcalorie diet is best obtained from carbohydrates because of their palatability. Fats increase the workload of the liver. QUESTION 71 Which of the following nursing orders should be included in the plan of care for a client with hepatitis C? A. The nurse should use universal precautions when obtaining blood samples. B. Total bed rest should be maintained until the client is asymptomatic. C. The client should be instructed to maintain a low semi-Fowler position when eating meals. D. The nurse should administer an alcohol backrub at bedtime. Correct Answer: A Section: (none) Explanation Explanation/Reference: Explanation: (A) The source of infection with hepatitis C is contaminated blood products. (B) Modified bed rest should be maintained while the client is symptomatic. Routine activities can be slowly resumed once the client is asymptomatic. (C) Nausea and vomiting occur frequently with hepatitis C. A high Fowler position may decrease the tendency to vomit. (D) The buildup of bilirubin in the client's skin may cause pruritus. Alcohol is a drying agent. QUESTION 72 A 27-year-old man was diagnosed with type I diabetes 3 months ago. Two weeks ago he complained of pain, redness, and tenderness in hisright lower leg. He is admitted to the hospital with a slight elevation of temperature and vague complaints of "not feeling well." At 4:30 PM on the day of his admission, his blood glucose level is 50 mg; dinner will be served at 5:00 PM. The best nursing action would be to:A. Give him 3 tbsp of sugar dissolved in 4 oz of grape juice to drink B. Ask him to dissolve three pieces of hard candy in his mouth C. Have him drink 4 oz of orange juice D. Monitor him closely until dinner arrives Correct Answer: C Section: (none) Explanation Explanation/Reference: Explanation: (A) The combination of sugar and juice will increase the blood sugar beyond the normal range. (B) Concentrated sweets are not absorbed as fast as juice; consequently, they elevate the blood sugar beyond the normal limit. (C) Four ounces of orange juice will act immediately to raise the blood sugar to a normal level and sustain it for 30 minutes until supper is served. (D) There is an increased potential for the client's blood sugar to decrease even further, resulting in diabetic coma. QUESTION 73 A male client receives 10 U of regular human insulin SC at 9:00 AM. The nurse would expect peak action from this injection to occur at: A. 9:30 AM B. 10:30 AM C. 12 noon D. 4:00 PM Correct Answer: C Section: (none) Explanation Explanation/Reference: Explanation: (A) This is too early for peak action to occur. (B) This is too early for peak action to occur. (C) Regular insulin peak action occurs 24 hours after administration. (D) This is too late for peak action to occur. QUESTION 74 The physician has ordered that a daily exercise program be instituted by a client with type I diabetes following his discharge from the hospital. Discharge instructions about exercise should include which of the following?A. Exercise should be performed 30 minutes before meals. B. A snack may be needed before and/or during exercise. C. Hyperglycemia may occur 24 hours after exercise. D. The blood glucose level should be 100 mg or below before exercise is begun. Correct Answer: B Section: (none) Explanation Explanation/Reference: Explanation: (A) Exercise should not be performed before meals because the blood sugar is usually lower just prior to eating; therefore, there is an increased risk for hypoglycemia. (B) Exercise lowers blood sugar levels; therefore, a snack may be needed to maintain the appropriate glucose level. (C) Exercise lowers blood sugar levels. (D) Exercise lowers blood sugar levels. If the blood glucose level is 100 mg or below at the start of exercise, the potential for hypoglycemia is greater. QUESTION 75 A 74-year-old female client is 3 days postoperative. She has an indwelling catheter and has been progressing well. While the nurse is in the room, the client states, "Oh dear, I feel like I have to urinate again!" Which of the following is the most appropriate initial nursing response? A. Assure her that this is most likely the result of bladder spasms. B. Check the collection bag and tubing to verify that the catheter is draining properly. C. Instruct her to do Kegel exercises to diminish the urge to void. D. Ask her if she has felt this way before. Correct Answer: B Section: (none) Explanation Explanation/Reference: Explanation: (A) Although this may be an appropriate response, the initial response would be to assure the patency of the catheter. (B) The most frequent reason for an urge to void with an indwelling catheter is blocked tubing. Thisresponse would be the best initial response. (C) Kegel exercises while a retention catheter is in place would not help to prevent a voiding urge and could irritate the urethral sphincter. (D) Though the nurse would want to ascertain whether the client has felt the same urgeto void before, the initial response should be to assure the patency of the catheter. QUESTION 76 Nursing interventions designed to decrease the risk of infection in a client with an indwelling catheter include:A. Cleanse area around the meatus twice a day B. Empty the catheter drainage bag at least daily C. Change the catheter tubing and bag every 48 hours D. Maintain fluid intake of 12001500 mL every day Correct Answer: A Section: (none) Explanation Explanation/Reference: Explanation: (A) Catheter site care is to be done at least twice daily to prevent pathogen growth at the catheter insertion site. (B) Catheter drainage bags are usually emptied every 8 hours to prevent urine stasis and pathogen growth. (C) Tubing and collection bags are not changed this often, because research studies have not demonstrated the efficacy of this practice. (D) Fluid intake needs to be in the 20002500 mL range if possible to help irrigate the bladder and prevent infection. QUESTION 77 A client tells the nurse that she has had a history of urinary tract infections. The nurse would do further health teaching if she verbalizes she will: A. Drink at least 8 oz of cranberry juice daily B. Maintain a fluid intake of at least 2000 mL daily C. Wash her hands before and after voiding D. Limit her fluid intake after 6 PM so that there is not a great deal of urine in her bladder while she sleeps Correct Answer: D Section: (none) Explanation Explanation/Reference: Explanation: (A) Cranberry juice helps to maintain urine acidity, thereby retarding bacterial growth. (B) A generous fluid intake will help to irrigate the bladder and to preventbacterial growth within the bladder. (C) Hand washing is an effective means of preventing pathogen transmission. (D) Restricting fluid intake would contribute to urinary stasis, which in turn would contribute to bacterial growth. QUESTION 78 The nurse provides a male client with diet teaching so that he can help prevent constipation in the future. Which food choices indicate that this teaching has been understood?A. Omelette and hash browns B. Pancakes and syrup C. Bagel with cream cheese D. Cooked oatmeal and grapefruit half Correct Answer: D Section: (none) Explanation Explanation/Reference: Explanation: (A) Eggs and hash browns do not provide much fiber and bulk, so they do not effectively prevent constipation. (B) Pancakes and syrup also have little fiber and bulk, so they do not effectively prevent constipation. (C) Bagel and cream cheese do not provide intestinal bulk. (D) A combination of oatmeal and fresh fruit will provide fiber and intestinal bulk. QUESTION 79 The nurse teaches a male client ways to reduce the risks associated with furosemide therapy. Which of the following indicates that he understands this teaching? A. "I'll be sure to rise slowly and sit for a few minutes after lying down." B. "I'll be sure to walk at least 23 blocks every day." C. "I'll be sure to restrict my fluid intake to four or five glasses a day." D. "I'll be sure not to take any more aspirin while I amon this drug." Correct Answer: A Section: (none) Explanation Explanation/Reference: Explanation: (A) This response will help to prevent the occurrence of postural hypotension, a common side effect of this drug and a common reason for falls. (B) Although walking is an excellent exercise, it is not specific to the reduction of risks associated with diuretic therapy. (C) Clients on diuretic therapy are generally taught to ensure that their fluid intake is at least 20003000 mL daily, unless contraindicated. (D) Aspirin is a safe drug to take along with furosemide. QUESTION 80 The nurse would be sure to instruct a client on the signs and symptoms of an eye infection and hemorrhage. These signs and symptoms would include: A. Blurred vision and dizzinessB. Eye pain and itching C. Feeling of eye pressure and headache D. Eye discharge and hemoptysis Correct Answer: B Section: (none) Explanation Explanation/Reference: Explanation: (A) Although blurred vision may occur, dizziness would not be associated with an infection or hemorrhage. (B) Eye pain is a symptom of hemorrhage within the eye, and itching is associated with infection. (C) Nausea and headache would not be usual symptoms of eye hemorrhage or infection. (D) Some eye discharge might be anticipated if an infection is present; hemoptysis would not. QUESTION 81 With a geriatric client, the nurse should also assess whether he has been obtaining a yearly vaccination against influenza. Why is this assessment important? A. Influenza is growing in our society. B. Older clients generally are sicker than others when stricken with flu. C. Older clients have less effective immune systems. D. Older clients have more exposure to the causative agents. Correct Answer: C Section: (none) Explanation Explanation/Reference: Explanation: (A) Although influenza is common, the elderly are more at risk because of decreased effectiveness of their immune system, not because the incidence is increasing. (B) Older clients have the same degree of illness when stricken as other populations. (C) As people age, their immune system becomes less effective, increasing their risk for influenza. (D) Older clients have no more exposure to the causative agents than do school-age children, for example. QUESTION 82 Which of the following menu choices would indicate that a client with pressure ulcers understands the role diet plays in restoring her albumin levels? A. Broiled fish with rice B. Bran flakes with fresh peachesC. Lasagna with garlic bread D. Cauliflower and lettuce salad Correct Answer: A Section: (none) Explanation Explanation/Reference: Explanation: (A) Broiled fish and rice are both excellent sources of protein. (B) Fresh fruits are not a good source of protein. (C) Foods in the bread group are not high in protein. (D) Most vegetables are not high in protein; peas and beans are the major vegetables higher in protein. QUESTION 83 The nurse observes that a client has difficulty chewing and swallowing her food. A nursing response designed to reduce this problem would include: A. Ordering a full liquid diet for her B. Ordering five small meals for her C. Ordering a mechanical soft diet for her D. Ordering a puréed diet for her Correct Answer: C Section: (none) Explanation Explanation/Reference: Explanation: (A) Full liquids would be difficult to swallow if the muscle control of the swallowing act is affected; this is a probable reason for her difficulties, given her medical diagnosis of multiple sclerosis. (B) Five small meals would do little if anything to decrease her swallowing difficulties, other than assure that she tires less easily. (C) A mechanical soft diet should be easier to chew and swallow, because foods would be more evenly consistent. (D) A pureed diet would cause her to regress morethan might be needed; the mechanical soft diet should be tried first. QUESTION 84 When a client with pancreatitis is discharged, the nurse needs to teach him how to prevent another occurrence of acute pancreatitis. Which of the following statements would indicate he has an understanding of his disease? A. "I will not eat any raw or uncooked vegetables." B. "I will limit my alcohol to one cocktail per day."C. "I will look into attending Alcoholics Anonymous meetings." D. "I will report any changes in bowel movements to my doctor." Correct Answer: C Section: (none) Explanation Explanation/Reference: Explanation: (A) Raw or uncooked vegetables are all right to eat postdischarge. (B) This client must avoid any alcohol intake. (C) The client displays awareness of the need to avoid alcohol. (D) This action would be pertinent only if fatty stools associated with chronic hepatitis were the problem. QUESTION 85 A 54-year-old client is admitted to the hospital with a possible gastric ulcer. He is a heavy smoker. When discussing his smoking habits with him, the nurse should advise him to: A. Smoke low-tar, filtered cigarettes B. Smoke cigars instead C. Smoke only right after meals D. Chew gum instead Correct Answer: C Section: (none) Explanation Explanation/Reference: Explanation: (A, B, D) Cigarettes, cigars, and chewing gum would stimulate gastric acid secretion. (C) Smoking on a full stomach minimizes effect of nicotine on gastric acid. QUESTION 86 Iron dextran (Imferon) is a parenteral iron preparation.The nurse should know that it: A. Is also called intrinsic factor B. Must be given in the abdomen C. Requires use of the Z-track method D. Should be given SCCorrect Answer: C Section: (none) Explanation Explanation/Reference: Explanation: (A) Intrinsic factor is needed to absorb vitamin B12.(B) Iron dextran is given parenterally, but Z- track in a large muscle. (C) A Ztrack method of injection isrequired to prevent staining and irritation of the tissue. (D) An SC injection is not deep enough and may cause subcutaneous fat abscess formation. QUESTION 87 A nasogastric (NG) tube inserted preoperatively is attached to low, intermittent suctions. A client with an NG tube exhibits these symptoms: He is restless; serum electrolytes are Na 138, K 4.0, blood pH 7.53. This client is most likely experiencing: A. Hyperkalemia B. Hyponatremia C. Metabolic acidosis D. Metabolic alkalosis Correct Answer: D Section: (none) Explanation Explanation/Reference: Explanation: (A) Sodium level is within normal limits. (B) Sodium level is within normal limits. (C) pH level is consistent with alkalosis. (D) With an NG tube attached to low, intermittent suction, acids are removed and a client will develop metabolic alkalosis. QUESTION 88 A client is experiencing muscle weakness and lethargy. His serum K+is 3.2. What other symptoms might he exhibit? A. Tetany B. Dysrhythmias C. Numbness of extremities D. Headache Correct Answer: B Section: (none)Explanation Explanation/Reference: Explanation: (A) Tetany is seen with low calcium. (B) Low potassium causes dysrhythmias because potassium is responsible for cardiac muscle activity. (C) Numbness of extremities is seen with high potassium. (D) Headache is not associated with potassium excess or deficiency. QUESTION 89 Following a gastric resection, which of the following actions would the nurse reinforce with the client in order to alleviate the distress from dumping syndrome? A. Eating three large meals a day B. Drinking small amounts of liquids with meals C. Taking a long walk after meals D. Eating a low-carbohydrate diet Correct Answer: D Section: (none) Explanation Explanation/Reference: Explanation: (A) Six small meals are recommended. (B) Liquids after meals increase the time food empties from the stomach. (C) Lying down after meals is recommended to prevent gravity from producing dumping. (D) A low-carbohydrate diet will prevent a hypertonic bolus, which causes dumping. QUESTION 90 Azulfidine (Sulfasalazine) may be ordered for a client who has ulcerative colitis. Which of the following is a nursing implication for this drug? A. Limit fluids to 500 mL/day. B. Administer 2 hours before meals. C. Observe for skin rash and diarrhea. D. Monitor blood pressure, pulse. Correct Answer: C Section: (none) Explanation Explanation/Reference: Explanation:(A) Fluids up to 25003000 mL/day are needed to prevent kidney stones. (B) The client should be instructed to take oral preparations with meals or snacks to lessen gastric irritation. (C) Sulfasalazine causes skin rash and diarrhea. (D) Blood pressure and pulse are not altered by sulfasalazine. QUESTION 91 Other drugs may be ordered to manage a client's ulcerative colitis. Which of the following medications, if ordered, would the nurse question? A. Methylprednisolone sodium succinate (Solu-Medrol) B. Loperamide (Imodium) C. Psyllium D. 6-Mercaptopurine Correct Answer: D Section: (none) Explanation Explanation/Reference: Explanation: (A) Methylprednisolone sodium succinate is used for its anti-inflammatory effects. (B) Loperamide would be used to control diarrhea. (C) Psyllium may improve consistency of stools by providing bulk. (D) An immunosuppressant such as 6-mercaptopurine is used for chronic unrelenting Crohn's disease. QUESTION 92 A male client is scheduled for a liver biopsy. In preparing him for this test, the nurse should: A. Explain that he will be kept NPO for 24 hours before the exam B. Practice with him so he will be able to hold his breath for 1 minute C. Explain that he will be receiving a laxative to prevent a distended bowel from applying pressure on the liver D. Explain that his vital signs will be checked frequently after the test Correct Answer: D Section: (none) Explanation Explanation/Reference: Explanation: (A) There is no NPO restriction prior to a liver biopsy. (B) The client would need to hold his breath for 510 seconds. (C) There is no pretest laxative given. (D) Following the test, the client is watched for hemorrhage and shock.QUESTION 93 After a liver biopsy, the best position for the client is: A. High Fowler B. Prone C. Supine D. Right lateral Correct Answer: D Section: (none) Explanation Explanation/Reference: Explanation: (A) This position does not help to prevent bleeding. (B) This position does not help to prevent bleeding. (C) This position does not help to prevent bleeding. (D) The right lateral position would allow pressure on the liver to prevent bleeding. QUESTION 94 A complication for which the nurse should be alert following a liver biopsy is: A. Hepatic coma B. Jaundice C. Ascites D. Shock Correct Answer: D Section: (none) Explanation Explanation/Reference: Explanation: (A) Hepatic coma may occur in liver disease due to the increased NH3levels, not due to liver biopsy. (B) Jaundice may occur due to increased bilirubin levels, not due to liver biopsy. (C) Ascites would occur due to portal hypertension, not due to liver biopsy. (D) Hemorrhage and shock are the most likely complications after liver biopsy because of already existing bleeding tendencies in the vascular makeup of the liver. QUESTION 95 Which nursing implication is appropriate for a client undergoing a paracentesis?A. Have the client void before the procedure. B. Keep the client NPO. C. Observe the client for hypertension following the procedure. D. Place the client on the right side following the procedure. Correct Answer: A Section: (none) Explanation Explanation/Reference: Explanation: (A) A full bladder would impede withdrawal of ascitic fluid. (B) Keeping the client NPO is not necessary. (C) The client may exhibit signs and symptoms of shock and hypertension. (D) No position change is needed after the procedure. QUESTION 96 The nurse would assess the client's correct understanding of the fertility awareness methods that enhance conception, if the client stated that: A. "My sexual partner and I should have intercourse when my cervical mucosa is thick and cloudy." B. "At ovulation, my basal body temperature should rise about 0.5F." C. "I should douche immediately after intercourse." D. "My sexual partner and I should have sexual intercourse on day 14 of my cycle regardless of the length of the cycle." Correct Answer: B Section: (none) Explanation Explanation/Reference: Explanation: (A) At ovulation, the cervical mucus is increased, stretchable, and watery clear. (B) Under the influence of progesterone, the basal body temperature increases slightly after ovulation. (C) To enhance fertility, measures should be taken that promote retention of sperm rather than removal. (D) Ovulation, the optimal time for conception, occurs 14+2 days before the next menses; therefore, the date of ovulation is directly related to the length of the menstrual cycle. QUESTION 97 A couple is planning the conception of their first child. The wife, whose normal menstrual cycle is 34 days in length, correctly identifies the time that she is most likely to ovulate if she states that ovulation should occur on day: A. 14+2 daysB. 16+2 days C. 20+2 days D. 22+2 days Correct Answer: C Section: (none) Explanation Explanation/Reference: Explanation: (A) Ovulation is dependent on average length of menstrual cycle, not standard 14 days. (B) Ovulation occurs 14+2 days before next menses (34 minus 14 does not equal 16). (C) Ovulation occurs 14+2 days before next menses (34 minus 14 equals 20). (D) Ovulation occurs 14+2 days before next menses (34 minus 14 does not equal 22). QUESTION 98 A client is pregnant with her second child. Her last menstrual period began on January 15. Her expected date of delivery would be: A. October 8 B. October 15 C. October 22 D. October 29 Correct Answer: C Section: (none) Explanation Explanation/Reference: Explanation: (A) Incorrect application of Nägele's rule: correctly subtracted 3 months but subtracted 7 days rather than added. (B) Incorrect application of Nägele's rule: correctly subtracted 3 months but did not add 7 days. (C) Correct application of Nägele's rule: correctly subtracted 3 months and added 7 days. (D) Incorrect application of Nägele's rule: correctly subtracted 3 months but added 14 days instead of 7 days. QUESTION 99 The nurse instructs a pregnant client (G2P1) to rest in a side-lying position and avoid lying flat on her back. The nurse explains that this is to avoid "vena caval syndrome," a condition which: A. Occurs when blood pressure increases sharply with changes in positionB. Results when blood flow from the extremities is blocked or slowed C. Is seen mainly in first pregnancies D. May require medication if positioning does not help Correct Answer: B Section: (none) Explanation Explanation/Reference: Explanation: (A) Blood pressure changes are predominantly due to pressure of the gravid uterus. (B) Pressure of the gravid uterus on the inferior vena cava decreases blood return from lower extremities. (C) Inferior vena cava syndrome is experienced in the latter months of pregnancy regardless of parity. (D) There are no medications useful in the treatment of interior vena cava syndrome; alleviating pressure by position changes is effective. QUESTION 100 A pregnant client comes to the office for her first prenatal examination at 10 weeks. She has been pregnant twice before; the first delivery produced a viable baby girl at 39 weeks 3 years ago; the second pregnancy produced a viable baby boy at 36 weeks 2 years ago. Both children are living and well. Using the GTPAL system to record her obstetrical history, the nurse should record: http://www.gratisexam.com/ A. 3-2-0-0-2 B. 2-2-0-2-2 C. 3-1-1-0-2 D. 2-1-1-0-2 Correct Answer: C Section: (none) Explanation Explanation/Reference: Explanation:(A) This answer is an incorrect application of the GTPAL method. One prior pregnancy was a preterm birth at 36 weeks (T =1, P= 1; not T = 2). (B) This answer is an incorrect application of the GTPAL method. The client is currently pregnant for the third time (G = 3, not 2), one prior pregnancy was preterm (T= 1, P= 1; not T= 2), and she has had no prior abortions (A =0). (C) This answer is the correct application of GTPAL method. The client is currently pregnant for the third time (G =3), her first pregnancy ended at term (>37 weeks) (T = 1), her second pregnancy ended preterm 2033 weeks) (P = 1), she has no history of abortion (A=0), and she has two living children (L = 2). (D) This answer is an incorrect application of the GTPAL method. The client is currently pregnant for the third time (G =3, not 2). QUESTION 101 A pregnant client comes to the office for her first prenatal examination at 10 weeks. She has been pregnant twice before; the first delivery produced a viable baby girl at 39 weeks 3 years ago; the second pregnancy produced a viable baby boy at 36 weeks 2 years ago. Both children are living and well. Using the gravida and para system to record the client's obstetrical history, the nurse should record: A. Gravida 3 para 1 B. Gravida 3 para 2 C. Gravida 2 para 1 D. Gravida 2 para 2 Correct Answer: B Section: (none) Explanation Explanation/Reference: Explanation: (A) This answer is an incorrect application of gravida and para. The client has had two prior deliveries of more than 20 weeks' gestation; therefore, para equals 2, not 1. (B) This answer is the correct application of gravida and para. The client is currently pregnant for the third time (G = 3), regardless of the length of the pregnancy, and has had two prior pregnancies with birth after the 20th week (P = 2), whether infant was alive or dead. (C) This answer is an incorrect application of gravida and para. The client is currently pregnant for the third time (G = 3, not 2); prior pregnancies lasted longer than 20 weeks (therefore, P = 2, not 1). (D) This is an incorrect application of gravida and para. Client is currently pregnant for third time (G = 3, not 2). QUESTION 102 A gravida 2 para 1 client is hospitalized with severe preeclampsia. While she receives magnesium sulfate(MgSO4) therapy, the nurse knows it is safe to repeat the dosage if: A. Deep tendon reflexes are absent B. Urine output is 20 mL/hr C. MgSO4serum levels are>15 mg/dL D. Respirations are>16 breaths/minCorrect Answer: D Section: (none) Explanation Explanation/Reference: Explanation: (A) MgSO4is a central nervous system depressant. Loss of reflexes is often the first sign of developing toxicity. (B) Urinary output at <25 mL/hr or 100 mL in 4 hours may result in the accumulation of toxic levels of magnesium. (C) The therapeutic serum range for MgSO4is 68 mg/dL. Higher levels indicate toxicity. (D) Respirations of>16 breaths/min indicate that toxic levels of magnesium have not been reached. Medication administration would be safe. QUESTION 103 Prenatal clients are routinelymonitored for early signs of pregnancy-induced hypertension (PIH). For the prenatal client, which of the following blood pressure changes from baseline would be most significant for the nurse to report as indicative of PIH? A. 136/88 to 144/93 B. 132/78 to 124/76 C. 114/70 to 140/88 D. 140/90 to 148/98 Correct Answer: C Section: (none) Explanation Explanation/Reference: Explanation: (A) These blood pressure changes reflect only an 8 mm Hg systolic and a 5 mm Hg diastolic increase, which is insufficient for blood pressure changes indicating PIH. (B) These blood pressure changes reflect a decrease in systolic pressure of 8 mm Hg and diastolic pressure of 2 mm Hg; these values are not indicative ofblood pressure increases reflecting PIH. (C) The definition of PIH is an increase in systolic blood pressure of 30 mm Hg and/or diastolic blood pressure of 15 mm Hg. These blood pressures reflect a change of 26 mm Hg systolically and 18mm Hg diastolically. (D) These blood pressures reflect a change of only 8 mm Hg systolically and 8 mm Hg diastolically, which is insufficient for blood pressure changes indicating PIH. QUESTION 104 In assisting preconceptual clients, the nurse should teach that the corpus luteum secretes progesterone, which thickens the endometrial lining in which of the phases of the menstrual cycle? A. Menstrual phase B. Proliferative phase C. Secretory phaseD. Ischemic phase Correct Answer: C Section: (none) Explanation Explanation/Reference: Explanation: (A) Menses occurs during the menstrual phase, during which levels of both estrogen and progesterone are decreased. (B) The ovarian hormone responsible for the proliferative phase, during which the uterine endometrium enlarges, is estrogen. (C) The ovarian hormone responsible for the secretory phase is progesterone, which is secreted by the corpus luteum and causes marked swelling in the uterine endometrium. (D) The corpus luteum begins to degenerate in the ischemic phase, causing a fall in both estrogen and progesterone. QUESTION 105 A client decided early in her pregnancy to breast-feed her first baby. She gave birth to a normal, full-term girl and is now progressing toward the establishment of successful lactation. To remove the baby from her breast, she should be instructed to: A. Gently pull the infant away B. Withdraw the breast from the infant's mouth C. Compress the areolar tissue until the infant drops the nipple from her mouth D. Insert a clean finger into the baby's mouth beside the nipple Correct Answer: D Section: (none) Explanation Explanation/Reference: Explanation: (A) In pulling the infant away from the breast without breaking suction, nipple trauma is likely to occur. (B) In pulling the breast away from the infant without breaking suction, nipple trauma is likely to occur. (C) Compressing the maternal tissue does not break the suction of the infant on the breast and can cause nipple trauma. (D) By inserting a finger into the infant's mouth beside the nipple, the lactating mother can break the suction and the nipple can be removed without trauma. QUESTION 106 A gravida 2 para 1 client delivered a full-term newborn 12 hours ago. The nurse finds her uterus to be boggy, high, and deviated to the right. The most appropriate nursing action is to: A. Notify the physician B. Place the client on a pad countC. Massage the uterus and re-evaluate in 30 minutes D. Have the client void and then re-evaluate the fundus Correct Answer: D Section: (none) Explanation Explanation/Reference: Explanation: (A) The nurse should initiate actions to remove the most frequent cause of uterine displacement, which involves emptying the bladder. Notifying the physician is an inappropriate nursing action. (B) The pad count gives an estimate of blood loss, which is likely to increase with a boggy uterus; but this action does not remove themost frequent cause of uterine displacement, which is a full bladder. (C) Massage may firm the uterus temporarily, but if a full bladder is not emptied, the uterus will remain displaced and is likely to relax again. (D) The most common cause of uterine displacement is a full bladder. QUESTION 107 A client delivered her first-born son 4 hours ago. She asks the nurse what the white cheeselike substance is under the baby's arms. The nurse should respond: A. "This is a normal skin variation in newborns. It will go away in a few days." B. "Let me have a closer look at it. The baby may have an infection." C. "This material, called vernix, covered the baby before it was born. It will disappear in a few days." D. "Babies sometimes have sebaceous glands that get plugged at birth. This substance is an example of that condition." Correct Answer: C Section: (none) Explanation Explanation/Reference: Explanation: (A) This response identifies the fact that vernix is a normal neonatal variation, but it does not teach the client medical terms that may be useful in understanding other healthcare personnel. (B) This response may raise maternal anxiety and incorrectly identifies a normal neonatal variation. (C) This response correctly identifies this neonatal variation and helps the client to understand medical terms as well as the characteristics of her newborn. (D) Blocked sebaceous glandsproduce milia, particularly present on the nose. QUESTION 108 A client is in early labor. Her fetus is in a left occipitoanterior (LOA) position; fetal heart sounds are best auscultated just: A. Below the umbilicus toward left side of mother's abdomen B. Below the umbilicus toward right side of mother's abdomenC. At the umbilicus D. Above the umbilicus to the left side of mother's abdomen Correct Answer: A Section: (none) Explanation Explanation/Reference: Explanation: (A) LOA identifies a fetus whose back is on its mother's left side, whose head is the presenting part, and whose back is toward its mother's anterior. It is easiest to auscultate fetal heart tones (FHTs) through the fetus's back. (B) The identified fetus's back is on its mother's left side, not right side. It is easiest to auscultate FHTs through the fetus's back. (C) In an LOA position, the fetus's head is presenting with the back to the left anterior side of the mother. The umbilicus is too high of a landmark for auscultating the fetus's heart rate through its back. (D) This is the correct auscultation point for a fetus in the left sacroanterior position, where the sacrum is presenting, not LOA. QUESTION 109 In performing the initial nursing assessment on a client at the prenatal clinic, the nurse will know that which of the following alterations is abnormal during pregnancy? A. Striae gravidarum B. Chloasma C. Dysuria D. Colostrum Correct Answer: C Section: (none) Explanation Explanation/Reference: Explanation: (A) Striae gravidarum are the normal stretch marks that frequently occur on the breasts, abdomen, and thighs as pregnancy progresses. (B) Chloasma is the "mask of pregnancy" that normally occurs in many pregnant women. (C) Dysuria is an abnormal danger sign during pregnancy and may indicate a urinary tract infection. (D) Colostrum is a yellow breast secretion that is normally present during the last trimester of pregnancy. QUESTION 110 A 35-weeks-pregnant client is undergoing a nonstress test (NST). During the 20-minute examination, the nurse notes three fetal movements accompanied by accelerations of the fetal heart rate, each 15 bpm, lasting 15 seconds. The nurse interprets this test to be:A. Nonreactive B. Reactive C. Positive D. Negative Correct Answer: B Section: (none) Explanation Explanation/Reference: Explanation: (A) In a nonreactive NST, the criteria for reactivity are not met. (B) A reactive NST shows at least two accelerations of FHR with fetal movements, each 15 bpm, lasting 15 seconds or more, over 20 minutes. (C, D) This term is used to interpret a contraction stress test (CST), or oxytocin challenge test, not an NST. QUESTION 111 The nurse is caring for a laboring client. Assessment data include cervical dilation 9 cm; contractions every 12 minutes; strong, large amount of "bloody show." The most appropriate nursing goal for this client would be: A. Maintain client's privacy. B. Assist with assessment procedures. C. Provide strategies to maintain client control. D. Enlist additional caregiver support to ensure client's safety. Correct Answer: C Section: (none) Explanation Explanation/Reference: Explanation: (A) Privacy may help the laboring client feel safer, but measures that enhance coping take priority. (B) The frequency of assessments do increase in transition, but helping the client to maintain control and cope with this phase of labor takes on importance. (C) This laboring client is in transition, the most difficult part of the first stage of labor because of decreased frequency, increased duration and intensity, and decreased resting phase of the uterine contraction. The client's ability to cope is most threatened during this phase of labor, and nursing actions aredirected toward helping the client to maintain control. (D) Safety is a concern throughout labor, but helping the client to cope takes on importance in transition. QUESTION 112 A client is admitted to the labor unit. On vaginal examination, the presenting part in a cephalic presentation was at station plus two. Station 12 means that the:A. Presenting part is 2 cm above the level of the ischial spines B. Biparietal diameter is at the level of the ischial spines C. Presenting part is 2 cm below the level of the ischial spines D. Biparietal diameter is 5 cm above the ischial spines Correct Answer: C Section: (none) Explanation Explanation/Reference: Explanation: (A) Station is the relationship of the presenting part to an imaginary line drawn between the ischial spines. If the presenting part is above the ischial spines, the station is negative. (B) When the biparietal diameter is at the level of the ischial spines, the presenting part is generally at a +4 or +5 station. (C) Station is the relationship of the presenting part to an imaginary line drawn between the ischial spines. If the presenting part is below the ischial spines, the station is positive. Thus, 2 cm below the ischial spines is the station +2. (D) When the biparietal diameter is above the ischial spines by 5 cm, the presenting part is usually engaged or at station 0. QUESTION 113 A pregnant client is at the clinic for a third trimester prenatal visit. During this examination, it has been determined that her fetus is in a vertex presentation with the occiput located in her right anterior quadrant. On her chart this would be noted as: A. Right occipitoposterior B. Right occipitoanterior C. Right sacroanterior D. LOA Correct Answer: B Section: (none) Explanation Explanation/Reference: Explanation: (A) The fetus in the right occipitoposterior position would be presenting with the occiput in the maternal right posterior quadrant. (B) Fetal position is defined by the location of the fetal presenting part in the four quadrants of the maternal pelvis. The right occipitoanterior is a fetus presenting with the occiput in mother's right anterior quadrant. (C) The fetus in right sacroanterior position would be presenting a sacrum, not an occiput. (D) The fetus in left occipitoanterior position would be presenting with the occiput in the mother's left anterior quadrant.QUESTION 114 Assessment of parturient reveals the following: cervical dilation 6 cm and station 22; no progress in the last 4 hours. Uterine contractions decreasing in frequency and intensity. Marked molding of the presenting fetal head is described. The physician orders, "Begin oxytocin induction at 1 mU/min." The nurse should: A. Begin the oxytocin induction as ordered B. Increase the dosage by 2 mU/min increments at15-minute intervals C. Maintain the dosage when duration of contractions is 4060 seconds and frequency is at 21/2 4 minute intervals D. Question the order Correct Answer: D Section: (none) Explanation Explanation/Reference: Explanation: (A) Oxytocin stimulates labor but should not be used until CPD (cephalopelvic disproportion) is ruled out in a dysfunctional labor. (B) This answer is the correct protocol for oxytocin administration, but the medication should not be used until CPD is ruled out. (C) This answer is the correct manner to interpret effective stimulation, but oxytocin should not be used until CPD is ruled out. (D) This answer is the appropriate nursing action because the scenario presents adysfunctional labor pattern that may be caused by CPD. Oxytocin administration is contraindicated in CPD. QUESTION 115 A client in active labor asks the nurse for coaching with her breathing during contractions. The client has attended Lamaze birth preparation classes. Which of the following is the best response by the nurse? A. "Keep breathing with your abdominal muscles as long as you can." B. "Make sure you take a deep cleansing breath as the contractions start, focus on an object, and breathe about 1620 times a minute with shallow chest breaths." C. "Find a comfortable position before you start a contraction. Once the contraction has started, take slow breaths using your abdominal muscles." D. "If a woman in labor listens to her body and takes rapid, deep breaths, she will be able to deal with her contractions quite well." Correct Answer: B Section: (none) Explanation Explanation/Reference: Explanation: (A) Lamaze childbirth preparation teaches the use of chest, not abdominal, breathing. (B) In Lamaze preparation, every patterned breath is preceded by a cleansing breath; as labor progresses, shallow, paced breathing is found to be effective. (C) It is important to assume a comfortable position in labor, but the Lamazeprepared laboring woman is taught to breathe with her chest, not abdominal, muscles. (D) When deep chest breathing patterns are used in Lamazepreparation, they are slowly paced at a rate of 69 breaths/min. QUESTION 116 A client is being discharged and will continue enteral feedings at home. Which of the following statements by a family member indicates the need for further teaching? A. "If he develops diarrhea lasting for more than 23 days, I will contact the doctor or nurse." B. "I should anticipate that he will gain about 1 lb/day now that he is on continuous feedings." C. "It is important to keep the head of his bed elevated or sit him in the chair during feedings." D. "I should use prepared or open formula within 24 hours and store unused portions in the refrigerator." Correct Answer: B Section: (none) Explanation Explanation/Reference: Explanation: (A) Diarrhea is a complication of tube feedings that can lead to dehydration. Diarrhea may be the result of hypertonic formulas that can draw fluid into the bowel. Other causes of diarrhea may be bacterial contamination, fecal impaction, medications, and low albumin. (B) A consistent weight gain of more than 0.22 kg/day (1/2 lb/day) over several days should be reported promptly. The client should be evaluated for fluid volume excess. (C) Elevating the client's head prevents reflux and thus formula from entering the airway. (D) Bacteria proliferate rapidly in enteral formulas and can cause gastroenteritis and even sepsis. QUESTION 117 A 74-year-old obese man who has undergone open reduction and internal fixation of the right hip is 8 days postoperative. He has a history of arthritis and atrial fibrillation. He admits to right lower leg pain, described as "a cramp in my leg." An appropriate nursing action is to: A. Assess for pain with plantiflexion B. Assess for edema and heat of the right leg C. Instruct him to rub the cramp out of his leg D. Elevate right lower extremity with pillows propped under the knee Correct Answer: B Section: (none) Explanation Explanation/Reference: Explanation: (A) Calf pain with dorsiflexion of the foot (Homans' sign) can be a sign of a deep venous thrombosis; however, it is not diagnostic of the condition. (B) Swelling andwarmth along the affected vein are commonly observed clinical manifestations of a deep venous thrombosis as a result of inflammation of the vessel wall. (C) Rubbing or massaging of the affected leg is contraindicated because of the risk of the clot breaking loose and becoming an embolus. (D) A pillow behind the knee can be constricting and further impair blood flow. QUESTION 118 A male client is started on IV anticoagulant therapy with heparin. Which of the following laboratory studies will be ordered to monitor the therapeutic effects of heparin? A. Partial thromboplastin time B. Hemoglobin C. Red blood cell (RBC) count D. Prothrombin time Correct Answer: A Section: (none) Explanation Explanation/Reference: Explanation: (A) Partial thromboplastin time is used to monitor the effects of heparin, and dosage is adjusted depending on test results. It is a screening test used to detectdeficiencies in all plasma clotting factors except factors VII and XIII and platelets. (B) Hemoglobin is the main component of RBCs. Its main function is to carry O2from the lungs to the body tissues and to transport CO2back to the lungs. (C) RBC count is the determination of the number of RBCs found in each cubic millimeter of whole blood. (D) PT is used to monitor the effects of oral anticoagulants, e.g., coumarintype anticoagulants. QUESTION 119 A client is being discharged on warfarin (Coumadin), an oral anticoagulant. The nurse instructs him about using this drug. Which following response by the client indicates the need for further teaching? A. "I should shave with my electric razor while on Coumadin." B. "I will inform my dentist that I am on anticoagulant therapy before receiving dental work." C. "I will continue with my usual dosage of aspirin for my arthritis when I return home." D. "I will wear an ID bracelet stating that I am on anticoagulants." Correct Answer: C Section: (none) Explanation Explanation/Reference: Explanation:(A) Using an electric razor prevents the risk of cuts while shaving. (B) Any physician or dentist should be informed of anticoagulant therapy because of the risk of bleeding due to a prolonged PT. (C) The client should be instructed to consult with his physician. Aspirin is avoided because it potentiates the affects of oral anticoagulants by interfering with platelet aggregation. (D) Identification bracelets are necessary to direct treatment, especially in an emergency situation. QUESTION 120 A 68-year-old woman is admitted to the hospital with chronic obstructive pulmonary disease (COPD). She is started on an aminophylline infusion. Three days later she is breathing easier. A serum theophylline level is drawn. Which of the following values represents a therapeutic level? A. 14 µ g/mL B. 25 µ g/mL C. 4 µ g/mL D. 30 µ g/mL Correct Answer: A Section: (none) Explanation Explanation/Reference: Explanation: (A) The therapeutic blood level range of theophylline is 1020 mg/mL. Therapeutic drug monitoring determines effective drug dosages and prevents toxicity. (B, D) This value is a toxic level of the drug. (C) This value is a nontherapeutic level of the drug. QUESTION 121 A client is being discharged with albuterol (Proventil) and beclomethasone dipropionate (Vanceril) to be administered via inhalation three times a day and at bedtime. Client teaching regarding the sequential order in which the drugs should be administered includes: A. Glucocorticoid followed by the bronchodilator B. Bronchodilator followed by the glucocorticoid C. Alternate successive administrations D. According to the client's preference Correct Answer: B Section: (none) Explanation Explanation/Reference: Explanation:(A) The client would not receive therapeutic effects of the glucocorticoid when it is inhaled through constricted airways. (B) Bronchodilating the airways first allows for the glucocorticoid to be inhaled through open airways and increases the penetration of the steroid for maximum effectiveness of the drug. (C) Inac- Inaccurate use of the inhalers will lead to decreased effectiveness of the treatment. (D) Client teaching regarding the use and effects of inhalers will promote client understanding and compliance. QUESTION 122 To prevent fungal infections of the mouth and throat, the nurse should teach clients on inhaled steroids to: A. Rinse the plastic holder that aerosolizes the drug with hydrogen peroxide every other day B. Rinse the mouth and gargle with warm water after each use of the inhaler C. Take antacids immediately before inhalation to neutralize mucous membranes and prevent infection D. Rinse the mouth before each use to eliminate colonization of bacteria Correct Answer: B Section: (none) Explanation Explanation/Reference: Explanation: (A) It is sufficient to rinse the plastic holders with warm water at least once per day. (B) It is important to rinse the mouth after each use to minimize the risk of fungal infections by reducing the droplets of the glucocorticoid left in the oral cavity. (C) Antacids act by neutralizing or reducing gastric acid, thus decreasing the pH of the stomach. "Neutralizing" the oral mucosa prior to inhalation of a steroid inhaler does not minimize the risk of fungal infections. (D) Rinsing prior to the use of the glucocorticoid will not eliminate the droplets left on the oral mucous membranes following the use of the inhaler. QUESTION 123 Which of the following would indicate the need for further teaching for the client with COPD? The client verbalizes the need to: A. Eat high-calorie, high-protein foods B. Take vitamin supplementation C. Eliminate intake of milk and milk products D. Eat small, frequent meals Correct Answer: C Section: (none) Explanation Explanation/Reference: Explanation: (A) Protein is vital for the maintenance of muscle to aid in breathing. A high-calorie diet using higher fat than carbohydrate content is given because clients areunable to breathe off the excess CO2that is an end product of carbohydrate metabolism. (B) Inadequate nutritional status, in particular, deficiencies in vitamins A and C, decreases resistance to infection. (C) Milk does not make mucus thicker. It may coat the back of the throat and make it feel thicker. Rinsing the mouth with water after drinking milk will prevent this problem. (D) Small, frequent meals minimize a fullness sensation and reduce pressure on the diaphragm. The work of breathing and SOB are also reduced. QUESTION 124 A dose of theophylline may need to be altered if a client with COPD: A. Is allergic to morphine B. Has a history of arthritis C. Operates machinery D. Is concurrently on cimetidine for ulcers Correct Answer: D Section: (none) Explanation Explanation/Reference: Explanation: (A) The effects of morphine or an allergic response to the drug will not affect theophylline clearance. (B) Xanthines are used cautiously in clients with severe cardiac disease, liver disease, cor pulmonale, hypertension, or hyperthyroidism. Arthritis does not influence the dosage of theophylline. (C) Theophylline does not cause sedation or drowsiness. Conversely, its side effects may be exhibited by central nervous system stimulation. (D) Cimetidine decreases theophylline clearance from the system and increases theophylline levels in the blood, thus increasing the risk of toxicity. QUESTION 125 The nurse working in a prenatal clinic needs to be alert to the cardinal signs and symptoms of PIH because: A. Immediate treatment of mild PIH includes the administration of a variety of medications B. Psychological counseling is indicated to reduce the emotional stress causing the blood pressure elevation C. Self-discipline isrequired to control caloric intake throughout the pregnancy D. The client may not recognize the early symptoms of PIH Correct Answer: D Section: (none) Explanation Explanation/Reference: Explanation:(A) Mild PIH is not treated with medications. (B) Emotional stress is not the cause of blood pressure elevation in PIH. (C) Excessive caloric intake is not the cause of weight gain in PIH. (D) The client most frequently is not aware of the signs and symptoms in mild PIH. QUESTION 126 Which of the following changes in blood pressure readings should be of greatest concern to the nurse when assessing a prenatal client? A. 130/88 to 144/92 B. 136/90 to 148/100 C. 150/96 to 160/104 D. 118/70 to 130/88 Correct Answer: D Section: (none) Explanation Explanation/Reference: Explanation: (A, B, C) The individual's systolic and diastolic changes are more significant than the relatively high initial blood pressure readings. (D) The systolic pressure went up 12 mm Hg and the diastolic pressure 18 mm Hg. This is a more significant rise than the increases in AC choices, and client should receive more frequent evaluations and care. QUESTION 127 A 16-year-old client comes to the prenatal clinic for her monthly appointment. She has gained 14 lb from her 7th to 8th month; her face and hands indicate edema. She is diagnosed as having PIH and referred to the high-risk prenatal clinic. The client's weight increase is most likely due to: A. Overeating and subsequent obesity B. Obesity prior to conception C. Hypertension due to kidney lesions D. Fluid retention Correct Answer: D Section: (none) Explanation Explanation/Reference: Explanation: (A) Overeating can lead to obesity, but not to edema. (B) There is no indication of obesity prior to pregnancy. PIH is more prevalent in the underweight than in the obese in this age group. (C) Hypertension can be due to kidney lesions, but it would have been apparent earlier in the pregnancy. (D) The weight gain in PIH is dueto the retention of sodium ions and fluid and is one of the three cardinal symptoms of PIH. QUESTION 128 MgSO4 is ordered IV following the established protocol for a client with severe PIH. The anticipated effects of this therapy are anticonvulsant and: A. Vasoconstrictive B. Vasodilative C. Hypertensive D. Antiemetic Correct Answer: B Section: (none) Explanation Explanation/Reference: Explanation: (A) An anticonvulsant effect is the goal of drug therapy for PIH. However, we would not want to increase the vasoconstriction that is already present. This would make the symptoms more severe. (B) An anticon-vulsant effect and vasodilation are the desired outcomes when administering this drug. (C) An anticonvulsant effect is the goal of drug therapy for PIH; however, hypertensive drugs would increase the blood pressure even more. (D) An anticonvulsant effect is the goal of drug therapy for PIH. MgSO4is not classified as an antiemetic. Antiemetics are not indicated for PIH treatment. QUESTION 129 A nurse should carefully monitor a client for the following side effect of MgSO4: A. Visual blurring B. Tachypnea C. Epigastric pain D. Respiratory depression Correct Answer: D Section: (none) Explanation Explanation/Reference: Explanation: (A, C) The nurse should provide good distractors because these symptoms indicate that PIH has become more severe and may precede the convulsive or eclamptic phase. (B) This is the oppositeside effect of this medication. (D) This is a common side effect of this medication and needs to be monitored and recorded frequently.QUESTION 130 MgSO4 blood levels are monitored and the nurse would be prepared to administer the following antidote for MgSO4 side effects or toxicity: A. Magnesium oxide B. Calciumhydroxide C. Calcium gluconate D. Naloxone (Narcan) Correct Answer: C Section: (none) Explanation Explanation/Reference: Explanation: (A, B) These drugs are not antidotes for MgSO4. (C) This drug is the standard antidote and should always be readily available when MgSO4is being administered. (D) This drug is an antidote for narcotics, not MgSO4. QUESTION 131 A client with severe PIH receiving MgSO4 is placed in a quiet, darkened room. The nurse bases this action on the following understanding: A. The client is restless. B. The elevated blood pressure causes photophobia. C. Noise or bright lights may precipitate a convulsion. D. External stimuli are annoying to the client with PIH. Correct Answer: C Section: (none) Explanation Explanation/Reference: Explanation: (A) The client may be anxious and hyperresponsive to stimuli but not necessarily restless. (B) This is not a physiological response to an elevated blood pressure in PIH. (C) The nurse must know the nursing measures that decrease the potential for convulsions. A quiet, darkened room decreases stimuli and promotes rest. (D) External stimuli might induce a convulsion but are not annoying to the client with PIH. QUESTION 132 A 26-year-old client is admitted to the labor, delivery, recovery, postpartum unit. The nurse completes her assessment and determines the client is in the first stageof labor. The nurse should instruct her: A. To hold her breath during contractions B. To be flat on her back C. Not to push with her contractions D. To push before becoming fully dilated Correct Answer: C Section: (none) Explanation Explanation/Reference: Explanation: (A) This nursing action may cause hyperventilation. (B) This nursing action could cause inferior vena cava syndrome. (C) The client is allowed to push only after complete dilation during the second stage of labor. The nurse needs to know the stages of labor. (D) If the client pushes before dilation, it could cause cervical edema and/or edema to the fetal scalp; both of these could contribute to increased risk of complications. QUESTION 133 In addition to changing the mother's position to relieve cord pressure, the nurse may employ the following measure (s) in the event that she observes the cord out of the vagina: A. Immediately pour sterile saline on the cord, and repeat this every 15 minutes to prevent drying. B. Cover the cord with a wet sponge. C. Apply a cord clamp to the exposed cord, and cover with a sterile towel. D. Keep the cord warm and moist by continuous applications of warm, sterile saline compresses. Correct Answer: D Section: (none) Explanation Explanation/Reference: Explanation: (A) Saline should be warmed; waiting 15 minutes may not keep the cord moist. (B) This choice does not specify what the sponge was "wet" with. (C) This measure would stop circulation to the fetus. (D) The cord should be kept warm and moist to maintain fetal circulation. This measure is an accepted nursing action. QUESTION 134 Which of the following signs might indicate a complication during the labor process with vertex presentation?A. Fetal tachycardia to 170 bpm during a contraction B. Nausea and vomiting at 810 cm dilation C. Contraction lasting 60 seconds D. Appearance of dark-colored amniotic fluid Correct Answer: D Section: (none) Explanation Explanation/Reference: Explanation: (A) Fetal tachycardia may indicate fetal hypoxia; however, 170 bpm is onlymild tachycardia. (B) Nausea and vomiting occur frequently during transition and are not a complication. (C) Contractions frequently last 6090 seconds during the transitional phase of labor and are not considered a complication as long as the uterus relaxes completely between contractions. (D) Passage of meconium in a vertex presentation is a sign of fetal distress; this may be normal in a breech presentation owing to pressure on the presenting part. QUESTION 135 A client is admitted to the hospital for an induction of labor owing to a gestation of 42 weeks confirmed by dates and ultrasound. When she is dilated 3 cm, she has a contraction of 70 seconds. She is receiving oxytocin. The nurse's first intervention should be to: A. Check FHT B. Notify the attending physician C. Turn off the IV oxytocin D. Prepare for the delivery because the client is probably in transition Correct Answer: C Section: (none) Explanation Explanation/Reference: Explanation: (A) FHT should be monitored continuously with an induction of labor; this is an accepted standard of care. (B) The physician should be notified, but this is not thefirst intervention the nurse should do. (C) The standard of care for an induction according to the Association of Women's Health, Obstetric, and Neonatal Nurses and American College of Obstetrics and Gynecology is that contractions should not exceed 60 seconds in an induction. Inductions should simulate normal labor; 70-second contractions during the latent phase (3 cm) are not the norm. The next contractions can be longer and increase risks to the mother and fetus. (D) Contractions lasting 6090 seconds during transition are typical; this provides a good distractor. The nurse needs to be knowledgeable of the phases and stages of labor.QUESTION 136 During a client's first postpartum day, the nurse assessed that the fundus was located laterally to the umbilicus.This may be due to: A. Endometritis B. Fibroid tumor on the uterus C. Displacement due to bowel distention D. Urine retention or a distended bladder Correct Answer: D Section: (none) Explanation Explanation/Reference: Explanation: (A, B) Endometritis, urine retention, or bladder distention provide good distractors because they may delay involution but do not usually cause the uterus to be lateral. (C) Bowel distention and constipation are common in the postpartum period but do not displace the uterus laterally. (D) Urine retention or bladder distention commonly displaces the uterus to the right and may delay involution. QUESTION 137 The nurse would be concerned if a client exhibited which of the following symptoms during her postpartum stay? A. Pulse rate of 5070 bpm by her third postpartum day B. Diuresis by her second or third postpartum day C. Vaginal discharge or rubra, serosa, then rubra D. Diaphoresis by her third postpartum day Correct Answer: C Section: (none) Explanation Explanation/Reference: Explanation: (A) Bradycardia is an expected assessment during the postpartum period. (B) Diuresis can occur during labor and the postpartum period and is an expected physiological adaptation. (C) A return of rubra after the serosa period may indicate a postpartal complication. (D) Diaphoresis, especially at night, is an expected physiological change and does not indicate an infectious process. Bradycardia, diuresis, and diaphoresis are normal postpartum physiological responses to adjust the cardiac output and blood volume to the nonpregnant state.QUESTION 138 A postpartum client complains of rectal pressure and severe pain in her perineum; this may be indicative of: A. Afterbirth pains B. Constipation C. Cystitis D. A hematoma of the vagina or vulva Correct Answer: D Section: (none) Explanation Explanation/Reference: Explanation: (A) Afterbirth pains are a common complaint in the postpartum client, but they are located in the uterus. (B) Constipation may cause rectal pressure but is notusually associated with "severe pain." (C) Cystitis may cause pain, but the location is different. (D) Hematomas are frequently associated with severe pain and pressure. Further assessments are indicated for this client. QUESTION 139 After a 10-year-old child with insulin-dependent diabetes mellitusreceives her dinner tray, she tells the nurse that she hates broccoli and wants some corn on the cob. The nurse's appropriate response is: A. "No vegetable exchanges are allowed." B. "Corn and other starchy vegetables are considered to be bread exchanges." C. "Yes, you may exchange any vegetable for any other vegetable." D. "Yes, but only one-half ear is allowed." Correct Answer: B Section: (none) Explanation Explanation/Reference: Explanation: (A) Sites for injection need to be rotated, including abdominal sites, to enhance insulin absorption. (B) The pinch technique is the most effective method for obtaining skin tightness to allow easy entrance of the needle to subcutaneous tissues. (C) Massaging the site of injectionfacilitates absorption of the insulin. (D) Changing the needle will break the sterility of the system. It has become acceptable practice to reuse disposable needles and syringes for 37 days.QUESTION 140 The nurse is teaching a 10-year-old insulin-dependent diabetic how to administer insulin. Which one of the following steps must be taught for insulin administration? A. Never use abdominal site for a rotation site. B. Pinch the skin up to form a subcutaneous pocket. C. Avoid applying pressure after injection. D. Change needles after injection. Correct Answer: B Section: (none) Explanation Explanation/Reference: Explanation: (A) Sites for injection need to be rotated, including abdominal sites, to enhance insulin absorption. (B) The pinch technique is the most effective method for obtaining skin tightness to allow easy entrance of the needle to subcutaneous tissues. (C) Massaging the site of injection facilitates absorption of the insulin. (D) Changing the needle will break the sterility of the system. It has become acceptable practice to reuse disposable needles and syringes for 37 days. QUESTION 141 In assessing the nature of the stool of a client who has cystic fibrosis, what would the nurse expect to see? A. Clay-colored stools B. Steatorrhea stools C. Dark brown stools D. Blood-tinged stools Correct Answer: B Section: (none) Explanation Explanation/Reference: Explanation: (A) Clay-colored stools indicate dysfunction of the liver or biliary tract. (B) In the early stages of cystic fibrosis, fat absorption is primarily affected resulting in fat, foul, frothy, bulky stools. (C) Dark brown stools indicate normal passage through the colon. (D) Blood-tinged stools indicate dysfunction of the gastrointestinal (GI) tract. QUESTION 142 A group of nursing students at a local preschool day care center are going to screen each child's fine and gross motor, language, and social skills. The students will use which one of the most widely used screening tests?A. Revised Prescreening Developmental Questionnaire B. Goodenough Draw-a-Person Screening Test C. Denver Development Screening Test D. Caldwell Home Inventory Correct Answer: C Section: (none) Explanation Explanation/Reference: Explanation: (A) The Revised Prescreening Developmental Questionnaire is more age appropriate and offers simplified parent scoring and easier comparison. It is used by parents instead of professionals. (B) The Goodenough Draw-a-Person test is used to assess intellectual development. (C) The Denver Developmental ScreeningTest is one of the most widely used screening tests. It offers a concise, easy-to-administer, systematic approach to assessing the preschool child. It is widely used because of its reliability and validity. (D) The Caldwell Home Inventory is used to assess the home environment in areas of social, emotional, and cognitive supports. QUESTION 143 A mother came to the pediatric clinic with her 17- month-old child. The mother would like to begin toilet training. What should the nurse teach her about implementing toilet training? A. Take two or three favorite toys with the child. B. Have a child-sized toilet seat or training potty on hand. C. Explain to the child she is going to "void" and "defecate." D. Show disapproval if she does not void or defecate. Correct Answer: B Section: (none) Explanation Explanation/Reference: Explanation: (A) Giving her toys will distract her and interfere with toilet training because of inappropriate reinforcement. (B) A child-sized toilet seat or training potty gives a child a feeling of security. (C) She should use words that are age appropriate for the child. (D) Children should be praised for cooperative behavior and/or successful evacuation. QUESTION 144 A mother is unsure about the type of toys for her 17-month-old child. Based on knowledge of growth and development, what toy would the nurse suggest?A. A pull toy to encourage locomotion B. A mobile to improve hand-eye coordination C. A large toy with movable parts to improve pincer grasp D. Various large colored blocks to teach visual discrimination Correct Answer: A Section: (none) Explanation Explanation/Reference: Explanation: (A) Increased locomotive skills make push-pull toys appropriate for the energetic toddler. (B) Infants progress from reflex activity through simple repetitive behaviors to imitative behavior. Hand-eye coordination forms the foundation of other movements. (C) At age 8 months, infants begin to have pincer grasp. Toys that help infants develop the pincer grasp are recommended for this age group. (D) Various large colored blocks are suggested toys for infants 612 months of age to help visual stimulation. QUESTION 145 A 16-year-old student has a long history of bronchial asthma and has experienced several severe asthmatic attacks during the school year. The school nurse is required to administer 0.2 mL of 1/1000 solution of epinephrine SC during an asthma attack. How does the school nurse evaluate the effectiveness of this intervention? A. Increased pulse rate B. Increased expectorate of secretions C. Decreased inspiratory difficulty D. Increased respiratory rate Correct Answer: C Section: (none) Explanation Explanation/Reference: Explanation: (A) A side effect of epinephrine is fatal ventricular fibrillation owing to its effects on cardiac stimulation. (B) Medications used to treat asthma are designed to decrease bronchospasm, not to increase expectorate of secretions. (C) Epinephrine decreased inspiratory difficulty by stimulating -, 1, and 2-receptors causing sympathomimetic stimulation (e.g., bronchodilation). (D) The person with asthma fights to inspire sufficient air thus increasing respiratory rate. QUESTION 146Respiratory function is altered in a 16-year-old asthmatic. Which of the following is the cause of this alteration? A. Altered surfactant production B. Paradoxical movements of the chest wall C. Increased airway resistance D. Continuous changes in respiratory rate and depth Correct Answer: C Section: (none) Explanation Explanation/Reference: Explanation: (A) Altered surfactant production is found in sudden infant death syndrome. (B) Paradoxical breathing occurs when a negative intrathoracic pressure is transmitted to the abdomen by a weakened, poorly functioning diaphragm. (C) Asthma is characterized by spasm and constriction of the airways resulting in increased resistance to airflow. (D) If the pulmonary tree is obstructed for any reason, inspired air has difficulty overcoming the resistance and getting out. The rate of respiration increases in order to compensate, thus increasing air exchange. QUESTION 147 A mother frantically calls the emergency room (ER) asking what to do about her 3-year-old girl who was found eating pills out of a bottle in the medicine cabinet. The ER nurse tells the motherto: A. Give the child 15 mL of syrup of ipecac. B. Give the child 10 mL of syrup of ipecac with a sip of water. C. Give the child 1 cup of water to induce vomiting. D. Bring the child to the ER immediately. Correct Answer: D Section: (none) Explanation Explanation/Reference: Explanation: (A) Before giving any emetic, the substance ingested must be known. (B) At least 8 oz of water should be administered along with ipecac syrup to increase volume in the stomach and facilitate vomiting. (C) Water alone will not induce vomiting. An emetic is necessary to facilitate vomiting. (D) Vomiting should never be induced in an unconscious client because of the risk of aspiration.QUESTION 148 A mother brings her 3-year-old child who is unconscious but breathing to the ER with an apparent drug overdose. The mother found an empty bottle of aspirin next to her child in the bathroom. Which nursing action is the most appropriate? A. Put in a nasogastric tube and lavage the child's stomach. B. Monitor muscular status. C. Teach mother poison prevention techniques. D. Place child on respiratory assistance. Correct Answer: A Section: (none) Explanation Explanation/Reference: Explanation: (A) The immediate treatment for drug overdose is removal of the drug from the stomach by either forced emesis or gastric lavage. The child's unconscious state prohibits forced emesis. (B) Toxic amounts of salicylates directly affect the respiratory system, which could lead to respiratory failure. (C) The mother's anxiety is probably so high that preventive guidance will be ineffective. (D) Respiratory assistance is not needed if the child's respiratory function is unaltered. QUESTION 149 A parent told the public health nurse that her 6-year-old son has been taking tetracycline for a chronic skin condition. The parent asked if this could cause any problems for the child. What should the nurse explain to the parent? A. Giving tetracycline to a child younger than 8 years may cause permanent staining of his teeth. B. If you give tetracycline with milk, it may be absorbed readily. C. The medication should be given to adults, not children. D. Secondary infections of chronic skin disorders do not respond to antibiotics. Correct Answer: A Section: (none) Explanation Explanation/Reference: Explanation: (A) Tetracycline should be avoided during tooth development because it interferes with enamel formation and dental pigmentation. (B) Milk interferes with the absorption of tetracyclines. (C) Children older than 9 years or past the tooth development stage may be given tetracycline. (D) Secondary infections of chronic skin disorders may respond to antibiotics such as penicillin or tetracyclines.QUESTION 150 A 6-month-old infant has developmental delays. His weight falls below the 5th percentile when plotted on a growth chart. A diagnosis of failure to thrive is made. What behaviors might indicate the possibility of maternal deprivation? A. Responsive to touch, wants to be held B. Uncomforted by touch, refuses bottle C. Maintains eye-to-eye contact D. Finicky eater, easily pacified, cuddly Correct Answer: B Section: (none) Explanation Explanation/Reference: Explanation: (A) Normal infant attachment behaviors include responding to touch and wanting to be held. (B) Maternal deprivation behaviors include poor feeding, stiffening and refusal to eat, and inconsistencies in responsiveness. (C) Attachment behavior includes maintaining eye contact. (D) Maternal deprivation behaviors include displeasure with touch and physical contact. QUESTION 151 A mother continues to breast-feed her 3-month-old infant. She tells the nurse that over the past 3 days she has not been producing enough milk to satisfy the infant. The nurse advises the mother to do which of the following? A. "Start the child on solid food." B. "Nurse the child more frequently during this growth spurt." C. "Provide supplements for the child between breastfeeding so you will have enough milk." D. "Wait 4 hours between feedings so that your breasts will fill up." Correct Answer: B Section: (none) Explanation Explanation/Reference: Explanation: (A) Solid foods introduced before 46 months of age are not compatible with the abilities of the GI tract and the nutritional needs of the infant. (B) Production of milk is supply and demand. A common growth spurt occurs at 3 months of age, and more frequent nursing will increase the milk supply to satisfy the infant. (C) Supplementation will decrease the infant's appetite and in turn decrease the milk supply. When the infant nurses less often or with less vigor, the amount of milk produced decreases. (D) Rigid feeding schedules lead to a decreased milk supply, whereas frequent nursing signals the mother's body to produce acorrespondingly increased amount of milk. QUESTION 152 An 8-week-old infant has been diagnosed with gastroesophageal reflux. The nurse is teaching the infant's mother to care for the infant at home. Which one of the following statements by the nurse is appropriate regarding the infant's home care? A. "Lay the infant flat on her left side after feeding." B. "Feed the infant every 4 hours with half-strength formula." C. "Antacids need to be given an hour before feeding." D. "Play activities should be carried out before instead of after feedings." Correct Answer: D Section: (none) Explanation Explanation/Reference: Explanation: (A) Elevating the child's head to a 30-degree angle is the recommended position for gastroesophageal reflux. The supine position predisposes the child to aspiration. (B) Small, frequent feedings with thickened formula are recommended to minimize vomiting. (C) Antacids should be given at the same time as the feeding to improve their buffering action. (D) The infant should be kept still after feedings to reduce the risk of vomiting and aspiration. Vigorous activities should be carried out before feedings. QUESTION 153 The nurse is preparing a 6-year-old child for an IV insertion. Which one of the following statements by the nurse is appropriate when preparing a child for a potentially painful procedure? A. "Some say this feels like a pinch or a bug bite. You tell me what it feels like." B. "This is going to hurt a lot; close your eyes and hold my hand." C. "This is a terrible procedure, so don't look." D. "This will hurt only a little; try to be a big boy." Correct Answer: A Section: (none) Explanation Explanation/Reference: Explanation: (A) Educating the child about the pain may lessen anxiety. The child should be prepared for a potentially painful procedure but avoid suggesting pain. The nurseshould allow the child his own sensory perception and evaluation of the procedure. (B) The nurse should avoid absolute descriptive statements and allow the child his own perception of the procedure. (C) The nurse should avoid evaluative statements or descriptions and give the child control in describing his reactions. (D) False statements regarding a painful procedure will cause a loss of trust between the child and the nurse. QUESTION 154 The nurse enters the playroom and finds an 8-year-old child having a grand mal seizure. Which one of the following actions should the nurse take? A. Place a tongue blade in the child's mouth. B. Restrain the child so he will not injure himself. C. Go to the nurses station and call the physician. D. Move furniture out of the way and place a blanket under his head. Correct Answer: D Section: (none) Explanation Explanation/Reference: Explanation: (A) The nurse should not put anything in the child's mouth during a seizure; this action could obstruct the airway. (B) Restraining the child's movements could cause constrictive injury. (C) Staying with the child during a seizure provides protection and allows the nurse to observe the seizure activity. (D) The nurse should providesafety for the child by moving objects and protecting the head. QUESTION 155 A six-month-old infant is receiving ribavirin for the treatment of respiratory syncytial virus. Ribavirin is administered via which one of the following routes? A. Oral B. IM C. IV D. Aerosol Correct Answer: D Section: (none) Explanation Explanation/Reference: Explanation: (A) Ribavirin is not supplied in an oral form. (B) Ribavirin is administered by aerosol in order to decrease the duration of viral shedding within the infected tissue. (C) Ribavirin is not approved for IV use to treat respiratory syncytial virus. (D) Ribavirin is a synthetic antiviral agent supplied as a crystalline powder that is reconstituted with sterile water. A Small Aerosol Particle Generator unit aerosolizes the medication for delivery by oxygen hood, croup tent, or aerosol mask.QUESTION 156 A 5-year-old child has suffered second-degree thermal burns over 30% of her body. Forty-eight hours after the burn injury, the nurse must begin to monitor the child for which one of the following complications? A. Fluid volume deficit B. Fluid volume excess C. Decreased cardiac output D. Severe hypotension Correct Answer: B Section: (none) Explanation Explanation/Reference: Explanation: (A) Fluid volume deficit resulting from fluid shifts to the interstitial spaces occurs in the first 48 hours. (B) Forty-eight hours to 72 hours after the burn injury and fluid resuscitation, capillary permeability is restored and fluid requirements decrease. Interstitial fluid returns rapidly to the vascular compartment, and the nurse must monitor the child for signs and symptoms of hypervolemia. (C) Increased cardiac output results as fluids shift back to the vascular compartment. (D) Hypertensionis the result of hypervolemia. QUESTION 157 Which one of the following is considered a reliable indicator for assessing the adequacy of fluid resuscitation in a 3-year-old child who suffered partial- and fullthickness burns to 25% of her body? A. Urine output B. Edema C. Hypertension D. Bulging fontanelle Correct Answer: A Section: (none) Explanation Explanation/Reference: Explanation: (A) Urinary output is a reliable indicator of renal perfusion, which in turn indicates that fluid resuscitation is adequate. IV fluids are adjusted based on the urinary output of the child during fluid resuscitation. (B) Edema is an indication of increased capillary permeability following a burn injury. (C) Hypertension is an indicator of fluid volume excess. (D) Fontanelles close by 18 months of age.QUESTION 158 A 4-year-old child is being discharged from the hospital after being treated for severe croup. Which one of the following instructions should the nurse give to the child's mother for the home treatment of croup? A. Take him in the bathroom, turn on the hot water, and close the door. B. Give him a dose of antihistamine. C. Give large amounts of clear liquids if drooling occurs. D. Place him near a cool mist vaporizer and encourage crying. Correct Answer: A Section: (none) Explanation Explanation/Reference: Explanation: (A) Initial home treatment of croup includes placing the child in an environment of high humidity to liquefy and mobilize secretions. (B) Antihistamines should be avoided because they can cause thickening of secretions. (C) Drooling is a characteristic sign of airway obstruction and the child should be taken directly to the emergency room. (D) Crying increases respiratory distress and hypoxia in the child with croup. The nurse should promote methods that will calm the child. QUESTION 159 A 7-year-old child is brought to the ER at midnight by his mother after symptoms appeared abruptly. The nurse's initial assessment reveals a temperature of 104.5F (40.3C), difficulty swallowing, drooling, absence of a spontaneous cough, and agitation. These symptoms are indicative of which one of the following? A. Acute tracheitis B. Acute spasmodic croup C. Acute epiglottis D. Acute laryngotracheobronchitis Correct Answer: C Section: (none) Explanation Explanation/Reference: Explanation: (A) Clinical manifestations of acute tracheitis include a 23 day history of URI, croupy cough, stridor, purulent secretions, high fever. (B) Clinical manifestations of spasmodic croup include a history of URI, croupy cough, stridor, dyspnea, low-grade fever, and a slow progression. The age group most affected is 3 months to 3 years. (C) Three clinical observations have been found to be predictive of epiglottitis: the presence of drooling, absence of spontaneous cough, andagitation.Epiglottitis has a rapid onset that is accompanied by high fever and dysphagia. (D) Clinical manifestations of acute laryngotracheobronchitis (LTB) include slow onset with a history of URI, low-grade fever, stridor, brassy cough, and irritability. QUESTION 160 The nurse is teaching a mother care of her child's spica cast. The mother states that he complains of itching under the edge of the cast. One nonpharmacological technique the nurse might suggest would be: A. "Blowing air under the cast using a hair dryer on cool setting often relieves itching." B. "Slide a ruler under the cast and scratch the area." C. "Guide a towel under and through the cast and moveit back and forth to relieve the itch." D. "Gently thump on cast to dislodge dried skin that causes the itching." Correct Answer: A Section: (none) Explanation Explanation/Reference: Explanation: (A) Cool air will often relieve pruritus without damaging the cast or irritating the skin. (B) The nurse should never force anything under the cast, because the cast may become damaged and skin breakdown may occur. (C) Forcing an object under the cast could lead to cast damage and skin breakdown. The object may become lodged under the cast necessitating cast removal. (D) This technique does not dislodge skin cells. It could damage the cast and cause skin breakdown. QUESTION 161 A 30-year-old client has just been treated in the ER for bruises and abrasions to her face and a broken arm from domestic violence, which has been increasing in frequency and intensity over the last few months. The nurse assesses her as being very anxious, fearful, bewildered, and feeling helpless as she states, "I don't know what to do, I'm afraid to go home." The best response by the nurse to the client would be: A. "I wouldn't want to go home either; call a friend who could help you." B. "Did you do something that could have made him so angry?" C. "Let's talk about people and resources available to you so that you don't have to go home." D. "I'll call the police and they will take care of him, and you can go home and get some rest." Correct Answer: C Section: (none) Explanation Explanation/Reference: Explanation:(A) A person in crisis needs support, assistance, and direction from a caregiver rather than just an instruction. (B) A battered person may feel guilt and think that they cause the abuser's behavior; however, the abuser has the problem and goes through phases of violence. (C) The nurse should provide support and guidance to the client in crisis by offering alternatives and assist in referrals. (D) Focusing on help from law enforcement may be a very temporary solution, because the victim may be fearful of pressing charges. This answer does not address the crisis of going home. QUESTION 162 A 26-year-old client is in a treatment center for aprazolam (Xanax) abuse and continues to manifest moderate levels of anxiety 3 weeks into the rehabilitation program, often requesting medication for "his nerves." Included in the client's plan of care is to identify alternate methods of coping with stress and anxiety otherthan use of medication. After intervening with assistance in stress reduction techniques, identifying feelings and past coping, the nurse evaluates the outcome as being met if: A. Client promises that he will not abuse aprazolam after discharge B. Client demonstrates use of exercise or physical activity to handle nervous energy following conflicts of everyday life C. Client is able to verbalize effects of substance abuse on the body D. Client hasremained substance free during hospitalization and is discharged Correct Answer: B Section: (none) Explanation Explanation/Reference: Explanation: (A) This client response does not address stress reduction techniques. Verbal response focuses only on the problem. (B) Exercise or physical activity is a common strategy or coping technique used to reduce stress and anxiety. (C) Verbalizing effects of substance abuse on the body may help with insight and break through denial, but it is not a strategy to reduce anxiety. (D)Remaining substance-free does indicate motivation to change lifestyle of substance abuse or dependence, and it is not a stress reduction strategy in itself. QUESTION 163 A 23-year-old borderline client is admitted to an inpatient psychiatric unit following an impulsive act of self-mutilation. A few hours after admission, she requests special privileges, and when these are not granted, she stands up and angrily shouts that the people on the unit do not care, and she storms across the room. The nurse should respond to this behavior by: A. Placing her in seclusion until the behavior is under control B. Walking up to the client and touching her on the arm to get her attention C. Communicating a desire to assist the client to regain control, offering a one-to-one session in a quiet area D. Confronting the client, letting her know the consequences for getting angry and disrupting the unit Correct Answer: C Section: (none)Explanation Explanation/Reference: Explanation: (A) Threatening a client with punitive action is violating a client's rights and could escalate the client's anger. (B) Angry clients need respect for personal space, and physical contact may be perceived as a threatening gesture escalating anger. (C) Client lacks sufficient self-control to limit own maladaptive behavior; she may need assistance from staff. (D) Confronting an angry client may escalate her anger to further acting out, and consequences are for acting out anger aggressively, not for getting angry or feeling angry. QUESTION 164 A 56-year-old client is admitted to the psychiatric unit in a state of total despair. She feels hopeless and worthless, has a flat affect and very sad appearance, and is unable to feel pleasure from anything. Her husband has been assisting her at home with the housework and cooking; however, she has not been eating much, lies around or sits in a chair most of the day, and is becoming confused and thinks her family does not want her around anymore. In assessing the client, the nurse determines that her behavior is consistent with: A. Transient depression B. Mild depression C. Moderate depression D. Severe depression Correct Answer: D Section: (none) Explanation Explanation/Reference: Explanation: (A) Transient depression manifests as sadness or the "blues" as seen with everyday disappointments and is not necessarily dysfunctional. (B) Mild depression manifests as symptoms seen with grief response, such as denial, sadness, withdrawal, somatic symptoms, and frequent or continuous thoughts of the loss. (C) Moderate depression manifests as feelings of sadness, negativism; low self-esteem; rumination about life's failures; decreased interest in grooming and eating; and possibly sleep disturbances. These symptoms are consistent with dysthymia. (D) Severe depression manifests as feelings of total despair, hopelessness, emptiness, inability to feel pleasure; possibly extreme psychomotor retardation; inattention to hygiene; delusional thinking; confusion; self-blame; and suicidal thoughts. These symptoms are consistent with major depression. QUESTION 165 A 56-year-old psychiatric inpatient has had recurring episodes of depression and chronic low self-esteem. She feels that her family does not want her around, experiences a sense of helplessness, and has a negative view of herself. To assist the client in focusing on her strengths and positive traits, a strategy used by the nurse would be to: A. Tell the client to attend all structured activities on the unitB. Encourage or direct client to attend activities that offer simple methods to attain success C. Increase the client's self-esteem by asking that she make all decisions regarding attendance in group activities D. Not allow any dependent behaviors by the client because she must learn independence and will have to ask for any assistance from staff Correct Answer: B Section: (none) Explanation Explanation/Reference: Explanation: (A) The nurse should encourage activities gradually, as client's energy level and tolerance for shared activities improve. (B) Activities that focus on strengths and accomplishments, with uncomplicated tasks, minimize failure and increase self-worth. (C) Asking a client to set a goal to make all decisions about attending group activities is unrealistic, and such decisions are not always under the client's control; this sets up the client for further failure and possibly decreased self-worth. (D) Encouragement toward independence does promote increased feelings of selfworth; however, clients may need assistance with decision making and problem solving for various situations and on an individual basis. QUESTION 166 A 42-year-old client on an inpatient psychiatric unit comments that he was brought to the hospital by his wife because he had taken too many pills and states, "I just couldn't take it anymore." The nurse's best response to this disclosure would be: A. "You shouldn't do things like that, just tell someone you feel bad." B. "Tell me more about what you couldn't take anymore." C. "I'm sure you probably didn't mean to kill yourself." D. "How long have you been in the hospital." Correct Answer: B Section: (none) Explanation Explanation/Reference: Explanation: (A) Disapproving gives the impression that the nurse has a right to pass judgment on the client's thoughts, actions, or ideas. (B) Giving a broad opening gives the client encouragement to continue with verbalization. (C) Failing to acknowledge the client's feelings conveys a lack of understanding and empathy. (D) Changing the subject takes the conversation away from the client and is indicative of the nurse's anxiety or insensitivity. QUESTION 167 A 42-year-old client with bipolar disorder has been hospitalized on the inpatient psychiatric unit. She is dancing around, talking incessantly, and singing. Much of the time the client is anorexic and eats very little from her tray before she is up and about again. The nurse's intervention would be to:A. Confront the client with the fact that she will have to eat more from her tray to sustain her B. Try to get the client to focus on her eating by offering a detailed discussion on the importance of nutrition C. Let her have snacks and drinks anytime that she wants them because she will not eat at regular meal times D. Not expect the client to sit down for complete meals; monitor intake, offering snacks and juice frequently Correct Answer: D Section: (none) Explanation Explanation/Reference: Explanation: (A) The manic client's mood may easily change from euphoric to irritable. The nurse should avoid confrontation and let the client know what she can do, rather than what she cannot. (B) Although helpful to refocus or redirect the manic client to discuss only one topic at a time, distractibility is very high and it's best to avoid long discussions. (C) Manic clients have a tendency to manipulate persons in their environment. Staff should monitor intake, including at mealtime and snacks, and be consistent in their approach to meeting nutritional needs. (D) Manic clients may not sit and eat complete meals, but they can carry foods and liquids from regular meals with them. Staff can monitor and give high-caloric and high-energy snacks and liquids. QUESTION 168 Assessment of severe depression in a client reveals feelings of hopelessness, worthlessness; inability to feel pleasure; sleep, psychomotor, and nutritional alterations; delusional thinking; negative view of self; and feelings of abandonment. These clinical features of the client's depression alert the nurse to prioritize problems and care by addressing which of the following problems first: A. Nutritional status B. Impaired thinking C. Possible harm to self D. Rest and activity impairment Correct Answer: C Section: (none) Explanation Explanation/Reference: Explanation: (A) Anorexia and weight loss are problems that need attention in severe depression, but they can be addressed secondary to immediate concerns. (B) Impaired thinking and confusion are problems in severe depression that are addressed with administration of medication, through group and individual psychotherapy, and through activity therapy as motivation and interest increase. (C) Possible harm to self as with suicidal ideation; a suicide plan, means to execute plan; and/or overt gestures or an attempt must be addressed as an immediate concern and safety measures implemented appropriate to the risk of suicide. (D) Rest and activityimpairment may take time and further assessment to determine client's sleep pattern and amount of psychomotor retardation with the more immediate concern for safety present. Topic 3, Questions Set C QUESTION 169 The nurse is assessing and getting a history from a client treated for depression with a monoamine oxidase (MAO) antidepressant. The most serious side effect associated with this antidepressant and the ingestion of tyramine in aged foods may be: A. Hypertensive crisis B. Severe rash C. Severe hypotension D. Severe diarrhea Correct Answer: A Section: (none) Explanation Explanation/Reference: Explanation: (A) The most serious adverse reactions of MAO inhibitors involve blood pressure and ingestion of tyramine-containing foods, which may provoke a hypertensive crisis. (B) MAO inhibitors cause adverse reactions affecting the central nervous system and serious adverse reactions involving blood pressure. (C) MAO inhibits false neurotransmitters (phenylalanines) and may produce hypotensive reactions from gradual accumulation of these neurotransmitters. (D) The most serious adverse reactions of MAO inhibitors involve blood pressure. QUESTION 170 A cardinal symptom of the schizophrenic client is hallucinations. A nurse identifies this as a problem in the category of: A. Impaired communication B. Sensory-perceptual alterations C. Altered thought processes D. Impaired social interaction Correct Answer: B Section: (none) Explanation Explanation/Reference: Explanation:(A) Impaired communication refers to decreased ability or inability to use or understand language in an interaction. (B) In sensory-perceptual alterations an individual has distorted, impaired, or exaggerated responses to incoming stimuli (i.e., a hallucination, which is a false sensory perception that is not associated with real external stimuli). (C) An altered thought processes problem statement is used when an individual experiences a disruption in cognitive operations and activities (i.e., delusions, loose associations, ideas of reference). (D) In impaired social interaction, the individual participates too little or too much in social interactions. QUESTION 171 A schizophrenic client who is experiencing thoughts of having special powers states that "I am a messenger from another planet and can rule the earth." The nurse assesses this behavior as: A. Ideas of reference B. Delusions of persecution C. Thought broadcasting D. Delusions of grandeur Correct Answer: D Section: (none) Explanation Explanation/Reference: Explanation: (A) Clients experiencing ideas of reference believe that information from the environment (e.g., the television) is referring to them. (B) Clients experiencing delusions of persecution believe that others in the environment are plotting against them. (C) Clients experiencing thought broadcasting perceive that others can hear their thoughts. (D) Clients experiencing delusions of grandeur think that they are omnipotent and have superhuman powers. QUESTION 172 A client experiencing delusions states, "I came here because there were people surrounding my house that wanted to take me away and use my body for science." The best response by the nurse would be: A. "Describe the people surrounding your house that want to take you away." B. "I need more information on why you think others want to use your body for science." C. "There were no people surrounding your house, your relatives brought you here, and no one really wants your body for science." D. "I know that must be frightening for you; let the staff know when you are having thoughts that trouble you." Correct Answer: D Section: (none) Explanation Explanation/Reference: Explanation:(A) Focusing on the delusional content does not reinforce reality. (B) Pursuing details or more information on the delusion reinforces the false belief and further distances the client from reality. (C) Challenging the client's delusional system may force the client to defend it, and you cannot change the delusion through logic. (D) Focusing on the feeling can reinforce reality and discourage the false belief. Seeking out staff when thoughts are troublesome can help to decrease anxiety. QUESTION 173 A 16-year-old client with anorexia nervosa is on an inpatient psychiatric unit. She has a fear of gaining weight and is refusing to eat sufficient amounts to maintain body weight for her age, height, and stature. To assist with the problem of powerlessness and plan for the client to no longer need to withhold food to feel in control, the nurse uses the following strategy: A. Establish a structured environment with routine tasks and activities. Also, serve meals at the same time each day. B. Distract the client during meals to get her to eat because she must take in sufficient amounts to keep from starving. C. Do frequent room checks to be sure that the client is not hiding food or throwing it away. D. Listen attentively and participate in in-depth discussions about food, because these actions may encourage her to eat. Correct Answer: A Section: (none) Explanation Explanation/Reference: Explanation: (A) Anorexia nervosa clients feel out of control. Providing a structured environment offers safety and comfort and can help them to develop internal control, thus reducing their need to control by self-starvation. (B) Distraction does not focus on the client's need for control. (C) Doing frequent room checks reinforces feelings of powerlessness and the need to continue with the dysfunctional behavior. (D) Participating in long discussions about food does not make the client want to eat, but rather this strategy allows her to indulge in her preoccupation and to continue with the dysfunctional behavior. QUESTION 174 A 45-year-old male client was admitted to a chemical dependency treatment center following legal problems related to alcohol abuse. He states, "I know that alcohol is a problem for some people, but I can stop whenever I want to. I'm never sick or miss work, and no one can complain about me." During the initial assessment, the best response by the nurse would be: A. "The fact is you are an alcoholic or you wouldn't be here." B. "I understand it took strength to admit yourself to the unit, and I will do my part to help you to stay alcohol- free." C. "If you can stop drinking when you want to, why don't you stop?" D. "It's good that you can stop drinking when you want to." Correct Answer: B Section: (none) ExplanationExplanation/Reference: Explanation: (A) Direct confrontation initially is nontherapeutic and may result in the client becoming frustrated and wanting to leave. (B) A positive, supportive attitude builds trust, and identifying positive strength raises self-esteem. Offering help allows the client to feel that he is not alone in dealing with problems. (C) Asking the client why or to give an explanation for his behavior puts him in a position of having to justify his behavior to the nurse. (D) Giving approval or placing a value on feelings or a behavior may limit the client's freedom to behave in a way that may displease another. This response may lead to seeking praise instead of progress. QUESTION 175 A 79-year-old client with Alzheimer's disease is exhibiting significant memory impairment, cognitive impairment, extremely impaired judgment in social situations, and agitation when placed in a new situation or around unfamiliar people. The nurse should include the following strategy in the client's care: A. Maintain routines and usual structure and adhere to schedules. B. Encourage the client to attend all structured activities on the unit, whether she wants to or not. C. Ask the client to go to an activity once. If she gives no response right away, change the question around, asking the same thing. D. Give the client two or three choices to decide what she wants to do. Correct Answer: A Section: (none) Explanation Explanation/Reference: Explanation: (A) Alzheimer's clients cope poorly with changes in routine because of memory deficits. Schedule changes cause confusion and frustration, whereas adhering to schedules is helpful and supports orientation. (B) Insisting that the client go to all unit activities may antagonize her and increase her agitation because of cognitive impairments. It may be better to allow the client time for calming down or distraction rather than to insist that she attend every activity. (C) When repeating a question, allow time first for a response; then use the same words the second time to avoid further confusion. (D) The nurse should avoid giving several choices at once. Cognitively impaired clients will become more frustrated with making decisions. QUESTION 176 The nurse working with a client who is out of control should follow a model of intervention that includes which of the following? A. Approach the client on a continuum of least restrictive care. B. Challenge client's behavior immediately with steps to prevent injury to self or others. C. Leave the aggressive client to himself or herself, and take other clients away. D. To ensure safety of other clients, place client in seclusion immediately when he or she begins shouting. Correct Answer: A Section: (none)Explanation Explanation/Reference: Explanation: (A) Approaching a client's aggressive behavior on a continuum of least restrictive care is in agreement with his or her rights (i.e., verbal methods to help maintain control, medication, seclusion, and restraints, as necessary). (B) Approaching a client in a challenging manner is threatening and inappropriate. A nonchallenging and calm approach reflects staff in control and may increase client's internal control. (C) It is inappropriate to leave an aggressive client who is acting out alone. The nurse should acquire qualified help to prevent client from harm or injury to self or others. (D) Moving a client to seclusion immediately for shouting is inappropriate. The nurse should offer the client an opportunity to control self with limit setting. The client should understand that the staff will assist with control if necessary (i.e., quietly accompany out of environment to decrease stimulation and allow for verbalization) employing the least restrictive care model of intervention. QUESTION 177 When planning care for the passive-aggressive client, the nurse includes the following goal: A. Allow the client to use humor, because this may be the only way this client can express self. B. Allow the client to express anger by using "I" messages, such as "I was angry when . . .," etc. C. Allow the client to have time away from therapeutic responsibilities. D. Allow the client to give excuses if he forgets to give staff information. Correct Answer: B Section: (none) Explanation Explanation/Reference: Explanation: (A) Ceasing to use humor and sarcasm is a more appropriate goal, because this client uses these behaviors covertly to express aggression instead of being open with anger. (B) Use of "I" messages demonstrates proper use of assertive behavior to express anger instead of passive- aggressive behavior. (C) Client is expected to complete share of work in therapeutic community because he has often obstructed other's efforts by failing to do his share. (D) Client has used conveniently forgetting or withholding information as a passive-aggressive behavior, which is not acceptable. QUESTION 178 A client calls the prenatal clinic to schedule an appointment. She states she has missed three menstrual periods and thinksshe might be pregnant. During her first visit to the prenatal clinic, it is confirmed that she is pregnant. The registered nurse (RN) learns that her last menstrual period began on June 10. According to Nägele's rule, the estimated date of confinement is: A. March 17 B. June 3 C. August 30D. January 10 Correct Answer: A Section: (none) Explanation Explanation/Reference: Explanation: (A) Using Nägele's rule, count back 3 calendar months from the first day of the last menstrual period. The answer is March 10. Then add 7 days and 1 year, which would be March 17 of the following year. (B, C, D) This date is incorrect. QUESTION 179 At 16 weeks' gestation, a pregnant client is admitted to the maternity unit to have a McDonald procedure (cerclage) done. She tells the RN who is admitting her to the unit that her physician had explained what this procedure was, but that she did not understand. The RN explains to the client that the purpose for this procedure is to: A. Reinforce an incompetent cervix B. Repair the amniotic sac C. Evaluate cephalopelvic disproportion D. Dilate the cervix Correct Answer: A Section: (none) Explanation Explanation/Reference: Explanation: (A) The treatment most commonly uses the Shirodkar-Barter procedure (McDonald procedure) or cerclage to enforce the weakened cervix by encircling it with a suture at the level of the internal os. (B) There is no known procedure that is used to repair the amniotic sac. (C) Cephalopelvic disproportion is evaluated later in pregnancy. It is not related to this procedure. (D) No procedure is done to dilate the cervix at 16 weeks' gestation unless the pregnancy is to be terminated. QUESTION 180 A pregnant client continues to visit the clinic regularly during her pregnancy. During one of her visits while lying supine on the examining table, she tells the RN that she is becoming light- headed. The RN notices that the client has pallor in her face and is perspiring profusely. The first intervention the RN should initiate is to: A. Place the examining table in the Trendelenburg position B. Assess the client to see if she is having vaginal bleeding C. Obtain the client's vital signs immediatelyD. Help the client to a sitting position Correct Answer: D Section: (none) Explanation Explanation/Reference: Explanation: (A) This position would cause the gravid uterus to bear the increased pressure of the vena cava, which could lead to maternal hypotension, in turn causing the client to continue to have pallor and to feel light-headed. (B) This would not be the first intervention the RN should initiate. TheRN should understand the supine position and its effect on the gravid uterus and vena cava. (C) The RN's first intervention should be one that helps to alleviate the client's symptoms. Obtaining her vital signs will not alleviate her symptoms. (D) This would move the gravid uterus off of the client's vena cava, which would alleviate the maternal hypotension that is the cause of her symptoms. QUESTION 181 At 30 weeks' gestation, a client is admitted to the unit in premature labor. Her contractions are every 5 minutes and last 60 seconds, her cervix is closed, and thesuture placed around her cervix during her 16th week of gestation, when she had the MacDonald procedure, can still be felt by the physician. The amniotic sac isstill intact. She is very concerned about delivering prematurely. She asks the RN, "What is the greatest risk to my baby if it is born prematurely?" The RN's answer should be: A. Hyperglycemia B. Hypoglycemia C. Lack of development of the intestines D. Lack of development of the lungs Correct Answer: D Section: (none) Explanation Explanation/Reference: Explanation: (A) Any infant would be at risk for hyperglycemia because the infant's liver is missing the islets of Langerhans, which secrete insulin to break down glucose for cellular use. Prematurity is not an added risk for hyperglycemia. (B) Both premature and mature infants can be at risk for hypoglycemia if their mother had gestational diabetes during pregnancy or entered the pregnancy with diabetes mellitus. These infants are exposed to high levels of maternal glucose while in utero, which causes the islets of Langerhans in the infant's liver to produce insulin. After birth when the umbilical cord is severed, the generous amount of maternal blood glucose is eliminated; however, there is continued islet cell hyperactivity in the infant'sliver, which can lead to excessive insulin levels and depleted blood glucose. (C) Mature infants are born with an immature GI system. The nervous control of the stomach is incomplete at birth, salivary glands are immature at birth, and the intestinal tract is sterile. This is not the greatest risk to the premature infant. (D) Infants born before 37 weeks' gestation are at greatest risk for an insufficient amount of surfactant in the alveoli system of the lungs. Surfactant helps to prevent lung collapse and ensures stability of the respiratory system so that the infantcan maintain his own respirations once the umbilical cord is severed at birth. QUESTION 182 At 30 weeks' gestation, a client is admitted to the unit in premature labor. Her physician orders that an IV be started with 500 mL D5W mixed with 150 mg of ritodrine stat. The RN prepares the IV solution with the medication. The RN knows that clientsreceiving the medication ritodrine IV should be observed closely for which one of the following side effects: A. Hypoglycemia B. Hyperkalemia C. Tachycardia D. Increase in hematocrit and hemoglobin Correct Answer: C Section: (none) Explanation Explanation/Reference: Explanation: (A) Ritodrine is a sympathomimetic 2-adrenergic agonist that can cause an elevation of blood glucose and plasma insulin in pregnant women. Hyperglycemia can occur in women with abnormal carbohydrate metabolism because of their inability to release more insulin. (B) Hypokalemia can occur resulting from the action of the _-mimetics. It results from a displacement of the extracellular potassium into the intracellular space. (C) Ritodrine causes vasodilation of vessel walls, which can lead to hypotension. The body compensates by increasing heart rate and pulse pressure. (D) There is a lowering of serum iron resulting from the action of _- mimetics to activate hematopoiesis. QUESTION 183 At 32 weeks' gestation, a client is scheduled for a fetal activity test (nonstress test). She calls the clinic and asks the RN, "How do I prepare for the test I am scheduled for?" The RN will most likely inform her of the following instructions to help prepare her for the test: A. "You need to know that an IV is always started before the test." B. "You will need to drink 6 to 8 glasses of water to fill your bladder." C. "Do not eat any food or drink any liquids before the test is started." D. "You will have to remain as still as you possibly can." Correct Answer: D Section: (none) Explanation Explanation/Reference: Explanation:(A) An IV line is not started in a nonstress test, because this test is used as an indicator of fetal well-being. This test measures fetal activity and heart rate acceleration. (B) The bladder does not have to be full prior to this test. It is not a sonogram test where a full bladder enables other structures to be scanned. (C) It has been proved that eating or drinking liquids prior to the test can assist in increasing fetal activity. (D) Any maternal activity will interfere with the results of the test. QUESTION 184 After the fetal activity test (nonstress test) is completed, the RN is looking at the test results on the monitor strip. The RN observes that the fetal heart accelerated 5 beats/min with each fetal movement. The accelerations lasted 15 seconds and occurred 3 times during the 20- minute test. The RN knows that these test results will be interpreted as: A. A reactive test B. A nonreactive test C. An unsatisfactory test D. A negative test Correct Answer: A Section: (none) Explanation Explanation/Reference: Explanation: (A) A nonstress test that shows at least two accelerations of the fetal heart rate of 15 bpm with fetal activity, lasting 15 seconds over a 20-minute period. (B)Reactive criteria are not met. The accelerations of the fetal heart rate are not at least 15 bpm and do not last 15 seconds. This could mean fetal well-being is compromised. Usually a contraction stress test is ordered if the nonstress test results are negative. (C) An unsatisfactory test means the data cannot be interpreted, or there was inadequate fetal activity. If this happens, usually the test is ordered to be done at a later date. (D) A negative test is a term used todescribe the results of a contraction stress test. QUESTION 185 At 38 weeks' gestation, a client is in active labor. She is using her Lamaze breathing techniques. The RN is coaching her breathing and encouraging her to relax and work with her contractions. Which one of the following complaints by the client will alert the RN that she is beginning to hyperventilate with her breathing? A. "I am cold." B. "I have a backache." C. "I feel dizzy." D. "I am nauseous." Correct Answer: C Section: (none) ExplanationExplanation/Reference: Explanation: (A) Cold is not a symptom of hyperventilation. This could be due to the temperature of the room. (B) Backache is not a symptom of hyperventilation. This is probably due to the gravid uterus and its effect on the back muscles, or it may be due to the client's position in bed. (C) Dizziness is the first symptom of hyperventilation. It occurs because the body is eliminating too much CO2. (D) Nausea is not a symptom of hyperventilation. It could be a symptom of pain. QUESTION 186 After performing a sterile vaginal exam on a client who has just been admitted to the unit in active labor and placed on an electronic fetal monitor, the RN assesses that the fetal head is at 21 station. She documents this on the monitor strip. Fetal head at 21 station means that the fetal head is located where in the pelvis? A. One centimeter below the ischial spines B. One centimeter above the ischial spines C. Has not entered the pelvic inlet yet D. Located in the pelvic outlet Correct Answer: B Section: (none) Explanation Explanation/Reference: Explanation: (A) The ischial spines are located on both sides of the midpelvis. These spines mark the diameter of the narrowest part of the pelvis that the fetus will encounter.They are not sharp protrusions that will harm the fetus. Station refers to the relationship between the ischial spines in the pelvis and the fetus. The ischial spines are designated at 0 station. If the presenting part of the fetus is located above the ischial spines, a negative number is assigned, noting the number of centimeters above the ischial spines. Therefore, 1 centimeter below the ischial spines is designated as +1 station. (B) See explanation in A. One centimeter above the ischialspines is designated as +1 station. (C) The pelvic inlet is the first part of the pelvis that the fetus enters in routine delivery. The midpelvis is the second part of thepelvis to be entered by the fetus. The ischial spines are located on both sides of the midpelvis. (D) The pelvic outlet is the last part of the pelvis that the fetus will enter. When the fetus reaches this part of the pelvis, birth is near. QUESTION 187 A client has been admitted to the labor and delivery unit in active labor. After assessing her, the RN notes that the client's fetus position is left occipital posterior. Which of the following statements best describes what this means to the labor process:http://www.gratisexam.com/ A. Decreases the overall time of the labor process B. Prolongs the client's first stage of labor C. Decreases the time of the client's first stage of labor D. Prolongs the client's third stage of labor Correct Answer: B Section: (none) Explanation Explanation/Reference: Explanation: (A) Posterior position causes a larger diameter of the fetal head to enter the pelvis than an anterior position. Pressure on the sacral nerves is increased, and it takes the fetus a longer time to enter the pelvic inlet. (B) This position will prolong the first stage of labor. When the larger diameter of the fetal head enters the pelvis first, it will have a more difficult time accommodating to the pelvis; therefore, it will take a longer time for the fetus to move through the pelvis. (C) It will increase the time of labor because the larger diameter of the fetal head will have a more difficult time accommodating to the pelvic inlet and thus will move through the pelvis slower. (D) In the third stage of labor the placenta is delivered; therefore, the infant has been delivered. QUESTION 188 A client is in active labor and has been admitted to the labor and delivery unit. The RN has just done a sterile vaginal exam and determines that the client is dilated 5 cm, effaced 85%, and the fetus's head is at 0 station. She asks if she could have a lumbar epidural now. The epidural is started, and the anesthetic agent used is bupivacaine (Marcaine). After the client has received her lumbar epidural, it is important for the RN to monitor her for which of the following side effects: A. Hypertension B. Hypotension C. Hypoglycemia D. Hyperglycemia Correct Answer: B Section: (none) ExplanationExplanation/Reference: Explanation: (A) The medication bupivacaine will cause vasodilation in the vascular system, and this does not result in elevation of the ma-ternal blood pressure. (B) The medication bupivacaine will cause vasodilation in the vascular system, and this will result in lowering the maternal blood pressure. (C) Bupivacaine does not interfere with the functioning of the endocrine system. (D) Bupivacaine does not interfere with the functioning of the endocrine system. QUESTION 189 A client has been in labor for 10 hours. Her contractions have become hypoactive and slowed in duration. The fetus is at 0 station, cervix is dilated 8 cm and effaced 90%. The physician orders an oxytocin (Pitocin) infusion to be started at once. The RN begins the oxytocin infusion. It is important that the RN discontinue the infusion if which one of the following occur? A. The client's contractions are <2 minutes apart. B. Duration of the contractions are 60 seconds. C. The uterusrelaxes between contractions. D. The client complains that she is tired. Correct Answer: A Section: (none) Explanation Explanation/Reference: Explanation: (A) It is very important that there is a resting phase or relaxation period between the contractions. During this period, the uterus, placenta, and umbilical vessels re- establish blood flow. No resting phase between contractions can lead to fetal bradycardia, fetal hypoxia, and acidosis. It can also result in a tetanic contraction, which can cause uterine rupture. (B) The goal of the oxytocin infusion is to help establish a contraction pattern lasting 4560 seconds occurring every 2 minutes and a uterine tonus of 6070 mm Hg. (C) This choice is correct. The uterus has time to recover from the contraction. (D) The client's tiring is no indication to stop the infusion. She will be tired even without the infusion. QUESTION 190 The client has been in active labor for the last 12 hours. During the last 3 hours, labor has been augmented with oxytocin because of hypoactive uterine contractions. Her physician assesses her cervix as 95% effaced, 8 cm dilated, and the fetus is at 0 station. Her oral temperature is 100.2F at this time. The physician orders that she be prepared for a cesarean delivery. In preparing the client for the cesarean delivery, which one of the following physician's orders should the RN question? A. Administer meperidine (Demerol) 100 mg IM 1 hour prior to the delivery. B. Discontinue the oxytocin infusion.C. Insert an indwelling Foley catheter prior to delivery. D. Prepare abdominal area from below the nipples to below the symphysis pubis area. Correct Answer: A Section: (none) Explanation Explanation/Reference: Explanation: (A) Meperidine is a narcotic analgesic medication that crosses the placental barrier and reaches the fetus, causing respiratory depression in the fetus. A narcoticmedication should never be included in the preoperative order for a cesarean delivery. (B) Oxytocin infusion would be discontinued if client is being prepared for a cesarean delivery because the medication would not be needed. (C) The bladder is always emptied prior to and during the surgical intervention to prevent the urinary bladder from accidentally being incised while the uterine incision is made. (D) The abdominal area is always prepared to rid the area of hair before the abdominal incision is made. Abdominal hair cannot be sterilized and could become a source for postoperative incisional infection. QUESTION 191 After an infant is delivered by cesarean delivery and placed on the warmer, the RN dries and assesses the infant. At 1 and 5 minutes after birth, the RN does the Apgar scoring of the infant. The RN knows that because this infant was delivered by cesarean section, he is at increased risk for having which one of the following: A. Cold stress B. Cyanosis C. Respiratory distress syndrome D. Seizures Correct Answer: C Section: (none) Explanation Explanation/Reference: Explanation: (A) The infant is placed on the warmer and dried after birth. Cold stress occurs when the infant is not dried and kept warm. (B) The fact that this infant was born by cesarean delivery does not place him at a greater risk for cyanosis than an infant delivered vaginally. Cyanosis occurs when infants cannot oxygenate their blood after the umbilical cord is severed. (C) Infants born by cesarean delivery are at a higher risk for developing respiratory distress syndrome because these infants do not pass through the pelvis, where the chest is compressed and fluid is able to escape from the lungs. (D) Cesarean-delivered infants are not at greater risk for seizures than infants delivered vaginally. QUESTION 192 After the RN is finished the initial assessment of a newborn baby and after the initial bonding between the newborn and the mother has taken place in the deliveryroom, the RN will bring the newborn to the well-baby nursery. Before the newborn is taken from the delivery room and brought to the well-baby nursery, the RN makes sure that which of the following interventions was completed? A. The physician verifies the exact time of birth. B. The nurse counts the instruments and sponges with the scrub nurse. C. The nurse instills prophylactic ointment in the conjunctival sacs of the newborn's eyes. D. The nurse makes sure the mother and her newborn have been tagged with identical bands. Correct Answer: D Section: (none) Explanation Explanation/Reference: Explanation: (A) The delivery room personnel are responsible for verifying time of birth. (B) The scrub and circulating nurses count sponges and instruments. (C) This intervention is done in the nursery. (D) Tagging the mother and infant with identical bands is of utmost importance. The mother wears one band, and the newborn wears two. Identical numbers on the three bands provide identification for the newborn and the birth mother. Every time the newborn is brought to the mother after delivery, those bands are checked to be sure that the numbers are identical. QUESTION 193 On a mother's 2nd postpartum day after having a vaginal delivery, the RN is preparing to assess her perineum and anus as part of her daily assessment. The best position for the client to be placed in for this assessment is: A. Sims' B. Fowler's C. Prone D. Any position that the RN chooses Correct Answer: A Section: (none) Explanation Explanation/Reference: Explanation: (A) The Sims' position is the best position for assessment of the perineum and anus. The top leg is placed over the bottom leg, and the RN raises the upper buttocks to fully expose the perineum and anus. (B) Fowler's position is a sitting position, and the perineum and anus would not be exposed. (C) The prone position would have the mother on her back, and her perineum and anus would not be exposed. (D) The position of choice should always be the Sims'.QUESTION 194 While the RN is assessing a mother's perineum on her 2nd postpartum day after having a vaginal delivery, the RN notes a large ecchymotic area located to the left of the mother's perineum. Which one of the following interventions should the RN initiate at this time? A. Have the client expose the area to air. B. Apply ice to the perineum. C. Encourage the client to take warm sitz baths. D. Inform the physician. Correct Answer: C Section: (none) Explanation Explanation/Reference: Explanation: (A) The area is bruised and painful. This action would do nothing to help with the healing process of the perineum or to provide comfort. (B) Ice is effective immediately after birth to reduce edema and discomfort, but not on the 2nd postpartum day. (C) Sitz baths are useful if the perineum has been traumatized, because the moist heat increases circulation to the area to promote healing, relaxes tissue, and decreases edema. (D) The physician is not notified of bruising, but if a hematoma is present, then the physician is notified. QUESTION 195 A mother who is breast-feeding her newborn asks the RN, "How can I express milk from my breasts manually?" The RN tells her that the correct method for manual milk expression includes using the thumb and the index finger to: A. Alternately compress and release each nipple B. Roll the nipple and gently pull the nipple forward C. Slide the thumb and index finger forward from the outer border of the areola toward the end of the nipple D. Compress and release each breast at the outer border of the areola Correct Answer: D Section: (none) Explanation Explanation/Reference: Explanation: (A) Manipulation of nipples will cause soreness and trauma. (B) Pulling the nipples will cause discomfort and soreness. (C) Sliding the thumb and index finger forward over the nipple will cause soreness. (D) The best method to express milk from the breast is to position the thumb and index finger at the outer border of the areola and compress. This is the location of the milk sinuses.QUESTION 196 A client had a vaginal delivery 3 days ago and is discharged from the hospital on the 2nd day postpartum. She told the RN, "I need to start exercising so that I can get back into shape. Could you suggest an exercise I could begin with?'' The RN could suggest which one of the following? A. Push-ups B. Jumping jacks C. Leg lifts D. Kegel exercises Correct Answer: D Section: (none) Explanation Explanation/Reference: Explanation: (A, B, C) This exercise is too strenuous at this time. (D) This exercise is recommended for the first few days after delivery. It helps to stimulate muscle tonus in the area of the perineum and the area around the urinary meatus and vagina. QUESTION 197 A 60-year-old male client was hospitalized 3 days ago with the diagnosis of acute anterior wall myocardial infarction. Today he has been complaining of increasing weakness and shortness of breath. Crackles in both lung bases are audible on auscultation. He is developing: A. An extension of his myocardial infarction B. Pneumonia C. Pulmonary edema D. Pulmonary emboli Correct Answer: C Section: (none) Explanation Explanation/Reference: Explanation: (A) Extensions of his myocardial infarction would be chest pain unrelieved with nitroglycerin, cardiac enzyme elevations, and electrocardiographic changes. (B) Persons with pneumonia may complain of weakness and shortness of breath and have crackles in their lung bases. However, they would also have sputum production and leukocytosis. (C) Persons who have had myocardial infarctions (especially anterior wall) are at risk of developing left ventricular heart failure, which is a major cause of pulmonary edema. Pulmonary edema is manifest by shortness of breath, weakness, and crackles on auscultation of the lung fields. (D)Pulmonary emboli may be accompanied by shortness of breath, weakness, and crackles. However, the pulmonary hypertension that accompanies pulmonary emboli results in signs of increased systemic venous pressure as well. QUESTION 198 On admission, the client has signs and symptoms of pulmonary edema. The nurse places the client in the most appropriate position for a client in pulmonary edema, which is: A. High Fowler B. Lying on the left side C. Sitting in a chair D. Supine with feet elevated Correct Answer: A Section: (none) Explanation Explanation/Reference: Explanation: (A) High Fowler position decreases venous return to the heart and permits greater lung expansion so that oxygenation is maximized. (B) Lying on the left side may improve perfusion to the left lung but does not promote lung expansion. (C) Sitting in a chair will decrease venous return and promote maximal lung expansion. However, clients with pulmonary edema can deteriorate quickly and require intubation and mechanical ventilation. If a client is sitting in achair when this deterioration happens, it will be difficult to intervene quickly. (D) The supine with feet elevated position increases venous return and will worsen pulmonary edema. QUESTION 199 A male client has been hospitalized with congestive heart failure. Medical management of heart failure focuses on improving myocardial contractility. This can be achieved by administering: A. Digoxin (Lanoxin) 0.25 mg po every day B. Furosemide (Lasix) 40 mg po every morning C. O22 L/min via nasal cannula D. Nitroglycerin (Nitrol) 1 inch topically every 4 hours Correct Answer: A Section: (none) Explanation Explanation/Reference: Explanation: (A) Digoxin is a cardiac glycoside given to clients in heart failure to improve their myocardial contractility. (B) Furosemide is a loop diuretic given to clients in heartfailure to promote diuresis. (C) O2is given to clients in heart failure to increase oxygenation and to prevent or treat hypoxemia. (D) Nitroglycerin is a nitrate given to clients in heart failure to increase their cardiac output by decreasing the peripheral resistance that the left ventricle must pump against. QUESTION 200 A client's congestive heart failure has been treated, and he will soon be discharged. Discharge teaching should include instruction to call the physician if he notices a 2-lb weight gain in a 24- hour period. Increased weight gain may indicate: A. A diet too high in calories and saturated fat B. Decreasing cardiac output C. Decreasing renal function D. Development of diabetes insipidus Correct Answer: B Section: (none) Explanation Explanation/Reference: Explanation: (A) Increased calories may result in weight gain, but there is no indication in this question that this man's diet has changed in a way that would result in increased calories. (B) Decreasing cardiac output stimulates the renin-angiotensin-aldosterone cycle and results in fluid retention, which is reflected by weight gain. (C) Decreasing renal function may result in fluid retention, but this question gives no indication that this man has any renal problems. (D) Profound diuresis occurs with diabetes insipidus, which results in weight loss. QUESTION 201 A client was not using his seat belt when involved in a car accident. He fractured ribs 5, 6, and 7 on the left and developed a left pneumothorax. Assessment findings include: A. Crackles and paradoxical chest wall movement B. Decreased breath sounds on the left and chest pain with movement C. Rhonchi and frothy sputum D. Wheezing and dry cough Correct Answer: B Section: (none) Explanation Explanation/Reference: Explanation:(A) Crackles are caused by air moving through moisture in the small airways and occur with pulmonary edema. Paradoxical chest wall movement occurs with flail chest when a segment of the thorax moves outward on inspiration and inward on expiration. (B) Decreased breath sounds occur when a lung is collapsed or partially collapsed. Chest pain with movement occurs with rib fractures. (C) Rhonchi are caused by air moving through large fluid-filled airways. Frothy sputum may occur with pulmonary edema. (D) Wheezing is caused by fluid in large airways already narrowed by mucus or bronchospasm. Dry cough could indicate a cardiac problem. QUESTION 202 A client has a chest tube placed in his left pleural space to re-expand his collapsed lung. In a closed-chest drainage system, the purpose of the water seal is to: A. Prevent air from entering the pleural space B. Prevent fluid from entering the pleural space C. Provide a means to measure chest drainage D. Provide an indicator of respiratory effort Correct Answer: A Section: (none) Explanation Explanation/Reference: Explanation: (A) A chest tube extends from the pleural space to a collection device. The tube is placed below the surface of the saline so that air cannot enter the pleural space. (B) Fluid may enter the pleural space as a result of injury or disease. A chest tube may drain fluid from the pleural space, but the water seal is not involved in this. (C) Chest drainage should be measured, but the water seal is not involved in this. (D) Fluctuations in the tube in the water-sealed bottle will give an indication of respiratory effort, but that is not the purpose of the water seal. QUESTION 203 A client was admitted with rib fractures and a pneumothorax, which were sustained as a result of a motor vehicle accident. A chest tube was placed on the left side to reinflate his lung, and he was transferred to a client unit. Twenty-four hours after admission he continues to have bloody sputum, develops increasing hypoxemia, and his chest x-ray shows patchy infiltrates. The nurse analyzes these symptoms as being consistent with: A. Pneumonia B. Pulmonary contusions C. Pulmonary edema D. Tension pneumothorax Correct Answer: B Section: (none) ExplanationExplanation/Reference: Explanation: (A) Pneumonia may be reflected by patchy infiltrates. In addition, fever, an increasing white blood cell count, and copious sputum production would be present. (B) Blunt chest traumacauses a bruising process in which interstitial and alveolar edema and hemorrhage occur. This is manifest by gradual deterioration over 24 hours of arterial blood gases and the continued production of bloody sputum. Patchy infiltrates are evident on chest xray 24 hours postinjury. (C) Pulmonary edema usually results from left heart failure. It is manifest by pink, frothy sputum; increasing dyspnea; tachycardia; and crackles on auscultation. (D) Tension pneumothorax is a potential complication for someone with rib fractures and a chest tube. It is manifest by diminished breath sounds on the affected side, rapidly deteriorating arterial blood gases in the presence of an open airway, and shock that is unexplained by other injuries. QUESTION 204 A 66-year-old female client has smoked 2 packs of cigarettes per day for 20 years. Her arterial blood gases on room air are as follows: pH 7.35; PO2 70 mm Hg; PCO2 55 mm Hg; HCO3 32 mEq/L. These blood gases reflect: A. Compensated metabolic acidosis B. Compensated respiratory acidosis C. Compensated respiratory alkalosis D. Uncompensated respiratory acidosis Correct Answer: B Section: (none) Explanation Explanation/Reference: Explanation: (A) In compensated metabolic acidosis, the pH level is normal, the PCO2level is decreased, and the HCO3level is decreased. The client's primary alteration is an inability to remove excess acid via the kidneys. The lungs compensate by hyperventilating and decreasing PCO2. (B) In compensated respiratory acidosis, the pH level is normal, the PCO2level is elevated, and the HCO3level is elevated. The client's primary alteration is an inability to remove CO2from the lungs, so over time,the kidneys increase reabsorption of HCO3to buffer the CO2. (C) In compensated respiratory alkalosis, the pH level is normal, the PCO2level is decreased, and the HCO3level is decreased. The client's primary alteration is hyperventilation, which decreases PCO2. The client compensates by increasing the excretion of HCO3from the body. (D) In uncompensated respiratory acidosis, the pH level is decreased, the PCO2level is increased, and the HCO3level is normal. The client's primary alteration is an inability to remove CO2from the lungs. The kidneys have not compensated by increasing HCO3reabsorption. QUESTION 205 A female client who has chronic obstructive pulmonary disease (COPD) has presented in the emergency department with cough productive of yellow sputum and increasing shortness of breath. On room air, her blood gases are as follows: pH 7.30 mm Hg, PCO2 60 mm Hg, PO2 55 mm Hg, HCO3 32 mEq/L. These arterial blood gases reflect: A. Compensated respiratory acidosisB. Normal blood gases C. Uncompensated metabolic acidosis D. Uncompensated respiratory acidosis Correct Answer: D Section: (none) Explanation Explanation/Reference: Explanation: (A) In compensated respiratory acidosis, the pH level is normal, the PCO2level is elevated, and the HCO3level is elevated. The client's primary alteration is an inability to remove CO2from the lungs, so over time, the kidneys increase reabsorption of HCO3to buffer the CO2. (B) Normal ranges for arterial blood gases for adults and children are as follows: pH 7.357.45, PO280100 mm Hg, PCO23545 mm Hg, HCO32128 mEq/L. (C) In uncompensated metabolic acidosis the pH level is decreased, the PCO2level is normal, and the HCO3level is decreased. The client's primary alteration is an inability to remove excess acid via the kidneys. The lungs are unable to clear the increased acid. (D) In uncompensated respiratory acidosis, the pH level is decreased, the PCO2level is increased, and the HCO3level is normal. In a person with long-standing COPD, the HCO3level will rise gradually over time to compensate for the gradually increasing PCO2, and the person's pH level will be normal. When a person with COPD becomes acutely ill, the kidneys do not have time to increase the reabsorption of HCO3, so the person's pH level will reflect acidosis even though the HCO3is elevated. QUESTION 206 A 19-year-old client has sustained a C-7 fracture, which resulted in his spinal cord being partially transected. By 2 weeks' postinjury, his neck has been surgically stabilized, and he has been transferred from the intensive care unit. A potential life-threatening complication the nurse monitors the client for is: A. Autonomic dysreflexia B. Bradycardia C. Central cord syndrome D. Spinal shock Correct Answer: A Section: (none) Explanation Explanation/Reference: Explanation: (A) Autonomic dysreflexia is the exaggerated sympathetic nervous system response to various stimuli in the anesthetized area. Sympathetic stimulation results in severe, uncontrolled hypertension, which may result in myocardial infarction or cerebral hemorrhage. (B) Bradycardia occurs as a result of sympathetic blockade in the immediate postinjury period. After spinal shock recedes, cardiovascular stability returns, but the client will be bradycardiac for life. (C) Central cord syndrome is a specific type of spinal cord injury that occurs as a result of either hyperextension injuries or disrupted blood flow to the spinal cord. (D) Spinal shock occurs in theimmediate postinjury phase and usually resolves in approximately 72 hours. QUESTION 207 A 17-year-old client has a T-4 spinal cord injury. At present, he is learning to catheterize himself. When he says, "This is too much trouble. I would rather just have a Foley.'' An appropriate response for the RN teaching him would be: A. "I know. It is a lot to learn. In the long run, though, you will be able to reduce infections if you do an intermittent catheterization program.'' B. "It is not too much trouble. This is the best way to manage urination.'' C. "OK. I'll ask your physician if we can replace the Foley.'' D. "You need to learn this because your doctor ordered it.'' Correct Answer: A Section: (none) Explanation Explanation/Reference: Explanation: (A) This response acknowledges the client's feelings, gives him factual information, and acknowledges that the final decision is his. (B) This response is judgmental and discourages the client from expressing his feelings about the procedure. (C) Catheterization is a procedure thattakes time to learn, but which, for the spinal cordinjured client, can significantly reduce the incidence of urinary tract infections. A young client with a T-4 injury has the hand function to learn this procedure fairly easily. (D) The final decision about bladder elimination management ultimately rests with the client and not the physician. QUESTION 208 A client's physician has prescribed theophylline (Theo- Dur) to help control the bronchospasm associated with the client's COPD. Instructions that should be given to the client include: A. "Call your physician if you develop palpitations, dizziness, or restlessness.'' B. "Cigarette smoking may significantly increase the risk for theophylline toxicity.'' C. "Take this medication on an empty stomach.'' D. "Do not take your medicine if your pulse is less than 60 beats per minute.'' Correct Answer: A Section: (none) Explanation Explanation/Reference: Explanation: (A) Indications of theophylline toxicity include palpitations, dizziness, restlessness, nausea, vomiting, shakiness, and anorexia. (B) Cigarette smoking significantly lowers theophylline plasma levels. (C) Theophylline should be taken with food to decrease stomach upset. (D) These instructions are appropriate for someonetaking digoxin. QUESTION 209 A 19-year-old client fell off a ladder approximately 3 ft to the ground. He did not lose consciousness but was taken to the emergency department by a friend to have a scalp laceration sutured. The nurse instructs the client to: A. Clean the sutured laceration twice a day with povidone- iodine (Betadine) B. Remove his scalp sutures after 5 days C. Return to the hospital immediately if he develops confusion, nausea, or vomiting D. Take meperidine 50 mg po q46h prn for headache Correct Answer: C Section: (none) Explanation Explanation/Reference: Explanation: (A) Povidone-iodine is very irritating to skin and should not be routinely used. (B) Sutures should not be removed by the client. (C) Confusion, nausea, vomiting,and behavioral changes may indicate increasing intracranial pressure as a result of intracerebral bleeding. (D) Use of a narcotic opiate such as meperidine is not recommended in clients with a possible head injury because it may produce sedation, pupil changes, euphoria, and respiratory depression, which may mask the signs of increasing intracranial pressure. QUESTION 210 A male client has asthma and his physician has prescribed beclomethasone (Vanceril) 3 puffs tid in addition to his other medications. After taking his beclomethasone, the client should be instructed to: A. Clean his inhaler with warm water and soak it in a10% bleach solution B. Drink a glass of water C. Sit and rest D. Use his bronchodilator inhaler Correct Answer: B Section: (none) Explanation Explanation/Reference: Explanation: (A) Inhalers should be cleaned once a day. They should be taken apart, washed in warm water, and dried according to manufacturer's instructions. Soaking inbleach is inappropriate. (B) A common side effect of inhaled steroid preparations is oral candidal infection. This can be prevented by drinking a glass of water or gargling after using a steroid inhaler. (C) There is nothing wrong with sitting and resting after using a steroid inhaler, but it is not necessary. (D) If a person is using a steroid inhaler as well as a bronchodilator inhaler, the bronchodilator shouldalways be used first. The reason for this is that the bronchodilator opens up the person's airways so that when the steroid inhaler is used next, there will be better distribution of medication. QUESTION 211 A 70-year-old client has pneumonia and has just had a respiratory arrest. He has just been intubated with an 8- mm endotracheal tube. During auscultation of his chest, breath sounds were found to be absent on the left side. The nurse identifies the most likely cause of this as: A. Inappropriate endotracheal tube size B. Left-sided pneumothorax C. Right mainstem bronchus intubation D. Pneumonia Correct Answer: C Section: (none) Explanation Explanation/Reference: Explanation: (A) Appropriate endotracheal tube sizes for adults range from 7.08.5 mm. (B) Pneumothorax could be indicated by an absence of breath sounds on the affected side. However, in a recently intubated client, the first priority would be to consider tube malposition. (C) During intubation, the right mainstem bronchus can be inadvertently entered if the endotracheal tube is inserted too far. Left mainstem bronchus intubation almost never occurs because of the angle of the left mainstem bronchus. (D) Breath sounds for someone with pneumonia may be decreased over the areas of consolidation. However, in a recently intubated client, the first priority would be to consider tube malposition. QUESTION 212 A 55-year-old client is unconscious, and his physician has decided to begin tube feeding him using a smallbore silicone feeding tube (Keofeed, Duo-Tube). After the tube is inserted, the nurse identifies the most reliable way to confirm appropriate placement is to: A. Aspirate gastric contents B. Auscultate air insufflated through the tube C. Obtain a chest x-ray D. Place the tip of the tube under water and observe for air bubbles Correct Answer: C Section: (none) ExplanationExplanation/Reference: Explanation: (A) Aspiration of gastric contents is usually a reliable way to verify tube placement. However, if the client has dark respiratory secretions from bleeding, tube feedings could be mistaken for respiratory secretions; in other words, aspirating an empty stomach is less reliable in this instance. In addition, it is common for small-bore feeding tubes to collapse when suction pressure is applied. (B) Insufflation of air into large-bore nasogastric tubes can usually be clearly heard. In small- bore tubes, it is more difficult to hear air, and it is difficult to distinguish between air in the stomach and air in the esophagus. (C) A chest x-ray is the most reliable means to determine placement of small-bore nasogastric tubes. (D) Observing for air bubbles when the tip is held under water is an unreliable means to determine correct tube placement for all types of nasogastric tubes. Air may come from both the respiratory tract and the stomach, and the client who is breathing shallowly may not force air out of the tube into the water. QUESTION 213 A 70-year-old client is almost finished receiving her second unit of packed red blood cells. The client, who weighs 80 lb, has started complaining of being short of breath and now has crackles in the bases of her lungs. After slowing or stopping the transfusion, the most appropriate initial nursing action would be to: A. Raise the client's head and place her feet in a dependent position B. Notify the physician C. Place the client on 2 liters of O2 via nasal cannula D. Administer furosemide (Lasix) 20 mg IV push Correct Answer: A Section: (none) Explanation Explanation/Reference: Explanation: (A) Raising the client's head and placing her feet in a dependent position is an independent nursing action that can be taken to decrease venous return and to reduce pulmonary congestion. (B) Notifying the physician is an appropriate action that should be taken after the client is positioned to maximize her respiratorystatus. (C) Placing the client on O2may be done with a physician's order or according to an institution's standing orders; however, other actions should be taken first. (D) Furosemide 20 mg IV push is an appropriate medication for the client, but it must be ordered by her physician. QUESTION 214 A 52-year-old client's abdominal aortic aneurysm ruptured. She received rapid massive blood transfusions for bleeding. One potential complication of blood administration for which she is especially at risk is: A. Air embolus B. Circulatory overload C. Hypocalcemia D. HypokalemiaCorrect Answer: C Section: (none) Explanation Explanation/Reference: Explanation: (A) Air embolism is a potential complication of blood administration, but it is fairly rare and can be prevented by using good IV technique. (B) Circulatory overload is a potential complication of blood administration, but because this client is actively bleeding, she is not at high risk for overload. (C) Hypocalcemia is a potential complication of blood administration that occurs in situations where massive transfusion has occurred over a short period of time. It occurs because the citrate in stored blood binds with the client's calcium. Another potential complication for which this client is especially at risk is hypothermia, which can be prevented by using a blood warmer to administer the blood. (D) Hypokalemia is not a complication of blood administration. QUESTION 215 A 67-year-old client will be undergoing a coronary arteriography in the morning. Client teaching about postprocedure nursing care should include that: A. Bed rest with bathroom privileges will be ordered B. He will be kept NPO for 812 hours C. Some oozing of blood at the arterial puncture site is normal D. The leg used for arterial puncture should be keptstraight for 812 hours Correct Answer: D Section: (none) Explanation Explanation/Reference: Explanation: (A) Bed rest will be ordered for 812 hours postprocedure. Flexing of the leg at the arterial puncture site will occur if the client gets out of bed, and this is contraindicated after arteriography. (B) The client will be able to eat as soon as he is alert enough to swallow safely and that will depend on what medications areused for sedation during the procedure. (C) Oozing at the arterial puncture site is not normal and should be closely evaluated. (D) The leg where the arterial puncture occurred must be kept straight for 812 hours to minimize the risk of bleeding. QUESTION 216 A client had a myocardial infarction 5 days ago. His physician has ordered an echocardiogram to determine how his myocardial infarction has affected his ventricular wall motion. When the client asks if this test is painful, an appropriate response is: A. "No, but you must be able to ride on a stationary bicycle while the test is being performed." B. "No, but you will have to lie still and the gel that is used may be cool."C. "Yes, but your physician will be there and will order pain medicine for you." D. "Your physician has ordered medicine, which you will be given before you go for the test, which will make you sleepy." Correct Answer: B Section: (none) Explanation Explanation/Reference: Explanation: (A) Riding a stationary bicycle or walking on a treadmill is done during a stress test. (B) During an echocardiogram, the client must lie supine while a technician performs the test. To perform the test, the technician uses a conductive gel and a transducer to obtain ultrasound tracings of the heart. (C) A physician need not be present during an echocardiogram, and it is neither invasive nor painful. (D) There is no premedication required for an echocardiogram. QUESTION 217 A 55-year-old man has recently been diagnosed with hypertension. His physician orders a low- sodium diet for him. When he asks, "What does salt have to do with high blood pressure?'' the nurse's initial response would be: A. "The reason is not known why hypertension is associated with a high-salt diet." B. "Large amounts of salt in your diet can cause you to retain fluid, which increases your blood pressure." C. "Salt affects your blood vessels and causes your blood pressure to be high." D. "Salt is needed to maintain blood pressure, but too much causes hypertension." Correct Answer: B Section: (none) Explanation Explanation/Reference: Explanation: (A) This response is untrue. (B) Decreasing salt intake reduces fluid retention and decreases blood pressure. (C) Salt does not have an effect on the blood vessels themselves, but on fluid retention, which accompanies salt intake. (D) This response is untrue. QUESTION 218 A client has consented to have a central venous catheter placed. The best position in which to place the client is the Trendelenburg position. The reason is that the Trendelenburg position: A. Allows the physician to visualize the subclavian vein B. Reduces the possibility of air embolism C. Reduces the possibility of hematoma formationD. Makes the procedure more comfortable for the client Correct Answer: B Section: (none) Explanation Explanation/Reference: Explanation: (A) The subclavian vein is not visible during central line insertion regardless of the client's position. (B) The Trendelenburg position reduces the possibility of air embolism because it places slight positive pressure on the central veins. It also distends the veins, and distention facilitates insertion. (C) This response is untrue; it has no effect on hematoma formation. (D) This position is not necessarily more comfortable for the client, and many clients, especially those who may be short of breath, may find the position uncomfortable and difficult to maintain. QUESTION 219 A newborn infant is exhibiting signs of respiratory distress. Which of the following would the nurse recognize as the earliest clinical sign of respiratory distress? A. Cyanosis B. Increased respirations C. Sternal and subcostal retractions D. Decreased respirations Correct Answer: C Section: (none) Explanation Explanation/Reference: Explanation: (A) Cyanosis is a late clinical sign of respiratory distress. (B) Rapid respirations are normal in a newborn. (C) The newborn has to exert an extra effort for ventilation, which is accomplished by using the accessory muscles of ventilation. The diaphragm and abdominal muscles are immature and weak in the newborn. (D) Decreased respirations are a late clinical sign. In the newborn, decreased respirations precede respiratory failure. QUESTION 220 A nurse is performing a vaginal exam on a client in active labor. An important landmark to assess during laborand delivery are the ischial spines because: A. Ischial spines are the narrowest diameter of the pelvis B. Ischial spines are the widest diameter of the pelvis C. They represent the inlet of birth canalD. They measure pelvic floor Correct Answer: A Section: (none) Explanation Explanation/Reference: Explanation: (A) The fetal descent, or station, is determined by the relationship of the presenting part to the spine. (B) Ischial spines are the narrowest measurement. (C) Ischial spines measure the pelvic outlet. (D) Pelvic floor measurement is not related to fetal descent. QUESTION 221 The nurse instructs a client on the difference between true labor and false labor. The nurse explains, "In true labor: A. Uterine contractions will weaken with walking." B. Uterine contractions will strengthen with walking." C. The cervix does not dilate." D. The fetus does not descend." Correct Answer: B Section: (none) Explanation Explanation/Reference: Explanation: (A) Uterine contractions increase with activity. (B) Walking will increase the strength and regularity of uterine contractions in true labor. (C) Uterine contractions that are strong and regular facilitate cervical dilation. (D) Regular, strong uterine contractions, as in true labor, result in fetal descent. QUESTION 222 A first-trimester primigravida is diagnosed with anemia. The nurse should suspect that this anemia is a result of: A. Mother's increased blood volume B. Mother's decreased blood volume C. Fetal blood volume increase D. Increase in iron absorption Correct Answer: A Section: (none)Explanation Explanation/Reference: Explanation: (A) Maternal blood volume increases at the end of the first trimester leading to a dilutional anemia. (B) Maternal blood volume increases. (C) Fetal blood volume is minimal in the first trimester. (D) Increased iron absorption would facilitate the manufacturing of erythrocytes and decrease anemia. QUESTION 223 In client teaching, the nurse should emphasize that fetal damage occurs more frequently with ingestion of drugs during: A. First trimester B. Second trimester C. Third trimester D. Every trimester Correct Answer: A Section: (none) Explanation Explanation/Reference: Explanation: (A) Organogenesis occurs in the first trimester. Fetus is most susceptible to malformation during this period. (B) Organogenesis has occurred by the second trimester. (C) Fetal development is complete by this time. (D) The dangerous period for fetal damage is the first trimester, not the entire pregnancy. QUESTION 224 A laboring client presents with a prolapsed cord. The nurse should immediately place the client in what position? A. Reverse Trendelenburg B. Fowler's C. Trendelenburg D. Sims' Correct Answer: C Section: (none) Explanation Explanation/Reference: Explanation:(A) Reverse Trendelenburg position increases pressure on the perineum. This position will not relieve cord pressure. (B) Fowler's position increases perineal pressure. Cord pressure would not be relieved. (C) Trendelenburg position will decrease perineal pressure. Cord compression will be decreased and increase in fetal blood flow occurs. (D) Sims' position does not relieve pressure on cord or perineum. QUESTION 225 A client suspects that she is pregnant. She reports two missed menstrual periods. The first day of her last menstrual period was August 3. Her estimated date of confinement would be: A. November 7 B. November 10 C. May 7 D. May 10 Correct Answer: D Section: (none) Explanation Explanation/Reference: Explanation: (A) Wrong calculation (B) Wrong calculation (C) Wrong calculation (D) Nägele's rule is: Expected Date of Confinement = Last Menstrual Period - 3 months + 7 days + 1 year QUESTION 226 An elective saline abortion has been performed on a 3- week primigravida. Following the procedure, the nurse should be alert for which early side effect? A. Water satiety B. Thirst C. Edema D. Diabetes insipidus Correct Answer: B Section: (none) Explanation Explanation/Reference: Explanation: (A) If the client is experiencing water satiety, there is no more desire for water. (B) Absorption of saline into circulation rather than into amniotic sac increasesserum sodium and desire for water. (C) Edema can be a late side effect caused by water intoxication. (D) Diabetes insipidus occurs as a result of deficient antidiuretic hormone. QUESTION 227 Assessment of a newborn for Apgar scoring includes observation for: A. Pupil response B. Respiratory rate C. Heart rate D. Babinski'sreflex Correct Answer: C Section: (none) Explanation Explanation/Reference: Explanation: (A) Pupil response should be assessed but is not part of Apgar scoring. (B) Respiratory effort is an essential part of Apgar scoring, not respiratory rate. (C) Heart rate is the most critical component of Apgar scoring. (D) Assessment of Babinski's reflex is not a component of Apgar scoring. QUESTION 228 Painless vaginal bleeding in the last trimester may be caused by: A. Menstruation B. Abruptio placentae C. Placenta previa D. Polyhydramnios Correct Answer: C Section: (none) Explanation Explanation/Reference: Explanation: (A) Menstruation should not occur during pregnancy. (B) Abruptio placentae is marked by painful vaginal bleeding following a premature placental detachment after 20th week of gestation. (C) A low-lying placenta separates from the uterine wall as the uterus contracts and cervix dilates. This separation causes painless bleeding in the 7th-8th month. (D) Polyhydramnios is excessive amniotic fluid.QUESTION 229 The nurse should facilitate bonding during the postpartum period. What should the nurse expect to observe in the taking-hold phase? A. Mother is concerned about her recovery. B. Mother calls infant by name. C. Mother lightly touches infant. D. Mother is concerned about her weight gain. Correct Answer: B Section: (none) Explanation Explanation/Reference: Explanation: (A) This observation can be made during the taking-in phase when the mother's needs are more important. (B) This observation can be made during the taking- hold phase when the mother is actively involved with herself and the infant. (C, D) This observation can be made during the taking-in phase. QUESTION 230 The physician is preparing to induce labor on a 40-week multigravida. The nurse should anticipate the administration of: A. Oxytocin (Pitocin) B. Progesterone C. Vasopressin (Pitressin) D. Ergonovine maleate Correct Answer: A Section: (none) Explanation Explanation/Reference: Explanation: (A) Oxytocin is a hormone secreted by the neurohypophysis during suckling and parturition that produces strong uterine contractions. (B) Progesterone has a quiescence effect on the uterus. (C) Vasopressin is an antidiuretic hormone that promotes water reabsorption by the renal tubules. (D) Ergonovine producesdystocia as a result of sustained uterine contractions. QUESTION 231 A primigravida is at term. The nurse can recognize the second stage of labor by the client's desire to:A. Push during contractions B. Hyperventilate during contractions C. Walk between contractions D. Relax during contractions Correct Answer: A Section: (none) Explanation Explanation/Reference: Explanation: (A) The second stage of labor is characterized by uterine contractions, which cause the client to bear down. (B) Slow, deep, rhythmic breathing facilitates the laboring process. Hyperventilation is abnormal breathing resulting from loss of pain control. (C) The client should remain on bed rest during labor. (D) Contractions result in discomfort. QUESTION 232 A pregnant client during labor is irritable and feels the urge to vomit. The nurse should recognize this as the: A. Fourth stage of labor B. Third stage of labor C. Transition stage of labor D. Second stage of labor Correct Answer: C Section: (none) Explanation Explanation/Reference: Explanation: (A) The fourth stage begins after expulsion of the placenta. Client symptoms are: fatigue; chills; scant, bloody vaginal discharge; and nausea. (B) The third stage is from birth to expulsion of placenta. Client symptoms are uterine contractions, gush of blood, and perineal pain. (C) The transition stage is characterized by strong uterine contractions and cervical dilation. Clientsymptoms are irritability, restlessness, belching, muscle tremors, nausea, and vomiting. (D) The second stage is characterized by full dilation of cervix. Client symptoms are perineal bulge, pushing with contractions, great irritability, and leg cramps. QUESTION 233 A pregnant client experiences spontaneous rupture of membranes. The first nursing action is to:A. Assess the client's respirations B. Notify the physician C. Auscultate fetal heart rate D. Transfer to delivery suite Correct Answer: C Section: (none) Explanation Explanation/Reference: Explanation: (A) Immediately following membrane rupture, the fetus is at risk for complications, not necessarily the mother. (B) The physician is notified after the nurse completes an assessment of the mother's and fetus's conditions. (C) Rupture of membranes facilitates fetal descent. A potential complication is cord prolapse, which is assessed by auscultating fetal heart rate. (D) Rupture of membranes does not necessarily indicate readiness to deliver. QUESTION 234 A pregnant client experiences a precipitous delivery. The nursing action during a precipitous delivery is to: A. Control the delivery by guiding expulsion of fetus B. Leave the room to call the physician C. Push against the perineum to stop delivery D. Cross client's legs tightly Correct Answer: A Section: (none) Explanation Explanation/Reference: Explanation: (A) Controlling the rapid delivery will reduce the risk of fetal injury and perineal lacerations. (B) The nurse should always remain with a client experiencing aprecipitous delivery. (C) Pushing against the perineum may cause fetal distress. (D) Crossing of legs may cause fetal distress and does not stop the delivery process. QUESTION 235 Following a vaginal delivery, the postpartum nurse should observe for: A. Dystocia, kraurosisB. Chadwick's sign C. Fatigue, hemorrhoids D. Hemorrhage and infection Correct Answer: D Section: (none) Explanation Explanation/Reference: Explanation: (A) Dystocia is difficult labor. The delivery has occurred. Kraurosis is atrophy and dryness of skin and any mucous membrane (vulva). (B) Chadwick's sign is a bluish color of vaginal mucosa suggestive of pregnancy. (C) Fatigue is a common symptom in the postpartal period. Hemorrhoids may occur with pregnancy. (D) Hemorrhage and infection are potential complications of vaginal delivery. Hemorrhage may result from retained placental fragments or soft uterus. Infection may occur from the introduction of organisms into the uterus during the delivery. QUESTION 236 A client who is 7 months pregnant is diagnosed with pyelonephritis. The nurse anticipates the physician ordering: A. Oxytocin B. Magnesium sulfate (MgSO4) C. Ampicillin D. Tetracycline Correct Answer: C Section: (none) Explanation Explanation/Reference: Explanation: (A) Oxytocin is prescribed to stimulate uterine contractions. (B) MgSO4is a central nervous system depressant prescribed to prevent and control convulsions related to preeclampsia. (C) Ampicillin is a penicillin derivative with no known teratogenic effects. Thisis the safest antibiotic during pregnancy. (D) Tetracycline stains teeth yellow and is not as safe as ampicillin during pregnancy. QUESTION 237 A newborn is admitted to the newborn nursery with tremors, apnea periods, and poor sucking reflex. The nurse should suspect: A. Central nervous system damageB. Hypoglycemia C. Hyperglycemia D. These are normal newborn responses to extrauterine life Correct Answer: B Section: (none) Explanation Explanation/Reference: Explanation: (A) Central nervous system damage presents as seizures, decreased arousal, and absence of newborn reflexes. (B) In a diabetic mother, the infant is exposed to high serum glucose. The fetal pancreas produces large amounts of insulin, which causes hypoglycemia after birth. (C) Hypoglycemia is a common newborn problem. Increased insulin production causes hypoglycemia, not hyperglycemia. (D) These are not normal adaptive behaviors to extrauterine life. QUESTION 238 A premature infant needs supplemental O2 therapy. A nursing intervention that reduces the risk of retrolental fibroplasia is to: A. Maintain O2at <40% B. Maintain O2at>40% C. Give moist O2at>40% D. Maintain on 100% O2 Correct Answer: A Section: (none) Explanation Explanation/Reference: Explanation: (A) Retrolental fibroplasia is the result of prolonged exposure to high levels of O2in premature infants. Complications are hemorrhage and retinal detachment. (B, C, D) O2concentration is too high. QUESTION 239 A client presents to the psychiatric unit crying hysterically. She is diagnosed with severe anxiety disorder. The first nursing action is to: A. Demand that she relax B. Ask what is the problem C. Stand or sit next to herD. Give her something to do Correct Answer: C Section: (none) Explanation Explanation/Reference: Explanation: (A) This nursing action is too controlling and authoritative. It could increase the client's anxiety level. (B) In her anxiety state, the client cannot rationally identify a problem. (C) This nursing action conveys a message of caring and security. (D) Giving the client a task would increase her anxiety. This would be a late nursing action. QUESTION 240 A schizophrenic is admitted to the psychiatric unit. What affect would the nurse expect to observe? A. Anger B. Apathy and flatness C. Smiling D. Hostility Correct Answer: B Section: (none) Explanation Explanation/Reference: Explanation: (A) Anger is an emotion that is not necessarily present in schizophrenia. (B) Lack of response to or involvement with environment and distancing are characteristic of schizophrenia. (C) Euphoria is more characteristic of manic-depressive disorder (bipolar disorder). (D) Hostility is an emotion that is not necessarily present in schizophrenia. QUESTION 241 A 16-year-old client reports a weight loss of 20% of her previous weight. She has a history of food binges followed by self-induced vomiting (purging). The nurse should suspect a diagnosis of: A. Anorexia nervosa B. Anorexia hysteria C. BulimiaD. Conversion reaction Correct Answer: C Section: (none) Explanation Explanation/Reference: Explanation: (A) Anorexia nervosa is characterized by self-starvation. (B) Anorexia hysteria is not a known disease or disorder. (C) Bulimia is characterized by food binges and self-induced vomiting. (D) Conversion reaction is a defense mechanism. QUESTION 242 A 24-year-old client presents to the emergency department protesting "I am God." The nurse identifies this as a: A. Delusion B. Illusion C. Hallucination D. Conversion Correct Answer: A Section: (none) Explanation Explanation/Reference: Explanation: (A) Delusion is a false belief. (B) Illusion is the misrepresentation of a real, external sensory experience. (C) Hallucination is a false sensory perception involving any of the senses. (D) Conversion is the expression of intrapsychic conflict through sensory or motor manifestations. QUESTION 243 A 30-year-old client has a history of several recent traumatic experiences. She presents at the physician's office with a complaint of blindness. Physical exam and diagnostic testing reveal no organic cause. The nurse recognizes this as: A. Delusion B. Illusion C. Hallucination D. ConversionCorrect Answer: D Section: (none) Explanation Explanation/Reference: Explanation: (A) The client's blindness is real. Delusion is a false belief. (B) Illusion is the misrepresentation of a real, external sensory experience. (C) Hallucination is a false sensory perception involving any of the senses. (D) Conversion is the expression of intrapsychic conflict through sensory or motor manifestations. QUESTION 244 A 40-year-old client is admitted to the hospital for tests to diagnose cancer. Since his admission, he has become dependent and demanding to the nursing staff. The nurse identifies this behavior as which defense mechanism? A. Denial B. Displacement C. Regression D. Projection Correct Answer: C Section: (none) Explanation Explanation/Reference: Explanation: (A) Denial is the disowning of consciously intolerable thoughts. (B) Displacement is the referring of a feeling or emotion from one person, object, or idea to another. (C) Regression isreturning to an earlier stage of development. (D) Projection is attributing one's own thoughts, feelings, or impulses to another person. QUESTION 245 A young boy tells the nurse, "I don't like my Dad to kiss or hug my Mom. I love my Mom and want to marry her." The nurse recognizes this stage of growth and development as: A. Electra complex B. Oedipus complex C. Superego D. Ego Correct Answer: B Section: (none)Explanation Explanation/Reference: Explanation: (A) The Electra complex is the erotic attachment of the female child to the father. (B) The Oedipus complex is characterized by jealousy toward the parent of the same sex and erotic attachment to the parent of the opposite sex. (C) The superego as described by Freud is the part of personality that is associated with internalized parental and societal control. (D) The ego as described by Freud is the part of personality that is associated with reality assessment. QUESTION 246 A client was prescribed a major tranquilizer 2 months ago. One month ago she was placed on benztropine (Cogentin). What would indicate that benztropine therapy is effective? A. Smooth, coordinated voluntarymovement B. Tremors C. Rigidity D. Muscle weakness Correct Answer: A Section: (none) Explanation Explanation/Reference: Explanation: (A) Benztropine is prescribed to decrease or alleviate extrapyramidal side effects of major tranquilizers. Smooth, coordinated voluntarymovement indicates minimal extrapyramidal side effects. (B) Tremors are an extrapyramidal side effect. (C) Rigidity is an extrapyramidal side effect. (D) Muscle weakness is an extrapyramidal side effect. QUESTION 247 A client is diagnosed with organic brain disorder. The nursing care should include: A. Organized, safe environment B. Long, extended family visits C. Detailed explanations of procedures D. Challenging educational programs Correct Answer: A Section: (none) ExplanationExplanation/Reference: Explanation: (A) A priority nursing goal is attending to the client's safety and well-being. Reorient frequently, remove dangerous objects, and maintain consistent environment. (B) Short, frequent visits are recommended to avoid overstimulation and fatigue. (C) Short, concise, simple explanations are easier to understand. (D) Mental capability and attention span deficits make learning difficult and frustrating. QUESTION 248 A 4-year-old child has Down syndrome. The community health nurse has coordinated a special preschool program. The nurse's primary goal is to: A. Provide respite care for the mother B. Facilitate optimal development C. Provide a demanding and challenging educational program D. Prepare child to enter mainstream education Correct Answer: B Section: (none) Explanation Explanation/Reference: Explanation: (A) Respite care for the family may be needed, but it is not the primary goal of a preschool program. (B) Facilitation of optimal growth and development is essential for every child. (C) A demanding and challenging educational program may predispose the child to failure. Children with retardation should begin with simple and challenging educational programs. (D) Mental retardation associated with Down syndrome may not permit mainstream education. A preschoolprogram's primary goal is not preparation for mainstream education but continuation of optimal development. QUESTION 249 A 13-year-old hemophiliac is hospitalized for hemarthrosis of hisright knee. To relieve the pain, the nurse should: A. Place on bed rest; elevate and splint the right knee B. Apply moist heat to the right knee C. Administer aspirin for pain D. Encourage active range of motion to right knee Correct Answer: A Section: (none) ExplanationExplanation/Reference: Explanation: (A) Immobilization, splinting, and bed rest will reduce the bleeding. Once bleeding is reduced or stopped, the pain will subside. (B) Moist heat causes vasodilation and bleeding. Ice or cold compresses should be applied. (C) Aspirin decreases platelet aggregation, which causes bleeding. (D) Active range of motion aggravates bleeding and damages the synovial sac during bleeding episodes. QUESTION 250 A 3-month-old infant has had a unilateral cleft lip repair. He has resumed feedings of oral formula. The nurse should feed the infant with: A. Gavage tube B. Nipple and bottle C. A straw and cup D. Syringe Correct Answer: D Section: (none) Explanation Explanation/Reference: Explanation: (A) A gavage tube may damage suture line. It is the most invasive and should be the last measure. (B) A nipple and bottle require sucking, which may damage sutures. (C) A 3- month-old infant is not able to drink from a straw. (D) A syringe allows for the formula to be placed to the side and back of the mouth. This minimizes the amount of sucking needed. QUESTION 251 A 3-year-old child is admitted with a diagnosis of possible noncommunicating hydrocephalus. What is the first symptom that indicates increased intracranial pressure? A. Bulging fontanelles B. Seizure C. Headache D. Ataxia Correct Answer: C Section: (none) Explanation Explanation/Reference:Explanation: (A) Bulging fontanelles are a symptom of increased intracranial pressure in infants. (B) Seizure is a late sign of increased intracranial pressure. (C) Headache is a very early symptom of increased intracranial pressure in the child. (D) Ataxia is a late sign of increased intracranial pressure. QUESTION 252 What is the appropriate nursing action for a child with increased intracranial pressure? A. Head of bed elevated 45 degrees with child's head maintained in a neutral position B. Child lying flat C. Head turned to side D. Frequent visitation for stimulation Correct Answer: A Section: (none) Explanation Explanation/Reference: Explanation: (A) Elevation of head of bed and neutral head position promote drainage of cerebrospinal fluid. (B) Flat position increases intracranial pressure and impedes cerebrospinal fluid drainage. (C) Head turned to either side impedes cerebrospinal fluid drainage. (D) Child should be in a calm, quiet environment with minimal stimulation. QUESTION 253 A client is 2 hours post ventriculoperitoneal shunt placement. How should the nurse position the client? A. Head of bed elevated 30 degrees on nonoperative side B. Head of bed elevated 30 degrees on operative side C. Bed flat on operative side D. Bed flat on nonoperative side Correct Answer: D Section: (none) Explanation Explanation/Reference: Explanation: (A) Elevation of head on nonoperative side would be the position for the late postoperative period. (B) Positioning on operative side puts pressure on the suturelines and on the shunt valve. Elevation of head in immediate postoperative period may cause rapid reduction of cerebrospinal fluid. (C) Placement on operative side puts pressure on the suture lines and shunt valve. (D) Flat position on nonoperative side in the immediate postoperative period prevents pressure on shunt valve and rapid reduction in cerebrospinal fluid. QUESTION 254 A type I diabetic client delivers a male newborn. The newborn is 45 minutes old. What is the primary nursing goal in the nursery during the first hours for this newborn? A. Bonding B. Maintain normal blood sugar C. Maintain normal nutrition D. Monitor intake and output Correct Answer: B Section: (none) Explanation Explanation/Reference: Explanation: (A) Bonding is necessary but would not be the priority with this newborn in the nursery. (B) The infant will be at risk for hypoglycemia because of excess insulin production. (C) Normal nutrition is a goal for all newborns. (D) Monitoring intake and output is necessary but is not the most critical nursing goal. QUESTION 255 A 16-month-old infant is being prepared for tetralogy of Fallot repair. In the nursing assessment, which lab value should elicit further assessment and requires notification of physician? A. pH 7.39 B. White blood cell (WBC) count 10,000 WBCs/mm3 C. Hematocrit 60% D. Bleeding time of 4 minutes Correct Answer: C Section: (none) Explanation Explanation/Reference: Explanation: (A) Normal pH of arterial blood gases for an infant is 7.357.45. (B) Normal white blood cell count in an infant is 6,00017,500 WBCs/mm3. (C) Normal hematocrit ininfant is 28%42%. A 60% hematocrit may indicate polycythemia, a common complication of cyanotic heart disease. (D) Normal bleeding time is 27 minutes. QUESTION 256 A 2-month-old infant is receiving IV fluids with a volume control set. The nurse uses this type of tubing because it: A. Prevents administration of other drugs B. Prevents entry of air into tubing C. Prevents inadvertent administration of a large amount of fluids D. Prevents phlebitis Correct Answer: C Section: (none) Explanation Explanation/Reference: Explanation: (A) A volume control set has a chamber that permits the administration of compatible drugs. (B) Air may enter a volume control set when tubing is not adequately purged. (C) A volume control set allows the nurse to control the amount of fluid administered over a set period. (D) Contamination of volume control set may cause phlebitis. QUESTION 257 Which type of insulin can be administered by a continuous IV drip? A. Humulin N B. NPH insulin C. Regular insulin D. Lente insulin Correct Answer: C Section: (none) Explanation Explanation/Reference: Explanation: (A) Humulin N cannot be administered IV. (B) NPH insulin cannot be administered IV. (C) Regular insulin is the only insulin that can be administered IV. (D) Lente insulin cannot be administered IV.QUESTION 258 A physician's order reads: Administer furosemide oral solution 0.5 mL stat. The furosemide bottle dosage is 10 mg/mL. What dosage of furosemide should the nurse give to this infant? A. 5 mg B. 0.5 mg C. 0.05 mg D. 20 mg Correct Answer: A Section: (none) Explanation Explanation/Reference: Explanation: (A) 1 mg = 0.1 mL, then 0.5 mL X= 55 mg. (B) Thisanswer is a miscalculation. (C) This answer is a miscalculation. (D) This answer is a miscalculation. QUESTION 259 A physician's order reads: Administer KCl 10% oral solution 1.5 mL. The KCl bottle reads 20 mEq/15 mL.What dosage should the nurse administer to the infant? A. 1 mEq B. 1.13 mEq C. 2 mEq D. Not enough information to calculate Correct Answer: C Section: (none) Explanation Explanation/Reference: Explanation: (A) This answer is a miscalculation. (B) This answer is a miscalculation. (C) 1.33 mEq = 1 mL, then 1.5 mL X=1.99, or 2 mEq. (D) Information is adequate for calculation. QUESTION 260 A 1000-mL dose of lactated Ringer's solution is to be infused in 8 hours. The drop factor for the tubing is 10 gtt/mL. How many drops per minute should the nurse administer?A. 125 gtt/min B. 48 gtt/min C. 20 gtt/min D. 21 gtt/min Correct Answer: D Section: (none) Explanation Explanation/Reference: Explanation: (A) This answer is a miscalculation. (B) This answer is a miscalculation. (C) This answer has not been rounded off to an even number. (D) 20.8, or 21 gtt/min. QUESTION 261 A 1000-mL dose of D5W 1/2 normal saline is to be infused in 8 hours. The drop factor for the tubing is 60 gtt/min. How many drops per minute should the nurse administer? A. 75 gtt/min B. 100 gtt/min C. 125 gtt/min D. 150 gtt/min Correct Answer: C Section: (none) Explanation Explanation/Reference: Explanation: (A) This answer is a miscalculation. (B) This answer is a miscalculation. (C)125 gtt/min. (D) This answer is a miscalculation. QUESTION 262 A physician's order reads: 0.25 normal saline at 50 mL/hr until discontinued. The nurse is using a microdrip tubing set. How many drops per minute should the nurse administer? A. 1 gtt/min B. 5 gtt/min C. 50 gtt/minD. 100 gtt/min Correct Answer: C Section: (none) Explanation Explanation/Reference: Explanation: (A) This answer is a miscalculation. (B) This answer is a miscalculation. (C)50 gtt/min. (D) This answer is a miscalculation. QUESTION 263 A 6-year-old child is attending a pediatric clinic for a routine examination. What should the nurse assess for while conducting a vision screening? A. Hearing test B. Gait C. Strabismus D. Papilledema Correct Answer: C Section: (none) Explanation Explanation/Reference: Explanation: (A) Hearing should be assessed separately. (B) Gait should be assessed separately. Client usually remains in one place for vision screening. Gait is part of neurological assessment. (C) Strabismus is crossing of eyes or outward deviation, which may cause diplopia or ambylopia. It is easily assessed during vision screening. (D) Papilledema is assessed by an ophthalmoscopic examination, which follows vision screening. It is part of neurological assessment. QUESTION 264 An 11-year-old boy hasreceived a partial-thickness burn to both legs. He presents to the emergency room approximately 15 minutes after the accident in excruciating pain with charred clothing to both legs. What is the first nursing action? A. Apply ice packs to both legs. B. Begin débridement by removing all charred clothing from wound. C. Apply Silvadene cream (silver sulfadiazine). D. Immerse both legs in cool water.Correct Answer: D Section: (none) Explanation Explanation/Reference: Explanation: (A) Ice creates a dramatic temperature change in the tissue, which can cause further thermal injury. (B) Charred clothing should not be removed from wound first. This creates further tissue damage. Débridement is not the first nursing action. (C) Applying silver sulfadiazine cream first insulates heat in injured tissue and increases potential for infection. (D) Emergency care of a thermal burn is immersing both legs in cool water. Cool water permits gradual temperature change andprevents further thermal damage. QUESTION 265 A burn victim's immunization history is assessed by the nurse. Which immunization is of priority concern? A. Oral poliovirus vaccine B. Inactivated poliovirus vaccine C. Tetanus toxoid D. Hepatitis B vaccine Correct Answer: C Section: (none) Explanation Explanation/Reference: Explanation: (A) Oral poliovirus vaccine is given to prevent polio. Polio is transmitted by direct contact with an infected person. (B) Inactivated poliovirus vaccine is given to adults and immunosuppressed individuals. Polio is transmitted by direct contact with an infected person. (C) Tetanus toxoid prevents tetanus. Tetanus is transmitted through contaminated wounds. (D) Hepatitis B vaccine prevents hepatitis B infection. Hepatitis B is transmitted through contact with infected blood or body fluids. QUESTION 266 A newborn has been delivered with a meningomyelocele. The nursery nurse should position the newborn: A. Prone B. Supine C. Side lying D. Semi-FowlerCorrect Answer: A Section: (none) Explanation Explanation/Reference: Explanation: (A) The prone position reduces pressure and tension on the sac. Primary nursing goals are to prevent trauma and infection of the sac. (B) The supine position exerts pressure on the sac. (C) Newborns usually cannot maintain side-lying position. (D) The semi- Fowler position exerts pressure on the sac. QUESTION 267 Nursing care of the infant prior to surgical closure of a meningomyelocele would include: A. Cover sac with dry sterile dressing B. Cover sac with saline-soaked sterile dressing C. Do not apply dressing; keep sac open to air D. Aspirate any fluid from sac Correct Answer: B Section: (none) Explanation Explanation/Reference: Explanation: (A) A dry, sterile dressing would adhere to the sac, causing tissue damage. (B) A saline-soaked sterile dressing protects the sac from contamination by air and prevents drying. (C) A sac open to air causes drying and potential for contamination. (D) This intervention is not an independent nursing action. QUESTION 268 A 35-year-old client is admitted to the hospital with diabetic ketoacidosis. Results of arterial blood gases are pH 7.2, PaO2 90, PaCO2 45, and HCO3 16. The nursing assessment of arterial blood gases indicate the presence of: A. Respiratory alkalosis B. Respiratory acidosis C. Metabolic alkalosis D. Metabolic acidosis Correct Answer: D Section: (none) ExplanationExplanation/Reference: Explanation: (A) Respiratory alkalosis is determined by elevated pH and low PaCO2. (B) Respiratory acidosis is determined by low pH and elevated PaCO2. (C) Metabolic alkalosis is determined by elevated pH and HCO3.(D) Metabolic acidosis is determined by low pH and HCO3. Topic 4, Questions Set D QUESTION 269 A client presents to the emergency room with cyanosis, coughing, tachypnea, and tachycardia. She has a history of asthma. Arterial blood gas values are pH 7.28, PaO2 54, PaCO2 60, and HCO3 24. The nursing assessment of arterial blood gases indicate the presence of: A. Respiratory alkalosis B. Respiratory acidosis C. Metabolic alkalosis D. Metabolic acidosis Correct Answer: B Section: (none) Explanation Explanation/Reference: Explanation: (A) Respiratory alkalosis is determined by elevated pH and low PaCO2. (B) Respiratory acidosis is determined by low pH and elevated PaCO2. (C) Metabolic alkalosis is determined by elevated pH and HCO3. (D) Metabolic acidosis is determined by low pH and HCO3. QUESTION 270 A 40-year-old client is admitted to the coronary care unit with chest pain and shortness of breath. The physician diagnosed an anterior wall myocardial infarction. What tests should the nurse anticipate? A. Reticulocyte count, creatinine phosphokinase (CPK) B. Aspartate transaminase, alanine transaminase C. Sedimentation rate, WBC count D. Lactic dehydrogenase, CPK Correct Answer: D Section: (none) ExplanationExplanation/Reference: Explanation: (A) Reticulocyte count measures the number of immature erythrocytes. CPK is an enzyme released from injured myocardial tissue. (B) Aspartate transaminase is an enzyme released from injured myocardial tissue. Alanine transaminase is an enzyme released for general tissue destruction, which is specific for liver injury. (C) Sedimentation rate is a nonspecific test for inflammation. (D) Lactic dehydrogenase and CPK are enzymes released from injured myocardial tissue. QUESTION 271 The nurse needs to be aware that the most common early complication of a myocardial infarctionis: A. Diabetes mellitus B. Anaphylactic shock C. Cardiac hypertrophy D. Cardiac dysrhythmia Correct Answer: D Section: (none) Explanation Explanation/Reference: Explanation: (A) Diabetes mellitus is not a common complication of myocardial infarction. (B) Anaphylactic shock is an allergic reaction. (C) Cardiac hypertrophy is a late potential complication. It is a common complication of congestive heart failure. (D) Myocardial infarction causes tissue damage, which may interrupt electrical impulses. Myocardial irritability results from lack of oxygenated tissue. QUESTION 272 A client is being treated for congestive heart failure. His medical regimen consists of digoxin (Lanoxin) 0.25 mg po daily and furosemide 20 mg po bid. Which laboratory test should the nurse monitor? A. Intake and output B. Calcium C. Potassium D. Magnesium Correct Answer: C Section: (none) ExplanationExplanation/Reference: Explanation: (A) Intake and output are not laboratory tests. (B) Serum calcium levels are not affected by digoxin or furosemide. (C) Furosemide is a nonpotassium-sparing loop diuretic. Hypokalemia is a common side effect of furosemide and may enhance digoxin toxicity. (D) Serum magnesium levels are not affected by digoxin or furosemide. QUESTION 273 In the coronary care unit, a client has developed multifocal premature ventricular contractions.The nurse should anticipate the administration of: A. Furosemide B. Nitroglycerin C. Lidocaine D. Digoxin Correct Answer: C Section: (none) Explanation Explanation/Reference: Explanation: (A) Furosemide is a loop diuretic. (B) Nitroglycerin is a vasodilator. (C) Lidocaine is the drug of choice to treat ectopic ventricular beats. (D) Digoxin slows down the electrical impulses and increases ventricular contractions, but it does not rapidly correct ventricular ectopy. QUESTION 274 A client has received digoxin 0.25 mg po daily for 2 weeks. Which of the following digoxin levels indicates toxicity? A. 0.5 ng/mL B. 1.0 ng/mL C. 2.0 ng/mL D. 3.0 ng/mL Correct Answer: D Section: (none) Explanation Explanation/Reference:Explanation: (A) 0.5 ng/mL of digoxin is a subtherapeutic level, not a toxic one. (B) 1.0 ng/mL is a therapeutic level. (C) 2.0 ng/mL is a therapeutic level. (D) Digoxin's therapeutic level is 0.82.0 ng/mL. Digoxin's toxic level is >2.0 ng/mL. QUESTION 275 A client has developed congestive heart failure secondary to his myocardial infarction. Discharge diet instructions should emphasize the reduction or avoidance of: A. Fresh vegetables and fruit B. Canned vegetables and fruit C. Breads, cereals, and rice D. Fish Correct Answer: B Section: (none) Explanation Explanation/Reference: Explanation: (A) Fresh vegetables and fruits are excellent sources of essential vitamins. (B) Canned and frozen foods have a high sodium content. Labels of all canned foods should be read to determine if sodium is used in any form. (C) Bread, cereal, and rice are excellent sources of carbohydrates. (D) Fish is an excellent source of protein. QUESTION 276 A client takes warfarin (Coumadin) 15 mg po daily. To evaluate the medication's effectiveness, the nurse should monitor the: A. prothrombin time (PT) B. partial thromboplastin time (PTT) C. PTT-C D. Fibrin split products Correct Answer: A Section: (none) Explanation Explanation/Reference: Explanation: (A) PT evaluates adequacy of extrinsic clotting pathway. Adequacy of warfarin therapy is monitored by PT. (B) PTT evaluates adequacy of intrinsic clottingpathway. Adequacy of heparin therapy is monitored by PTT. (C) There is no such laboratory test. (D) Fibrin split products indicate fibrinolysis. This is a screening test for disseminated intravascular coagulation. Heparin therapymay increase fibrin split products. QUESTION 277 Prior to administering digoxin to a client with congestive heart failure, the nurse needs to assess: A. Respiratory rate for 1 minute B. Radial pulse for 1 minute C. Radial pulse for 2 minutes D. Apical pulse for 1 minute Correct Answer: D Section: (none) Explanation Explanation/Reference: Explanation: (A) Respiratory rate is not directly affected by digoxin therapy. (B) A radial pulse is not as accurate as an apical pulse. Dysrhythmias may not be detected. (C) A radial pulse is not as accurate as an apical pulse, regardless of assessment time. (D) Apical pulse should be measured for 1-minute prior to digoxin administration. Digoxin decreases the heart rate. Digoxin should be withheld if apical rates are <60 bpm or >120 bpm. QUESTION 278 A client is diagnosed with Mycobacterium tuberculosis. He is placed in respiratory isolation, intubated, and receives mechanical ventilation. When performing suctioning, the nurse should: A. Suction for a maximum of 20 seconds B. Hyperoxygenate before and after suctioning C. Suction for a maximum of 30 seconds D. Maintain clean technique during suctioning Correct Answer: B Section: (none) Explanation Explanation/Reference: Explanation: (A) The maximum time for suctioning is 1015 seconds. (B) Supplemental O2should be administered before and after suctioning to reduce hypoxia. (C) Themaximum time for suctioning is 1015 seconds. (D) Strict sterile technique should be used during suctioning. QUESTION 279 The physician prescribes a medical regimen of isoniazid, rifampin, and vitamin B6 for a tuberculosis client. The nurse instructs the client that B6 is given because it: A. Increases activity of isoniazid B. Increases activity of rifampin C. Improves nutritional status D. Reduces peripheral neuropathy Correct Answer: D Section: (none) Explanation Explanation/Reference: Explanation: (A) Vitamin B6does not enhance the activity of isoniazid. (B) Vitamin B6does not enhance the activity of rifampin. (C) A vitamin alone does not improve nutritional status. (D) Isoniazid leads to Vitamin B6deficiency, which is manifested as peripheral neuropathy. QUESTION 280 Which of the following nursing actions is essential to prevent drug-resistant tuberculosis? A. Monitor liver function. B. Monitor renal function. C. Assess knowledge of respiratory isolation. D. Monitor compliance with drug therapy. Correct Answer: D Section: (none) Explanation Explanation/Reference: Explanation: (A) Monitoring liver function will not prevent the development of drug-resistant organisms. (B) Monitoring renal function will not prevent the development of drug- resistant organisms. (C) Knowledge of respiratory isolation will reduce transmission of tuberculosis but will not prevent development of drug-resistant organisms. (D) Noncompliance with prescribed antituberculosis drug regimen is the primary cause of drug-resistant organisms. Noncompliance permits the mutation of organisms.QUESTION 281 To facilitate maximum air exchange, the nurse should position the client in: A. High Fowler B. Orthopneic C. Prone D. Flat-supine Correct Answer: B Section: (none) Explanation Explanation/Reference: Explanation: (A) The high Fowler position does increase air exchange, but not to the extent of orthopneic position. (B) The orthopneic position is a sitting position that allows maximum lung expansion. (C) The prone position places pressure on diaphragm and does not promote maximum air exchange. (D) The flat-supine position places pressure on diaphragm by abdominal organs and does not promote maximum air exchange. QUESTION 282 A client has been diagnosed with congestive heart failure. His fluid intake and output are strictly regulated. For lunch, he drank 8 oz of milk, 4 oz of tea, and 6 oz of coffee. His intake would be recorded as: A. 500 mL B. 540 mL C. 600 mL D. 655 mL Correct Answer: B Section: (none) Explanation Explanation/Reference: Explanation: (A, C, D) This answer is a miscalculation. (B) 1 oz = 30 mL; therefore, 18 oz x. QUESTION 283 The client tells the nurse, "I have pain in my left shoulder." This is considered:A. Evaluation process B. Objective information C. Subjective information D. Complaining Correct Answer: C Section: (none) Explanation Explanation/Reference: Explanation: (A) Evaluation process follows a nursing intervention. (B) Objective information can be measured. (C) Subjective information is provided by a person. (D) Client is reporting a symptom that needs to be assessed. QUESTION 284 Before completing a nursing diagnosis, the nurse must first: A. Write goals and objectives B. Perform an assessment C. Plan interventions D. Perform evaluation Correct Answer: B Section: (none) Explanation Explanation/Reference: Explanation: (A) Goals and objectives are based on a nursing assessment and diagnosis. (B) Assessment is the first step of nursing process. (C) Interventions are nursing actions to meet goals and objectives. (D) Evaluation process follows nursing interventions. QUESTION 285 A 70-year-old homeless woman is admitted with pneumonia. She is weak, emaciated, and febrile. The physician orders enteral feedings intermittently by nasogastric tube. When inserting the nasogastric tube, once the tube passes through the oropharynx, the nurse will instruct the client to: A. Tilt her head backwards B. Swallow as tube passesC. Hold breath as tube passes D. Cough as tube passes Correct Answer: B Section: (none) Explanation Explanation/Reference: Explanation: (A) Head should be tilted slightly forward to facilitate insertion. (B) Swallowing assists with insertion of tube and closes off airway. (C) Client should be swallowing as tube passes; holding the breath facilitates nothing. (D) Coughing may expel tube. QUESTION 286 When assessing residual volume in tube feeding, the feeding should be delayed if the amount of gastric contents (residual) exceeds: A. 20 mL B. 25 mL C. 30 mL D. 50 mL Correct Answer: D Section: (none) Explanation Explanation/Reference: Explanation: (A) A residual volume of 20 mL is not excessive. (B) A residual volume of 25 mL is not excessive. (C) A residual volume of 30 mL is not excessive. (D) Tube feedings should be withheld and physician notified for residual volumes of 50100 mL. QUESTION 287 A client has a history of alcoholism. He is currently diagnosed with cirrhosis of the liver. The nurse would expect him to be on which type of diet? A. High protein and high calorie B. High calorie and high carbohydrate C. Low-fat 2-g sodium diet D. High protein and high fatCorrect Answer: B Section: (none) Explanation Explanation/Reference: Explanation: (A) A high-protein diet is contraindicated in hepatic disease. (B) High carbohydrates provide high-caloric content to prevent tissue catabolism. (C) A low-fat 2-g sodium diet is a cardiac diet; however, a low-fat diet would be beneficial. (D) A high-protein and high-fat diet is contraindicated in hepatic disease. QUESTION 288 A client has ascites, which is caused by: A. Decreased plasma proteins B. Electrolyte imbalance C. Decreased renal function D. Portal hypertension Correct Answer: A Section: (none) Explanation Explanation/Reference: Explanation: (A) A decrease in plasma proteins causes a decrease in intravascular osmotic pressure resulting in leakage of fluid into peritoneal cavity. (B) Fluid and electrolyte imbalance may occur as a result of the ascites. (C) Ascites is a result of hepatic malfunction, not renal malfunction. (D) Portal hypertension causes esophageal varices, not ascites. QUESTION 289 A common complication of cirrhosis of the liver is prolonged bleeding. The nurse should be prepared to administer? A. Vitamin C B. Vitamin K C. Vitamin E D. Vitamin A Correct Answer: B Section: (none) ExplanationExplanation/Reference: Explanation: (A) Vitamin C does not directly affect clotting. (B) Vitamin K is a fat-soluble vitamin that depends on liver function for absorption. Vitamin K is essential for clotting. (C) Vitamin E does not directly affect clotting. (D) Vitamin A does not directly affect clotting. QUESTION 290 A 45-year-old client has a permanent colostomy. Which of the following foods should he avoid? A. Peanut butter and jelly sandwich and milk B. Corn beef and cabbage and boiled potatoes C. Oatmeal, whole-wheat toast, and milk D. Tuna on whole-wheat bread and iced tea Correct Answer: B Section: (none) Explanation Explanation/Reference: Explanation: (A, C, D) These foods are allowed with a colostomy. (B) Gasforming foods such as cabbage should be avoided. QUESTION 291 On an assessment of a client's mouth, the nurse notices white patches on the buccal mucosa. The nurse tries to obtain a sample for a culture, but the lesion cannot be rubbed off. The nurse would suspect that this lesion is: A. Xerosteromia B. Candidiasis C. Leukoplakia D. Stomatitis Correct Answer: C Section: (none) Explanation Explanation/Reference: Explanation: (A) Xerostomia is dry mouth. (B) Candidiasis can be rubbed off, but it will bleed. (C) Leukoplakia cannot be rubbed off. (D) Stomatitis is caused by candidiasis andgram-negative bacteria. QUESTION 292 A client on the infectious disease unit is discussing transmission of human immunodeficiency virus (HIV).The nurse would need to provide more client education based on which client statement? A. "HIV is a virus transmitted by sexual contact." B. "Condoms reduce the transmission of HIV." C. "HIV is a virus that is easily transmitted by casual contact." D. "HIV can be transmitted to an unborn infant." Correct Answer: C Section: (none) Explanation Explanation/Reference: Explanation: (A) HIV is transmitted through unprotected sexual contact. (B) Condoms are an effective barrier to prevent HIV transmission. (C) HIV is not easily transmitted by casual contact. (D) HIV can be transmitted intrauterinely at the time of delivery, and by breast-feeding. QUESTION 293 A 26-year-old client is diagnosed with an astrocytoma, a benign brain tumor. From the nurse's knowledge of the central nervous system, the nurse knows that benign tumors: A. Can be just as dangerous as malignant tumors B. Grow more rapidly than malignant tumors C. Do not warrant concern because they do not become malignant tumors D. Can be removed surgically Correct Answer: A Section: (none) Explanation Explanation/Reference: Explanation: (A) Both a benign and a malignant tumor can displace or destroy nearby structures or increase intracranial pressure. (B) Benign or malignant brain tumors grow at different rates depending on the type of tumor. (C) Some benign tumors do become malignant tumors. (D) Whether or not a tumor is operable depends on its location and the amount of damage its removal will cause.QUESTION 294 A 55-year-old client is admitted with a diagnosis of renal calculi. He presented with severe right flank pain, nausea, and vomiting. The most important nursing action for him at this time is: A. Intake and output measurement B. Daily weights C. Straining of all urine D. Administration of O2 therapy Correct Answer: C Section: (none) Explanation Explanation/Reference: Explanation: (A) Intake and output measurements are important but must be accompanied by straining urine. (B) Daily weights would not provide for identification of calculi. (C) Straining urine provides for assessment of calculi and evaluation of calculi descent through ureters and urethra. (D) O2therapy should not be necessary for renal calculi. QUESTION 295 A client'srenal calculi are identified as consisting of calcium phosphate. Which of the following diets would be appropriate? A. High calcium, low phosphorus B. Low calcium, high phosphorus C. Two-gram sodium diet D. Low calcium and phosphorus, acid ash Correct Answer: D Section: (none) Explanation Explanation/Reference: Explanation: (A) The stones consist of calcium and phosphorus; therefore, these minerals should be avoided. A high-calcium diet is contraindicated. (B) A high-phosphorus diet is contraindicated. (C) A 2-g sodium diet is a cardiac diet. (D) A low-calcium and phosphorus diet will reduce further calculi formation. QUESTION 296A client is admitted to the hospital with diabetic ketoacidosis. The emergency room nurse should anticipate the administration of: A. Humulin N B. Humulin R C. Humulin U D. Humulin L Correct Answer: B Section: (none) Explanation Explanation/Reference: Explanation: (A) Intermediate-acting insulin is not indicated in an emergency. (B) Regular insulin is rapid acting and indicated in an emergency situation. (C) Long-acting insulin is not indicated in an emergency situation. (D) Intermediate-acting insulin is not indicated in an emergency situation. QUESTION 297 A client is diagnosed with diabetic ketoacidosis. The nurse should be prepared to administer which of the following IV solutions? A. D5in normal saline B. D5W C. 0.9 normal saline D. D5in lactated Ringer's Correct Answer: C Section: (none) Explanation Explanation/Reference: Explanation: (A) D5in normal saline would increase serum glucose. (B) D5W would increase serum glucose. (C) A concentration of 0.9 NS is used to correct extracellular fluid depletion. (D) D5in Ringer's lactate would increase serum glucose. QUESTION 298 The nurse is caring for a client who has diabetes insipidus. The nurse would describe this client's urineoutput pattern as:A. Anuria B. Oliguria C. Dysuria D. Polyuria Correct Answer: D Section: (none) Explanation Explanation/Reference: Explanation: Explanation (A)Anuriais defined as absence of urine output, which is not indicative of the urinary pattern of diabetes insipidus. (B)Oliguriais defined as <500 mL of urine per day, which is not a urinary output pattern associated with diabetes insipidus. (C)Dysuriais defined as difficult urination. Clients with diabetes insipidus do not have dysuria as a symptom of their disease. (D) Polyuria is a primary symptom of diabetes insipidus. These clients have decreased or absent vasopressin secretion, which causes water loss in the urine and sodium increases. QUESTION 299 A male client was involved in a motor vehicle accident earlier in the day. The nurse caring for him on evenings notices that on admission to the hospital, he lost a lot of blood and required multiple blood transfusions. The nurse would anticipate which blood product would be ordered when a large blood loss has occurred? A. Whole blood B. Platelets C. Fresh frozen plasma D. Packed red blood cells Correct Answer: A Section: (none) Explanation Explanation/Reference: Explanation: (A) Whole blood is the transfusion component of choice when large volumes of blood need to be replaced. Whole blood contains all blood components that are lost during active bleeding. (B) Platelet therapy is indicated for thrombocytopenia if the client's platelet count is below 15,000/mm3. (C) Infusion of fresh frozen plasma is required when the prothrombin time and partial thromboplastic time are prolonged. (D) Packed red blood cells are transfused in instances of anemia with decreases in hematocrit and hemoglobin. QUESTION 300An expected response to sodium polystyrene sulfonate (Kayexalate) is: A. Increase in serum magnesium B. Increase in serum HCO3 C. Decrease in serum potassium D. Decrease in serum calcium Correct Answer: C Section: (none) Explanation Explanation/Reference: Explanation: (A) Sodium polystyrene sulfonate administration will not increase serum magnesium. Hypermagnesemia is virtually unknown except for clients in renal failure. (B) Sodium polystyrene sulfonate administration is not known to increase serum bicarbonate. (C) Decrease in serum potassium, the expected response of sodium polystyrene sulfonate, is secondary to the binding of this drug and potassium in the colon, and potassium is removed through the feces. (D) Serum calcium may actually increase with sodium polystyrene sulfonate administration, especially if calcium chloride is administered concurrently with this drug. QUESTION 301 Cheyne-Stokes respiratory pattern can be associated with which of the following conditions? A. Diabetic ketoacidosis B. Fever C. Increased intracranial pressure D. Spinal meningitis Correct Answer: C Section: (none) Explanation Explanation/Reference: Explanation: Explanation (A) Kussmaul'srespirations are associated with diabetic ketoacidosis, severe hemorrhage, peritonitis, renal failure, and uremia. (B) Tachypnea (respiratory rate >25 breaths/min) is often associated with fever. (C) Cheyne-Stokes respiratory pattern is most often associated with increased intracranial pressure secondary to changes in pressure in the cerebral and cerebellar areas. (D) Biot's breathing is most frequently associated with spinal meningitis. QUESTION 302When providing dietary teaching to an individual who has diabetes mellitus, type II, the nurse discusses the importance of consuming the recommended daily allowance of which of the following electrolytes? A. Potassium B. Magnesium C. Sodium D. HCO3 Correct Answer: B Section: (none) Explanation Explanation/Reference: Explanation: (A) Potassium intake that meets the recommended daily allowance is important, especially in clients who have a history of cardiac disease. (B) Low levels of magnesium can cause an increase in resistance to insulin and can lead to carbohydrate intolerance. (C) Sodium is an important electrolyte for all clients but has no direct effect on diabetes mellitus. (D) Bicarbonate plays an important role in acid-base balance. It is equally necessary for maintenance of all body functions. QUESTION 303 A physician tells the nurse that he wants to orally intubate a client with a No. 8 endotracheal tube. The finding of normal breath sounds on the right side of the chest and diminished, distant breath sounds on the left side of the chest of a newly intubated client is probably due to: A. A left hemothorax B. A right hemothorax C. Intubation of the right mainstem bronchus D. An inadequate mechanical ventilator Correct Answer: C Section: (none) Explanation Explanation/Reference: Explanation: (A) Although a left hemothorax could cause diminished and distant breath sounds, it is irrelevant to this situation. (B) A right hemothorax will not cause diminished and distant breath sounds on the left side of the chest. (C) The right mainstem bronchus is most frequently intubated in error because the angle of the right mainstem bronchus is very small as compared with that of the left mainstem bronchus. Because ventilation is only occurring on the right side, the nurse would auscultate diminished and distant breath sounds on the left. (D) An inadequate mechanical ventilator has no relationship to this situation.QUESTION 304 Which of the following blood gas parameters primarily reflects respiratory function? A. PCO2 B. CO2 content of the blood C. HCO3 D. Base excess Correct Answer: A Section: (none) Explanation Explanation/Reference: Explanation: (A) The lungs are responsible for regulation of CO2, and this parameter primarily reflects respiratory function. (B) CO2 content of the blood is an indirect measureof respiratory function. (C) HCO3 is a measure of kidney function only and is important in acid-base balance. (D) Base excess represents the excess of HCO3 and is not reflective of respiratory function. QUESTION 305 Endotracheal tube cuff pressure should never exceed: A. 10 mm Hg B. 20 mm Hg C. 45 mm Hg D. 60 mm Hg Correct Answer: B Section: (none) Explanation Explanation/Reference: Explanation: (A) Pressure<10 mm Hg places the client at risk for aspiration. (B) Pressure in the endotracheal tube cuff should never exceed 20 mm Hg, because higher pressure places the client at risk for tracheal erosion. (C) A pressure of 45 mm Hg is an extremely high pressure in the endotracheal tube cuff. This places the client at great risk for tracheal erosion. (D) A pressure of 60 mm Hg is an extremely high pressure in the endotracheal tube cuff. This places the client at great risk for tracheal erosion. QUESTION 306 The physician prescribes phenytoin (Dilantin) for a client with seizure disorders. Phenytoin can only be mixed with which of the following solutions?A. Ringer's lactate B. D5 in water C. D5 with Ringer's lactate D. Normal saline Correct Answer: D Section: (none) Explanation Explanation/Reference: Explanation: (A) Phenytoin will precipitate if mixed with Ringer's lactate and should not be administered. (B, C) Phenytoin will precipitate if mixed with D5 in Ringer's lactate and should not be administered. (D) Phenytoin is compatible only with normal saline and should be mixed only with normal saline for administration. QUESTION 307 A client sustained second- and third-degree burns to his face, neck, and upper chest. Which of the following nursing diagnoses would be given the highest priority in the first 8 hours' postburn? A. Fluid volume deficit secondary to alteration in skin integrity B. Alteration in comfort secondary to alteration in skin integrity C. Alteration in sensation secondary to third-degree burn D. Alteration in airway integrity secondary to edema of neck and face, which in turn is secondary to alteration in skin integrity Correct Answer: D Section: (none) Explanation Explanation/Reference: Explanation: (A) Fluid deficit is a high priority not only during the first 8 hours postburn, but also during the first 36 hours postburn. (B) Alteration in comfort is a high priority during the entire length of the client's hospitalization and on discharge. (C) Alteration in sensation is a high priority during the first 4872 hours postburn. Lack of sensation may be indicative of lack of circulation. (D) Alteration in airway integrity is the highest priority for this client in the first 8 hours postburn. Failure to continually assess this client's airway status could result in poor ventilation and oxygenation, in addition to an inability to intubate the client secondary to excessive edema formation in the neck. QUESTION 308 A post-lung surgery client is placed on a chest tube drainage system. When explaining to the family how the system works, the nurse states that the water-sealbottle of a three-bottle chest drainage system serves which of the following purposes? A. Collection bottle for drainage B. Pressure regulator C. Preventing accumulation of blood around the heart D. Preventing air from entering the chest upon inspiration Correct Answer: D Section: (none) Explanation Explanation/Reference: Explanation: (A) There is a separate collection bottle for drainage as part of a chest drainage system. (B) In a three-bottle chest drainage system, one bottle serves only as a pressure regulator. (C) Mediastinal chest tubes prevent accumulation of blood around the heart immediately following heart surgery. (D) The purpose of the water- seal bottle in any chest drainage setup is to allow air out of the chest, but not back in. This negative pressure promotes lung expansion. QUESTION 309 Which of the following serum laboratory values would the nurse monitor during gentamicin therapy? A. Creatinine B. Sodium C. Calcium D. Potassium Correct Answer: A Section: (none) Explanation Explanation/Reference: Explanation: (A) A common side effect of gentamicin is nephrotoxicity. The serum laboratory test that best reflects kidney function is serum creatinine. (B) Serum sodium has no relationship to gentamicin. (C) Serum calcium has no relationship to gentamicin. (D) Serum potassium has no relationship to gentamicin. If a client has impaired renal function secondary to gentamicin administration, he or she may also have hyperkalemia as a secondary disorder. QUESTION 310 While changing the dressing on a client's central line, the nurse notices redness and warmth at the needle insertion site. Which of the following actions would be appropriate to implement based on this finding?A. Discontinue the central line. B. Begin a peripheral IV. C. Document in the nurse's notes and notify the physician after redressing the site. D. Clean the site well and redress. Correct Answer: C Section: (none) Explanation Explanation/Reference: Explanation: (A) The nurse may never discontinue a central line without a physician's order. (B) The nurse may never initiate a peripheral IV without a physician's order except in an emergency situation. (C) The nurse should always document findings and alert the physician to the findings as well. The physician may then initiate a new central line and order the current central line to be discontinued. (D) Besides cleaning and redressing, the nurse should always document the findings. QUESTION 311 The nurse is caring for a client who has had a tracheostomy for 7 years. The client is started on a fullstrength tube feeding at 75 mL/hr. Prior to starting the tube feeding, the nurse confirms placement of the tube in the stomach. The hospital policy states that all tube feeding must be dyed blue. On suctioning, the nurse notices the sputum to be a blue color. This is indicative of which of the following? A. The client aspirated tube feeding. B. The nurse has placed the suction catheter in the esophagus. C. This is a normal finding. D. The feeding is infusing into the trachea. Correct Answer: A Section: (none) Explanation Explanation/Reference: Explanation: (A) Once the feeding tube placement is confirmed in the stomach, aspiration can occur if the client's stomach becomes too full. When suctioning the trachea, if secretions resemble tube feeding, the client has aspirated the feeding. (B) Because the trachea provides direct access to a client's airway, it would not be possible to place the catheter in the esophagus. (C) Blue-colored sputum is never considered a normal finding and should be reported and documented. (D) The nurse confirmed placement of the feeding tube in the stomach prior to initiating the tube feeding; therefore, it is highly unlikely that the feeding tube would be located in the trachea.QUESTION 312 The nurse is caring for a client with pancreatitis. Which of the following IV medications would the nurse expect the physician to prescribe for control of pain in this client? A. Morphine sulfate B. Kerolac tromethamine (Toradol) C. Promethazine (Phenergan) D. Meperidine (Demerol) Correct Answer: D Section: (none) Explanation Explanation/Reference: Explanation: (A) Morphine sulfate is contraindicated in clients with pancreatitis because it may cause spasms of the sphincter of Oddi and increase pancreatic pain. (B) Ketorolac tromethamine is currently not approved by the Food and Drug Administration for IV use. (C) Promethazine is a medication that has no analgesic properties. (D) Meperidine is the drug of choice for clients with pancreatitis. It will not cause spasms at the sphincter of Oddi, which can lead to increasedpancreatic pain. QUESTION 313 The nurse begins morning assessment on a male client and notices that she is unable to palpate either of his dorsalis pedis pulses in his feet. What is the first nursing action after assessing this finding? A. Palpate these pulses again in 15 minutes. B. Use a Doppler to determine presence and strength of these pulses. C. Document the finding that the pulses are not palpable. D. Call the physician and notify the physician of this finding. Correct Answer: B Section: (none) Explanation Explanation/Reference: Explanation: (A) Palpating these pulses again in 15 minutes may only result in the same findings. (B) Any time during an assessment that the nurse is unable to palpate pulses, the nurse should then obtain a Doppler and assess for presence or absence of the pulse and pulse strength, if a pulse is present. (C) Pulses may be present andassessed through use of a Doppler. Absence of palpable pulses does not indicate absence of blood flow unless pulses cannot be located with a Doppler. (D) Thenurse would only call the physician after determining that the pulses are absent by both palpation and Doppler. QUESTION 314 The physician has prescribed metoclopramide (Reglan). When assessing the client, the nurse would expect to find which of the following responses? A. Increase in gastric secretions B. Increase in peristalsis C. Disorientation D. Drowsiness Correct Answer: B Section: (none) Explanation Explanation/Reference: Explanation: (A) Metoclopramide does not stimulate gastric secretions. (B) This response is expected with metoclopramide, in addition to increasing gastric emptying. (C) Disorientation is a symptom of metoclopramide overdose. The drug should be discontinued. (D) Drowsiness is a symptom of metoclopramide overdose and the drug should be discontinued. QUESTION 315 A 33-year-old client was brought into the emergency room unconscious, and it is determined that surgery is needed. Informed consent must be obtained from his next of kin. The sequence in which the next of kin would be asked for the consent would be: A. Parent, spouse, adult child, sibling B. Spouse, adult child, parent, sibling C. Spouse, parent, sibling, adult child D. Parent, spouse, sibling, adult child Correct Answer: B Section: (none) Explanation Explanation/Reference: Explanation: (A) Spouse and adult child would be asked before a parent. (B) The order of kin relationship for an adult, as determined from legal intestate succession, is usuallyspouse, adult child, parent, sibling. (C) Parent and sibling would be asked after adult child. (D) Spouse and adult child would be asked before parent. Sibling would be asked last.QUESTION 316 A client had abdominal surgery this morning. The nurse notices that there is a small amount of bloody drainage on his surgical dressing. The nurse would document this as what type of drainage? A. Serosanguinous B. Purulent C. Sanguinous D. Catarrhal Correct Answer: C Section: (none) Explanation Explanation/Reference: Explanation: (A) Drainage from a surgical incision usually proceeds from sanguinous to serosanguinous. (B) Purulent drainage usually indicates infection and should not be seen initially from a surgical incision. (C) Drainage from a surgical incision is initially sanguinous, proceeding to serosanguinous, and then to serous. (D) Catarrhal is a type of exudate seen in upper respiratory infections, not in surgical incisions. QUESTION 317 A client had a hemicolectomy performed 2 days ago. Today, when the nurse assesses the incision, a small part of the abdominal viscera is seen protruding through the incision. This complication of wound healing is known as: A. Excoriation B. Dehiscence C. Decortication D. Evisceration Correct Answer: D Section: (none) Explanation Explanation/Reference: Explanation: (A) Excoriation is abrasion of the epidermis or of the coating of any organ of the body by trauma, chemicals, burns, or other causes. (B) Dehiscence is a partial or complete separation of the wound edges with no protrusion of abdominal tissue. (C) Decortication is removal of the surface layer of an organ or structure. It is a type of surgery, such as removing the fibrinous peel from the visceral pleura in thoracic surgery. (D) Evisceration occurs when the incision separates and thecontents of the cavity spill out. QUESTION 318 The nurse documents a client's surgical incision as having red granulated tissue. This indicates that the wound is: A. Infected B. Not healing C. Necrotic D. Healing Correct Answer: D Section: (none) Explanation Explanation/Reference: Explanation: (A) The wound is not infected. An infected wound would contain pus, debris, and exudate. (B) The wound is healing properly. (C) A necrotic wound would appear black or brown. (D) The wound is healing properly and is filled with red granulated tissue and fragile capillaries. QUESTION 319 A client has returned to the unit following a left femoral popliteal bypass graft. Six hours later, his dorsalis pedis pulse cannot be palpated, and his foot is cool and dusky. The nurse should: A. Continue to monitor the foot B. Notify the physician immediately C. Reposition and reassess the foot D. Assure the client that his foot is fine Correct Answer: B Section: (none) Explanation Explanation/Reference: Explanation: (A) The client is losing blood supply to his left foot. Continuing to monitor the foot will not help restore the blood supply to the foot. (B) The physician should be notified immediately because the client is losing blood supply to his left foot and is in danger of losing the foot and/or leg. (C) The presenting symptoms are of an emergency nature and require immediate intervention. (D) This action would be giving the client false assurance. QUESTION 320A client is to have a coronary artery bypass graft performed in the morning using a saphenous vein. He wants to know why the physician does not use the internal mammary artery for his bypass graft because his friend's physician uses this artery. The nurse tells the client that the internal mammary artery: A. Takes more time to remove B. Has a greater risk of becoming reoccluded C. Is smaller in diameter D. Has too many valves Correct Answer: A Section: (none) Explanation Explanation/Reference: Explanation: (A) It does take more time to remove the internal mammary artery, and this is one reason why some physicians do not use it. (B) There is not a greater risk ofreocclusion. In fact, it may actually stay patent longer. (C) The internal mammary artery is actually larger in diameter than the saphenous vein. (D) The internal mammary artery does not have too many valves. QUESTION 321 A client returns to the cardiovascular intensive care unit following his coronary artery bypass graft. In planning his care, the most important electrolyte the nurse needs to monitor will be: A. Chloride B. HCO3 C. Potassium D. Sodium Correct Answer: C Section: (none) Explanation Explanation/Reference: Explanation: (A) Chloride, HCO3, and sodium will need to be monitored, but monitoring these electrolytes is not as important as potassium monitoring. (B) Chloride, HCO3, and sodium will need to be monitored, but monitoring these electrolytes is not as important as potassium monitoring. (C) Potassium will need to be closely monitored because of its effects on the heart. Hypokalemia could result in supraventricular tachyarrhythmias. (D) Chloride, HCO3, and sodium will need to be monitored, butmonitoring these electrolytes is not as important as potassium monitoring.QUESTION 322 A client is being discharged from the hospital today. The discharge teaching for care of her colostomy included which of the following basic principles for protecting the skin around her stoma: A. Taping a pouch that is leaking B. Cutting the skin barrier 11/2 inches larger than the stoma C. Changing the pouch only when leakage occurs D. Using a skin sealant under pouch adhesives Correct Answer: D Section: (none) Explanation Explanation/Reference: Explanation: (A) When a pouch seal leaks, the pouch should be immediately changed, not taped. Stool held against the skin can quickly result in severe irritation. (B) The skin barrier should be cut only slightly larger than the stoma (one-half inch). (C) The client should be taught to change pouches whenever possible before leakage occurs. (D) When skin sealant is used under the tape, the outermost layer of the epidermis remains intact. When no skin sealant is used, this layer is removed when the tape is removed. QUESTION 323 A client is being discharged from the hospital tomorrow following a colon resection with a left colostomy. The nurse knows that the client understands the discharge teaching about care of her colostomy when she says: A. "I know that I am not supposed to irrigate my colostomy." B. "My stool will be soft like paste." C. "My stoma should be red and slightly raised." D. "The skin around my stoma may become irritated from the enzymes in my stool." Correct Answer: C Section: (none) Explanation Explanation/Reference: Explanation: (A) A left colostomy indicates an ascending colon resection. This type of colostomy can be irrigated. (B) The stool from an ascending colon resection should beformed. (C) The healthy stoma should be red and slightly raised. If it begins to turn dark or blue, the client should see the physician immediately. (D) The stool in the ascending colon does not usually have many enzymes in it. Stool from an ileostomy has more enzymes and is more irritating to the skin.QUESTION 324 A client had a right below-the-knee amputation 4 days ago. He is complaining of pain in hisright lower leg. The nurse should: A. Remind the client that he no longer has that part of his leg and assure him he will be OK B. Call the physician to request a psychological consultation for the client C. Turn on the television to distract the client's attention from his amputated leg D. Give the client his order of Demerol 50 mg IM prn Correct Answer: D Section: (none) Explanation Explanation/Reference: Explanation: (A) The nurse is ignoring the client's pain. Telling the client that he will be OK will not relieve his phantom pain. (B) The client does not need a psychological consultation. Phantom pain is a normal sensation experienced by clients with amputations. (C) Using the television as a distractor will not relieve the client's phantom pain. (D) Phantom pain is a normal, very real experience for an amputee and should be treated with pain medication. QUESTION 325 A client has returned to the unit from the recovery room after having a thyroidectomy. The nurse knows that a major complication after a thyroidectomy is: A. Respiratory obstruction B. Hypercalcemia C. Fistula formation D. Myxedema Correct Answer: A Section: (none) Explanation Explanation/Reference: Explanation: (A) Respiratory obstruction due to edema of the glottis, bilateral laryngeal nerve damage, or tracheal compression from hemorrhage is a major complication after a thyroidectomy. (B) Hypocalcemia accompanied by tetany from accidental removal of one or more parathyroid glands is a major complication, not hypercalcemia. (C) Fistula formation is not a major complication associated with a thyroidectomy. It is a major complication with a laryngectomy.(D) Myxedema is hypothyroidism that occurs in adults and is not a complication of a thyroidectomy. A thyroidectomy client tends to develop thyroid storm, which is excess production of thyroid hormone.QUESTION 326 A client had a transurethral resection of the prostate yesterday. He is concerned about the small amount of blood that is still in his urine. The nurse explains that the blood in his urine: A. Should not be there on the second day B. Will stop when the Foley catheter is removed C. Is normal and he need not be concerned about it D. Can be removed by irrigating the bladder Correct Answer: C Section: (none) Explanation Explanation/Reference: Explanation: (A) Some hematuria is usual for several days after surgery. (B) The client will continue to have a small amount of hematuria even after the Foley catheter is removed. (C) Some hematuria is usual for several days after surgery. The client should not be concerned about it unless it increases. (D) Irrigating the bladder will not remove the hematuria. Irrigation is done to remove blood clots and facilitate urinary drainage. QUESTION 327 A 72-year-old male client had the Foley catheter that was inserted during the transurethral resection of his prostate removed today. He is concerned about the urinary incontinence he is having since removal of the Foley catheter. The nurse explains that: A. He should not be concerned about it because it will resolve quickly B. This is usually temporary C. The nurse will keep him dry, and he should notify the nurse when this happens D. This is related to the bladder spasms and will soon stop Correct Answer: B Section: (none) Explanation Explanation/Reference: Explanation: (A) This problem is temporary, but it may take some time to resolve, especially in an older man. (B) This problem is usually temporary, but it may take some time to resolve. (C) Keeping the client dry will not relieve his anxiety about his incontinence. (D) The bladder spasms are not the cause of the client's incontinence. QUESTION 328A 48-year-old female client is going to have a cholecystectomy in the morning. In planning for her postoperative care, the nurse is aware that a priority nursing diagnosis for her will be high risk for: A. Knowledge deficit B. Urinary retention C. Impaired physical mobility D. Ineffective breathing pattern Correct Answer: D Section: (none) Explanation Explanation/Reference: Explanation: (A) The client may have a knowledge deficit, but reducing the risk for knowledge deficit is not a priority nursing diagnosis postoperatively. (B) The client will have a Foley catheter for a day or two after surgery. Urinary retention is usually not a problem once the Foley catheter is removed. (C) A client having a cholecystectomy should not be physically impaired. In fact, the client is encouraged to begin ambulating soon after surgery. (D) Because of the location of the incision, the client having a cholecystectomy is reluctant to breathe deeply and is at risk for developing pneumonia. These clients have to be reminded and encouraged to take deep breaths. QUESTION 329 A client is having a pneumonectomy done today, and the nurse is planning her postoperative care. Nursing interventions for a postoperative left pneumonectomy would include: A. Monitoring the chest tubes B. Positioning the client on the right side C. Positioning the client in semi-Fowler position with a pillow under the shoulder and back D. Monitoring the right lung for an increase in rales Correct Answer: D Section: (none) Explanation Explanation/Reference: Explanation: (A) Chest tubes are usually not necessary in a pneumonectomy because there is no lung to re- expand on the operative side. (B) The pneumonectomy client should be positioned on the back or operated side because the sutured bronchial stump may open, allowing fluid to drain into the unoperated side and drown the client. (C) The client should not have a pillow under the shoulder and back because of the subscapular incision. (D) Rales are commonly heard over the base of theremaining lung, but an increase could indicate circulatory overload and therefore should be closelymonitored. QUESTION 330 A client returned to the unit following a pneumonectomy. As the nurse is assessing her incision, she notices fresh blood on the dressing. The nurse should first: A. Reinforce the dressing. B. Continue to monitor the dressing. C. Notify the physician. D. Note the time and amount of blood. Correct Answer: C Section: (none) Explanation Explanation/Reference: Explanation: (A) The dressing should not be reinforced without first notifying the physician. The decision may be made by the physician to reinforce the dressing after assessing the amount of bleeding. (B) Blood on the dressing is unusual and should make the nurse aware that something more than continuing to monitor the dressing should be done. (C) The physician should be notified immediately, because if the bleeding persists, the client may have to be taken back to surgery. (D) The time and amount of blood do need to be recorded after the physician is notified. QUESTION 331 A client had a renal transplant 3 months ago. He has suddenly developed graft tenderness, an increased white blood cell count, and malaise. The client is experiencing which type of rejection? A. Acute B. Chronic C. Hyperacute D. Hyperchronic Correct Answer: A Section: (none) Explanation Explanation/Reference: Explanation: (A) The sudden development of fever, graft tenderness, increased white blood count, and malaise are signs and symptoms of an acute rejection that commonly occurs at 3 months. (B) Chronic rejection occurs slowly over a period of months to years and mimics chronic renal failure. (C) Hyperacute rejection occursimmediately after surgery up to 48 hours postoperatively. (D) Hyperchronic rejection is not a type of rejection. QUESTION 332 A client hasreceived preoperative teaching for the vertical partial laryngectomy that he is scheduled to have in the morning. The nurse determines that the teaching has been effective when the client states: A. "I know I will need special swallowing training after my surgery." B. "The quality of my voice will be excellent after surgery." C. "I will have very little difficulty swallowing after surgery." D. "I may also have to have a radical neck dissection done." Correct Answer: C Section: (none) Explanation Explanation/Reference: Explanation: (A) A client with a supraglottic (horizontal partial) laryngectomy would require special swallowing training, not a vertical partial laryngectomy. (B) The quality of the client's voice will be altered but adequate for communication. (C) The client will have minimal difficulty swallowing. (D) A radical neck dissection may be done with a total laryngectomy, but not with a partial laryngectomy. QUESTION 333 A client is having a vertical partial laryngectomy, and the nurse is planning his postoperative care. A priority postoperative nursing diagnosis for a client having a vertical partial laryngectomy would be: A. Activity intolerance B. Ineffective airway clearance C. High risk for infection D. Altered oral mucous membrane Correct Answer: B Section: (none) Explanation Explanation/Reference: Explanation: (A) The laryngectomy client should be able to gradually increase activities without difficulty. (B) The laryngectomy client may have copious amounts of secretions and require suctioning for the first 2448 hours. The cannula will require cleaning even after the first 24 hours because mucus collects in it. (C) The client does havea potential for infection, but it is not a more importantnursing priority than the ineffective airway clearance. (D) This problem is not a more important nursing priority than ineffective airway clearance. The client's mouth may become dry, but good oral care should take care of the dryness. QUESTION 334 A client is going to have a pneumonectomy in the morning. She had a previous negative surgical experience, is talking rapidly, and has an increased pulse and respiratory rate. Nursing interventions for this client should include: A. Providing opportunities to ask questions and talk about concerns B. Providing distractors such asreading or watching television C. Telling her that she should not be so nervous and assuring her that everything will be OK D. Reminding her that this surgery is not as extensive as her past surgery was Correct Answer: A Section: (none) Explanation Explanation/Reference: Explanation: (A) This intervention will help to clarify any misunderstandings about the surgery and give the client an opportunity to verbalize concerns about the surgery. (B) Distractors will not alleviate the preoperative anxiety that the client is experiencing. Thisresponse actually denies the client's anxiety. (C) This intervention is false assurance and denies that anxiety is a normal response to the threat of surgery. (D) Psychological responses are not directly related to the extent of the surgery, because they are influenced by the client's past experiences. QUESTION 335 On admission to the inpatient unit, a 34-year-old client is able to follow simple directions, but with great difficulty. He is worried about how he can keep clean in such a public place and repeatedly dusts his bureau, straightens his bed, and adjusts the clothes in his closet. The client is experiencing a severe level of anxiety. Which response by the nurse would be most therapeutic in initially attempting to reduce his anxiety? A. "You will not be allowed to remain in your room if you continue to bother things." B. "I can see how uncomfortable you are, but I would like you to walk with me so I can show you around the unit." C. "Tell me why your room needs to be so clean." D. "I've inspected this room and it is perfectly clean." Correct Answer: B Section: (none) Explanation Explanation/Reference: Explanation:(A) This statement is punitive. (B) Acknowledging the anxiety and channeling it into some positive activity is therapeutic. (C) The client cannot say "why"; this statement puts the client on the defensive. (D) A rational approach, especially a judgmental one, is nontherapeutic. QUESTION 336 The physician prescribes amitriptyline (Elavil) for a client. What does the patient need to know about this medication? A. Prolonged use of this medication will result in extrapyramidal side effects. B. When the medication is effective, he will experience no anxiety. C. The medication should relieve his symptoms of depression. D. Blood must be drawn weekly to test for toxicity. Correct Answer: C Section: (none) Explanation Explanation/Reference: Explanation: (A) Phenothiazines cause extrapyramidal symptoms. (B) No amount of medication can relieve all anxiety in all cases. (C) The purpose of amitriptyline is to relieve the symptoms of depression because it is an antidepressant. It increases the action of norepinephrine and serotonin on nerve cells. (D) Periodic blood tests are done when lithium is prescribed. QUESTION 337 The health team needs to realize that the compulsive concern with cleanliness that a client with severe anxiety exhibits is most likely an attempt to: A. Reduce his anxiety B. Avoid going to psychotherapy C. Manipulate the health team members D. Increase his self-image by showing higher standards than the fellow clients Correct Answer: A Section: (none) Explanation Explanation/Reference: Explanation: (A) These behaviors are attempts to relieve anxiety. (B) Avoidance is not a pattern in the obsessive client. (C) Although these behaviors may seem to manipulate others, that is not the purpose behind the activity. (D) Inflated self-esteem is not a characteristic of the severely anxious client.QUESTION 338 A successful executive left her job and became a housewife after her marriage to a plastic surgeon. She started doing volunteer work for a charity organization. She developed pain in her legs that advanced to the point of paralysis. Her physicians can find no organic basis for the paralysis. The client's behavior can be described as: A. Housework phobia B. Malingering C. Conversion reaction D. Agoraphobia Correct Answer: C Section: (none) Explanation Explanation/Reference: Explanation: (A) A typical phobia does not result in physical symptoms (i.e., paralysis). (B) Malingering is pretending to be ill. This person has a true paralysis. (C) A conversion reaction is a physical expression of an emotional conflict. It has no organic basis. (D) Agoraphobia is fear of public places. QUESTION 339 A 28-year-old client performs a long, involved ritual in getting up and preparing for the day. He became unable to get to his job before noon. His family, in desperation, has admitted him to the hospital's psychiatric unit. On the unit, he is always late for breakfast, which is served at 8 am. The nurse identifies that the best approach to this problem is to: A. Allow him to eat late B. Suggest that he do the rituals after breakfast C. Get him up early so that he can complete the ritual before breakfast D. Ask him to get all the other clients up so that he will forget about his ritual Correct Answer: C Section: (none) Explanation Explanation/Reference: Explanation: (A) Allowing him to eat late is not a solution to the problem because the ritual affects more than just this meal. (B) He is helpless to change this behavior because the rituals occur as a result of an irrational effort to control his anxiety. (C) To interfere with the ritual will increase anxiety. Until the basic problem isresolved, and inturn his need for the ritual relieved, arrange the schedule so that essential activities may be included (such as meals with the group). (D) This approach would be very disruptive to the other clients and would not serve to relieve the anxiety of the client. QUESTION 340 A 25-year-old lawyer who is married with three young children works long hours in an effort to become a partner in the law firm. Following a recent hospitalization for a bleeding ulcer, he was referred for therapy to treat this psychophysiological disorder. On meeting with the therapist, he informed him or her that he was a busy man and did not have much time for this "psych stuff." When guiding the client to ventilate his feelings, the therapist can expect him to express feelings of: A. Guilt B. Shame C. Despair D. Anger Correct Answer: D Section: (none) Explanation Explanation/Reference: Explanation: (A) Guilt relates to depression. (B) Shame is not associated with psychophysiological disorders. (C) Despair relates to depression. (D) Repressed anger is associated with psychophysiological disorders. QUESTION 341 Plans for the care of a client with an ulcer caused by emotional problems need to take into consideration that: A. His priority needs are limited to medical management B. There is no real psychological basis for his illness C. The disorder is a threat to his physical well-being D. He is unable to participate in planning his care Correct Answer: C Section: (none) Explanation Explanation/Reference: Explanation: (A) There may be a medical emergency that takes top priority; however, the basis of the problem is emotional. (B) The problem is a physical manifestation of anemotional conflict. (C) The bleeding ulcer can be life threatening. (D) For lifestyle change to occur, the client must participate in the planning of his care so that he is committed to changes that will have positive results. QUESTION 342 A client has been uncomfortable in crowds all her life. After the birth of her child, she has been housebound unless her husband can accompany her to the grocery store and for medical appointments. His schedule will not allow for this, and he has insisted that she must be more independent. Her anxiety has increased to the point of panic. The client has been diagnosed with agoraphobia. Which statement is true about this disorder? A. The behavior is not considered disabling. B. More men suffer from agoraphobia than women. C. The fears are persistent, and avoidance is used as the coping mechanism. D. Agoraphobia moves into remission when treated with chlorpromazine. Correct Answer: C Section: (none) Explanation Explanation/Reference: Explanation: (A) Agoraphobia is the most pervasive and serious phobic disorder. (B) Women compose 70% 85% of agoraphobia sufferers. (C) Agoraphobia is an acute disorder that immobilizes the sufferer with extreme anxiety. (D) Chlorpromazine is not a drug used to treat phobias. QUESTION 343 A 22-year-old single woman was admitted to the psychiatric hospital by her mother, who reported bizarre behavior. Except for going to work, she spends all her time in her room and expresses concern over neighbors spying on her. She has fears of the telephone being "bugged." Her diagnosis is schizophrenia. One nurse per shift is assigned to work with the client. The primary reason for this plan would be to: A. Protect her from suicide B. Enable her to develop trust C. Supervise her medication regimen D. Involve her in groups for social interaction Correct Answer: B Section: (none) Explanation Explanation/Reference: Explanation:(A) Suicide is a greater risk in depression than in schizophrenia. (B) The client is suspicious and needs help to develop trust, which is basic to her improvement. (C) Although she will be taking medication, drug therapy would not necessitate consistency in the nurses assigned. (D) A suspicious client should have limited exposure to groups, because group participation increases discomfort. QUESTION 344 The 4th of July holiday comes while a client is in the hospital being treated for schizophrenia. She is taking chlorpromazine and has improved to the point of being allowed to go with a group to the park for a picnic. The side effect of chlorpromazine that the nurse needs to keep in mind during this outing is: A. Hypotension B. Photosensitivity C. Excessive appetite D. Dryness of the mouth Correct Answer: B Section: (none) Explanation Explanation/Reference: Explanation: (A) A decrease in blood pressure sometimes occurs with chlorpromazine. It would not be a factor influenced by a picnic in the park. (B) Protection from the sun is important in clients taking phenothiazines because they burn easily and severely. (C) An appetite increase sometimes occurs with chlorpromazine. It would not be affected by a picnic. (D) Dryness of the mouth may occur at any time and is not affected by the picnic outing. QUESTION 345 Except for initial explosiveness on admission, a client diagnosed with schizophrenia stays in her room. She continues to believe other people are out to get her. A nursing intervention basic to improving withdrawn behavior is: A. Assigning her to occupational therapy B. Having her sit with the nurses while they chart C. Helping her to make friends D. Facilitating communication Correct Answer: D Section: (none) Explanation Explanation/Reference: Explanation: (A) The nurse does not make this assignment. (B) One-to-one observation is not appropriate. It does not focus on the client or encourage communication. (C) Theclient is too suspicious to accomplish this goal. (D) The withdrawn individual must learn to communicate on a one-to-one level before moving on to more threatening situations. QUESTION 346 A 32-year-old mother of two was brought to the hospital by her husband. He reported that his wife could no longer manage the house and children. She does not sleep and talks day and night. She has purchased some very expensive clothes. The nurse noted that the client speaks rapidly and changes the subject irrationally. This is an example of: A. Flight of ideas B. Delusions C. Hallucinations D. Echolalia Correct Answer: A Section: (none) Explanation Explanation/Reference: Explanation: (A) Rapidly moving from one topic to another without following any logical sequence is called flight of ideas. (B) False beliefs are delusions. (C) False sensory perceptions are hallucinations ("hearing voices"). (D) Repeating words is called echolalia. QUESTION 347 A client is placed on lithium therapy for her manicdepressive illness. When monitoring the client, the nurse assesses the laboratory blood values. Toxicity may occur with lithium therapy when the blood level is above: A. 1.0 mEq/L B. 2.2 mEq/L C. 0.03 mEq/L D. 1.5 mEq/L Correct Answer: D Section: (none) Explanation Explanation/Reference: Explanation: (A) This value is a low blood level. (B) This value is a toxic blood level. (C) This value is a low blood level. (D) This value is the level at which most clients aremaintained, and toxicity may occur if the level increases. The client should be monitored closely for symptoms, because some clients become toxic even at this level. QUESTION 348 A client's behavior is annoying other clients on the unit. He is meddling with their belongings and dominating the group. The best approach by the nurse is to: A. Seclude him in his room. B. Set limits on his behavior. C. Have his medication increased. D. Ignore him and tell the other clients that these behaviors are due to his illness and that they should understand. Correct Answer: B Section: (none) Explanation Explanation/Reference: Explanation: (A) This action by the nurse would be punitive. (B) Consistent limit setting will help the client to know what is acceptable behavior. (C) This action is not within the nurse's scope of practice. (D) This could be dangerous to the client and to others and violates other clients' rights. QUESTION 349 A client is hyperactive and not sleeping. She will not remain at the table during mealtime. She is getting very limited calories and is using a lot of energy in her hyperactive state. The most therapeutic nursing action is to: A. Insist that she remain at the table and eat a balanced diet. B. Order a high-calorie diet with supplements. C. Provide nutritious finger foods several times a day. D. Offer to go to the dining room with her and allow her to open the food and inspect what she eats. Correct Answer: C Section: (none) Explanation Explanation/Reference: Explanation: (A) The client is not able to sit for long periods. Forcing her to remain at the table will increase her anxiety and cause her to become hostile. (B) This action will not ensure that the client eats what is ordered. Dietary orders are not within the nurse's scope of practice. (C) Providing finger foods increases the likelihood of eating for hyperactive persons. They may be eating "on the run." (D) These clients are not suspicious of the food or insecure in moving about the unit alone.QUESTION 350 A hyperactive client is experiencing flight of ideas. The most therapeutic activity for him wouldbe: A. Doing crafts in occupational therapy B. Working a 1000-piece puzzle C. Playing bridge with three other clients D. Playing basketball in the gym Correct Answer: D Section: (none) Explanation Explanation/Reference: Explanation: (A) This activity requires motor skills and therefore would be difficult for a hyperactive client. (B) This activity would take too long, and the client would have difficulty concentrating owing to a limited attention span. (C) This client would not be able to concentrate enough to play card games. He would respond to all the stimuli in the area, become distracted, and leave the table. (D) This activity would allow the client to channel his energy in a positive way. QUESTION 351 A client is a depressed, 48-year-old salesman. A serious concern for the nurse working with depressed clients is the potential of suicide. The time that suicide is most likely to occur is: A. In the acutely depressed state B. When the depression starts to lift C. In the denial phase D. During a manic episode Correct Answer: B Section: (none) Explanation Explanation/Reference: Explanation: (A) The client may be too disorganized in the acute phase to make a workable plan. (B) When the depression starts to lift, the client is able to make a workable plan. (C) There usually is not a significant denial phase related to depression. Suicide occurs in a state of despair and hopelessness. (D) Suicide is uncommon in the manic state. In this state, clients do not feel hopeless, but euphoric and overly confident.QUESTION 352 Succinylcholine chloride (Anectine) is ordered prior to electroconvulsive therapy treatment for depressed clients. The nurse explains that the purpose of the drug is to: A. Relax muscles B. Relieve anxiety C. Reduce secretions D. Act as an anesthetic Correct Answer: A Section: (none) Explanation Explanation/Reference: Explanation: (A) Succinylcholine chloride relaxes muscles and decreases the intensity of the seizure. (B) Succinylcholine chloride does not relieve anxiety. (C) Atropine is given to reduce secretions. (D) Thiamylal sodium (Surital) or other phenobarbital preparations are used as brief anesthetics. QUESTION 353 The nurse teaches a pregnant client that a high-risk symptom occurring at any time during pregnancy that needs to be reported immediately to a healthcare provider is: A. Constipation B. Urinary frequency C. Breast tenderness D. Abdominal pain Correct Answer: D Section: (none) Explanation Explanation/Reference: Explanation: (A) Constipation is a result of decreased peristalsis due to smooth muscle relaxation related to changing progesterone levels that occur during pregnancy. (B) Urinary frequency is a common result of the increasing size of the uterus and the resulting pressure it places on the bladder. (C) With the increased vascularity and hypertrophy of the mammary alveoli due to estrogen and progesterone level changes, the breasts will increase in size and may become tender. (D) Abdominal pain may be an indication of early spontaneous abortion, preterm delivery, or a placental abruption.QUESTION 354 At her first prenatal visit, a 21-year-old woman who is gravida 2, para 0, ab 1, is currently at 32 weeks' gestation and has a history of drug abuse, smoking, and occasional ethyl alcohol use. Fetal ultrasound tests indicate poor fetal growth. The most likely reason for the infant's intrauterine growth retardation is: A. The client's young age B. The client's previous abortion C. The client's history of drug, ethyl alcohol, and tobacco use D. The client's late prenatal care Correct Answer: C Section: (none) Explanation Explanation/Reference: Explanation: (A) Although adolescents frequently have a higher incidence of low-birth-weight infants, this client is 21 years old. (B) Uncomplicated induced abortions have not been proved to influence the growth of infants of subsequent pregnancies. (C) Compounds in cigarettes and some illicit drugs cause maternal vasoconstriction and a subsequent reduction in O2 availability for the fetus owing to the resulting reduction in uteroplacental blood flow. As few as one or two drinks of alcohol per day will decrease birth weight. (D) Although early prenatal care has been shown to improve pregnancy outcomes, not seeking care until the second week of gestation does not, in and of itself, cause intrauterine growth retardation. QUESTION 355 When teaching a class of nursing students, the nurse asks why the embryonic period (weeks 48) of pregnancy is so critical. A. Duplication of genetic information takes place. B. Organogenesis occurs. C. Subcutaneous fat builds up steadily. D. Kidneys begin to secrete urine. Correct Answer: B Section: (none) Explanation Explanation/Reference: Explanation: (A) Duplication of genetic material occurs during the preembryonic period (weeks 13) following conception. The exact duplication of genetic material is essential for cell differentiation, growth, and biological maintenance of the organism. (B) Weeks 48, known as the embryonic period, are the time organogenesis occurs andpose the greatest potential for major congenital malformations. All major internal and external organs and systems are formed. (C) Subcutaneous fat does not develop until the latter weeks of gestation. (D) Kidneys begin to secrete urine during the 13th16th week. QUESTION 356 What specific hormone must be present in serum or urine laboratory tests used to diagnose pregnancy? http://www.gratisexam.com/ A. Human chorionic gonadotropin B. Estrogen C. -fetoprotein D. Sphingomyelin Correct Answer: A Section: (none) Explanation Explanation/Reference: Explanation: (A) Human chorionic gonadotropin is the biochemical basis for pregnancy tests. It is produced by the placenta to help maintain the corpus luteum. Its levels climb rapidly following conception, peaking at about 8 weeks and then gradually decreasing to low levels after 16 weeks. (B) Estrogen does steadily rise throughout pregnancy, increasing to 30 times that of prepregnancy levels. Although estrogen levels do change during pregnancy, it is not used as the main hormone of evaluation in pregnancy tests. (C) -Fetoprotein is the major protein in the serum of theembryo. It is initially produced by the yolk sac. (D) Lecithin and sphingomyelin are two phospholipids of which fetal lung surfactant is composed. Levels are evaluated to determine fetal lung maturity. QUESTION 357 A nurse is taking a maternal history for a client at her first prenatal visit. Her pregnancy test was positive, she has two living children, she had one spontaneous abortion, and one infant died at the age of 3 months. Which of the following best describes the client at the present? A. Gravida 4, para 2, ab 1 B. Gravida 5, para 3, ab 1 C. Gravida 5, para 4, ab 0 D. Gravida 4, para 3, ab 0Correct Answer: B Section: (none) Explanation Explanation/Reference: Explanation: (A) This individual has been pregnant four times, delivered two children, and had one abortion. (B) Your client has been pregnant five times, delivered three children, and had one abortion. (C) This individual has been pregnant five times, delivered four children, and has not had an abortion. (D) This individual has been pregnant four times, delivered three children, and has not had an abortion. QUESTION 358 A client is now pregnant for the second time. Her first child weighed 4536 g at delivery. The client's glucose tolerance test shows elevated blood sugar levels. Because she only shows signs of diabetes when she is pregnant, she is classified as having: A. Insulin-dependent diabetes B. Type II diabetes mellitus C. Type I diabetes mellitus D. Gestational diabetes mellitus Correct Answer: D Section: (none) Explanation Explanation/Reference: Explanation: (A) Insulin-dependent diabetes mellitus, also known as type I diabetes, usually appears before the age of 30 years with an abrupt onset of symptoms requiring insulin for management. It is not related to onset during pregnancy. (B) Non-insulindependent diabetes (type II diabetes) usually appears in older adults. It has a slow onset and progression of symptoms. (C) This type of diabetes is the same as insulin-dependent diabetes. (D) Gestational diabetes mellitus has its onset of symptoms during pregnancy and usually disappears after delivery. These symptoms are usuallymild and not life threatening, although they are associated with increased fetal morbidity and other fetal complications. QUESTION 359 Prior to an amniocentesis, a fetal ultrasound is done in order to: A. Evaluate fetal lung maturity B. Evaluate the amount of amniotic fluid C. Locate the position of the placenta and fetusD. Ensure that the fetus is mature enough to perform the amniocentesis Correct Answer: C Section: (none) Explanation Explanation/Reference: Explanation: (A) Amniocentesis can be performed to assess for lung maturity. Fetal ultrasound can be used for gestational dating, although it does not separately determine lung maturity. (B) Ultrasound can evaluate amniotic fluid volume, which may be used to determine congenital anomalies. (C) Amniocentesis involves removal of amniotic fluid for evaluation. The needle, inserted through the abdomen, is guided by ultrasound to avoid needle injuries, and the test evaluates the position of the placenta and the fetus. (D) Amniocentesis can be performed as early as the 15th17th week of pregnancy. QUESTION 360 A 17-year-old pregnant client who is gravida 1, para 0, is at 36 weeks' gestation. Based on the nurse's knowledge of the maternal physiological changes in pregnancy, which of these findings would be of concern? A. Complaints of dyspnea B. Edema of face and hands C. Pulse of 65 bpm at 8 weeks, 73 bpm at 36 weeks D. Hematocrit 39% Correct Answer: B Section: (none) Explanation Explanation/Reference: Explanation: (A) Dyspnea is a common complaint during the third trimester owing to the increasing size of the uterus and the resulting pressure against the diaphragm. (B) Edema of the face, hands, or pitting edema after 12 hours of bed rest may be indicative of preeclampsia and would be of great concern to the healthcare provider. (C) An increase in heart rate of 1015 bpm is a normal physiological change in pregnancy due to the multiple hemodynamic changes. (D) A hematocrit value of 39% is within the normal range. A value <35% would indicate anemia. QUESTION 361 Based on your knowledge of genetic inheritance, which of these statementsis true for autosomal recessive genetic disorders? A. Heterozygotes are affected. B. The disorder is always carried on the X chromosome.C. Only females are affected. D. Two affected parents always have affected children. Correct Answer: D Section: (none) Explanation Explanation/Reference: Explanation: (A) The term heterozygote refers to an individual with one normal and one mutant allele at a given locus on a pair of homologous chromosomes. An individual who is heterozygous for the abnormal gene does not manifest obvious symptoms. (B) Disorders carried on either the X or Y sex chromosome are referred to as sex- linked recessive. (C) Either sex may be affected by autosomal recessive genetic disorders because the responsible allele can be on any one of the 46 chromosomes. (D) If both parents are affected by the disorder and are not just carriers, then all their children would manifest the same disorder. QUESTION 362 Chorioamnionitis is a maternal infection that is usually associated with: A. Prolonged rupture of membranes B. Postterm deliveries C. Maternal pyelonephritis D. Maternal dehydration Correct Answer: A Section: (none) Explanation Explanation/Reference: Explanation: (A) Chorioamnionitis is an inflammation of the chorion and amnion that is generally associated with premature or prolonged rupture of membranes. (B) Postterm deliveries have not been shown to increase the risk of chorioamnionitis unless there has been prolonged rupture of membranes. (C) Pyelonephritis is a kidney infection that develops in 20%40% of untreated maternal UTIs. (D) Maternal dehydration, though of great concern, is not related to chorioamnionitis. QUESTION 363 A client has been diagnosed with thrombophlebitis. She asks, "What is the most likely cause of thrombophlebitis during my pregnancy?" The nurse explains: A. Increased levels of the coagulation factors and a decrease in fibrinolysis B. An inadequate production of platelets C. An inadequate intake of folic acid during pregnancyD. An increase in fibrinolysis and a decrease in coagulation factors Correct Answer: A Section: (none) Explanation Explanation/Reference: Explanation: (A) During pregnancy, the potential for thromboses increases owing to the increased levels of coagulation factors and a decrease in the breakdown of fibrin. (B) An inadequate production of platelets would result in thrombocytopenia with resulting signs and symptoms of bleeding such as petechiae, hematuria, or hematemesis. (C) A deficiency of folic acid during pregnancy produces a megaloblastic anemia. It is usually found in combination with iron deficiency. (D) This combination would result in bleeding disorders because more fibrin would be broken down and fewer clotting factors would be available. QUESTION 364 Following the delivery of a healthy newborn, a client has developed thrombophlebitis and is receiving heparin IV. What are the signs and symptoms of a heparin overdose for which the nurse would need to observe during postpartum care of the client? A. Dysuria B. Epistaxis, hematuria, dysuria C. Vertigo, hematuria, ecchymosis D. Hematuria, ecchymosis, and epistaxis Correct Answer: D Section: (none) Explanation Explanation/Reference: Explanation: (A) Dysuria is not a common symptom of heparin overdose. (B) Although epistaxis and hematuria are common symptoms of heparin overdose, dysuria is not. (C) Vertigo is not a common symptom of heparin overdose. (D) Hematuria, ecchymosis, and epistaxis are the most common signs and symptoms of a heparin overdose. Others are thrombocytopenia, elevated liver enzymes, and local injection site complications. QUESTION 365 A client who is a breast-feeding mother develops mastitis. The clinical signs and symptoms of mastitis include: A. Marked engorgement, elevated temperature, chills, and breast pain with an area that is red and hardened B. Marked engorgement and breast pain C. Elevated temperature and general malaiseD. Cracked nipple with complaints of soreness Correct Answer: A Section: (none) Explanation Explanation/Reference: Explanation: (A) Mastitis is a bacterial inflammation of the breast tissue found primarily in breast-feeding mothers. The bacteria usually enter the breast through a cracked nipple, or the infection results from stasis of milk behind a blocked duct. (B) With breast engorgement during breast-feeding, there may be marked breast pain. This is not necessarily a sign of infection. (C) Women may become ill during breast-feeding with other bacterial or viral infections that are not related to mastitis. (D) Improper care of the nipples or improper positioning of the infant during breastfeeding may result in cracked or sore nipples. QUESTION 366 A 34-year-old client who is gravida 1, para 0 has a history of infertility and conceived this pregnancy while taking fertility drugs. She is at 32 weeks' gestation and is carrying triplets. She is complaining of low back pain and a feeling of pelvic pressure. Her cervical exam reveals a long, closed cervix. The nurse notes that the client is experiencing mild uterine contractions every 78 minutes after the nurse has placed her on the fetal monitor. Her condition should indicate that: A. Her cervix shows she will likely deliver soon B. The nurse should not be alarmed because mild uterine activity is common at 32 weeks' gestation C. She may be in preterm labor because this is more common with multiple pregnancies D. She most likely has a urinary tract infection (UTI) because this is common with pregnancy Correct Answer: C Section: (none) Explanation Explanation/Reference: Explanation: (A) Her cervical exam is normal. There are no cervical changes at this time. (B) Braxton Hicks contractions may be common throughout pregnancy, but they arenot regular. (C) Rhythmical contractions in conjunction with low back pain and pelvic pressure at 32 weeks in a woman carrying triplets are of great concern. She may be in preterm labor. (D) UTIs are common in pregnancy due to the enlarging uterus compressing the ureters and the stasis of urine. The woman would be more likely to complain of urinary frequency and urgency, fever or chills, and malodorous urine with a UTI. QUESTION 367 The most frequent cause of early postpartum hemorrhage is: A. Hematoma B. Coagulation disordersC. Uterine atony D. Retained placental fragments Correct Answer: C Section: (none) Explanation Explanation/Reference: Explanation: (A) Hematomas, which are the result of damage to a vessel wall without laceration of the tissue, are a cause, though not the most frequent cause. (B) Coagulation disorders are among the causes of postpartal hemorrhage, but they are less common. (C) The most frequent causes of hemorrhage in the postpartal period are related to an interference with involution of the uterus. Uterine atony is the most frequent cause, occurring in the first 24 hours after delivery. (D) Retained placental fragments are also a cause, although these bleeds usually occur 714 days after delivery. QUESTION 368 A client has just been transferred to the floor from the labor and delivery unit following delivery of a stillborn term infant. She is very despondent. When the nurse attempts to take her vital signs, she responds in anger, stating, "You leave me alone. You don't care anything about me. It's people like you who let my baby die." The nurse's best course of action is to: A. Quietly leave her room, allowing her more private time to deal with her loss. B. Tell her that what happened was for the best and that she is still young and can have other children. C. Tell her how sorry you are, and let her know that her child is now a little angel in heaven. D. Tell her how sorry you are about the loss of her baby, and acknowledge her anger as being a normal stage of grief. Assure her that you are there to help her in any way you can. Correct Answer: D Section: (none) Explanation Explanation/Reference: Explanation: (A) Parents do need their privacy following a loss, but the nurse still has a responsibility to provide postpartum physical care. (B) This is a negative statement, which is not therapeutic. The client is not concerned about future children but is in the first stages of grief, denial, and anger. (C) This is a negative statement, which is not therapeutic. The client does not want to hear about her baby in heaven. She cannot believe that God could love or want her child more than she could. (D) Acknowledging that anger is normal and beneficial will help the client to understand the normal stages of grief. Expressing sorrow over her loss and assuring her that the support is there to take care of her physical and emotional needs will help to promote a trusting relationship. QUESTION 369When a client arrives on the labor and delivery unit, she informs the nurse that she has been having contractions for the last 5 hours. Now the pain is constant and not cyclical as it was earlier. The nurse considers the possibility of uterine rupture. Which of the following symptoms would be consistent with a uterine rupture? A. A large gush of clear fluid from the vagina B. Systolic hypertension C. Abdominal rigidity D. Increased fetal movements Correct Answer: C Section: (none) Explanation Explanation/Reference: Explanation: (A) This symptom would indicate a rupture of the membranes, which would be expected during labor. There would be no cause for alarm if the fluid were clear. (B) With uterine rupture and the risk of maternal shock secondary to blood loss, the most likely sign would be hypotension indicating hypovolemic shock. (C) In the event of a uterine rupture, an abdominal examination would likely reveal rigidity or tenderness. (D) The most likely finding would be a decrease in fetal movementrelated to fetal distress due to impaired uteroplacental blood flow. Maintaining the client on her left side would help to maximize uterine blood flow. QUESTION 370 A mother called the physician's office to ask if it would help relieve her small daughter's abdominal pain if she gave an enema and placed a heating pad on the abdomen. Her daughter has a fever and has vomited twice. The nurse's response is based on the knowledge that: A. The symptoms could easily have been caused by constipation, which an enema would relieve B. Heat would help to relax the abdominal muscles and relieve her pain C. Both heat and enemas stimulate intestinal motility and could increase the risk of perforation D. Complaints of stomach ache are common in young children and are generally best ignored Correct Answer: C Section: (none) Explanation Explanation/Reference: Explanation: (A) Constipation does not cause fever or vomiting but may cause anorexia. Risk of perforation outweighs the possible benefits of an enema. (B) Heat will not relieve her symptoms but will increase intestinal motility and increase the risk of perforation. (C) Heat and enemas are contraindicated where severe abdominal pain is suspected because they increase intestinal motility and the risk of perforation. (D) Complaints accompanied by physical symptoms such as pain, anorexia,and fever should never be ignored. QUESTION 371 An 8-year-old child is admitted to the hospital for surgery. She has had no previous hospitalizations, and both she and her family appear anxious and fearful. It will be most helpful for the nurse to: A. Take the child to her room and calmly and matter-offactly begin to get her ready to go to the operating room B. Take time to orient the child and her family to the hospital and the forthcoming events C. Explain that as soon as the child goes to the operating room she will have time to answer any questions the family has D. Tell the child and her family that there is nothing to worry about, that the operation will not take long, and she will soon be as "good as new" Correct Answer: B Section: (none) Explanation Explanation/Reference: Explanation: (A) This action does nothing to prepare the child and her family for what will happen or to relieve their anxiety and fear. (B) This action provides security by preparing the child and the family for what will happen and will help to relieve fear and anxiety. (C) This action does nothing to help prepare the child for what will happen and does not give the parents permission to ask questions until later. (D) This action provides possibly false reassurance and may prevent the child and/or the family from asking pressing questions. QUESTION 372 Following a bicycle accident, a 12-year-old client sustained a complete fracture of the left femur. He was placed in 90-90 skeletal traction with a pin in the distal end of the femur to achieve realignment and immobilization of the left femur. When providing nursing care, it is important for the nurse to remember that: A. The nurse may lift only the weights that are applying traction in order to reposition him in bed B. The client will need special skin care at the pin site according to hospital policy or the physician's preference C. The traction pull should result in an immediate increase in comfort and reduce the need for pain medication D. The client should be discouraged from participating in self-care activities to avoid the risk of disrupting the traction Correct Answer: B Section: (none) Explanation Explanation/Reference: Explanation: (A) Skeletal traction, including the weights that are applying the traction, is never released by the nurse. (B) It is necessary to keep the pin site clean and free frominfection. (C) When first placed in traction, the client may experience increased discomfort as a result of the traction pull fatiguing the muscle. (D) When the child intraction is allowed to participate in his care, it gives him a measure of control and helps him to cope with the situation. QUESTION 373 A 10-year-old client with a pin in the right femur is immobilized in traction. He is exhibiting behavioral changes including restlessness, difficulty with problem solving, inability to concentrate on activities, and monotony. Which of the following nursing implementations would be most effective in helping him cope with immobility? A. Providing him with books, challenging puzzles, and games as diversionary activities B. Allowing him to do as much for himself as he is able, including learning to do pin-site care under supervision C. Having a volunteer come in to sit with the client and to read him stories D. Stimulating rest and relaxation by gentle rubbing with lotion and changing the client's position frequently Correct Answer: B Section: (none) Explanation Explanation/Reference: Explanation: (A) These activities could be frustrating for the client if he is having difficulty with problem solving and concentration. (B) Selfcare is usually well received by the child, and it is one of the most useful interventions to help the child cope with immobility. (C) This may be helpful to the client if he has no visitors, but it does little to help him develop coping skills. (D) This will helpto prevent skin irritation or breakdown related to immobility but will not help to prevent behavioral changes related to immobility. QUESTION 374 Following a fracture of the left femur, a client develops symptoms of osteomyelitis. During the acute phase of osteomyelitis, nursing care is directed toward: A. Moving or turning the client's left leg carefully to minimize pain and discomfort B. Allowing the client out of bed only in a wheelchair or gurney to minimize weight bearing on the left leg C. Providing the client with a high-protein, high-fiber diet to promote healing D. Instituting physical therapy to ensure restoration of optimal functioning of the leg Correct Answer: A Section: (none) Explanation Explanation/Reference: Explanation: (A) Any movement of his affected limb will cause discomfort to the child. (B) No weight bearing will be allowed until healing is well underway to avoid pathological fractures. (C) The child will be anorexic and may experience vomiting. Diet should be simple and high caloric until appetite returns and symptoms subside. (D)Physical therapy is instituted only after infection subsides. QUESTION 375 Several months after antibiotic therapy, a child isreadmitted to the hospital with an exacerbation of osteomyelitis, which is now in the chronic stage. The mother appears anxious and asks what she could have done to prevent the exacerbation. The nurse's response is based on the knowledge that chronic osteomyelitis: A. Is caused by poor physical conditions or poor nutrition B. Often results from unhygienic conditions or an unclean environment C. Is directly related to sluggish circulation in the affected limb D. May develop from sinuses in the involved bone that retain infectious material Correct Answer: D Section: (none) Explanation Explanation/Reference: Explanation: (A) Poor nutrition and/or poor physical conditions are factorsthat predispose to the development of osteomyelitis but do not cause it. (B) An unclean or unhygienic environment may predispose to the development of chronic osteomyelitis, but it does not cause an exacerbation of the previous infection. (C) Sluggish circulation through the medullary cavity during acute osteomyelitis may delay healing, but it does not cause the disease to become chronic. (D) Areas of sequestrum may be surrounded by dense bone, become honeycombed with sinuses, and retain infectious organisms for a long time. QUESTION 376 A 5-year-old child was recently diagnosed as having acute lymphoid leukemia. She is hospitalized for additional tests and to begin a course of chemotherapy designed to induce a remission. She is scheduled to have a bone marrow aspiration tomorrow. She has had a bone marrow test previously and is apprehensive about having another. Which of the following interventions will be most effective in relieving her anxiety? A. Explain what will take place and what she will see, feel, and hear. B. Remind her that she has had this procedure before and that it is nothing to be afraid of. C. Tell her not to worry about it, that it will be over soon and she can join her friends in the playroom. D. Give her a big hug and tell her that she is a big girl now and that she will do just fine. Correct Answer: A Section: (none) Explanation Explanation/Reference: Explanation:(A) Even though the child has had the procedure before, she will probably need additional explanations and emotional support. (B) The fact that the child has had the procedure before and possibly found it painful or uncomfortable may increase, not relieve, her stress. (C) This intervention does nothing to reassure the child and may well prevent her from expressing her feelings. (D) This does not prepare the child for the test and burdens her with the expectation that she act bigger and braver than she is. QUESTION 377 Parents of children receiving chemotherapy should be warned that alopecia is a side effect and that: A. Children seldom show concern about losing their hair B. The hair will come out gradually, and the loss will not be noticeable for some time C. It is best for girls to choose a wig similar to their hair style and color before the hair falls out D. The parents will soon get used to seeing their children without hair, and it will no longer bother them Correct Answer: C Section: (none) Explanation Explanation/Reference: Explanation: (A) Children may become depressed with a changed appearance and not want to look at themselves or have others see them. (B) The hair will fall out in clumps, causing patchy baldness that is quite noticeable and traumatic to children and their families. (C) Having a wig that looks like a girl's own hair can be a psychological boost to children and is helpful in fostering later adjustments to hair loss. (D) Families may become accustomed to seeing their children without hair, but the loss is traumatic to them and will continue to bother them. QUESTION 378 A child receiving chemotherapeutic drugs experiences a loss of appetite directly related to the therapy. Which of the following strategies should be most effective in encouraging the child to eat? A. Provide a well-balanced diet at usual times, and restrict dessert if the child fails to eat well. B. Schedule procedures immediately after eating so that the child will not be tired or in pain at mealtime. C. Offer the child a diet with a wider variety of foods and with more seasoning than her usual diet. D. Offer the child smaller meals more frequently than usual, and include as many of her favorite foods as possible. Correct Answer: D Section: (none) Explanation Explanation/Reference: Explanation:(A) Because the child's appetite is capricious at best, regular servings may be overwhelming. Praise the child for what is eaten. (B) The child will soon learn that procedures follow meals and may play with food rather than eat it to avoid or delay the procedure. (C) Young children usually do not like highly seasoned foods and may need the security of usual foods. Such a change may actually increase anorexia. (D) Small servings appear more achievable to the child, and the inclusion offavorite foods can add a sense of security. QUESTION 379 A child becomes neutropenic and is placed on protective isolation. The purpose of protective isolation is to: A. Protect the child from infection B. Provide the child with privacy C. Protect the family from curious visitors D. Isolate the child from other clients and the nursing staff Correct Answer: A Section: (none) Explanation Explanation/Reference: Explanation: (A) The child no longer has normal white blood cells and is extremely susceptible to infection. (B) There are more appropriate ways to provide privacy, and there is no need to protect the child from healthy visitors. (C) Visitors and visiting hours may be at the client's and/or family's request without regard to the isolation precaution. (D) The child may have strong positive relationships with other clients or staff. As long as proper precautions are observed, there is no reason to isolate her from them. QUESTION 380 A client is experiencing mucosal cell damage secondary to chemotherapy. Because of mucosal ulcers, eating has become increasingly uncomfortable for her. Which of the following interventions would be most effective in getting her to eat? A. Local anesthetics or mouth washes applied to ulcers 30 minutes prior to meals B. A bland, moist, soft diet C. Staying with the client and providing distraction during meals D. Cleaning the mouth carefully with lemon glycerin swabs and milk of magnesia before meals Correct Answer: B Section: (none) Explanation Explanation/Reference:Explanation: (A) Local anesthetics do temporarily relieve the pain but leave an unpleasant taste and numb feeling that are not conductive to eating. (B) Such a diet is less irritating to the damaged mucosa and is easier for the child to tolerate. (C) This intervention is helpful if the child has only anorexia. It does not work if the type and texture of the food increase oral discomfort. (D) Lemon glycerin swabs and milk of magnesia dry the oral mucosa and should be avoided. QUESTION 381 A 15-year-old client was diagnosed as having cystic fibrosis at 8 months of age. He is in the hospital for a course of IV antibiotic therapy and vigorous chest physiotherapy. He has a poor appetite. The nurse can best help him to meet the desired outcome of consuming a prescribed number of calories by: A. Including the client in planning sessions to select the type of meal plan and foods for his diet B. Working with the nutritionist to devise a diet with significantly increased calories C. Selecting foods for the client's diet that are high in calories and instituting a strict calorie count D. Constantly providing him with chips, dips, and candies, because the number of calories consumed is more important than the quality of foods Correct Answer: A Section: (none) Explanation Explanation/Reference: Explanation: (A) The adolescent knows what he likes and will be more likely to eat if he has some control over his diet. (B) The nurses and nutritionist can plan an excellent diet, but it will not help the adolescent unless he eats it. (C) Eating is already a chore for this client. Adding a strict calorie count could make it even more burdensome. (D) Fats are particularly difficult for the cystic fibrosis client to digest. He does need a healthful diet, not just more calories. QUESTION 382 The most appropriate method of evaluating whether the diet of a child with cystic fibrosis is meeting his caloric needs is: A. Careful monitoring of weight loss or gain B. Carefully recording amounts and types of foods ingested C. Keeping a strict account of the number of calories ingested D. Keeping a careful account of the amount of pancreatic enzymes ingested Correct Answer: A Section: (none) Explanation Explanation/Reference: Explanation:(A) Consistent weight gain, even if it is slow, is an indication that the child is eating and digesting sufficient calories. (B) Recording how much the child eats is useful, but it is not an indicator of how well his body is using the foods consumed. (C) Counting calories will indicate how much he is eating, but it will not reflect whether or not the foods are properly digested. (D) Keeping track of the enzyme intake will indicate compliance with medication but not whether the child is getting sufficient calories. QUESTION 383 A client with cystic fibrosis exhibits activity intolerance related to the pulmonary problems associated with his disease. However, he needs to be encouraged to participate in daily physical exercise. The ultimate aim of exercise is to: A. Create a sense of well-being and self-worth B. Help him overcome respiratory infections C. Establish an effective, habitual breathing pattern D. Promote normal growth and development Correct Answer: C Section: (none) Explanation Explanation/Reference: Explanation: (A) Regular exercise does promote a sense of well-being and selfworth, but this is not the ultimate goal of exercise for this client. (B) Regular chest physiotherapy,not exercise per se, helps to prevent respiratory infections. (C) Physical exercise is an important adjunct to chest physiotherapy. It stimulates mucus secretion, promotes a feeling of well-being, and helps to establish a habitual breathing pattern. (D) Along with adequate nutrition and minimization of pulmonary complications, exercise does help promote normal growth and development. However, exercise is promoted primarily to help establish a habitual breathing pattern. QUESTION 384 As a nurse works with an adolescent with cystic fibrosis, the nurse begins to notice that he appears depressed and talks about suicide and feelings of worthlessness. This is an important factor to consider in planning for his care because: A. It may be a bid for attention and an indication that more diversionary activity should be planned for him B. No threat of suicide should be ignored or challenged in any way C. He needs to be observed carefully for signs that his depression has been relieved D. He needs to be confronted with his feelings and forced to work through them Correct Answer: B Section: (none) ExplanationExplanation/Reference: Explanation: (A) Threats of suicide should always be taken seriously. (B) This client has a life-threatening chronic illness. He may be concerned about dying or he may actually be contemplating suicide. (C) Sometimes clients who have made the decision to commit suicide appear to be less depressed. (D) Forcing him to look at his feelings may cause him to build a defense against the depression with behavioral or psychosomatic disturbances. QUESTION 385 A 9-month-old infant is being examined in the general pediatric clinic for a routine well-child checkup. His immunizations are up to date, and his mother reports that he has had no significant illnesses or injuries. Which of the following signs would lead the nurse to believe that he has had a cerebral injury? A. Hyperextension of the neck with evidence of pain on flexion B. Holding the head to one side and pointing the chin toward the other side C. Holding the head erect and in the midline when in a vertical position D. Significant head lag when raised to a sitting position Correct Answer: D Section: (none) Explanation Explanation/Reference: Explanation: (A) This position is indicative of a possible meningeal irritation or infection such as meningitis. (B) This position is seen most frequently in infants who have had aninjury to the sternocleidomastoid muscle. (C) Most infants aged 4 months and older are able to maintain this position. (D) Infants older than 6 months of age should not have significant head lag. This is a sign of cerebral injury and should be referred for further evaluation. QUESTION 386 During an examination, the nurse notes that an infant has diaper rash on the convex surfaces of his buttocks, inner thighs, and scrotum. Which of the following nursing interventions will be most effective in resolving the condition? A. Coating the inflamed areas with zinc oxide B. Using talcum powder on the inflamed areas to promote drying C. Removing the diaper entirely for extended periods of time D. Cleaning the inflamed area thoroughly with disposable wet "wipes" at each diaper change Correct Answer: C Section: (none) ExplanationExplanation/Reference: Explanation: (A) Zinc oxide is not usually applied to inflamed areas because it contributes to sweat retention. (B) Talcum powder is of questionable benefit and poses a hazard of accidental inhalation. (C) Removing the diaper and exposing the area to air and light facilitate drying and healing. (D) Infants may be sensitive to one or more agents in the wet "wipes." It is better to simply clean with a wet cloth. QUESTION 387 A 10-month-old infant's mother says that he takes fresh whole milk eagerly, but that when she offered him baby foods at 6 months of age, he pushed them out of his mouth. Because he has gained weight appropriately, she has quit trying to get him to eat other foods. The nurse's response is based on the knowledge that: A. Milk intake should be limited to no more than four 8-oz bottles per day and should be followed by iron-enriched cereal or other solid foods or juices B. Milk is an excellent food and will meet his nutritional needs adequately until he isready to eat solid foods C. It is acceptable to continue to give him whole milk and to delay giving solid foods as long as he takes a vitamin supplement daily D. He should be started on iron-enriched cereal, meat, vegetables, fruits, and juices prior to bottle feeds. Milk intake should be limited to 1 qt/day Correct Answer: D Section: (none) Explanation Explanation/Reference: Explanation: (A) If the infant is given the bottle first, he will be less likely to be hungry enough to eat the solid foods. (B) Milk is deficient in iron, vitamin C, zinc, and fluoride. It does not provide an adequate diet. (C) The vitamin supplement will help, but the infant needs an iron supplement. (D) Giving the solid food when the infant is hungriest will increase the likelihood that he will eat. The more solid food he takes, the less milk he will desire. QUESTION 388 A 9-month-old infant was diagnosed with nonorganic failure to thrive. During her hospitalization, primary nurses were assigned to initiate all infant feedings. The infant's parents question why they cannot feed their own child. Which of the following responses would be most appropriate by the nurse? A. By assigning the same nurses to the child, the nurses can begin to learn the infant's cues and feeding behaviors. B. The same nurses will prevent parental fatigue and frustration. C. The same nurses will prevent infant fatigue and frustration. D. Primary nurses will ensure privacy. Correct Answer: A Section: (none) ExplanationExplanation/Reference: Explanation: (A) Consistent primary care nurses can better interpret infant cues and note feeding behaviors. (B) In nonorganic failure to thrive the parent-infant dyad has already experienced difficulties in the relationship. These parents may already feel dissatisfied and frustrated. The primary nurse would be unable to prevent this. (C) Assigning a primary nurse does not ensure that infant fatigue and frustration will not occur or can be prevented. (D) Providing privacy does not ensure a change infeeding behavior. QUESTION 389 The parents of a 2-year-old child are ready to begin toilet training activities with him. His parents feel he is ready to train because he is now 2 years old. What would the nurse identify as readiness in this child? A. Patience by the child when wearing soiled diapers B. Communicating the urge to defecate or urinate C. The child awakening wet from his naps D. The age at which the child's siblings were trained Correct Answer: B Section: (none) Explanation Explanation/Reference: Explanation: (A) Children experience impatience with soiled diapers when readiness for training is apparent. They often desire to be changed immediately. (B) A child must be able to use verbal or nonverbal skills to communicate needs. (C) A readiness indicator would be awaking dry from naps. (D) The age at which a sibling was toilet trained has no implications for training this child. QUESTION 390 A 5-year-old child is hospitalized for an acute illness. The nurse encourages the family to bring her favorite objects from home. What is the nurse's rationale? A. To reduce fear of the unknown B. To keep the child calm C. To establish a trusting relationship D. To prevent or minimize separation anxiety Correct Answer: D Section: (none) ExplanationExplanation/Reference: Explanation: (A) Objects from home do not reduce fear of the unknown. Children need explanations, reassurance, and preparation for the unknown. Also, parental presence can promote comfort and feelings of security. (B) A calm, relaxed, and reassuring manner will assist in calming the child. The child's objects from home will not assist in calming the child. (C) A trusting relationship is based on the quality of the nurse-client relationship. Objects from home have no impact. (D) Favorite objects from home assist in creating a familiar setting. Also, these objects may prevent or minimize separation from the child's usual routine and family support. QUESTION 391 A 3-year-old child was hospitalized for acute laryngotracheobronchitis. During her hospitalization, the child was placed under an oxygen mist tent. The nurse's frequent monitoring of the child's temperature frightened her parents. Which response by the nurse would be most appropriate? A. Monitoring the temperature prevents undue chilling. B. Rapid temperature elevations can occur in children. C. Checking the temperature will prevent febrile seizures. D. Taking the child's temperature can prevent airway obstruction. Correct Answer: A Section: (none) Explanation Explanation/Reference: Explanation: (A) The refrigerated cool mist tent creates a cool, moist environment. The child as well as bedding and clothing may become dampened. Monitoring the temperature of the child will ensure warmth and prevent chilling. (B) Only a low-grade fever is expected in laryngotracheobronchitis. (C) Febrile seizures are not expected with the low-grade fever. (D) Inflammation of the mucosal lining in the respiratory tract can cause airway obstruction. However, monitoring the child'stemperature would not prevent airway obstruction. QUESTION 392 A school-age child with asthma is ready for discharge from the hospital. His physician has written an order to continue the theophylline given in the hospital as an oral home medication. Immediately prior to discharge, he complains of nausea and becomes irritable. His vital signs were normal except for tachycardia. What first nursing actions would be essential in this situation? A. Hold the child's discharge for 1 hour. B. Notify the physician immediately. C. Discharge the child as the physician ordered. D. Administer an antiemetic as necessary. Correct Answer: BSection: (none) Explanation Explanation/Reference: Explanation: (A) Holding the child's discharge alone does not address the client's problem. (B) Nausea, tachycardia, and irritability are all symptoms of theophylline toxicity. The physician should benotified immediately so that a serum theophylline level can be ordered. Theophylline dose should be withheld until the physician is notified. (C) The child must be evaluated for theophylline toxicity before any discharge. (D) Cause of the nausea should be investigated before the administration of an antiemetic. QUESTION 393 A neonate was admitted to the hospital with projectile vomiting. According to the parents, the baby had experienced vomiting episodes after feeding for the last 2 days. A medical diagnosis of hypertrophic pyloric stenosis was made. On assessment, the infant had poor skin turgor, sunken eyeballs, dry skin, and weight loss. Identify the number-one priority nursing diagnosis. A. Fluid volume deficit B. Altered nutrition C. Altered bowel elimination D. Anxiety Correct Answer: A Section: (none) Explanation Explanation/Reference: Explanation: (A) Fluid volume deficit is the major problem. Symptoms of dehydration are evident. The effects of fluid and electrolyte balance may be life threatening. Rehydration can be accomplished effectively through IV fluids and electrolytes. (B) Vomiting may also signal a nutritional problem. However, the nutritional problem would be secondary to fluid and electrolyte disturbances. The infant may also be placed on NPO status. (C) With vomiting, a decrease in the size and number of stools is expected. (D) The infant cannot verbalize feelings of anxiety. Anxiety would not be an appropriate diagnosis. QUESTION 394 A baby who was diagnosed with pyloric stenosis has continued to have projectile vomiting. With prolonged vomiting, the infant is prone to: A. Respiratory acidosis B. Respiratory alkalosis C. Metabolic acidosisD. Metabolic alkalosis Correct Answer: D Section: (none) Explanation Explanation/Reference: Explanation: (A) Respiratory acidosis is the result of problematic ventilation. Plasma pH decreases, while plasma PCO2 and plasma HCO3 increase. (B) Respiratory alkalosis results from increased respiratory rate and depth. Plasma pH increases, while plasma PCO2 and plasma HCO3 decrease. (C) Metabolic acidosis occurs when there is strong acid gain in the body. Plasma pH, PCO2, and HCO3 decrease. (D) Increased risk for metabolic alkalosis is due to a loss of hydrogen ions; depletion of potassium, sodium, and chloride when vomiting occurs. Plasma pH and plasma PCO2 increase; plasma HCO3 may decrease and then increase to compensate. QUESTION 395 Parents of young children often need anticipatory guidance from the nurse. Parents may have little knowledge regarding growth and development. Which of the following toys and activities would the nurse suggest as appropriate for a toddler? A. Cutting, pasting, string beads, music, dolls B. Mobiles, rattle, squeeze toys C. Pull-toys, large ball, dolls, sand and water play, music D. Simple card games, puzzles, bicycle, television Correct Answer: C Section: (none) Explanation Explanation/Reference: Explanation: (A) These activities are suited for the preschool-age child (35 years old). The activities are not safe for a toddler. (B) Infants (01 year) like these toys. (C) These activities provide the toddler (13 years old) with a variety of physical activities for play. (D) The toddler lacks the physical and cognitive abilities for these activities. The tasks are far better suited for the school-age child. QUESTION 396 A 5-year-old child cries continually in her bed. Her parents have been unsuccessful in assisting her in expressing her feelings. Which activity should the nurse provide the child to assist her in expressing her feelings? A. Books with colorful pictures B. Music C. Riding toysD. Puppets Correct Answer: D Section: (none) Explanation Explanation/Reference: Explanation: (A) Books increase cognition, assist with fine motor skills, and augment language development. (B) Music provides auditory stimulation and large-muscle activity. (C) Riding toys provide large-muscle activity. (D) Puppets allow expression of feelings and fearsthat otherwise could not be directly communicated. QUESTION 397 During his hospitalization, a 3-year-old child has become unusually aggressive in his play activities. His parents report this change in behavior to the primary nurse. How could the nurse explain the child's change in behavior? A. Deep-seated feelings of hostility B. A lack of interest in socializing C. Usual behavior for this child D. A coping response Correct Answer: D Section: (none) Explanation Explanation/Reference: Explanation: (A) Unusually aggressive behavior does not indicate a deepseated problem. (B) A lack of social interest results in poor participation in play activities with peers. Aggression would not be an expected behavior. (C) The aggressive behavior was newly developed and not a routine behavior. (D) Play provides the child with opportunities for coping and adaptation. Aggression during the play activities would indicate a coping response. QUESTION 398 Following her surgery, a 5-year-old child will return to the pediatric unit with a long-arm cast. She experienced a supracondylar fracture of the humerus near the elbow. Which nursing action is most essential during the first 24 hours after surgery and cast application? A. Mobilization of the child B. Discharge teaching C. Pain managementD. Assessment of neurovascular status Correct Answer: D Section: (none) Explanation Explanation/Reference: Explanation: (A) Mobilization is important but not absolutely essential. (B) Discharge teaching should be initiated prior to surgery as well as during the postoperative period. (C) Assessment and management of pain are necessary and high in priority. (D) Neurovascular status of the extremity is of primary importance. The risk of circulatory impairment exists with any cast application. This type of fracture is common in children. A high incidence of neurovascular complications exists with fractures near the elbow. QUESTION 399 A 9-month-old infant visits her pediatrician for a routine visit. A developmental assessment was initiated by the nurse. Which skill would cause the nurse to be concerned about the infant's developmental progression? A. She sits briefly alone with assistance. B. She creeps and crawls. C. She pulls herself to her feet with help. D. She stands while holding onto furniture. Correct Answer: A Section: (none) Explanation Explanation/Reference: Explanation: (A) The 9-month-old infant can sit alone for long periods. By the age of 6 months, many infants can pull themselves to a sitting position. (B, C, D) This skill represents normal development. QUESTION 400 Children often experience visual impairments. Refractive errors affect the child's visual activity. The main refractive error seen in children is myopia. The nurse explains to the child's parents that myopia may also be described as: A. Cataracts B. Farsightedness C. NearsightednessD. Lazy eye Correct Answer: C Section: (none) Explanation Explanation/Reference: Explanation: (A) Cataracts are not considered refractive errors. Cataracts canbe described as opacity of the lens. (B)Hyperopiais the term forfarsightedness. One can see objects at a distance more clearlythan close objects. (C)Myopiais the term for nearsightedness.Objects that are close in distance are more clearly seen. (D) Lazyeye refers to strabismus or misalignment of the eyes. QUESTION 401 A client is experiencing visual problems at school. She has complained of difficulty seeing the blackboard and squinting. She no longer likes to participate in physical activities such as softball. The client has displayed possible classic symptoms of which refractive error? A. Astigmatism B. Hyperopia C. Myopia D. Amblyopia Correct Answer: C Section: (none) Explanation Explanation/Reference: Explanation: (A) Visual images are blurred and distorted. (B) Symptoms are headaches, burning eyes, fatigue, squinting, and difficulty reading. (C) These symptoms are classic for myopia. (D) Amblyopia is not a refractive error. It is a loss of vision in one or both eyes. QUESTION 402 An 18-year-old client enters the emergency room complaining of coughing, chest tightness, dyspnea, and sputum production. On physical assessment, the nurse notes agitation, nasal flaring, tachypnea, and expiratory wheezing. These signs should alert the nurse to: A. A tension pneumothorax B. An asthma attack C. PneumoniaD. Pulmonary embolus Correct Answer: B Section: (none) Explanation Explanation/Reference: Explanation: (A) A tension pneumothorax is an accumulation of air in the pleural space. Important physical assessment findings to confirm this condition include cyanosis, jugular vein distention, absent breath sounds on the affected side, distant heart sounds, and lowered blood pressure. (B) Asthma is a disorder in which there is an airflow obstruction in the bronchioles and smaller bronchi secondary to bronchospasm, swelling of mucous membranes, and increased mucus production.Physical assessment reveals some important findings: agitation, nasal flaring, tachypnea, and expiratory wheezing. (C) Pneumonia is an acute bacterial or viral infection that causes inflammation of the lung in the alveolar and interstitial tissue and results in consolidation. Specific assessment findings to confirm this condition include decreased chest expansion caused by pleuritic pain, dullness on percussion over consolidated areas, decreased breath sounds, and increased vocal fremitus. (D) A pulmonary embolus is the passage of a foreign substance (blood clot, fat, air, or amniotic fluid) into the pulmonary artery or its branches, with subsequent obstruction of blood supply to lung tissue. Specific assessment findings that confirm this condition include tachypnea, tachycardia, crackles (rales), transient friction rub, diaphoresis, edema, and cyanosis. QUESTION 403 The nurse is assisting a 4th-day postoperative cholecystectomy client in planning her meals for tomorrow's menu. Which vitamin is the most essential in promoting tissue healing? A. Vitamin C B. Vitamin B1 C. Vitamin D D. Vitamin A Correct Answer: A Section: (none) Explanation Explanation/Reference: Explanation: (A) Vitamin C (ascorbic acid) is essential in promoting wound healing and collagen formation. (B) Vitamin B1 (thiamine) maintains normal gastrointestinal (GI) functioning, oxidizes carbohydrates, and is essential for normal functioning of nervous tissue. (C) Vitamin D regulates absorption of calcium and phosphorus from the GI tract and helps prevent rickets. (D) Vitamin A is necessary for the formation and maintenance of skin and mucous membranes. It is also essential for normal growth and development of bones and teeth. QUESTION 404As a postoperative cholecystectomy client completes tomorrow's dinner menu, the nurse knows that one of the following meal choices will best provide the essential vitamin(s) necessary for proper tissue healing? A. Liver, white rice, spinach, tossed salad, custard pudding B. Fish fillet, carrots, mashed potatoes, butterscotch pudding C. Roast chicken, gelatin with sliced fruit D. Chicken breast fillet in tomato sauce, potatoes, mustard greens, orange and strawberry slices Correct Answer: D Section: (none) Explanation Explanation/Reference: Explanation: (A) This meal choice provides more of the vitamins A, D, and K than of vitamin C. (B) This meal choice provides more of the vitamins A, B12, and D than of vitamin C. (C) This meal choice provides more of the vitamins A, B1 (thiamine), niacin, and microminerals than of vitamin C. (D) This meal choice provides foods rich in vitamin C, which are essential in tissue healing. QUESTION 405 A 71-year-old client fell and injured her left leg while cooking in the kitchen. Her husband calls the ambulance, and she is taken to the emergency department at a local hospital. Xray reports confirm that she has an intertrochanteric fracture of the left femur. Her left leg will require skeletal traction initially and then surgery. The nurse knows that this type of traction will be used: A. By inserting pins to provide steady pull on the bone B. To suspend the leg in a sling without pull on the extremity C. Intermittently to place a pull over the pelvis and lower spine D. With weights at both ends of the bed to maintain pull on the upper extremity Correct Answer: A Section: (none) Explanation Explanation/Reference: Explanation: (A) Skeletal traction is the application of traction directly to bone with the use of pins and wires or tongs for the purpose of providing a strong, steady, continuous longitudinal pull on the bone. It is indicated for preoperative immobilization and positioning of hip and femur fractures. (B) A type of skeletal traction (balanced suspension with a Thomas splint and Pearson attachment) uses a sling to support the extremity, but it also uses weights to provide a strong, steady continuouspull on the extremity. A sling is used instead of pins. (C) Pelvic traction provides an intermittent pull over the pelvis and bone, whereas skeletal traction is continuous.Pelvic traction does not use pins. (D) Skeletal traction uses weights at the end of the bed to provide a continuous pull on long bones. Weights are not applied to both ends of the bed. QUESTION 406 Pin care is a part of the care plan for a client who is in skeletal traction. When assessing the site of pin insertion, which one of the following findings would the nurse know as an indicator of normal wound healing? A. Exudate B. Crust C. Edema D. Erythema Correct Answer: B Section: (none) Explanation Explanation/Reference: Explanation: (A) Exudate (moist, active drainage) is a clinical sign of wound infection. (B) Crust (dry, scaly) is part of the normal stages of wound healing and should not be removed from around the pin site. It usually sloughs off after the underlying tissue has healed. (C) Edema (swelling) is a clinical sign of wound infection. (D) Erythema (redness) is a clinical sign of wound infection. QUESTION 407 A 47-year-old client comes to the emergency department complaining of moderate flank, abdominal, and testicular pain with nausea of 4 hours' duration. After physical examination and obtaining the client's history, the physician suspects urethral obstruction by calculi. The nurse realizes that the physician will order which one of the following diagnostic studies to best confirm the diagnosis? A. Cystoscopy B. Kidneys, ureter, bladder, x-ray of abdomen C. Intravenous pyelogram with excretory urogram D. Ureterolithotomy Correct Answer: C Section: (none) Explanation Explanation/Reference: Explanation: (A) Cystoscopy is an endoscopic procedure that uses an instrument (a cystoscope) to visualize the internal bladder and ureter structures and to capture andremove an obstructing stone. (B) Kidney, ureter, bladder x-ray is used to outline gross structural changes in the kidneys, ureter, and bladder and will determine the general location of a stone. (C) An intravenous pyelogram with excretory urogram is used to visualize the kidneys, kidney pelvis, ureters, and bladder. This procedure is used specifically to determine whether urethral obstruction is partial or complete; it shows the exact location of the stone and dilation of the ureter above the stone. (D) Ureterolithotomy is a surgical procedure in which the ureter is incised and the stone is manually removed because the stone is unable to pass through the ureter independently. QUESTION 408 An obstructing stone in the renal pelvis or upper ureter causes: A. Radiating pain into the urethra with labia pain experienced in females or testicular pain in males B. Urinary frequency and dysuria C. Severe flank and abdominal pain with nausea, vomiting, diaphoresis, and pallor D. Dull, aching, back pain Correct Answer: C Section: (none) Explanation Explanation/Reference: Explanation: (A) Radiating pain in the urethra in both sexes, extending into the labia in females and into the testicle or penis in the male, indicates a stone in the middle or lower segment of the ureter. (B) Urinary frequency and dysuria are caused by a stone in the terminal segment of the ureter withinthe bladder wall. (C) An obstructing stone in the renal pelvis or upper ureter causes severe flank and abdominal pain with nausea, vomiting, diaphoresis, and pallor. (D) Dull and aching pain may indicate early stages of hydronephrosis. Also, a stone in the renal pelvis or upper ureter causes severe flank and abdominal pain. QUESTION 409 A client who has gout is most likely to form which type of renal calculi? A. Struvite stones B. Staghorn calculi C. Uric acid stones D. Calcium stones Correct Answer: C Section: (none) Explanation Explanation/Reference: Explanation:(A) The presence of urinary tract infection is a factor in the formation of struvite stones. (B) Staghorn calculi is the other name for struvite stones associated with urinary tract infection. (C) Clients who have gout form uric acid stones. (D) Clients who have increased urinary excretion of calcium form calcium stones. QUESTION 410 A 75-year-old client is hospitalized with pneumonia caused by gram-positive bacteria. Which one of the following best describes a gram-positive bacterial pneumonia? A. Klebsiellapneumonia B. Pneumococcal pneumonia C. Legionella pneumophilapneumonia D. Escherichia colipneumonia Correct Answer: B Section: (none) Explanation Explanation/Reference: Explanation: (A)Klebsiellapneumonia is caused by gram-negative bacteria. (B) Pneumococcal pneumonia is caused by gram-positive bacteria. (C)Legionella pneumophilapneumonia is a nonbacterial pneumonia. (D)E. colipneumonia is caused by gram-negative bacteria. QUESTION 411 The nurse caring for a client who has pneumonia, which is caused by a gram-positive bacteria, inspects her sputum. Because the client's pneumonia is caused by a gram-positive bacteria, the nurse experts to find the sputum to be: A. Bright red with streaks B. Rust colored C. Green colored D. Pink-tinged and frothy Correct Answer: B Section: (none) Explanation Explanation/Reference: Explanation: (A) Bright red sputum with streaks is associated with pneumonia caused by gram-negative bacteria, such asKlebsiellapneumonia. (B) Pneumococcal pneumonia,caused by gram-positive bacteria, has a characteristic productive cough with green or rust-colored sputum. (C) Green- colored sputum is more characteristic ofPseudomonasthan of gram-positive bacterial pneumonia. (D) Pink-tinged and frothy sputum is more characteristic of pulmonary edema than of gram- positive bacterial pneumonia. QUESTION 412 The nurse who is caring for a client with pneumonia assesses that the client has become increasingly irritable and restless. The nurse realizes that this is a result of: A. Prolonged bed rest B. The client's maintaining a semi-Fowler position C. Cerebral hypoxia D. IV fluids of 2.53 liters in 24 hours Correct Answer: C Section: (none) Explanation Explanation/Reference: Explanation: (A) Maintaining bed rest helps to decrease the O2 needs of the tissues, which decreases dyspnea and workload on the respiratory system. (B) The semi-Fowler orhigh-Fowler position is necessary to aid in lessening pressure on the diaphragm from the abdominal organs, which facilitates comfort and easier breathing patterns. (C) Cerebral hypoxia causes the client with pneumonia to be increasingly irritable and restless and results from the client not obtaining enough O2 to meet metabolic needs. (D) Proper hydration facilitates liquefaction of mucus trapped in the bronchioles and alveoli and enhances expectoration. Unless contraindicated, a reasonable amount of IV fluids to be administered is at least 2.53 liters in a 24-hour period. QUESTION 413 A 22-year-old client who is being seen in the clinic for a possible asthma attack stops wheezing suddenly as the nurse is doing a lung assessment. Which one of the following nursing interventions is most important? A. Place the client in a supine position. B. Draw a blood sample for arterial blood gases. C. Start O2 at 4 L/min. D. Establish a patent airway. Correct Answer: D Section: (none) Explanation Explanation/Reference:Explanation: (A) During impending respiratory failure or asthmatic complications, the client is placed in the high-Fowler position to facilitate comfort and promote optimal gas exchange. (B) Arterial blood gases are monitored in the treatment of respiratory failure during an asthma attack, but it is not an initial intervention. (C) O2 therapy is used during an asthma attack, but it is not the initial intervention. The usual prescribed amount is a cautiously low flow rate of 12 L/min. (D) Wheezing is a characteristic clinical finding during an asthma attack. If wheezing suddenlyceases, it usually indicates a complete airway obstruction and requires immediate treatment for respiratory failure or arrest. QUESTION 414 A 49-year-old obese woman has been admitted to the general surgery unit with choledocholithiasis. As the nurse is admitting her to the unit, she states, "The doctor said I have stones that need to be removed; where are they?" The nurse knows that the best explanation for this is to tell her that: A. There are stones present in her gallbladder B. There are stones present in her kidneys C. There are stones present in her common bile duct D. There are no stones, but her gallbladder is irritated and caused her nausea, vomiting, and pain Correct Answer: C Section: (none) Explanation Explanation/Reference: Explanation: (A)Cholelithiasisis the correct term used to describe the presence of stones in the gallbladder. (B)Nephrolithiasis,orrenal calculi,is the correct term used to describe the presence of stones in the kidney. (C)Choledocholithiasisis the correct term used to describe the presence of stones in the common bile duct. (D)Cholecystitisis the correct term used to describe inflammation of the gallbladder and can be associated with cystic duct obstructions from impacted stones. QUESTION 415 A 48-year-old client is being seen in her physician's office for complaints of indigestion, heartburn, right upper quadrant pain, and nausea of 4 days' duration, especially after meals. The nurse realizes that these symptoms may be associated with cholecystitis and therefore would check for which specific sign during the abdominal assessment? A. Cullen's sign B. Rebound tenderness C. Murphy's sign D. Turner's sign Correct Answer: C Section: (none)Explanation Explanation/Reference: Explanation: (A) This sign is a faint blue discoloration around the umbilicus found in clients who have hemorrhagic pancreatitis. (B) This sign indicates areas of inflammation within the peritoneum, such as with appendicitis. It is a deep palpation technique used on a nontender area of the abdomen, and when the palpating hand is removed suddenly, the client experiences a sharp, stabbing pain at an area of peritoneal inflammation. (C) This sign is considered positive with acute cholecystitiswhen the client is unable to take a deep breath while the right upper quadrant is being deeply palpated. The client will elicit a sudden, sharp gasp, which means the gallbladder is acutely inflamed. (D) This is a sign of acute hemorrhagic pancreatitis and manifests as a green or purple discoloration in the flanks. QUESTION 416 When caring for a postoperative cholecystectomy client, the nurse assesses patency and documents drainage of the T-tube. The nurse recognizes that the expected amount of drainage during the first 24 hours postoperatively is: A. 50100 mL B. 200300 mL C. 300500 mL D. 10001200 mL Correct Answer: C Section: (none) Explanation Explanation/Reference: Explanation: (A) During the first 24 hours after surgery, the drainage is normally 300500 mL and then decreases to about 200 mL in 24 hours during the next 34 days. (B) This range is the amount of drainage after the first 24 hours postoperatively. During the first 24 hours, it is 300-500 mL. (C) During the first 24 hours after surgery, thisrange is the expected amount of drainage. (D) The expected amount of drainage during the first 24 hours is 300500 mL. An output of >500 mLshould be reported to the physician, because an occlusion of some type, caused by a retained gallstone or an inflammatory process within the biliary drainage system, is evident. QUESTION 417 The nurse recognizes that a client with the diagnosis of cholecystitis and cholelithiasis would expect to have stools that are: A. Clay or gray colored B. Watery and loose C. Bright-red streaked D. BlackCorrect Answer: A Section: (none) Explanation Explanation/Reference: Explanation: (A) Clients who have obstruction in the biliary tract so that bile is not released into the duodenum experience a change in stools from brown to gray or clay colored. (B) This type of stool can occur with other GI problems, such as bacterial or viral infections, and other disease problems, and is not a common finding with biliaryobstructions such as cholecystitis and cholelithiasis. (C) This type of stool is usually associated with a GI or bowel problem, such as lower GI bleeding, rather than with biliary obstructions. (D) This type of stool is usually associated with a GI or bowel problem, such as upper GI bleeding, rather than with biliary obstructions. QUESTION 418 A 52-year-old client who underwent an exploratory laparotomy for a bowel obstruction beginsto complain of hunger on the third postoperative day. His nasogastric (NG) tube was removed this morning, and he has an IV of D5W with 0.45% normal saline running at 125 mL/hr. He asks when he can get rid of his IV and start eating. The nurse recognizes that he will be able to begin taking oral fluids and nourishment when: A. It is determined that he has no signs of wound infection B. He is able to eat a full meal without evidence of nausea or vomiting C. The nurse can detect bowel sounds in all four quadrants D. His blood pressure returns to its preoperative baseline level or greater Correct Answer: C Section: (none) Explanation Explanation/Reference: Explanation: (A) The absence of wound infection is related to his surgical wound and not to postoperative GI functioning and return of peristalsis. (B) Routine postoperative protocol involves detection of bowel sounds and return of peristalsis before introduction of clear liquids, followed by progression of full liquids and a regular diet versus a full regular meal first. (C) Routine postoperative protocol for bowel obstruction is to assess for the return of bowel sounds within 72 hours after major surgery, because that is when bowel sounds normally return. If unable to detect bowel sounds, the surgeon should be notified immediately and have the client remain NPO. (D) Routine postoperative protocol for bowel obstruction and other major surgeries involves frequent monitoring of vital signs in the immediate postoperative period (in recovery room) and then every 4 hours, or more frequently if the client is unstable, on the nursing unit. This includes assessing for signs of hypovolemic shock. Vital signs usually stabilize within the first 24 hours postoperatively. QUESTION 419 A 47-year-old client has been admitted to the general surgery unit for bowel obstruction. The doctor has ordered that an NG tube be inserted to aid in bowel de- compression. When preparing to insert a NG tube, the nurse measures from the:A. Lower lip to the shoulder to the upper sternum B. Tip of the nose to the lower lip to the umbilicus C. End of the tube to the first measurement line on the tube D. Tip of the nose to the ear lobe to the xiphoid process or midepigastric area Correct Answer: D Section: (none) Explanation Explanation/Reference: Explanation: (A) This measurement is _50 cm (4849 cm). Fifty centimeters is considered the length necessary for the distal end of the tube to be in place in the stomach. This measurement is too short. (B) This measurement is _50 cm (4748 cm). Fifty centimeters is considered the lengthnecessary for the distal end of the tube to be in place in the stomach. This measurement is too short. (C) This measurement gives an approximate indication of the length necessary for the distal end of the tube to be in place in the stomach, but it is not as accurate as actually measuring the client (nose-earxiphoid). (D) This is the correct measurement of 50 cm from the tip of the client's nose to the tip of the earlobe to the xiphoid process (called the NEX [nose-ear-xiphoid] measurement). It is approximately equal to the distance necessary for the distal end of the tube to be located in the correct position in the stomach. QUESTION 420 A 65-year-old client who has a new colostomy is preparing for discharge from the hospital. As part of the instructions on colostomy care, the nurse explains to the client that to regulate the bowel, colostomy irrigation should be performed at the same time each day. The best time is: A. After meals B. Before meals C. Every 2 hours D. At bedtime Correct Answer: A Section: (none) Explanation Explanation/Reference: Explanation: (A) Bowel movements should be regulated at a specific time each day to prevent "accidents." Irrigating after meals takes advantage of the gastrocolic reflex and time of increased peristalsis, so better results may be produced. After meals is the normal time that peristalsis begins in most persons and evacuation of feces occurs. (B) Irrigating before meals may cause poor results because of decreased gastrocolic reflex and decreased peristalsis. (C) Irrigating a colostomy every 2 hours may produce hyperactivity of the bowel, leading to irritation and diarrhea. This would not aid in regulation of the bowel. (D) If irrigation of a colostomy were done at bedtime, there is greater chance of having an "accident" during sleep. This would not be an advantageous practice of bowel regulation.QUESTION 421 A 72-year-old client with a new colostomy is being evaluated at the clinic today for constipation. When discussing diet with the client, the nurse recognizes that which one of the following foods most likely caused this problem? A. Fried chicken B. Eggs C. Tapioca D. Cabbage Correct Answer: C Section: (none) Explanation Explanation/Reference: Explanation: (A) Fried, greasy food, such as fried chicken, will produce diarrhealike stools in individuals with all types of GI ostomies. (B) Eggs will cause odor-producing stoolsin individuals with all types of GI ostomies. (C) Tapioca and rice products will cause constipation in individuals with all types of GI ostomies. (D) Cabbage will cause odor-producing and flatusproducing stools in individuals with all types of GI ostomies. QUESTION 422 When giving discharge instructions to a 24-year-old client who had a short-arm cast applied for a fractured right ulna, the nurse recognizes the importance of telling him that the drying time for a plaster of Paris cast is approximately: A. 30 minutes B. 14 hours C. 1224 hours D. 2472 hours Correct Answer: D Section: (none) Explanation Explanation/Reference: Explanation: (A) Synthetic cast materials harden in 315 minutes. Weight bearing is permitted in 1530 minutes. Drying time for plaster of Paris is about 2472 hours. (B, C) Plaster of Paris cast materials are heavier than synthetic materials and require a drying time of 2472 hours. Synthetic materials dry within 30 minutes. (D) Plaster of Paris cast materials are heavier thansynthetic materials and require a longer period to set and dry. Even though setting time (hardening) is only 315 minutes, the dryingtime for plaster of Paris is 2472 hours. This depends on the size and thickness of the cast, exposure to air, and humidity in the air. QUESTION 423 A 58-year-old client on a general surgery unit is scheduled for transurethral resection of the prostate (TURP) in 2 hours. The nurse explains to the client that this procedure means: A. Removal of the prostate tissue by way of a lower abdominal midline incision through the bladder and into the prostate gland B. Removal of prostate tissue by a resectoscope that is inserted through the penile urethra C. Removal of the prostate tissue by an open surgical approach through an incision between the ischial tuberosities, the scrotum, and the rectum D. Removal of prostate tissue by an open surgical approach through a low horizontal incision, bypassing the bladder, to the prostate gland Correct Answer: B Section: (none) Explanation Explanation/Reference: Explanation: (A) This describes a suprapubic (transvesical) prostatectomy procedure. (B) This is the correct description of a TURP procedure. (C) This describes a perineal prostatectomy procedure. (D) This describes a retropubic (extravesical) prostatectomy procedure. QUESTION 424 A postoperative TURP client returns from the recovery room to the general surgery unit and is in stable condition. One hour later the nurse assesses him and finds him to be confused and disoriented. She recognizes that this is most likely caused by: A. Hypovolemic shock B. Hypokalemia C. Hypernatremia D. Hyponatremia Correct Answer: D Section: (none) Explanation Explanation/Reference: Explanation: (A) Early signs of hypovolemic shock include hypotension, tachycardia, tachypnea, pallor, and diaphoresis. (B) Early signs of potassium depletion include muscular weakness or paralysis, tetany, postural hypotension, weak pulse, shallow respirations, apathy, weak voice, and electrocardiographic changes. (C) Early signs of an elevated sodium level include dry oral mucous membranes, marked thirst, hypertension, tachycardia, oliguria or anuria, anxiety, and agitation. (D) This answer is correct. Important early clinical findings of a decreased sodium concentration include confusion and disorientation. Hyponatremia can occur after a TURP becauseabsorption during surgery through the prostate veins can increase circulating blood volume and decrease sodium concentration. QUESTION 425 A postoperative TURP client is ordered continuous bladder irrigations. Later in the evening on the first postoperative day, he complains of increasing suprapubic pain. When assessing the client, the nurse notes diminished flow of bloody urine and several large blood clots in the drainage tubing. Which one of the following should be the initial nursing intervention? A. Call the physician about the problem. B. Irrigate the Foley catheter. C. Change the Foley catheter. D. Administer a prescribed narcotic analgesic. Correct Answer: B Section: (none) Explanation Explanation/Reference: Explanation: (A) The physician should be notified as problems arise, but in this case, the nurse can attempt to irrigate the Foley catheter first and call the physician if irrigation is unsuccessful. Notifying the physician of problems is a subsequent nursing intervention. (B) This answer is correct. Assessing catheter patency and irrigating as prescribed are the initial priorities to maintain continuous bladder irrigation. Manual irrigation will dislodge blood clots that have blocked the catheter and prevent problems of bladder distention, pain, and possibly fresh bleeding. (C) The Foley catheter would not be changed as an initial nursing intervention, but irrigation of the catheter should be done as ordered to dislodge clots that interfere with patency. (D) Even though the client complains of increasing suprapubic pain, administration of a prescribed narcotic analgesic is not the initial priority. The effect of the medication may mask the symptoms of a distended bladder and lead to more serious complications. QUESTION 426 A postoperative prostatectomy client is preparing for discharge from the hospital the next morning. The nurse realizes that additional instructions are necessary when he states: A. "If I drink 10 to 12 glasses of fluids each day, that will help to prevent any clot formation in my urine." B. "The isometric exercises will help to strengthen my perineal muscles and help me control my urine." C. "If I feel as though I have developed a fever, I will take a rectal temperature, which is the most accurate." D. "I do not plan to do any heavy lifting until I visit my doctor again." Correct Answer: C Section: (none) ExplanationExplanation/Reference: Explanation: (A) This is correct health teaching. Drinking 1012 glasses of clear liquid will help increase urine volumes and prevent clot formation. (B) This is correct health teaching. These types of exercises are prescribed by physicians to assist postprostatectomy clients to strengthen their perineal muscles. (C) This action is not recommended post-TURP because of the close proximity of the prostate and rectum. (D) This is correct healthcare teaching. The client should limit walking long distances, lifting heavy objects, or driving a car until these activities are cleared by the physician at the first office visit. QUESTION 427 A 67-year-old postoperative TURP client has hematuria. The nurse caring for him reviews his postoperative orders and recognizes that which one of the following prescribed medications would best relieve this problem? A. Acetaminophen suppository 650 mg B. Meperidine 50 mg IM C. Promethazine 25 mg IM D. Aminocaproic acid (Amicar) 6 g/24 hr Correct Answer: D Section: (none) Explanation Explanation/Reference: Explanation: (A) Acetaminophen (Tylenol) has analgesic and antipyretic actions approximately equivalent to those of aspirin. It produces analgesia possibly by action on the peripheral nervous system. It reduces fever by direct action on the hypothalamus heat-regulating center with consequent peripheral vasodilation. It is generally used for temporary relief of mild to moderate pain, such as a simple headache, minor joint and muscle pains, and control of fever. (B) Meperidine is a narcotic agonist analgesic with properties similar to morphine except that it has a shorter duration of action and produces less depression of urinary retention and smooth muscle spasm. It is used for moderate to severe pain, for a preoperative medication, for support of anesthesia, and for obstetrical analgesia. In a postoperative TURP client, it would be used in conjunction with other medications for relief of moderate to severe pain, but not specifically for bladder spasms associated with TURP surgery. (C) Promethazine hydrochloride is an antihistamine, antiemetic preparation. It exerts antiserotonin, anticholinergic, and local anesthetic actions. It is used for symptomatic relief of various allergic conditions, motion sickness, nausea, and vomiting. It is used for preoperative, postoperative, and obstetrical sedation and as an adjunct to analgesics for control of pain. (D) This answer is correct because aminocaproic acid is prescribed specifically for hematuria. Aminocaproic acid is excreted in the urine. The nurse should be alert for possible signs of thrombosis, particularly in the extremities. QUESTION 428 A 52-year-old client is scheduled for a small-bowel resection in the morning. In conjunction with other preoperative preparation, the nurse is teaching her diaphragmatic breathing exercises. She will teach the client to: A. Inhale slowly and deeply through the nose until the lungs are fully expanded, hold the breath a couple of seconds, and then exhale slowly through the mouth. Repeat 23 more times to complete the series every 12 hours while awakeB. Purse the lips and take quick, short breaths approximately 1820 times/min C. Take a large gulp of air into the mouth, hold it for 1015 seconds, and then expel it through the nose. Repeat 45 times to complete the series D. Inhale as deeply as possible and then immediately exhale as deeply as possible at a rate of approximately 2024 times/min Correct Answer: A Section: (none) Explanation Explanation/Reference: Explanation: (A) This is the correct method of teaching diaphragmatic breathing, which allows full lung expansion to increase oxygenation, prevent atelectasis, and move secretions up and out of the lungs to decrease risk of pneumonia. (B) Quick, short breaths do not allow for full lung expansion and movement of secretions up and out of the lungs. Quick, short breaths may lead to O2 depletion, hyperventilation, and hypoxia. (C) Expelling breaths through the nose does not allow for full lung expansion and the use of diaphragmatic muscles to assist in moving secretions up and out of the lungs. (D) Inhaling and exhaling at a rate of 2024 times/min does not allow time for full lung expansion to increase oxygenation. This would most likely lead to O2 depletion and hypoxia. QUESTION 429 A 27-year-old healthy primigravida is brought to the labor and birthing room by her husband at 32 weeks' gestation. She experienced a sudden onset of painless vaginal bleeding. Following an ultrasound examination, the diagnosis of bleeding secondary to complete placenta previa is made. Expected assessment findings concerning the abdomen would include: A. A rigid, boardlike abdomen B. Uterine atony C. A soft relaxed abdomen D. Hypertonicity of the uterus Correct Answer: C Section: (none) Explanation Explanation/Reference: Explanation: (A) A rigid, boardlike abdomen is an assessment finding indicative of placenta abruptio. (B) A cause of postbirth hemorrhage is uterine atony. With placenta previa, uterine tone is within normal range. (C) The placenta is located directly over the cervical os in complete previa. Blood will escape through the os, resulting in the uterus and abdomen remaining soft and relaxed. (D) In placenta abruptio, hypertonicity of the uterus is caused by the entrapment of blood between the placenta and uterine wall, a retroplacental bleed. This does not exist in placenta previa. QUESTION 430A 27-year-old primigravida stated that she got up from the chair to fix dinner and bright red blood was running down her legs. She denies any pain previously or currently. The client is very concerned about whether her baby will be all right. Her vital signs include P 120 bpm, respirations 26 breaths/min, BP 104/58 mm Hg, temperature 98.2_F, and fetal heart rate 146 bpm. Laboratory findingsrevealed hemoglobin 9.0 g/dL, hematocrit 26%, and coagulation studies within normal range. On admission, the peripad she wore was noted to be half saturated with bright red blood. A medical diagnosis of placenta previa is made. The priority nursing diagnosis for this client would be: A. Decreased cardiac output related to excessive bleeding B. Potential for fluid volume excess related to fluid resuscitation C. Anxiety related to threat to self D. Alteration in parenting related to potential fetal injury Correct Answer: A Section: (none) Explanation Explanation/Reference: Explanation: (A) Based on the client's history, presence of bright red vaginal bleeding, and hemoglobin value on admission, the priority nursing diagnosis would be decreased cardiac output related to excessive bleeding. (B) This nursing diagnosis is a potential problem that does not exist at the present time, and therefore is not the priority problem. (C) The client's expressed anxiety is for her child. The fetus will remain physiologically safe if the decreased cardiac output is resolved. (D) Initialspontaneous bleeding with placenta previa is rarely life threatening to the mother or the fetus. Delivery of the fetus will be postponed until fetal maturity is achieved and survival is likely. QUESTION 431 A 27-year-old primigravida at 32 weeks' gestation has been diagnosed with complete placenta previa. Conservative management including bed rest is the proper medical management. The goal for fetal survival is based on fetal lung maturity. The test used to determine fetal lung maturity is: A. Dinitrophenylhydrazine B. Metachromatic stain C. Blood serum phenylalanine test D. Lecithin-sphingomyelin ratio Correct Answer: D Section: (none) Explanation Explanation/Reference: Explanation: (A) Dinitrophenylhydrazine is a laboratory test used to detect phenylketonuria, maple syrup urine disease, and Lowe's syndrome. (B) Metachromatic stain is alaboratory test that may be used to diagnose Tay-Sachs and other lipid diseases of the central nervous system. (C) The blood serum phenylalanine test is diagnostic of phenylketonuria and can be used for wide-scale screening. (D) A lecithin-sphingomyelin ratio of at least 2:1 is indicative of fetal lung maturity, and survival of the fetus is likely. QUESTION 432 The nurse is notified that a 27-year-old primigravida diagnosed with complete placenta previa is to be admitted to the hospital for a cesarean section. The client is now at 36 weeks' gestation and is presently having bright red bleeding of moderate amount. On admission, the nursing intervention that the nurse should give the highest priority to is: A. Shave the client's abdomen and arrange her lab work B. Determine the status of the fetus by fetal heart tones C. Start an IV infusion in the client's arm D. Insert an indwelling catheter into her bladder Correct Answer: B Section: (none) Explanation Explanation/Reference: Explanation: (A) These nursing actions are necessary prior to the cesarean section, but not immediately necessary to maintain physiological equilibrium. (B) Determining the physiological status of the fetus would constitute the highest priority in evaluating and maintaining fetal life. (C) These nursing actions are necessary prior to the cesarean section, but not immediately necessary to maintain physiological equilibrium. (D) These nursing actions are necessary prior to the cesarean section, but not immediately necessary to maintain physiological equilibrium. QUESTION 433 A 29-year-old client delivered her fifth child by the Lamaze method and developed a postpartal hemorrhage in the recovery room. What are the initial symptoms of shock that she may experience? A. Marked elevation in blood pressure, respirations, and pulse B. Decreased systolic pressure, cold skin, and anuria C. Rapid pulse; narrowed pulse pressure; cool, moist skin D. No urinary output, tachycardia, and restlessness Correct Answer: C Section: (none) Explanation Explanation/Reference:Explanation: (A) Early shock does not exhibit the symptom of marked elevation in blood pressure. A narrowing of the pulse pressure is indicative of early shock. (B) Anuria is a clinical finding in late shock. (C) All of these clinical findings are congruent with early shock. (D) Absence of urinary output is a clinical finding in the late phase ofshock. QUESTION 434 The nurse has been assigned a client who delivered a 6- lb, 12-oz baby boy vaginally 40 minutes ago. The initial assessment of greatest importance for this client would be: A. Length of her labor B. Type of episiotomy C. Amount of IV fluid to be infused D. Character of the fundus Correct Answer: D Section: (none) Explanation Explanation/Reference: Explanation: The length of labor has little bearing on the fourth stage of labor. The type of labor and delivery is significant. (B) The type of episiotomy will affect the client's comfort level. However, the nurse's assessment and implementations center on prevention of hemorrhage during the fourthstage of labor. The amount of bleeding from the episiotomy or hematoma formation is of higher priority than the type of episiotomy. (C) The amount of IV fluid to be infused is a nursing function to be attended to; however, it is lower in priority than determining if hemorrhaging is occurring. (D) Character of the fundus would be the priority nursing assessment because changes in uterine tone may identify possible postpartum hemorrhage. QUESTION 435 On the first postpartal day, a client tells the nurse that she has been changing her perineal pads every 1/2 hour because they are saturated with bright red vaginaldrainage. When palpating the uterus, the nurse assesses that it is somewhat soft, 1 fingerbreadth above the umbilicus, and midline. The nursing action to be taken is to: A. Gently massage the uterus until firm, express any clots, and note the amount and character of lochia B. Catheterize the client and reassess the uterus C. Begin IV fluids and administer oxytocic medication D. Administer analgesics as ordered to relieve discomfort Correct Answer: A Section: (none)Explanation Explanation/Reference: Explanation: (A) Gentle massage and expression of clots will let the fundus return to a state of firmness, allowing the uterus to function as the "living ligature." (B) A distended bladder may promote uterine atony; however, after determining the bladder is distended, the nurse would have the client void. Catheterization is only done if normal bladder function has not returned. (C) Oxytocic medications are ordered and administered if the uterus does not remain contracted after gentle massage and determining if the bladder is empty. (D) The client is not complaining of discomfort or pain; therefore, analgesics are not necessary. QUESTION 436 The nurse assesses a client on the second postpartum day and finds a dark red discharge on the peripad. The stain appears to be about 5 inches long. Which of the following correctly describes the character and amount of lochia? A. Lochia alba, light B. Lochia serosa, heavy C. Lochia granulosa, heavy D. Lochia rubra, moderate Correct Answer: D Section: (none) Explanation Explanation/Reference: Explanation: (A) Lochia alba occurs approximately 10 days after birth and is yellow to white. A discharge is classified as light when less than a 4-inch stain exists. (B) Lochia serosa is pink to brown and occurs 34 days after delivery. A stain is classified as heavy when a peripad is saturated in 1 hour. (C) Lochia granulosa is not a proper classification. (D) Lochia rubra is red, consisting mainly of blood, debris, and bacteria, and lasts from the time of delivery to 34 days afterward. A stain is classified as moderate when less than a 6-inch stain exists. QUESTION 437 A 2-day-old infant boy has been diagnosed with an atrial septal defect due to a persistent patent foramen ovale. When explaining the diagnosis to the mother, the nurse includes in the discussion the function of the foramen ovale. In fetal circulation, the foramen ovale allows a portion of the blood to bypass the: A. Left ventricle B. Pulmonary system C. Liver D. Superior vena cavaCorrect Answer: B Section: (none) Explanation Explanation/Reference: Explanation: (A) The foramen ovale permits a percentage of the blood to shunt from the right atrium to the left atrium. The blood then goes to the left ventricle, permitting systemic fetal circulation with blood containing a higher O2 saturation. (B) As the blood shunts from the right atrium to the left atrium, the pulmonary system is bypassed. The fetus receives O2 from the maternal circulation, thereby permitting the partial bypass of the pulmonary system. (C) The foramen ovale is locatedin the atrial septum of the heart and does not affect the liver. (D) The superior vena cava returns blood to the heart, bringing blood to the location of the foramen ovale. QUESTION 438 A client has been admitted to the nursing unit with the diagnosis of severe anemia. She is slightly short of breath, has episodes of dizziness, and complains her heart sometimes feels like it will "beat out of her chest." The physician has ordered her to receive 2 U of packed red blood cells. The most important nursing action to be taken is: A. Starting an 18-gauge IV infusion B. Having the consent form on the chart C. Administering the correct blood product to the correctclient D. Transfusing the blood in a 2-hour time frame Correct Answer: C Section: (none) Explanation Explanation/Reference: Explanation: (A) An 18-gauge IV is an appropriate size for administering blood; however, client safety demands that the right blood product must be administered. (B) The consent form is legally necessary to be on the chart, but client safety is maintained by giving the correct blood component to the correct client. (C) Administering the correct blood product to the correct client will maintain physiological safety and minimize transfusion reactions. (D) The blood administration should take place over the ordered time frame designated by the physician. QUESTION 439 A client diagnosed with severe anemia is to receive 2 U of packed red blood cells. Prior to starting the blood transfusion, the nurse must: A. Take a baseline set of vital signs B. Hang Ringer's lactate as the companion fluidC. Use microdrip tubing for the blood administration D. Have the registered nurse in charge assume responsibility for verifying the client and blood product information Correct Answer: A Section: (none) Explanation Explanation/Reference: Explanation: (A) A baseline set of vital signs is necessary to determine if any transfusion reactions occur as the blood product is being administered. (B) The only companion fluid to be used during a blood transfusion is normal saline. The calcium in Ringer's lactate can cause clotting. (C) Only a blood administration set should be used. A microdrip tube would cause lysis of the red blood cells. (D) Proper identification of the recipient and the blood product must be validated by at least two people. QUESTION 440 A client's transfusion of packed red blood cells has been infusing for 2 hours. She is complaining of a raised, itchy rash and shortness of breath. She is wheezing, anxious, and very restless. The nurse knows these assessment findings are congruent with: A. Hemolytic transfusion reaction B. Febrile transfusion reaction C. Circulatory overload D. Allergic transfusion reaction Correct Answer: D Section: (none) Explanation Explanation/Reference: Explanation: (A) A hemolytic transfusion reaction would be characterized by fever, chills, chest pain, hypotension, and tachypnea. (B) Fever, chills, and headaches are indicative of a febrile transfusion reaction. (C) Circulatory overload is manifest by dyspnea, cough, and pulmonary crackles. (D) Urticaria, pruritus, wheezing, and anxiety areindicative of an allergic transfusion reaction. QUESTION 441 Diagnostic assessment findings for an infant with possible coarctation of the aorta would include: A. A third heart sound B. A diastolic murmur C. Pulse pressure difference between the upper extremitiesD. Diminished or absent femoral pulses Correct Answer: D Section: (none) Explanation Explanation/Reference: Explanation: (A) S1 and S2 in an infant with coarctation of the aorta are usually normal. S3 and S4 do not exist with this diagnosis. (B) Either no murmur will be heard or a systolic murmur from an associated cardiac defect will be heard along the left upper sternal border. A diastolic murmur is not associated with coarctation of the aorta. (C) Pulse pressure differences of>20 mm Hg exist between the upper extremities and the lower extremities. It is important to evaluate the upper and lower extremities with the appropriate- sized cuffs. (D) Femoral and pedal pulses will be diminished or absent in infants with coarctation of the aorta. QUESTION 442 Decreased pulmonary blood flow, right-to-left shunting, and deoxygenated blood reaching the systemic circulation are characteristic of: A. Tetralogy of Fallot B. Ventricular septal defect C. Patent ductus arteriosus D. Transposition of the great arteries Correct Answer: A Section: (none) Explanation Explanation/Reference: Explanation: (A) Tetralogy of Fallot is the most common cyanotic heart defect, which includes a VSD, pulmonary stenosis, an overriding aorta, and ventricular hypertrophy. The blood flow is obstructed because the pulmonary stenosis decreases the pulmonary blood flow and shunts blood through the VSD, creating a right-to-left shunt that allows deoxygenated blood the reach the systemic circulation. (B) A VSD alone creates a left-to-right shunt. The pressure in the left ventricle is greater than that of the right; therefore, the blood will shunt from the left ventricle to the right ventricle, increasing the blood flow to the lungs. No deoxygenated blood will reach the systemic circulation. (C) In patent ductus arteriosus, the pressure in the aorta is greater than in the pulmonary artery, creating a left-to-right shunt. Oxygenated blood from the aorta flows into the unoxygenated blood of the pulmonary artery. (D) Transposition of the great arteries results in two separate and parallel circulatory systems. The only mixing or shunting of blood is based on the presence of associated lesions. QUESTION 443 A 2-year-old child will undergo a cardiac catheterization tomorrow to evaluate his ventricular septal defect. Based on his developmental stage, the nurse:A. Uses pictures to explain the procedure to the child and his parents that evening B. Explains the procedure using simple words and sentences just before the preoperative sedation C. Asks the parents to explain the procedure to the child after she explains it to them D. Asks the parents to leave the room while the preoperative medication and instructions are given Correct Answer: B Section: (none) Explanation Explanation/Reference: Explanation: (A) A toddler is not capable of conceptualizing about the inside of his body and is concerned about body intactness; therefore, diagrams would not be useful. Also, the previous evening is too far from the procedure for the toddler to remember the instructions. (B) A simple explanation the morning of the procedure is the best developmental strategy to use, because it focuses on the toddler's need for parental support, body intactness, and short attention span. (C) A relationship between the nurse and the child needs to develop. Also, misinformation may be given to the child if the parents explain the procedure to the child. (D) The parents are the child's support system and need to be there to strengthen the child. QUESTION 444 Home-care instructions for the child following a cardiac catheterization should include: A. Notify the physician if a slight bruise develops around the insertion site. B. Use sponge bathing until stitches are removed. C. Give aspirin if the child complains of pain at the insertion site. D. Keep a clean, dry dressing on the insertion site for 2 days. Correct Answer: B Section: (none) Explanation Explanation/Reference: Explanation: (A) A small bruise may develop around the insertion site and is not a reason for alarm. (B) It is best to keep the child out of the bathtub until the sutures are removed. (C) Acetaminophen, not aspirin, is the drug of choice if there is pain at the insertion site. (D) The insertion site should be kept clean and dry and open to air. QUESTION 445 Nursing care for the parents of a child with a congenital heart defect would include:A. Encouraging the parents not to tell the child about the seriousness of the congenital heart defect, so the child will function as normally as possible B. Acknowledging the fear and concern surrounding their child's health and assisting the parents through the grieving process as they mourn the loss of their fantasized healthy child C. Identifying anger and resentment as destructive emotions that serve no purpose D. Expressing to the parents after the corrective surgery has been completed successfully that all their grief feelings will resolve Correct Answer: B Section: (none) Explanation Explanation/Reference: Explanation: (A) It is important to discuss with parents the need to treat the child as they would any other children, but they must be truthful and honest with the child about the heart defect. As the child grows older, explanations can go into greater depth. (B) Parents of children with congenital heart defects go through a grieving process over the loss of their "healthy" child. The nurse needs to recognize these feelings and give the parents a role in the child's care when they are ready. (C) Anger and resentment are normal feelings that must be dealt with appropriately. (D) Parents may go through a second grieving process after the repair of the cardiac defect. During this grieving period, they mourn the loss of the "defective" child who now may be essentially "normal." QUESTION 446 An infant with a congenital heart defect is being discharged with an order for the administration of digoxin elixir every 12 hours. The parents need to be taught when administering digoxin to the infant that: A. If the infant vomits within 30 minutes of the digoxin administration, repeat the dose B. They need to mix it with formula so the infant swallows it easily C. If the infant vomits two or more consecutive doses or becomes listless or anorexic, notify thephysician D. If a dose of digoxin is skipped for more than 6 hours, a new timetable for administration must be developed Correct Answer: C Section: (none) Explanation Explanation/Reference: Explanation: (A) Occasionally the child may vomit. They should not repeat the dose because the amount of digoxin that was absorbed is un-known, and serum levels of digoxin that are too high are more dangerous than those that are temporarily too low. (B) To ensure that the entire dose of digoxin is received, never mix it with food or formula. (C) Vomiting, anorexia, and listlessness are all signs of digoxin toxicity and should be reported to the physician immediately. (D) If a dose is forgotten formore than 6 hours, the nurse should advise the parents to skip that dose and to continue the next dose as scheduled.http://www.gratisexam.com/ QUESTION 447 A 4-year-old child with a history of sickle cell anemia is admitted to the nursing unit with dizziness, shortness of breath, and pallor. Nursing assessment findings reveal tenderness in the abdomen. The child is most likely experiencing a/an: A. Aplastic crisis B. Vaso-occlusive crisis C. Dactylitis crisis D. Sequestration crisis Correct Answer: D Section: (none) Explanation Explanation/Reference: Explanation: (A) Aplastic anemia is characterized by a lack of reticulocytes in the blood. Platelet and white blood cell counts are usually not depressed. It is usually self-limiting, lasting 510 days. (B) Vaso-occlusive crisis is the most common type of crisis in sickle cell anemia. Sickled cells become clogged, leading to distal tissue hypoxia and infarction. Joints and extremities are the most commonly affected areas. (C) Dactylitis crisis, or "hand-foot syndrome," causes symmetrical infarction of the bones in the hands and feet, resulting in painful swelling in the soft tissues of the hands and feet. (D) Sequestration crisis occurs as enormous volumes of blood pool within the spleen. The spleen enlarges, causing tenderness. Signs of shock including pallor, tachypnea, and faintness result, related to the deficient intravascular volume. This type of crisis is potentially fatal. QUESTION 448 The primary focus of nursing interventions for the child experiencing sickle cell crisis is aimed toward: A. Maintaining an adequate level of hydration B. Providing pain relief C. Preventing infection D. O2 therapyCorrect Answer: A Section: (none) Explanation Explanation/Reference: Explanation: (A) Maintaining the hydration level is the focus for nursing intervention because dehydration enhances the sickling process. Both oral and parenteral fluids are used. (B) The pain is a result of the sickling process. Analgesics or narcotics will be used for symptom relief, but the underlying cause of the pain will be resolved with hydration. (C) Serious bacterial infections may result owing to splenic dysfunction. This is true at all times, not just during the acute period of a crisis. (D) O2 therapy is used for symptomatic relief of the hypoxia resulting from the sickling process. Hydration is the primary intervention to alleviate the dehydration that enhances the sickling process. QUESTION 449 A 30-year-old client is exhibiting auditory hallucinations. In working with this client, the nurse would be most effective if the nurse: A. Encourages the client to discuss the voices B. Attempts to direct the client's attention to the here and now C. Exhibits sincere interest in the delusional voices D. Gives the medication as necessary for the acting-out behavior Correct Answer: B Section: (none) Explanation Explanation/Reference: Explanation: (A) This answer is incorrect. Encouraging discussion of the voices will reinforce the delusion. (B) This answer is correct. The nurse should appropriately present reality. (C) This answer is incorrect. Showing interest would reinforce the delusional system. (D) This answer is incorrect. The statement only indicates that theclient is hearing voices. It does not state that the client is acting out. QUESTION 450 One week ago, a 21-year-old client with a diagnosis of bipolar disorder was started on lithium 300 mg po qid. A lithium level is ordered. The client's level is 1.3 mEq/L. The nurse recognizes that this level is considered to be: A. Within therapeutic range B. Below therapeutic range C. Above therapeutic rangeD. At a level of toxic poisoning Correct Answer: A Section: (none) Explanation Explanation/Reference: Explanation: (A) This answer is correct. The therapeutic range is 1.01.5 mEq/L in the acute phase. Maintenance control levels are 0.61.2 mEq/L. (B, C) This answer is incorrect. A level of 1.3 mEq/L is within therapeutic range. (D) This answer is incorrect. Toxic poisoning is usually at the 2.0 level or higher. QUESTION 451 A client was exhibiting signs of mania and was recently started on lithium carbonate. She has no known physical problems. A teaching plan for this client would include which of the following? A. Regular foods should be eaten, including those that contain salt, such as bacon, ham, V-8 juice, and tomato juice. B. Restrict fluids to 1000 mL/day. C. Restrict foods that contain salt or sodium. D. Discontinue the medication if nausea occurs. Correct Answer: A Section: (none) Explanation Explanation/Reference: Explanation: (A) This answer is correct. A balanced diet with adequate salt intake is necessary. (B) This answer is incorrect. The client must drink six to eight full glasses of fluidper day (20003000 mL/day). (C) This answer is incorrect. The client should be instructed to avoid fluctuations of sodium intake. Diet should be balanced, with an adequate salt intake. (D) This answer is incorrect. Nausea is a frequent side effect that can be minimized with administration of drug with meals or after eating food. QUESTION 452 A behavioral modification program isrecommended by the multidisciplinary team working with a 15-year-old client with anorexia nervosa. A nursing plan of care based on this modality would include: A. Role playing the client's eating behaviors B. Restriction to the unit until she has gained 2 lb C. Encouraging her to verbalize her feelings concerning food and food intakeD. Provision for a high-calorie, high-protein snack between meals Correct Answer: B Section: (none) Explanation Explanation/Reference: Explanation: (A) This answer is incorrect. Role playing is based on learning but is not based on the behavioral modification model. (B) This answer is correct. The behavioral modification model is based on negative and positive reinforcers to change behavior. (C) This answer is incorrect. Verbal catharsis is not an intervention based on behavioral modification. (D) This answer is incorrect. Although an acceptable nursing intervention, it is not based on behavioral modification. QUESTION 453 A 22-year-old client presents with a diagnosis of antisocial personality disorder and a history of using drugs, writing numerous checks with insufficient funds, and stealing. He appears charming and intelligent, and the other clients are impressed and want to be liked by him. The greatest problem that may arise from this situation is that: A. He will manipulate the other clients for his own benefit B. He will cause the other clients to become psychotic C. He will become delusional and hallucinate as a result of the excess attention given to him by peers D. He may exhibit self-mutilative behavior Correct Answer: A Section: (none) Explanation Explanation/Reference: Explanation: (A) This answer is correct. Persons with antisocial personality disorder typically are very manipulative. (B) This answer is incorrect. The client's behavior cannot cause another person to become psychotic. (C) This answer is incorrect. Psychosis is not a symptom of antisocial personality. One of the criteria for diagnosis of this disorder is that no psychosis be present. In addition, the client would love the attention. (D) This answer is incorrect. Self-mutilative behavior is characteristic of the borderline personality disorder. QUESTION 454 In admitting a client to the psychiatric unit, the nurse must explain the rules and regulations of the unit. A client with antisocial personality disorder makes the following remark, "Forget all those rules. I always get along well with the nurses." Which nursing response to him would be most effective? A. "OK, don't listen to the rules. See where you end up."B. "I'm pleased that you get along so well with the staff.You must still know and abide by the rules." C. "It is irrelevant whether you get along with the nurses." D. "I'm not the other nurses. You better read the rules yourself." Correct Answer: B Section: (none) Explanation Explanation/Reference: Explanation: (A) This answer is incorrect. A nurse should be an appropriate role model. Threats are not appropriate. No limit setting was stated. (B) This answer is correct. The nurse made a positive statement followed by a simple, clear, concise setting of limits. (C) This answer is incorrect. It appears to have a negative connotation. There was no limit setting. (D) This answer is incorrect. The nurse obviously responded in a negative manner. Learning takes place more readily when one is accepted, not rejected. No limits were set. QUESTION 455 A client was admitted to the hospital after falling in her home. At the time of admission, her blood alcohol level was 0.27 mg%. Her family indicates that she has been drinking a fifth of vodka a day for the past 9 months. She had her last drink 30 minutes prior to admission. Alcohol withdrawal symptoms would most likely be exhibited by her: A. Two to 4 hours after the last drink B. Six to 8 hours after the last drink C. Immediately on admission D. Twenty-four hours after the last drink Correct Answer: B Section: (none) Explanation Explanation/Reference: Explanation: (A) This answer is incorrect. Alcohol withdrawal usually begins approximately 68 hours after the last drink. (B) This answer is correct. It takes approximately 68 hours for metabolism of alcohol. (C) This answer is incorrect. The alcohol is still in the system, as indicated by the high blood alcohol level. (D) This answer isincorrect. Symptoms of alcohol withdrawal usually begin within 68 hours of the last drink. QUESTION 456 A client has begun to exhibit signs of alcohol withdrawal. Her blood pressure has risen from 120/60 to 190/100, pulse is increased from 88 to 110 bpm, and she is irritable and agitated and has gross motor tremors of the hands. The nurse notifies the doctor. The nurse can anticipate that the doctor will order which of thefollowing? A. An opiate such as propoxyphene napsylate (Darvocet) B. A benzodiazepine such as chlordiazepoxide (Librium) C. A tricyclic antidepressant such as amitriptyline (Elavil) D. A phenothiazine such as chlorpromazine (Thorazine) Correct Answer: B Section: (none) Explanation Explanation/Reference: Explanation: (A) This answer is incorrect. Benzodiazepines are drugs of choice for alcohol withdrawal. (B) This answer is correct. The drug has a sedative effect, is safe, and has an anticonvulsant effect.(C) This answer is incorrect. Amitriptyline is an antidepressant. (D) This answer is incorrect. Chlorpromazine is most effective in psychotic disorders. QUESTION 457 A 60-year-old woman exhibits forgetfulness, emotional lability, confusion, and decreased concentration. She has been unable to perform activities of daily living without assistance. After a thorough medical evaluation, a diagnosis of Alzheimer's disease was made. An appropriate nursing intervention to decrease the anxiety of this client would include: A. Allowing the client to perform activities of daily living as much as possible unassisted B. Confronting confabulations C. Reality testing D. Providing a highly stimulating environment Correct Answer: A Section: (none) Explanation Explanation/Reference: Explanation: (A) This answer is correct. The more the client is able to control her daily routine, the less anxiety she will experience. (B) This answer is incorrect. Confrontation tends to increase anxiety. (C) This answer is incorrect. Reality testing is an assessment tool. It does not decrease anxiety. (D) This answer is incorrect. A highlystimulating environment increases distractibility and anxiety. QUESTION 458A 74-year-old client seen in the emergency room is exhibiting signs of delirium. His family states that he has not slept, eaten, or taken fluids for the past 24 hours. The planning of nursing care for a delirious client is based on which of the following premises? A. The delirious client is capable of returning to his previous level of functioning. B. The delirious client isincapable of returning to his previous level of functioning. C. Delirium entails progressive intellectual and behavioral deterioration. D. Delirium is an insidious process. Correct Answer: A Section: (none) Explanation Explanation/Reference: Explanation: (A) This answer is correct. If the cause is removed, the delirious client will recover completely. (B) This answer is incorrect. The demented client is incapable of returning to previous level of functioning. The delirious client is capable of returning to previous functioning. (C) This answer is incorrect. The demented client, notthe delirious client, has progressive intellectual and behavioral deterioration. (D) This answer is incorrect. Delirium develops rapidly, whereas dementia is insidious. QUESTION 459 A 48-year-old client presents with a long history of severedepression unrelieved by medication. He is admitted to the hospital for electroconvulsive therapy. Familymembers are very concerned about this therapy and are requesting information about aftereffects of the treatment. The nurse informs the family that he will: A. Have transient memory loss, confusion, andheadache B. Be alert and oriented immediately after the treatment C. Have insomnia for the first few days D. Require no special care after the procedure Correct Answer: A Section: (none) Explanation Explanation/Reference: Explanation: (A) This answer is correct. The client will be confused and have a memory loss, which is usually temporary, after electroconvulsive shock therapy. (B) This answer is incorrect. The client will experience transient memory loss, look bewildered, and be confused initially. (C) This answer is incorrect. The client will sleep immediately following the treatment. (D) This answer is incorrect. Vital signs are taken at least hourly after treatment. The client is monitored for hypotension, tachycardia, respiratory problems, and possible seizure activity.QUESTION 460 An 80-year-old widow is living with her son and daughter- in-law. The home health nurse has been making weekly visits to draw blood for a prothrombin time test. The client is taking 5 mg of coumadin per day. She appears more debilitated, and bruises are noted on her face. Elder abuse is suspected. Which of the following are signs of persons who are at risk for abusing an elderly person? A. A family member who is having marital problems and is regularly abusing alcohol B. A person with adequate communication and coping skills who is employed by the family C. A friend of the family who wants to help but is minimally competent D. A lifelong friend of the client who is often confused Correct Answer: A Section: (none) Explanation Explanation/Reference: Explanation: (A) This answer is correct. Two risk factors are identified in this answer. (B) This answer is incorrect. Persons at risk tend to lack communication skills and effective coping patterns. (C) This answer is incorrect. Persons at risk are usually family members or those reluctant to provide care. (D) This answer is incorrect. This individual has a vested interest in providing care. QUESTION 461 A 16-year-old client with a diagnosis of oppositional defiant disorder is threatening violence toward another child. In managing a potentially violent client, the nurse: A. Must use the least restrictive measure possible to control the behavior B. Should put the client in seclusion until he promises to behave appropriately C. Should apply full restraints until the behavior is under control D. Should allow other clients to observe the acting out so that they can learn from the experience Correct Answer: A Section: (none) Explanation Explanation/Reference: Explanation: (A) This answer is correct. Least restrictive measures should always be attempted before a client is placed in seclusion or restraints. The nurse should first try a calm verbal approach, suggest a quiet room, or request that the client take "time-out" before placing the client in seclusion, givingmedication as necessary, or restraining. (B) This answer is incorrect. A calm verbal approach or requesting that a client go to his room should be attempted before restraining. (C) This answer is incorrect. Restraints should be applied only after all other measures fail to control the behavior. (D) This answer is incorrect. Other clients should be removedfrom the area. It is often very anxiety producing for other clients to see a peer out of control. It could also lead to mass acting- out behaviors. QUESTION 462 The nurse is planning a reality orientation program for a group of clients with organic brain syndrome at the mental health center. Props that could be used for this program are: A. Month-old magazines that are provided by volunteers B. Large maps and posters depicting area of current residence C. A litter of kittens for the clients to pet D. A library of biographical books Correct Answer: B Section: (none) Explanation Explanation/Reference: Explanation: (A) This answer is incorrect. Current magazines would be appropriate. (B) This answer is correct. Maps of the state and town and posters that depict current events in the area are appropriate props. (C) This answer is incorrect. Kittens would be appropriate for pet therapy, not reality therapy. (D) This answer is incorrect. Biographies depict a past, not a present, orientation. QUESTION 463 In working with a manipulative client, which of the following nursing interventions would be most appropriate? A. Bargaining with the client as a strategy to control the behavior B. Redirecting the client C. Providing a consistent set of guidelines and rules D. Assigning the client to different staff persons each day Correct Answer: C Section: (none) Explanation Explanation/Reference: Explanation: (A) This answer is incorrect. Bargaining is a manipulative act, which the nurse could expect from the client. (B) This answer is incorrect. Confrontation is an effective nursing strategy with manipulative behavior. Redirection is appropriate for the client who is out of touch with reality. (C) This answer is correct. Manipulative clients must abide by consistent rules. (D) This answer is incorrect. Manipulation is kept at a minimum if the same staff person is assigned to theclient. Often the client will attempt to play staff persons against each other. QUESTION 464 Primary nursing diagnoses for the antisocial client are: A. Alteration in perception and altered self-concept B. Impaired social interaction, ineffective individual coping, and altered self-concept C. Altered communication processes and altered recreational patterns D. Altered body image and altered thought processes Correct Answer: B Section: (none) Explanation Explanation/Reference: Explanation: (A) This answer is incorrect. Perception is not altered because the client is not psychotic. (B) This answer is correct. The antisocial client lacks responsibility, accountability, and social commitment; has impaired problem-solving ability; tends to overuse defense mechanisms; lies and steals; and is often grandiose concerning self. (C) This answer is incorrect. Altered communication processes do not characterize this client. The antisocial person communicates well and tends to have a charming personality. (D) This answer is incorrect. Altered thought processes refer to delusional thinking, which is bizarre and fixed, and do not characterize this client. QUESTION 465 A 25-year-old outpatient presents with a diagnosis of compulsive personality disorder. His coworkers become annoyed with hisrigid, perfectionistic manner and preoccupation with trivial details and schedules. A nursing intervention appropriate for this client would include: A. Encouraging him to engage in recreational activities B. Avoiding discussion of his annoying behavior C. Encouraging the client to set a time schedule and deadlines for himself D. Contracting with him for the amount of time he will spend on the compulsive behaviors Correct Answer: D Section: (none) Explanation Explanation/Reference: Explanation: (A) This answer is incorrect. The client will work hard at the activity instead of enjoying it. (B) This answer is incorrect. The nurse should allow the client to discussthese thoughts, within limits, not to avoid discussing them. (C) This answer is incorrect. The compulsive client tends to control time to excess. It should not be encouraged. (D) This answer is correct. A contract with the client regarding the amount of time that will be spent discussing the compulsive activities is appropriate. Time allotted should be gradually decreased. QUESTION 466 The serial sevens test is often used to determine delirium and dementia. This test aids in assessing which of the following? A. Abstract thinking B. Ability to focus and concentrate thoughts C. Judgment D. Memory Correct Answer: B Section: (none) Explanation Explanation/Reference: Explanation: (A) This answer is incorrect. The test measures the abilities to concentrate and calculate. The use of proverbs is the most common way to test abstraction. (B) This answer is correct. The serial sevens test is a common test of calculation ability. It is difficult for the demented or delirious client to perform. (C) This answer is incorrect. The test for judgment should predict whether the individual will behave in a socially accepted manner. (D) This answer is incorrect. In testingmemory, the nurse would attempt to get the client either to recall recent events or to think about past events. QUESTION 467 A 14-year-old client has a history of lying, stealing, and destruction of property. Personal items of peers have been found missing. After group therapy, a peer approaches the nurse to report that he has seen the 14- year-old with some of the missing items. The best response of the nurse is to: A. Request that he explain to the group why he took personal items from peers B. Approach him when he is alone to inquire about his involvement in the incident C. Imply to him that you doubt his involvement in the incident and request his denial D. Confront him openly in group and request an apology Correct Answer: B Section: (none) Explanation Explanation/Reference: Explanation: (A) This answer is incorrect. There is no proof that he removed the missing items. (B) This answer is correct. Anxiety and defensiveness are lessened if theindividual is approached in this manner. (C) This answer is incorrect. It is difficult for one to admit to wrongdoing with this approach. (D) This answer is incorrect. He has not yet been proved guilty. Confrontation will only increase defensiveness and anxiety. QUESTION 468 A 15-year-old client is admitted to the adolescent unit. The nurse recognizes that encouraging a client to speak openly depends on how clearly questions are phrased. Which of the following statements is most desirable in eliciting information from an adolescent client? A. "Do you get along well with your family?" B. "Do you communicate with your parents?" C. "You don't hate your family, do you?" D. "What is it like between you and your family?" Correct Answer: D Section: (none) Explanation Explanation/Reference: Explanation: (A, B) This statement can be answered with a simple yes or no. (C) This statement is asked in a negative manner and therefore has a negative connotation. (D) This statement is open ended and positively stated. QUESTION 469 A 37-year-old client has been taking antipsychotic medication for the past 10 days. The nurse observes her walking with a shuffling gait and postural rigidity and notes a masklike expression on her face. Which side effect is this client exhibiting? A. Dystonia B. Parkinsonism C. Tardive dyskinesia D. Akathesia Correct Answer: B Section: (none) Explanation Explanation/Reference: Explanation: (A) This answer is incorrect. Dystonia refers to severe, painful muscle contractions. (B) This answer is correct. Parkinsonism commonly occurs approximately 12 weeks after initiation of antipsychotic drug therapy. Traditional signs are masklike facies, postural rigidity, shuffling gait, and resting tremor. (C) This answer isincorrect. Tardive dyskinesia is characterized by involuntarymuscle movements of the face, jaw, and tongue. (D) This answer is incorrect. Akathesia is motor restlessness. QUESTION 470 Clientsreceiving antipsychotic drug therapy will often exhibit extrapyramidal side effects that are reversible with which of the following agents ordered by the physician? A. Phenothiazines B. Anticholinergics C. Anti-Parkinsonian drugs D. Tricyclic agents Correct Answer: B Section: (none) Explanation Explanation/Reference: Explanation: (A) This answer is incorrect. Phenothiazines are antipsychotic drugs and produce the symptoms. (B) This answer is correct. Anticholinergic agents are often used prophylactically for extrapyramidal symptoms. They balance cholinergic activity in the basal ganglia of the brain. (C) This answer is incorrect. Anti- Parkinsonian drugs would increase the symptoms. (D) This answer is incorrect. Tricyclic agents are used for symptoms of depression. QUESTION 471 A client who was started on antipsychotic medication 2 weeks ago is preparing for discharge from the hospital. Compliance with the medication regimen is important despite the mild side effects encountered. In order to increase the likelihood of medication compliance, the nurse would: A. Discuss the disease process and the importance of the medication in prevention of symptoms. B. Inform the client that additional side effects are to be expected and need not be reported. C. Discuss the importance of getting blood drawn weekly to determine medication therapeutics. D. Inform the client to cease taking the medication when all psychotic symptoms have cleared. Correct Answer: A Section: (none) Explanation Explanation/Reference: Explanation: (A) This answer is correct. If the client is well informed about what reactions to expect from her medication, she is more likely to follow the treatment regimen. (B)This answer is incorrect. There are many side effects that are reversible by medication, and these must be reported to the nurse or physician. There are also moresevere side effects, such as neuroleptic malignant syndrome, characterized by fever, tachycardia, and diaphoresis, which can be life threatening. (C) This answer is incorrect. There is no need for weekly blood tests if the drug regimen has been followed properly. (D) This answer is incorrect. The client should continue the medicationuntil the physician recommends any change in the drug regimen. Symptoms will usually reappear if medication is discontinued. QUESTION 472 A depressed client is seen at the mental health center for follow-up after an attempted suicide 1 week ago. She has taken phenelzine sulfate (Nardil), a monoamine oxidase (MAO) inhibitor, for 7 straight days. She states that she is not feeling any better. The nurse explains that the drug must accumulate to an effective level before symptoms are totally relieved. Symptom relief is expected to occur within: A. 10 days B. 24 weeks C. 2 months D. 3 months Correct Answer: B Section: (none) Explanation Explanation/Reference: Explanation: (A) This answer is incorrect. It can take up to 1 month for therapeutic effect of the medication. (B) This answer is correct. Because MAO inhibitors are slow to act, it takes 24 weeks before improvement of symptoms is noted. (C) This answer is incorrect. It can take up to 1 month for therapeutic effect of the medication. (D) Thisanswer is incorrect. Therapeutic effects of the medication are noted within 1 month of drug therapy. QUESTION 473 Because a client is taking an MAO inhibitor, it is necessary to discuss the need for adherence to a low-tyramine diet. Which of the following are foods that she should avoid? A. Pickled, aged, smoked, and fermented foods B. Fresh vegetables C. Broiled fresh fish and fowl D. Fresh fruit such as apples and oranges Correct Answer: A Section: (none) Explanation Explanation/Reference: Explanation:(A) These foods may produce elevation in blood pressure when consumed during MAO inhibition therapy. (B) These foods have not been pickled, fermented, smoked, or aged. They contain very little, if any, tyramine or tryptophan. (C) As long as the meat has not been aged or smoked, it is within the dietary regimen. (D) Fresh fruits can be consumed as desired. However, the consumption of bananas is limited. QUESTION 474 In working with mental health clients who are prescribed medication that must be taken on a routine basis, it is important for education to begin when the drug therapy is initiated. One of the first steps in the teaching process is to: A. Explain the side effects of the medication B. Discuss the danger of overmedication C. Distribute written material to supplement verbal instructions D. Explore the client's perception regarding medication therapy Correct Answer: D Section: (none) Explanation Explanation/Reference: Explanation: (A, B, C) The nurse must first obtain information regarding the client's perception of the medication regimen. (D) The first step in the teaching process is to determine the client's perception. QUESTION 475 A 29-year-old client is diagnosed with borderline personality disorder. He has aroused the nurse's anger by using a condescending tone of voice with other clients and staff persons. Which of the following statements from the nurse would be most appropriate in acknowledging feelings regarding the client's behavior? A. "I feel angry when I hear that tone of voice." B. "You make me angry when you talk to me that way." C. "Are you trying to get me angry?" D. "Why do you treat me that way?" Correct Answer: A Section: (none) Explanation Explanation/Reference: Explanation:The nurse appropriately states how he or she feels when the client speaks in a condescending manner. (B) This statement indicates that the client has control over the nurse. No one makes another person angry; each individual has a choice. (C) "Why" questions usually put a person on the defensive. In addition, the client cannot "make" the nurse angry. The client does not have that control. (D) Again, a "why" statement places the client on the defensive. QUESTION 476 The mother of a 7-year-old mental health center client reports that the client has refused to attend gymnastics for the past 2 weeks. Prior to that time, the child liked going to this class and was attending 3 times a week. In talking with the client, the nurse would: A. Ask her why she doesn't like gymnastics anymore B. Ask her to describe how things were at gymnastics before she started refusing to go C. Tell her that it is OK to be afraid of this activity D. Reassure her that things will get better once she begins the classes again Correct Answer: B Section: (none) Explanation Explanation/Reference: Explanation: (A) The child has not said that she dislikes gymnastics. (B) The nurse will be able to obtain information on what events occurred at gymnastics prior to her refusal to attend. The nurse will also gain information about the child's perception of the problem. (C) The child has not said she is afraid to go to gymnastics. (D) False reassurance is inappropriate. QUESTION 477 A 42-year-old client presents with a diagnosis of paranoid schizophrenia. She has become increasingly restless and verbally argumentative, and her speech has become pressured. She is exhibiting signs of: A. Depression B. Agitation C. Psychotic ideation D. Anhedonia Correct Answer: B Section: (none) Explanation Explanation/Reference: Explanation:(A) Signs of depression would include withdrawal, sadness, morbid thoughts, insomnia, early awakening, etc. (B) These clinical features are classic signs of agitation. (C) Psychotic ideation includes delusional thoughts, bizarre behavior, disorganized thinking, etc. (D) Anhedonia is the inability to experience pleasure. QUESTION 478 A 50-year-old depressed client hasrecently lost his job. He has been reluctant to leave his hospital room. Nursing care would include: A. Forcing the client to attend all unit activities B. Encouraging the client to discuss why he is so sad C. Monitoring elimination patterns D. Providing sensory stimulation Correct Answer: C Section: (none) Explanation Explanation/Reference: Explanation: (A) The client should be encouraged to attend the unit activities. The nurse and client should choose a few activities for the client to attend that will be positive experiences for him. (B) The nurse should encourage the client to discuss his feelings and to begin to deal with the depression. (C) Depressed persons often havelittle appetite and poor fluid intake. Constipation is common. (D) A calm, consistent level of stimuli is most effective. Sensory deprivation and overstimulation should be avoided. QUESTION 479 A client was admitted to the hospital for a TURP. Within 48 hours of admission and 12 hours postoperatively, both the blood pressure and pulse increased. He became agitated, thought snakes were crawling on his arms and legs, and generally became unmanageable. He pulled out his IV and urinary catheter in attempt to rid himself of the snakes. He was sweating profusely. The admission nurse's notes indicated that the client admitted to "having a few drinks now and then." He is probably experiencing which of the following? A. Major psychotic depression B. Delirium tremens C. Generalized anxiety disorder D. Adjustment disorder with mixed features Correct Answer: B Section: (none) Explanation Explanation/Reference: Explanation:(A) Symptoms of psychotic depression must exist for at least 2 weeks, and the symptoms must represent a change from previous functioning. (B) Delirium tremens occur approximately on the second or third day following cessation or reduction of alcohol intake. Symptoms would be all those described in the situation. (C) Symptoms exhibited by this client are not exhibited in clients with anxiety disorders, who manifest excessive or unrealistic worry about life circumstances for at least 6 months. (D) Symptoms for adjustment disorders with mixed emotional features (e.g., depression and anxiety) are different from those exhibited by the client in this situation. QUESTION 480 A 68-year-old client developed acute respiratory distress syndrome while hospitalized for pneumonia. After a respiratory arrest, an endotracheal tube was inserted. Several days later, numerous attempts to wean him from mechanical ventilation were ineffective, and a tracheostomy was created. For the first 24 hours following tracheostomy, it is important to minimize bleeding around the insertion site. The nurse can accomplish this by: A. Deflating the cuff for 10 minutes every other hour instead of 5 minutes every hour B. Avoiding manipulation of the tracheostomy including cuff deflation C. Reporting any signs of crepitus immediately to the physician D. Changing tracheostomy dressing only as necessary using one-half strength hydrogen peroxide to cleanse the site Correct Answer: B Section: (none) Explanation Explanation/Reference: Explanation: (A) The tracheal cuff should not be deflated within the first 24 hours following surgery. (B) To minimize bleeding, any manipulation, including cuff deflation, should be avoided. (C) Small amounts of crepitus are expected to occur; however, large amounts or expansion of the area of crepitus should be reported to the physician. (D) The tracheostomy site may be changed as often as necessary, but site care should be done with normal saline. QUESTION 481 A 43-year-old client is admitted to the hospital with a diagnosis of peripheral vascular disorder. She arrives in her room via stretcher and requires assistance to move to her bed. The nurse notes that her left leg is cold to touch. She complains of having recently experienced muscle spasms in that leg. To determine if these muscle spasms are indicative of intermittent claudication, the nurse would begin her assessment with the following question: A. "Would you describe the intensity, duration, and symptoms associated with your pain?" B. "Do you experience swelling at the end of the day in the affected and unaffected leg?" C. "Have you had any lesions of the affected leg that have been difficult to heal?" D. "Do your muscle spasms occur following rest, walking, or exercising?"Correct Answer: D Section: (none) Explanation Explanation/Reference: Explanation: (A) Describing pain is an important aspect of the assessment; however, assessing activity preceding muscle spasms is equally important. (B) Edema may occurwith peripheral vascular disease, but it is not of particular importance in assessing intermittent claudication. (C) Lesions may be present with peripheral vasculardisease, but they are not an indication of intermittent claudication. (D) With intermittent claudication, muscle spasms occur intermittently, mainly with walking and after exercising. Rest may relieve muscle spasms. QUESTION 482 A client had a ruptured abdominal aortic aneurysm that wasrepaired surgically. Her postoperative recovery progressed without complications, and she isready for discharge. Client education in preparation for discharge began 7 days ago on her admission to the nursing unit. Evaluation of nursing care related to client education is based on evaluation of expected outcomes. Which statement made by the client would indicate that she is ready for discharge? A. "I will not drive but ride in the front seat of the car with a seat belt on for my first doctor's appointment." B. "When I bathe tomorrow morning, I will be very careful not to get soap on my incision." C. "I am allowed to exercise by walking for short periods." D. "Teach my husband about the diet. He'll be doing all the cooking now." Correct Answer: C Section: (none) Explanation Explanation/Reference: Explanation: (A) Postoperatively, clients with major abdominal surgery are instructed to avoid driving, riding in the front seat, and wearing seat belts because any sudden impact may injure a fresh incision. She should ride in back seat without a seat belt. (B) Clients should not sit in the tub and allow the incision to soak in water because thismay predispose the client to infection. A short, cool shower would be preferable. Allowing soap to come in contact with the incision would not harm it and is frequently used as postoperative wound care at home on discharge from the hospital. (C) Activity instructions include: avoid sitting for long periods and get exercise by walking. Lifting more than 5 lb of weight is also prohibited. (D) The client must also learn her diet. Her husband cooking is probably a temporary measure unless he did the cooking prior to her hospitalization.A statement such as this may indicate the need for further exploration of feelings regarding her illness, dependence, and self-care expectations. QUESTION 483 A 67-year-old man had a physical examination prior to beginning volunteer work at the hospital. A routine chest x-ray demonstrated left ventricular hypertrophy. His blood pressure was 180/110. He is 45 lb overweight. His diet is high in sodium and fat. He has a strong family history of hypertension. The client is placed onantihypertensive medication; a low-sodium, low-fat diet; and an exercise regimen. On his next visit, compliance would best be determined by: A. A blood pressure reading of 130/70 with a 5-lb weight loss B. No side effects from antihypertensive medication and an accurate pill count C. No evidence of increased left ventricular hypertrophy on chest x-ray D. Serum blood levels of the antihypertensive medication within therapeutic range Correct Answer: A Section: (none) Explanation Explanation/Reference: Explanation: (A) A blood pressure within acceptable range best demonstrates compliance, but weight loss cannot be accomplished without adherence to medication, diet, and exercise. (B) Absence of side effects does not indicate compliance with medication. Pill counts can be misleading because the client can alter pill counts prior tovisit. (C) Left ventricular hypertrophy is not an accurate measure of compliance because hypertrophy frequently does not decrease even with pharmacological management. (D) Therapeutic blood levels measure the drug level at the time of the test. There is no indication of compliance several days before testing. QUESTION 484 A 55-year-old woman entered the emergency room by ambulance. Her primary complaint is chest pain. She is receiving O2 via nasal cannula at 2 L/min for dyspnea. Which of the following findings in the client's nursing assessment demand immediate nursing action? A. Associated symptoms of indigestion and nausea B. Restlessness and apprehensiveness C. Inability to tolerate assessment session with the admitting nurse D. History of hypertension treated with pharmacological therapy Correct Answer: B Section: (none) Explanation Explanation/Reference: Explanation: (A) Indigestion or nausea may accompany angina or myocardial infarction, but they do not indicate imminent danger for the client. (B) Restlessness and apprehensiveness require immediate nursing action because they are indicative of very low oxygenation of body tissues and are frequently the first indication ofimpending cardiac or respiratory arrest. (C) It is common for the cardiac client to experience fatigue and inability to physically tolerate long assessment sessions. (D) A history of hypertension requires no immediate nursing intervention. In the situation described, the blood pressure is not given and therefore cannot be assumed to be elevated.QUESTION 485 A 48-year-old client is in the surgical intensive care unit after having had three-vessel coronary artery bypass surgery yesterday. She is extubated, awake, alert and talking. She is receiving digitalis for atrial arrhythmias. This morning serum electrolytes were drawn. Which abnormality would require immediate intervention by the nurse after contacting the physician? A. Serum osmolality is elevated indicating hemoconcentration.The nurse should increase IV fluid rate. B. Serum sodium is low. The nurse should change IV fluids to normal saline. C. Blood urea nitrogen is subnormal. The nurse should increase the protein in the client's diet as soon as possible. D. Serum potassium is low. The nurse should administer KCl as ordered. Correct Answer: D Section: (none) Explanation Explanation/Reference: Explanation: (A) An elevated serum osmolality poses no immediate danger and is not corrected rapidly. (B) A low serum sodium alone does not warrant changing IV fluids to normal saline. Other assessment parameters, such as hydration status, must be considered. (C) A low serum blood urea nitrogen is not necessarily indicative of protein deprivation. It may also be the result of overhydration. (D)A low serum potassium potentiates the effects of digitalis, predisposing the client to dangerous arrhythmias. It must be corrected immediately. QUESTION 486 A male client received a heart-lung transplant 1 month ago at a local transplant center. While visiting the nursing center to have his blood pressure taken, he complains of recent weakness and fatigue. He also tells the nurse that he is considering stopping his cyclosporine because it is expensive and is causing his face to become round. He fears he will catch viruses and be more susceptible to infections. The nurse responds to this last statement by explaining that cyclosporine: A. Is given to prevent rejection and makes him less susceptible to infection than other oral corticosteroids B. Is available at discount pharmacies for a reduced price C. Is usually not necessary after the first year following transplantation D. May initially cause weakness, dizziness, and fatigue, but these side effects will gradually resolve themselves Correct Answer: A Section: (none) Explanation Explanation/Reference: Explanation: (A) Cyclosporine is the immunosuppressive drug of choice. It provides immunosuppression but does not lower the white blood cell count; therefore, the client isless susceptible to infection. (B) Cyclosporine is available at discount pharmacies. The cost may be absorbed by health insurance, or Medicare, if the client is eligible. However, this statement does not address the entire problem verbalized by the client. (C) Immunosuppressive agents will be taken for the client's entire life because rejection can occur at any time. (D) These side effects do not necessarily resolve in time; however, the client may adapt. QUESTION 487 A 23-year-old college student seeks medical attention at the college infirmary for complaints of severe fatigue. Her skin is pale, and she reports exertional dyspnea. She is admitted to the hospital with possible aplastic anemia. Laboratory values reflect anemia, and the client is prepared for a bone marrow biopsy. She refuses to sign the biopsy consent and states, "Can't you just get the doctor to give me a transfusion and let me go. This weekend begins spring break, and I have plans to go to Florida." At this time the nurse's greatest concern is that: A. The client may contract an infection as a result of being exposed to large crowds at spring break B. The client does not grasp the full impact of her illness C. The client may require transfusion before leaving for spring break D. The causative agent be identified and treatment begun Correct Answer: B Section: (none) Explanation Explanation/Reference: Explanation: (A) The client could contract an infection, but at this point it is not the most pertinent issue. (B) The client's statement indicates that she does not grasp the full impact of her illness. Further client education must be given, along with allowing her to express her feelings regarding her illness. (C) The client may require a transfusion, but this is a temporary measure because the causative agent has not been identified. Her feelings regarding her illness must be addressed in order for care to continue. (D) A bone marrow is done first to make a definitive diagnosis; then treatment may begin. QUESTION 488 A 68-year-old man was recently diagnosed with endstage renal disease. He has not yet begun dialysis but is experiencing severe anemia with associated symptoms of dyspnea on exertion and chest pain. Which statement best describes the management of anemia in renal failure? A. Hematocrit levels usually remain slightly below normalin clients with renal failure. B. Transfusion is often begun as early as possible to prevent complications of anemia such as dyspnea and angina. C. Anemia in renal failure is frequently caused by low serum iron and ferritin and corrected by oral iron and ferritin replacement therapy. D. The renal secretion of erythropoiesis is decreased. The bone marrow requires erythropoietin to mature red blood cells. Correct Answer: D Section: (none) ExplanationExplanation/Reference: Explanation: (A) Clients in renal failure typically have very low hematocrits, often in the range of 1622%. (B) Transfusion is avoided unless the client exhibits acute symptoms such as dyspnea, chest pain, tachycardia, and extreme fatigue. When the client is given a transfusion, the bone marrow adjusts by producing less red blood cells. (C) Anemia in renal failure is caused primarily by decreased erythropoietin. Low serum iron and ferritin may aggravate the anemia and require treatment. (D) Decreased secretion of erythropoietin by the kidney is the primary cause of anemia. The bone marrow requires this hormone to mature red blood cells. Treatment is with replacement therapy. QUESTION 489 A female client has married recently. A month ago she visited her physician with complaints of burning on urination. She was given a prescription for trimethoprimsulfamethoxazole (Bactrim) DS bid for 10 days. She was admitted through the emergency room on Saturday evening complaining of flank pain. Her temperature was 104_F. A preliminary urinalysis revealed 31 bacteria along with red and white blood cells in the urine. A preliminary diagnosis of pyelonephritis was made. During a nursing admission assessment, which statement by the client demonstrates a possible cause for pyelonephritis? A. "I have not been drinking six to eight glasses of water each day as the nurse had instructed." B. "I'm afraid I may have something wrong with my bladder because I have been getting bladder infections frequently since I've been married." C. "I took the Bactrim for 6 or 7 days. The burning stopped, so I saved the rest of the medication for the next time." D. "I recently had the flu, which could be settling in my kidneys now." Correct Answer: C Section: (none) Explanation Explanation/Reference: Explanation: (A) Although it is important that the client drink adequate fluids while treating a bladder infection with trimethoprimsulfamethoxazole, the failure to do so will not cause pyelonephritis. (B) A stricture or abnormality may cause the progression of bladder infection to urinary tract infection, but this is rare. There is no indication in this situation that this has occurred. (C) The most common cause of pyelonephritis is improper treatment of bladder infections. The client typically feels better afterseveral days, discontinues the medication, and saves the remainder forthe next occurrence of a bladder infection. For this reason, it is imperative to provide client education related to completion of the prescribed medication. (D) There is no evidence that infection in another body system could cause pyelonephritis. QUESTION 490 A male client was diagnosed 6 months ago with amyotrophic lateral sclerosis (ALS). The progression of the disease has been aggressive. He is unable to maintain his personal hygiene without assistance. Ambulation is most difficult, requiring him to use a wheelchair and rely on assistance for mobility. He recently has become severely dysphasic. Nursing interventions for dysphasia would be aimed toward prevention of: A. Loss of ability to speak and communicate effectively B. Aspiration and weight lossC. Secondary infection resulting from poor oral hygiene D. Drooling Correct Answer: B Section: (none) Explanation Explanation/Reference: Explanation: (A) Loss of ability to speak is not dysphasia. Although the client may have difficulty communicating, alternative measures can be developed to enhance communication. This goal, while important, is of a lesser priority. (B) Dysphasia is difficulty swallowing, which could result in aspiration of food and inability to eat, causing weight loss. (C) A secondary infection could result from poor oral hygiene, which could enhance the client's inability to eat, but this goal is of a lesser priority. (D) Drooling normally occurs in clients with amyotrophic lateral sclerosis and may require suctioning. Drooling, while aggravating for the client, does notpose an immediate danger. QUESTION 491 A 70-year-old female client is admitted to the medical intensive care unit with a diagnosis of cerebrovascular accident (CVA). She is semicomatose, responding to pain and change in position. She is unable to speak or cough. In planning her nursing care for the first 24 hours following a CVA, which nursing diagnosis should receive the highest priority? A. Ineffective airway clearance related to immobility, ineffective cough, and decreased level of consciousness B. Altered cerebral tissue perfusion related to pathophysiological changes that decrease blood flow C. Potential for injury related to impaired mobility and seizures D. Impaired verbal communication related to aphasia Correct Answer: A Section: (none) Explanation Explanation/Reference: Explanation: (A) An effective airway is necessary to prevent hypoxia and subsequent cardiac arrest. (B) Cerebral tissue perfusion is necessary to preserve remaining cerebral tissue, but this goal is secondary to maintenance of an effective airway. (C) While prevention of injury is important, it is secondary to maintaining an effective airway and cerebral tissue perfusion. (D) Impaired verbal communication is not life threatening in the acute phase of recovery. It is the lowest priority of the nursing diagnoses listed. QUESTION 492 A 32-year-old female client is being treated for Guillain- Barré syndrome. She complains of gradually increasing muscle weakness over the past several days. Shehas noticed an increased difficulty in ambulating and fell yesterday. When conducting a nursing assessment, which finding would indicate a need for immediate further evaluation? A. Complaints of a headache B. Loss of superficial and deep tendon reflexes C. Complaints of shortness of breath D. Facial paralysis Correct Answer: C Section: (none) Explanation Explanation/Reference: Explanation: (A) Headaches are not associated with Guillain-Barré syndrome. (B) Loss of superficial and deep tendon reflexes is expected with this diagnosis. (C) Complaints of shortness of breath must be further evaluated. Forty percent of all clients have some detectable respiratory weakness and should be prepared for a possible tracheostomy. Pneumonia is also a common complication of this syndrome. (D) Facial paralysis is expected and is not considered abnormal. QUESTION 493 A 19-year-old male client arrived via ambulance to the emergency room following a motorcycle accident. He is comatose. His face has evidence of dried blood. On assessment, the nurse notes an obvious injury to his left eye. The preferred positioning for a client with an obvious eye injury is: A. Reclining to control bleeding B. Any position in which the client is comfortable C. Side-lying, either left or right D. Sitting with head support Correct Answer: D Section: (none) Explanation Explanation/Reference: Explanation: (A) A reclining position can cause a penetrating object to advance further into the eye. (B) Prevention of further injury is the priority, not comfort. (C) A side-lying position may increase intraocular and intracranial pressure if an accompanying head injury is suspected. (D) A sitting position with the head supported will prevent further injury while allowing injury care to take place. QUESTION 494A female client has been hospitalized for several months following major abdominal surgery for a ruptured colon. A colostomy was created, and the large abdominal wound was left open and allowed to heal through granulation. She is receiving gentamicin IV for treatment of wound infection. Knowing this drug is ototoxic, the nurse would implement which of the following measures? A. Instruct the client to report any signs of tinnitus, dizziness or difficulty hearing. B. Advise the client to discontinue the drug at the first sign of dizziness. C. Order audiometric testing in order to determine if hearing loss is caused by an ototoxic drug or other cause. D. Instruct the client in Valsalva's maneuver to equalize middle ear pressure and to prevent hearing loss. Correct Answer: A Section: (none) Explanation Explanation/Reference: Explanation: (A) The first nursing measure is to instruct the client in which drug side effects to report. (B) Discontinuing the drug is not an independent nursing intervention and may compromise client care. (C) Audiometric testing will detect hearing loss, but it does not indicate a potential cause. (D) Equalizing middle ear pressure will not prevent hearing loss. QUESTION 495 A male client has experienced low back pain for several years. He is the primary support of his wife and six children. Although he would qualify for disability, he plans to continue his employment as long as possible. His back pain has increased recently, and he is unable to control it with non-steroidal anti-inflammatory agents. He refuses surgery and cannot take narcotics and remain alert enough to concentrate at work. His physician has suggested application of a transcutaneous electrical nerve stimulation (TENS) unit. Which of the following is an appropriate rationale for using a TENS unit for relief of pain? A. TENS units have an ultrasonic effect that relaxes muscles, decreases joint stiffness, and increases range of motion. B. TENS units produce endogenous opioids that affect the central nervous system with analgesic potency comparable to morphine. C. TENS units work on the gate-control theory of pain; biostimulation therapy of large fibers block painful stimuli. D. TENS units prevent muscle spasms, decrease the potential for further injury, and minimize pressure on joints. Correct Answer: C Section: (none) Explanation Explanation/Reference: Explanation: (A) TENS units do not have this effect, but whirlpool therapy does. (B) TENS units do not produce endogenous opioids, only the body can do that. (C) TENS units do work based on the gatecontrol theory of pain control. (D) TENS units do not have this effect, but possibly changing the client's position would.QUESTION 496 A male client had a right below-the-knee amputation 4 days ago. His incision is healing well. He has gotten out of bed several times and sat at the side of the bed. Each time after returning to bed, he has experienced pain as if it were located in his right foot. Which nursing measure indicates the nurse has a thorough understanding of phantom pain and its management? A. Phantom pain is entirely in the client's mind. The client should be instructed that the pain is psychological and should not be treated. B. The basis for phantom pain may occur because the nerves still carry pain sensation to the brain even though the limb has been amputated. The pain is real, intense, and should be treated. C. The cause of phantom pain is unknown. The nurse should provide the client with support, promote sleep, and handle the injured limb smoothly and gently. D. Phantom pain is caused by trauma, spasms, and edema at the incisional site. It will decrease when postoperative edema decreases. It should be treated with nonnarcotic medication whenever possible. Correct Answer: B Section: (none) Explanation Explanation/Reference: Explanation: (A) This statement is entirely false. (B) Phantom pain may be caused by nerves continuing to carry sensation to the brain even though the limb is removed. It is real, intense, and should be treated as ordinary pain would. (C) Although the cause of phantom pain is still unknown, thesemeasures may promote the relief of any type of pain, not just phantom pain. (D) Phantom pain is not caused by trauma, spasms, and edema and will not be relieved by decreasing edema. QUESTION 497 A 28-year-old woman was admitted to the hospital for a thyroidectomy. Postoperatively she is taken to the postanesthesia care unit for several hours. In preparing for the client's return to her room, which nursing measure best demonstrates the nurse's thorough understanding of possible postthyroidectomy complications? A. Dressings are placed at the bedside for dressing changes, which are to be done every 2 hours to best detect postoperative bleeding. B. Narcotics are readily available and administered when the client returnsto her room to prevent excruciating pain. C. A tracheostomy set, O2, and suction are available at the bedside. D. The nurse should instruct the client as soon as possible on alternative means of communication. Correct Answer: C Section: (none) Explanation Explanation/Reference: Explanation:(A) Dressing changes are done as necessary for bleeding. However, frequently, post- thyroidectomy bleeding may not be visible on the dressing, but blood may drain down the back of the neck by gravity. (B) Narcotics are administered for acute pain as necessary. They are not necessarily given on return of the client to her room. (C) The most serious postthyroidectomy complication is ineffective airway and breathing pattern related to tracheal compression and edema. A tracheostomy set, O2, and suction should be available at bedside for at least the first 24 hours postoperatively. (D) Impaired verbal communication may occur due to laryngeal edema or nerve damage, but most commonly, it occurs due to endotracheal intubation. The client is usually able to communicate but is hoarse. QUESTION 498 A male client is diagnosed with hypoparathyroidism. He has been on dialysis for several years. He is experiencing symptoms such as numbness of the lips, muscle weakness, carpopedal spasms, and wheezing. Given the client's symptoms, nursing assessment would focus on: A. Detection of tetany B. Detection of hypocalcemia to prevent seizures C. Evidence of depression D. Detection of premature cataract formation Correct Answer: A Section: (none) Explanation Explanation/Reference: Explanation: (A) Assessment should focus on detection of tetany, which is the most common symptom of hypoparathyroidism. Left undetected and untreated, tetany resulting from hypocalcemia can progress to seizures. (B) Hypocalcemia is difficult to detect on nursing assessment alone. Abdominal cramping may be an indication of hypocalcemia, but laboratory data are required to confirm diagnosis. (C) Depression can be a symptom of hypoparathyroidism, but it is not definitive. (D) Premature cataract formation can occur, but it also is not specific to parathyroidism and poses no immediate danger to the client. Topic 5, Questions Set E QUESTION 499 A male client has been an insulin-dependent diabetic for approximately 30 years. He frequently indulges in highsugar foods and forgets to take his insulin. He has not experienced acute diabetic emergencies over the years but is now beginning to demonstrate symptoms of diabetic peripheral neuropathy. This distresses him because dancing is one of his favorite pastimes. He decides to question his wife's home health nurse about diabetic peripheral neuropathy. The nurse points out his noncompliance to his diabetic diet and insulin regimen. The client answers the nurse, "It has been my experience that the diabetic diet is very difficult to follow. As far as the insulin, isn't a fellow allowed to forget now and then?" The client's actions and response best demonstrate: A. Depression B. Anger C. Denial D. BargainingCorrect Answer: C Section: (none) Explanation Explanation/Reference: Explanation: (A) Depression may be an underlying feature, but it is not evident from limited data presented here. (B) Anger is not exhibited in his response. (C) Denial is evident in the client's actions; through the years, he has had a casual approach to his illness. He only becomes concerned when bodily changes affect his present lifestyle, when in fact he should have been concerned all along. His verbal response also reflects denial. (D) There is no evidence of bargaining in the client's actions or verbal response. QUESTION 500 A female client was recently diagnosed with gastric cancer. She entered the hospital and had a total gastrectomy with esophagojejunostomy. Her postoperative recovery was uneventful. On conducting discharge teaching, the nurse discusses changes in bodily function and lifestyle changes with the client. In order to prevent pernicious anemia, the nurse stresses that the client must: A. Receive monthly blood transfusions B. Increase the amount of iron in her diet C. Eat small quantities several times daily until she is able to tolerate food in moderate portions D. Understand the need for Vitamin B12 replacement therapy Correct Answer: D Section: (none) Explanation Explanation/Reference: Explanation: (A) Monthly blood transfusions are not indicated postgastrectomy. (B) Increasing iron in the client's diet may cause irritation and will not alleviate pernicious anemia. (C) It may be necessary that the client eat small meals several times per day, but this measure has no relevance to prevention of pernicious anemia. (D) Pernicious anemia is caused by lack of Vitamin B12, and replacement therapy will be necessary because the client's stomach has been removed. QUESTION 501 A female client was employed as a client care technician in a hemodialysis unit. She recently began to experience extreme fatigue, being able to sleep for 1620 hours at a time. She also noted that her urine was tea colored, which she rationalized was a result of the vitamins she began taking to alleviate fatigue. She was diagnosed with hepatitis B. After a brief hospital stay, she is discharged to her parent's home. Her mother asks the nurse if any precautions are necessary to prevent transmission to the client's family. The nurse explains necessary precautions, which include:A. Isolation of the client from the remainder of the family B. Separate bathroom facilities if possible; if not, then cleansing daily of the facilities with a chloride solution C. No necessary precautions because she is beyond the contagious phase D. Laundering clothes separately in cold water with a chloride solution Correct Answer: B Section: (none) Explanation Explanation/Reference: Explanation: (A) Isolation is not necessary, even in the acute phase. (B) Separate bathroom facilities are recommended. If unavailable, daily cleansing with a chloride solution is recommended. (C) Precautions continue to be necessary while the client is in the active phase of hepatitis. (D) Clothes are to be laundered separately in hot water with a chloride solution. QUESTION 502 A male client is admitted to the medical-surgical unit from the emergency room with a diagnosis of acute pancreatitis. The nurse performs the admission nursing assessment. He is NPO with IV fluids infusing at 100 mL/hour. He is experiencing excruciating abdominal pain. Based on an analysis of these data, which nursing diagnosis would receive the highest priority? A. Pain related to stimulation of nerve endings associated with obstruction of the pancreatic tract B. Fluid volume deficit related to vomiting and nasogastric tube drainage C. Knowledge deficit related to treatment regimen D. Altered nutrition: less than body requirements, related to inadequate intake associated with current anorexia, nausea, vomiting, and digestive enzyme loss Correct Answer: A Section: (none) Explanation Explanation/Reference: Explanation:http://www.gratisexam.com/ (A) Relief of pain is the primary goal of nursing intervention because this client is experiencing acute pain. (B) Fluid volume deficit is being treated with IV fluid replacement. (C) Knowledge deficit will not be addressed at this time because a client in acute pain is not ready to learn. (D) Alteration in nutrition is the third priority after relief of pain and fluid volume deficit. QUESTION 503 A male client has burns over 90% of his body after an automobile accident resulting in a fire. He was trapped inside the auto and pulled out by a bystander. After several months in the hospital and over 20 surgeries, discharge planning has begun. Throughout his hospitalization the nursing staff has been aware of psychological changes the client faces after burns over a large portion of his body resulting in disfigurement. The nursing staff can best foster the client's self- esteem by: A. Adhering to a strict schedule of diet, exercise, and wound care B. Allowing him to go to physical therapy for whirlpool treatment when other clients were not in physical therapy C. Following a standardized plan of care for burn clients formulated by a world-renowned burn center D. Allowing him to plan, assist in, and perform his own care whenever possible Correct Answer: D Section: (none) Explanation Explanation/Reference: Explanation: (A) A regimented schedule, allowing no flexibility, will not foster the client's self-esteem. (B) Isolating the client may only enhance his feelings of social isolation due to his disfigurement. (C) Standardized care plans must be personalized and adapted to each client's situation. (D) Allowing the client control over his care will foster his self-esteem and prepare him for life outside of the hospital. QUESTION 504 A 20-year-old client presents to the obstetrics-gynecology clinic for the first time. She tells the nurse that she is pregnant and wants to start prenatal care. After collecting some initial assessment data, the nurse measures her fundal height to be at the level of the umbilicus. The nurse estimates the fetal gestational age to be approximately: A. 10 weeksB. 16 weeks C. 20 weeks D. 30 weeks Correct Answer: C Section: (none) Explanation Explanation/Reference: Explanation: (A) At 10 weeks, the fundus is located slightly above the symphysis pubis. (B) At 16 weeks, the fundus is halfway between the symphysis pubis and the umbilicus. (C) At 20 weeks, the fundus is located approximately at the umbilicus. (D) At 30 weeks, the fundal height is about 30 cm, or 10 cm above the umbilicus. QUESTION 505 A female client presents to the obstetric-gynecology clinic for a pregnancy test, the result which turns out to be positive. Her last menstrual period began December 10, 1993. Using Nägele's rule, the nurse estimates her date of delivery to be: A. September 17, 1994 B. September 10, 1994 C. September 3, 1994 D. August 17, 1994 Correct Answer: A Section: (none) Explanation Explanation/Reference: Explanation: (A) According to Nägele'srule, the estimated date of delivery is calculated by adding 7 days to the date of the first day of the normal menstrual period (December 10 + 7 days = December 17), and then by counting back 3 months (December 17 -3 mo = September 17). (B, C, D) These answers are incorrect. QUESTION 506 A female client comes for her second prenatal visit. The nurse-midwife tells her, "Your blood tests reveal that you do not show immunity to the German measles." Which notation will the nurse include in her plan of care for the client? "Will need . . . A. Rh-immune globulin at the next visit" B. Rh-immune globulin within 3 days of delivery"C. Rubella vaccine at the next visit" D. Rubella vaccine after delivery on the day of discharge" Correct Answer: D Section: (none) Explanation Explanation/Reference: Explanation: (A) Rh immune globulin is given to Rh-negative mothers to prevent the maternal Rh immune response. (B) Rh immune globulin is given to Rh-negative mothers to prevent the maternal Rh immune response. (C) The rubella vaccine is not given during pregnancy because of its teratogenicity. (D) Nonimmune mothers are vaccinated early in the postpartum period to prevent future infection with the rubella virus. QUESTION 507 A female client at 37 weeks' gestation has just undergone a nonstress test. The results were two fetal movements with a corresponding increase in fetal heart rate (FHR) of 15 bpm lasting 15 seconds within a 20-minute period. Her results would be classified as: A. Reactive; needs follow-up contraction stress test B. Reactive; no contraction stress test required C. Non-reactive; needs follow-up contraction stress test D. Non-reactive; no contraction stress test required Correct Answer: B Section: (none) Explanation Explanation/Reference: Explanation: (A) A contraction stress test is unnecessary following a reactive (normal) nonstress test. (B) The results are considered reactive, indicating that the fetus is not showing distress. Therefore, a contraction stress test, which is a more in-depth test for fetal distress, is unnecessary. (C) A nonreactive test would show fewer than two fetal movements or a failure of the FHR to increase at least 15 bpm with the movements in a 20-minute period. (D) A contraction stress test should follow a nonreactive nonstress test to validate fetal distress. QUESTION 508 A female client at 36 weeks' gestation has been treated successfully for premature labor for 4 weeks. She has begun having uterine contractions today and has been admitted to the labor and delivery suite. Her amniocentesis results reveal a lecithin/sphingomyelin (L/S) ratio of 2 and positive phosphatidylglycerol (PG).These lab values indicate:A. Placental maturity B. Suspected chronic asphyxia C. Cord compression D. Fetal lung maturity Correct Answer: D Section: (none) Explanation Explanation/Reference: Explanation: (A) Placental maturity is assessed by a biophysical profile. (B) L/S ratio and presence of phosphatidylglycerol are not used to determine fetal asphyxia. A biophysical profile score of6 may indicate this condition. (C) Cord compression is not reflected by the L/S ratio or presence of phosphatidylglycerol. Variable decelerations observed through electronic fetal monitoring could reflect umbilical cord compression. (D) An L/S ratio>2 and the presence of phosphatidylglycerol in amniotic fluid indicate fetal lung maturity. QUESTION 509 A primigravida with a blood type A negative is at 28 weeks' gestation. Today her physician has ordered a RhoGAM injection. Which statement by the client demonstrates that more teaching is needed related to this therapy? A. "I'm getting this shot so that my baby won't develop antibodies against my blood, right?" B. "I understand that if my baby is Rh positive I'll be getting another one of these injections." C. "This shot should help to protect me in future pregnancies if this baby is Rh positive, like my husband." D. "This shot will prevent me from becoming sensitized to Rh-positive blood." Correct Answer: A Section: (none) Explanation Explanation/Reference: Explanation: (A) RhoGAM is given to Rh-negative mothers to prevent the maternal Rh immune response to fetal Rh-positive antigens. (B) If the infant is Rh positive, the mother will receive another dose postdelivery to prevent maternal sensitization. (C) Prevention of maternal sensitization will protect future pregnancies because the mother's blood will be free of antibodies against her fetus. (D) RhoGAM prevents maternal sensitization to Rh-positive blood. QUESTION 510 At her monthly prenatal visit, a client reports experiencing heartburn. Which nursing measure should be included in her plan of care to help alleviate it?A. Restrict fluid intake. B. Use Alka-Seltzer as necessary. C. Eat small, frequent bland meals. D. Lie down after eating. Correct Answer: C Section: (none) Explanation Explanation/Reference: Explanation: (A) At least eight glasses of fluid per day are encouraged to help dilute stomach contents, thereby decreasing irritation. (B) Alka Seltzer contains aspirin, which is irritating to gastric mucosa, and therefore should be avoided. (C) Small, frequent bland meals help to decrease gastric pressure and to prevent reflux. (D) Lying down after meals may cause gastric reflux and prevents optimal gastric emptying. QUESTION 511 A client is dilated 8 cm and entering the transition phase of labor. Common behaviors of the laboring woman during transition are: A. Frustration, vague in communication B. Seriousness, some difficulty following directions C. Calmness, follows directions easily D. Excitement, openness to instructions Correct Answer: A Section: (none) Explanation Explanation/Reference: Explanation: (A) During the transition phase, the mother may become frustrated and unclear in her communication owing to severe pain and fear of loss of control. (B) These behaviors are common in the active phase of labor. (C) These behavioral clues are seen in the latent phase of labor. (D) These characteristics are observed in the latent phase of labor. QUESTION 512 The FHR pattern in a laboring client begins to show early decelerations. The nurse would best respond by: A. Notifying the physician B. Changing the client to the left lateral positionC. Continuing to monitor the FHR closely D. Administering O2 at 8 L/min via face mask Correct Answer: C Section: (none) Explanation Explanation/Reference: Explanation: (A) Early decelerations are reassuring and do not warrant notification of the physician. (B) Because early decelerations is a reassuring pattern, it would not be necessary to change the client's position. (C) Early decelerations warrant the continuation of close FHR monitoring to distinguish them from more ominous signs. (D) O2 is not warranted in this situation, but it is warranted in situations involving variable and/or late decelerations. QUESTION 513 A female client is admitted to the emergency department complaining of severe right-sided abdominal pain and vaginal spotting. She states that her last menstrual period was about 2 months ago. A positive pregnancy test result and ultrasonography confirm an ectopic pregnancy. The nurse could best explain to the client that her condition is caused by: A. Abnormal development of the embryo B. A distended or ruptured fallopian tube C. A congenital abnormality of the tube D. A malfunctioning of the placenta Correct Answer: B Section: (none) Explanation Explanation/Reference: Explanation: (A) The embryo itself may develop normally in the first several weeks of an ectopic pregnancy. (B) An ectopic pregnancy in the fallopian tube causes severe pain owing to the size of the growing embryo within the narrow lumen of the tube, causing distention and finally rupture within the first 12 weeks of pregnancy. (C) The Fallopian tube may either be normal or contain adhesions caused by a history of pelvic inflammatory disease or tubal surgeries, neither of which are congenital causes. (D) An ectopic pregnancy does not involve a dysfunctional placenta, but the implantation of the blastocyst outside the uterus. QUESTION 514 A female client at 10 weeks' gestation complains to her physician of slight vaginal bleeding and mild cramps. On examination, her physician determines that her cervix is closed. The client is exhibiting signs of:A. An inevitable abortion B. A threatened abortion C. An incomplete abortion D. A missed abortion Correct Answer: B Section: (none) Explanation Explanation/Reference: Explanation: (A) An inevitable abortion includes the signs of cervical dilation and effacement as well as pain and bleeding. (B) A threatened abortion is a condition in which intrauterine bleeding occurs early in pregnancy, the cervix remains undilated, and the uterine contents are not necessarily expelled. (C) An incomplete abortion occurs when some portions of the products of conception are expelled from the uterus. (D) A missed abortion occurs when the embryo dies in utero and is retained in the uterus. QUESTION 515 A female client at 36 weeks' gestation is experiencing preterm labor. Her physician has prescribed two doses of betamethasone 12 mg IM q24h. The nurse explains that she is receiving this drug to: A. Treat fetal respiratory distress syndrome B. Prevent uterine infection C. Promote fetal lung maturation D. Increase uteroplacental circulation Correct Answer: C Section: (none) Explanation Explanation/Reference: Explanation: (A) Respiratory distress syndrome occurs in the newborn, not the fetus. It may be treated postnatally with surfactant therapy. (B) Betamethasone is a corticosteroid, not an antiinfective drug; therefore, its use would not prevent uterine infection. (C) Betamethasone binds with glucocorticoid receptors in alveolar cells to increaseproduction of surfactant, thus increasing lung maturity in the preterm fetus. (D) Betamethasone does not affect uteroplacental circulatory exchange. QUESTION 516 A female client at 30 weeks' gestation is brought into the emergency department after falling down a flight of stairs. On examination, the physician notes a rigid, boardlike abdomen; FHR in the 160s; and stable vital signs. Considering possible abdominal trauma, which obstetric emergency must be anticipated?A. Abruptio placentae B. Ectopic pregnancy C. Massive uterine rupture D. Placenta previa Correct Answer: A Section: (none) Explanation Explanation/Reference: Explanation: (A) Abruptio placentae, the complete or partial separation of the placenta from the uterine wall, can be caused by external trauma. When hemorrhage is concealed, one sign is a rapid increase in uterine size with rigidity. (B) Ectopic pregnancy occurs when the embryo implants itself outside the uterine cavity. (C) Massive uterine rupture occurs during labor when the uterine contents are extruded through the uterine wall. It is usually due to weakness from a pre-existing uterine scar and trauma from instruments or an obstetrical intervention. (D) Placenta previa is the condition in which the placenta is implanted in the lower uterine segment and either completely or partially covers the cervical os. QUESTION 517 A 4 days postpartum client who is gravida 3, para 3, isexamined by the home health nurse during her first postpartum home visit. The nurse notes that she has a pink vaginal discharge with a serosanguineous consistency. The nurse would most accurately chart the client's lochia as: A. Rubra B. Rosa C. Serosa D. Alba Correct Answer: C Section: (none) Explanation Explanation/Reference: Explanation: (A) Lochia rubra is bloody with clots and occurs 13 days postpartum. (B) There is no such term as lochia rosa. (C) Lochia serosa is a pink-brown discharge with a serosanguineous consistency that occurs 49 days postpartum. (D) Lochia alba is yellow to white in color and occurs approximately 10 days postpartum. QUESTION 518 A primipara is assessed on arrival to the postpartum unit. The nurse finds her uterus to be boggy.The nurse's first action should be to: A. Call the physician B. Assess her vital signs C. Give the prescribed oxytocic drug D. Massage her fundus Correct Answer: D Section: (none) Explanation Explanation/Reference: Explanation: (A) The nurse should first implement independent and dependent measures to achieve uterine tone before calling the physician. (B) Assessment of vital signs will not help to restore uterine atony, which is the priority need. (C) Giving a prescribed oxytocic drug would be necessary ifthe uterus did not maintain tone with massage. (D) Fundal massage generally restores uterine tone within a few moments and should be attempted first. QUESTION 519 A female client plansto bottle-feed her newborn. Her physician has ordered bromocriptine (Parlodel) to suppress lactation. Which of the following instructions about bromocriptine should be given by the nurse? A. Bromocriptine stimulates the production of prolactin. B. Hypertension is a primary side effect. C. Bromocriptine is generally taken for 5 days. D. Her blood pressure must be stable before starting bromocriptine. Correct Answer: D Section: (none) Explanation Explanation/Reference: Explanation: (A) Bromocriptine inhibits the secretion of prolactin. (B) Hypotension is a side effect of this drug; hypertension is not. (C) Bromocriptine is generally taken for 14 days. (D) The administration of bromocriptine is delayed at least 4 hours postpartum and given only when the client's blood pressure is stable, because it can cause hypotension and syncope. QUESTION 520 The postpartum nurse should include which of the following instructions to breast-feeding mothers?A. Limit feeding times for several days to avoid nipple soreness. B. Wash the nipples with soap and water before and after each feeding. C. Daily caloric intake should be increased by 500 cal. D. Breast milk is totally digestible by the baby because it contains lactose. Correct Answer: C Section: (none) Explanation Explanation/Reference: Explanation: (A) Limiting initial feeding times will only delay nipple soreness as well as the establishment of the letdown reflex, thus encouraging engorgement from clogged ducts and ductules. (B) Soap should be avoided because it may be excessively drying, predisposing nipples to cracking. (C) For optimal milk production, an additional 500 kcal over maintenance levels are needed daily. (D) Lipase, not lactose, emulsifies the fat in breast milk, making it almost totally digestible by infants. QUESTION 521 At 12 hours postvaginal delivery, a female client is without complications. Which of the following assessment findings would warrant further nursing interventions? A. Apical pulse of 52 bpm B. Uterine fundus palpable left of midline C. No bowel movement since delivery D. Oral temperature of 100.4F Correct Answer: B Section: (none) Explanation Explanation/Reference: Explanation: (A) Bradycardia of 5070 bpm may be considered normal postpartally because the heart compensates for the decreased resistance in the pelvis. (B) The uterus isdisplaced from the midline by a full bladder. This condition could lead to a boggy uterus and increased risk of postpartal hemorrhage; therefore, the bladder should be kept empty. (C) Re-establishment of normal bowel function is delayed into the first postpartum week. (D) A postpartum woman's oral temperature may go as high as 100.4_F within 24 hours of delivery resulting from muscular exertion, dehydration, and hormonal changes. QUESTION 522 The nurse observes a client crying quietly. She has just experienced a spontaneous abortion at nine weeks' gestation. An appropriate response by the nurse would be:A. "It must be God's will and probably is for the best." B. "This must be a difficult time for you. Would you like to talk about it?" C. "I'm sure your other children will be a comfort for you." D. "Don't worry, you're still young. If I were you I'd just try again." Correct Answer: B Section: (none) Explanation Explanation/Reference: Explanation: (A) Thisresponse is nontherapeutic because it belittles the client's response and gives a meaningless rationalization. (B) This response acknowledges the client's feelings and demonstrates the therapeutic offering of self by the nurse. (C) This response is nontherapeutic because it does not focus on the client's feelings andoffers false reassurance. (D) This response is nontherapeutic because it belittles the client's feelings and offers her advice. QUESTION 523 A 48-hour-old male infant is ordered to have phototherapy. When his mother questions the nurse about its purpose, the nurse explainsthat phototherapy: A. Prevents the development of ophthalmia neonatorum B. Assists the baby's clotting mechanism C. Breaks down bilirubin in the skin into substances that can be excreted in stool or urine D. Increaseslevels of unconjugated bilirubin, thereby preventing kernicterus (brain damage) Correct Answer: C Section: (none) Explanation Explanation/Reference: Explanation: (A) The instillation of erythromycin ophthalmic preparation, not phototherapy, prevents ophthalmia neonatorum. (B) The administration of vitamin K (AquaMEPHYTON) assists the infant's clotting mechanism. (C) Excessive bilirubin accumulates when the infant's liver cannothandle the increased load caused by the breakdown of red blood cells postnatally. This excessive bilirubin seeps out of the blood and into the tissues, staining them yellow. Phototherapy accelerates the removal of bilirubin from the skin by breaking it down into substances that can be excreted in stool or urine. (D) Phototherapy decreases levels of unconjugated bilirubin, thereby preventing kernicterus. QUESTION 524 After instructing a female client on circumcision care, the nursery nurse asks her to restate some of the key points covered. Which statement shows that the client will properly care for her son's circumcision?A. "I'll make sure I soak the gauze with warm water first, before I take it off each time." B. "I'll make sure that I report any drainage around where they operated." C. "I'll apply alcohol to the area daily to clean it and prevent any infection." D. "I'll keep a close watch on it for a day or two." Correct Answer: A Section: (none) Explanation Explanation/Reference: Explanation: (A) Before petrolatum gauze is removed, it should be soaked with warm water to prevent trauma to adherent tissues. (B) A yellow exudate often forms normally over the surgical site. Only if it becomes foul-smelling and purulent would it need to be reported. (C) Alcohol should never be used on the site; this would be extremely painful to the infant. (D) Special care and observance should continue until the site is completely covered with clean, pink granulation tissue, which could take 710 days. QUESTION 525 A male infant is to be discharged home this morning. Which instruction related to his cord care should be included in his mother's discharge teaching plan? A. Keep the umbilical area moist with Vaseline until the stump falls off. B. Keep the umbilical area covered at all times with the diaper. C. Clean the umbilical cord with alcohol at each diaper change. D. Clean the umbilical cord daily with soap and water during the bath. Correct Answer: C Section: (none) Explanation Explanation/Reference: Explanation: (A) The umbilical area should be kept dry for healing to occur. Moisture is conducive to bacterial growth and therefore could lead to infection at the site. (B) The diaper should be folded below the cord to allow the cord stump to be exposed to the air for healing. (C) The umbilical cord should be swabbed with alcohol at eachdiaper change to remove urine and stool and to facilitate the desiccation process through drying. (D) Soap and water should not be used to clean the umbilical area because the area could retain moisture, thus making it susceptible to bacterial growth and infection. QUESTION 526 Which behavior by a female client feeding her newborn demonstrates that she needs more teaching related to safety and infant feeding?A. She uses the bulb syringe to help clear her baby's nose when milk is regurgitated. B. She places her infant on her right side after feeding her. C. She props the bottle in the crib to feed her baby,which allows her to write birth announcements and feed her baby at the same time. D. She burps her baby by placing her in a sitting position, supporting her head and neck and gently massaging her back. Correct Answer: C Section: (none) Explanation Explanation/Reference: Explanation: (A) This practice is the proper use of the bulb syringe to clear the infant's airway in case of regurgitation. (B) Placing the infant on either side or on the stomach prevents aspiration of regurgitated milk. (C) "Bottle propping" is an unsafe practice because it increases the likelihood of aspiration. (D) This practice is one correct way of burping an infant. QUESTION 527 Newborns are routinely screened for phenylketonuria. The nursery nurse ensures that this screening test is performed: A. Immediately after birth, because the most accurate result is obtained at this time B. After 23 days of milk ingestion C. At 23 days of age regardless of amount of milk feedings D. At 1 month, because the biochemical buildup of phenylalanine takes 1 month to detect Correct Answer: B Section: (none) Explanation Explanation/Reference: Explanation: (A) The infant has not ingested any protein immediately after birth, which is necessary to detect excessive serum phenylalanine. (B) It is important that the infant take in 23 full days of milk or formula feedings to preclude a false-negative reading. (C) At 23 days of age, inadequate milk could have been ingested owing to a delay in the initial feeding. (D) The biochemical buildup of serum phenylalanine is detectable after 23 days of milk or formula ingestion. QUESTION 528 A 3-year-old child has had symptoms of influenza including fever, productive cough, nausea, vomiting, and sore throat for the past several days. In caring for a young child with symptoms of influenza, the mother must be cautioned about:A. Giving aspirin and bismuth subsalicylate (Pepto-Bismol) to treat the symptoms B. Giving clear liquids too soon C. Allowing the child to come in contact with other children for 3 days D. The possibility of pneumonia as a complication Correct Answer: A Section: (none) Explanation Explanation/Reference: Explanation: (A) Aspirin should never be given to children with influenza because of the possibility of causing Reye's syndrome. Pepto- Bismol is also classified as a salicylate and should be avoided. (B) Depending on the severity of symptoms, the child may be receiving IV therapy or clear liquids. (C) The disease has a 13 day incubation period and affected children are most infectious 24 hours before and after the onset of symptoms. (D) Although viral pneumonia can be a complication of influenza, this would not be an initial priority. QUESTION 529 A 10-year-old boy has been diagnosed with Legg-Calvé Perthes disease. Which of the client'sresponses would indicate compliance during initial therapy? A. Drinking large amounts of milk B. Not bearing weight on affected extremity C. Walking short distances 3 times/day D. Putting self on weight reduction diet Correct Answer: B Section: (none) Explanation Explanation/Reference: Explanation: (A) This condition causes aseptic necrosis of the head of the femur in the acetabulum. Drinking large quantities of milk at this time cannot hasten recovery. (B) The aim of treatment is to keep the head of the femur in the acetabulum. Nonweight-bearing is essential. Activity causes microfractures of the epiphysis. (C) In addition to nonweight-bearing, clients are often placed on bedrest, which helps to reduce inflammation. Later, active motion is encouraged. (D) Weight is not generally an issue with this disease. Slipped femoral capital epiphysis, which is most frequently observed in obese pubescent children, usually requires a weight reduction diet. QUESTION 530 A 7-year-old girl has been diagnosed with juvenile arthritis and has been placed on daily aspirin. Which statement made by the parent indicates a need for further teaching?A. "My daughter takes her aspirin with her meals." B. "Her gums have been bleeding frequently. Maybe she is brushing too hard." C. "I give her aspirin on a regular schedule every day." D. "One sign of aspirin toxicity can be ringing in the ears." Correct Answer: B Section: (none) Explanation Explanation/Reference: Explanation: (A) Aspirin should not be given on an empty stomach because it is irritating to the mucosa. (B) Bleeding from decreased clotting capacity may be caused by aspirin toxicity. (C) A regular schedule of aspirin administration is important to maintain a satisfactory drug level in the body. (D) Aspirin toxicity may affect cranial nerve VIII, leading to tinnitus (ringing in the ears). QUESTION 531 A young child has been placed in a spica cast. The chief concern of the nurse during the first few hours is: A. Prevention of neurovascular complications B. Prevention of loss of muscle tone C. Immobilization of the affected limb D. Using heated fans to dry the cast Correct Answer: A Section: (none) Explanation Explanation/Reference: Explanation: (A) Because the extremity may continue to swell and the cast could constrict circulation, the nurse should elevate the limb and observe for capillary refill, warmth, mobility of toes and circulation. (B) Although muscle tone may diminish over time in the affected limb, this is not the immediate concern. (C) The limb has been immobilized already by the cast, and therefore immobilization is not a concern. (D) Heated fans and dryers are discouraged because the outside cast will dry quickly, yet the area beneath the cast remains wet and could cause burns. QUESTION 532 The pediatrician has diagnosed tinea capitis in an 8- year-old girl and has placed her on oral griseofulvin. The nurse should emphasize which of these instructions to the mother and/or child?A. Administer oral griseofulvin on an empty stomach for best results. B. Discontinue drug therapy if food tastes funny. C. May discontinue medication when the child experiences symptomatic relief. D. Observe for headaches, dizziness, and anorexia. Correct Answer: D Section: (none) Explanation Explanation/Reference: Explanation: (A) Giving the drug with or after meals may allay gastrointestinal discomfort. Giving the drug with a fatty meal (ice cream or milk) increases absorption rate. (B) Griseofulvin may alter taste sensations and thereby decrease the appetite. Monitoring of food intake is important, and inadequate nutrient intake should be reported to the physician. (C) The child may experience symptomatic relief after 4896 hours of therapy. It is important to stress continuing the drug therapy to prevent relapse (usually about 6 weeks). (D) The incidence of side effects is low; however, headaches are common. Nausea, vomiting, diarrhea, and anorexia may occur. Dizziness, although uncommon, should be reported to the physician. QUESTION 533 A 12-year-old girl has been diagnosed with insulindependent diabetes mellitus. Which of these principles would best guide her nutritional management? A. Concentrated sweets are taken during increased activity. B. Food restriction is imposed to reduce weight. C. Caloric distribution should be calculated to fit activity patterns. D. Fat requirements are increased owing to the possibility of ketoacidosis. Correct Answer: C Section: (none) Explanation Explanation/Reference: Explanation: (A) Concentrated sweets are eliminated from diet planning. Complex carbohydrates may be taken at the time of increased activity. (B) Food restriction is not usedfor diabetic control of growing children. Caloric restriction may be imposed for weight control if necessary. (C) Total caloric intake and proportions of basic nutrients should be consistent from day to day. Distribution of these calories should fit the activity pattern. Extra food is needed for increased activity. A balance of food, exercise, and insulin should be maintained. (D) Because of the increased risk of atherosclerosis, the fat percentage of the total caloric intake is reduced. QUESTION 534 A 5-year-old has just had a tonsillectomy and adenoidectomy. Which of these nursing measures should be included in the postoperative care?A. Encourage the child to cough up blood if present. B. Give warm clear liquids when fully alert. C. Have child gargle and do toothbrushing to remove old blood. D. Observe for evidence of bleeding. Correct Answer: D Section: (none) Explanation Explanation/Reference: Explanation: (A) The nurse should discourage the child from coughing, clearing the throat, or putting objects in his mouth. These may induce bleeding. (B) Cool, clear liquidsmay be given when child is fully alert. Warm liquids may dislodge a blood clot. The nurse should avoid red- or brown- colored liquids to distinguish fresh or old blood from ingested liquid should the child vomit. (C) Gargles and vigorous toothbrushing could initiate bleeding. (D) Postoperative hemorrhage, though unusual, may occur. The nurse should observe for bleeding by looking directly into the throat and for vomiting of bright red blood, continuous swallowing, and changes invital signs. QUESTION 535 A 4-week-old infant is admitted to the emergency room in respiratory distress. Which of the following statements indicates the nurse's knowledge of the anatomy of the respiratory system in pediatric clients? A. The diameter of the trachea is much smaller in children than in adults. B. The tongue is proportionally smaller in children than in adults. C. The pediatric airway is more rigid than that of the adults. D. The length of the pediatric airway is longer in children than in adults. Correct Answer: A Section: (none) Explanation Explanation/Reference: Explanation: (A) The airway in children is much smaller than it is in adults. The diameter of the trachea in the newborn is 4 mm and that of the adult is 20 mm. A small change in the diameter of the airway can make a major difference in the pediatric client. (B) The tongue is proportionally larger in children and fills most of the oral cavity, thereby decreasing air space. (C) The entire pediatric airway is elastic. Elasticity diminishes with age, however. (D) The distances between respiratory structures are shorter than that of adults, and therefore organisms are able to move more rapidly down the throat, leading to more extensive respiratory involvement.QUESTION 536 A 10-year-old has been diagnosed with acute poststreptococcal glomerulonephritis. The clinical findings were proteinuria, moderately elevated blood pressure, and periorbital edema. Which dietary plan is most appropriate for this client? A. Low-protein diet B. Low-sodium diet C. Increased fluid intake D. High-cholesterol diet Correct Answer: B Section: (none) Explanation Explanation/Reference: Explanation: (A) A high-protein diet is usually indicated because protein is excreted in urine. Protein restriction is usually prescribed with severe azotemia. (B) The kidneys usually enlarge in these children, and sodium and water are retained. (C) Fluid restriction may be ordered to help reduce edema; however, monitoring for dehydration is indicated. (D) A highcholesterol diet would not be indicated for any child, especially one with elevated blood pressure. QUESTION 537 A 6-year-old girl has been diagnosed with a urinary tract infection secondary to vesicoureteral reflux. Which statement by her mother indicates a need for further teaching? A. "I have taught her to wipe from front to back after urinating." B. "I make sure she drinks plenty of fluids every day." C. "She enjoys wearing nylon panties, but I make her change them everyday." D. "She tries to empty her bladder completely after she urinates, like I told her." Correct Answer: C Section: (none) Explanation Explanation/Reference: Explanation: (A) Wiping from front to back is wiping from an area of lesser contamination (urethra) to an area of greater contamination (rectum). (B) Generous fluid intake reduces the concentration of urine. (C) Cotton is a natural, absorbent fabric. Nylon often predisposes the client to urinary tract infections. Dark, warm, moist areas are excellent media for bacterial growth. (D) With vesicoureteral reflux, urine refluxes into the ureter(s) during voiding and then returns to the bladder (residual), which becomes a source for future infection.QUESTION 538 A 1-year-old child is to receive an IM injection ordered by his pediatrician. He has fallen asleep in his mother's arms when the nurse approaches. Which approach is most appropriate at this time? A. Give the injection in the vastus lateralis site before the child awakens. B. Awaken the child first and give the injection in the ventrogluteal site. C. Awaken the child first and give the injection in the dorsogluteal site. D. Ask the mother to place the child on the examination table and leave the room, and then give the injection in an appropriate site. Correct Answer: B Section: (none) Explanation Explanation/Reference: Explanation: (A) If awakened first, the child will know that nothing painful will be done without the child being alerted. (B) The ventrogluteal site is a safe site for children because it is a large muscle free of major nerves and blood vessels. (C) The dorsogluteal site is not recommended in children who have not been walking for at least 1 year because the muscle is not fully developed. (D) The parent will be able to offer support and comfort during and after the injection. QUESTION 539 The physician has ordered that ampicillin 250 mg IV be given over 30 minutes. The medication is diluted as recommended in 10 mL in the volume control chamber of a set that has a tubing of 12 mL. Which nursing measure is most accurate considering these facts? A. Infuse volume at 44 mL/hr. B. Infuse volume at 22 mL/hr. C. Infuse volume at 10 mL/hr. D. Infuse volume at 30 mL/hr. Correct Answer: A Section: (none) Explanation Explanation/Reference: Explanation: (A) The volume to be infused should be diluted medication volume added to the volume control chamber (10 mL) plus the tubing volume (12 mL). The general formula for calculating IV medications for children is: Rate = Volume to Be Infused X Administration Set Drop Factor (microdrop: 60 gtts/min)Desired Time to Infuse in Minutes Rate = (10 + 12)22 X 60 30 = 44 mL/hr. (B, C, D) These values are incorrect.QUESTION 540 An infant weighing 15 lb has just been treated for severe diarrhea in the hospital. Discharge instructions by the nurse will include maintenance fluid requirements for the pediatric client. Which of the following values best indicates the nurse's understanding of normal fluid requirements for this infant? A. 240 mL/day B. 680 mL/day C. 330 mL/day D. 960 mL/day Correct Answer: B Section: (none) Explanation Explanation/Reference: Explanation: (A, C, D) These answers are incorrect. (B) Normal fluid requirement for this pediatric client is based on the fact that 010 kg of weight equals 100 mL/kg per day. This infant weighs 15 pounds (6.8 kg). Thus, 100 mL X 6.8 = 680 mL/day. QUESTION 541 A normal 3-year-old child is suspected of having meningitis. The doctor has ordered a lumbar puncture. In light of this procedure and developmental characteristics of this age group, which nursing measure is most appropriate? A. Emphasize those aspects of the procedure that require cooperation. B. Tell the child not to cry or yell. C. Tell the child that he will get a "stick" in his back. D. Use medical terminology when explaining the procedure to the client. Correct Answer: A Section: (none) Explanation Explanation/Reference: Explanation: (A) The nurse should emphasize what is required to elicit cooperation and help to develop a sense of autonomy. (B) The child may express discomfort verbally and should be encouraged to express his feelings. (C) Selecting nonthreatening words to explain a procedure will prevent misinterpretation. (D) When explaining the procedure to the parent with the child present, the nurse should use words that the child can understand to avoid misunderstanding.QUESTION 542 A mother brings a 6-month-old infant and a 4-year-old child to the nursing clinic for routine examination and screening. Which of these plans by the nurse would be most successful? A. Examine the 4 year old first. B. Provide time for play and becoming acquainted. C. Have the mother leave the room with one child, and examine the other child privately. D. Examine painful areas first to get them "over with." Correct Answer: B Section: (none) Explanation Explanation/Reference: Explanation: (A) The 6 month old should be examined first. If several children will be examined, begin with the most cooperative and less anxious child to provide modeling. (B) Providing time for play and getting acquainted minimizes stress and anxiety associated with assessment of body parts. (C) Children generally cooperate best when their mother remains with them. (D) Painful areas are best examined last and will permit maximum accuracy of assessment. QUESTION 543 An 11-month-old infant is admitted with a possible diagnosis of pyloric stenosis. Which of the following best describes the characteristic clinical manifestations of pyloric stenosis? A. Pain, especially when eating B. Poor appetite and sucking reflex C. Increased frequency and quantity of stools D. Palpable olive-shaped mass in the epigastrium just right of the umbilical cord Correct Answer: D Section: (none) Explanation Explanation/Reference: Explanation: (A) There is no evidence of pain in infants with pyloric stenosis whether eating or not. (B) There are both good appetite and feeding habits in these children. (C) Because of regurgitation, there is usually decreased frequency and quantity of stools and also signs of dehydration and weight loss. (D) Along with upper abdominal distention, there is a characteristic palpable olive-shaped mass located to the right of the umbilicus.QUESTION 544 As soon as a child has been diagnosed as "hearing impaired," special education should begin. Which of the following special education tasks is the most difficult for a severely hearing- impaired child? A. Auditory training B. Speech C. Lip reading D. Signing Correct Answer: B Section: (none) Explanation Explanation/Reference: Explanation: (A) With the slight and mild hard of hearing, auditory training is beneficial. (B) Speech is the most difficult task because it is learned by visual and auditory stimulation, imitation, and reinforcement. (C, D) Lip reading and signing are aimed at establishing communicative skills, but they are learned more easily by visual stimulation. QUESTION 545 A 15-year-old child is admitted to the pediatric unit with a diagnosis of thalassemia. Which of the following would be included in educating the mother and child as part of discharge planning? A. Give oral iron medication every day. B. Have the child's blood pressure monitored every week. C. Know the signs and symptoms of iron overload. D. Keep exercise at a minimum to reduce stress. Correct Answer: C Section: (none) Explanation Explanation/Reference: Explanation: (A) Oral iron supplements are contraindicated in thalassemia. (B) Although heart failure may be an end result of this disease, this action is unnecessary. (C) Iron overload is a potential complication of frequent blood transfusions of children with thalassemia. (D) Children should be encouraged to pursue activities related to their exercise tolerance. QUESTION 546An 8-year-old boy has been diagnosed with hemophilia. Which of the following diagnostic blood studies is characteristically abnormal in this disorder? A. Partial thromboplastin time B. Platelet count C. Complete blood count D. Bleeding time Correct Answer: A Section: (none) Explanation Explanation/Reference: Explanation: (A) Partial thromboplastic time measures activity of thromboplastin, which depends on the intrinsic clotting factors deficient in children who are hemophiliacs. (B) Platelet counts are normal in hemophilia. (C) Hemophilia does not affect the complete blood count. (D) Bleeding times are normal in hemophiliacs. They measure the time interval for the bleeding from small superficial wounds to cease. QUESTION 547 A murmur has been discovered during the routine physical examination of a 1-year-old child. The parent is extremely concerned about this diagnosis. Which of the following explanations by the nurse indicates understanding of this dysfunction? A. The blood shifts from the right to the left atrium. B. Surgical closure by suture or patch is recommended before school age. C. Most atrial septal defects close spontaneously. D. The child can be treated medically with antibiotics to prevent bacterial endocarditis. Correct Answer: B Section: (none) Explanation Explanation/Reference: Explanation: (A) Because the left atrial pressure is greater than right atrial pressure, oxygenated blood flows from the left to the right atria. (B) Because of the risk of pulmonary obstructive diseases and congestive heart failure later in life, surgery is usually performed between age 4 and 6 years, with essentially no operative mortality or postoperative complications. (C) Many ventricular septal defects close spontaneously (2060%) as a result of growth and proliferation of the muscular septum or formation of a membrane across the opening. (D) This management is usually recommended with children with mild pulmonary stenosis. QUESTION 548 An alcoholic client who is completing the inpatient segment of a substance abuse program was placed on disulfiram (Antabuse) drug therapy. What should thenurse include in the discharge instructions? A. If disulfiram istaken and alcohol ingested, the client experiences nausea, vomiting and elevated blood pressure. B. Disulfiram is most effective when prescribed as late as possible in a recovery program. C. Disulfiram works on the desensitization principle. D. The effects of disulfiram can be triggered by alcohol 5 days to 2 weeks after the drug is discontinued. Correct Answer: D Section: (none) Explanation Explanation/Reference: Explanation: (A) When alcohol is ingested with disulfiram therapy, the client experiences nausea, vomiting, and a potentially serious drop in blood pressure. (B) Disulfiram is most successful when used early in the recovery process while the individual makes major lifestyle changes necessary for long-term recovery. (C) Disulfiram works on the classical conditioning principle. (D) The effects of disulfiram can be felt when alcohol is ingested 12 weeks after disulfiram is discontinued. QUESTION 549 An 82-year-old former restaurant owner walks to the nursing station and states, "I have to go. The restaurant opens at 11 am." Which response by the nurse is the most appropriate? A. "Go back to your room. You do not own a restaurant." B. "You are in the hospital now. Calm down." C. "You once owned a restaurant. Tell me about it." D. "It is snowing outside. The restaurant is closed." Correct Answer: C Section: (none) Explanation Explanation/Reference: Explanation: (A) This response cuts off communication with the client. It does not address her feelings. (B) Reality orientation frequently does not work alone. Feelings must be addressed. Telling a client to calm down is frequently ineffective. (C) Reminiscence is used here to reorient and recall past pleasant events. Talking about the restaurant will allay anxiety. (D) This response may confirm to the client that she indeed does still own a restaurant, buying into her confusion. Her feelings and anxiety require nursing intervention. QUESTION 550A 15-year-old female adolescent is frequently breaking the rules of the unit. She has left the unit and was found smoking in the bathroom and spending a large amount of time in the male ward. Which statement by the nurse would best explain to the teenager why she must follow the rules of the unit? A. "It is not easy, but the rules must be followed so that everyone can get a fair chance." B. "If you do not follow the rules, you will be transferred to the closed, locked unit." C. "You are not being fair to the other clients by getting them involved in your deviant behavior." D. "Break the rules, all you want, but don't get caught again!" Correct Answer: A Section: (none) Explanation Explanation/Reference: Explanation: (A) This statement acknowledges that it is difficult but is not threatening or punitive. (B) This statement is threatening and describes specific punishment for further deviant behavior. (C) This response elicits shame by blaming her for involving others. (D) This response gives her permission to break the rules but indicates thatgetting caught is wrong. QUESTION 551 A 45-year-old male client experiences a sense of depression because he has not yet achieved his life's goals. His career has not been satisfying. He is still looking for the right job. His wife spends too much money, and his children seem to ignore him while being very selfish. He is tired of all of their attitudes and is considering buying a red Corvette convertible. While obtaining these data concerning the client's feelings about his life, the nurse is able to determine he is experiencing what psychological crisis according to Erikson's stages? A. Identity versus role confusion B. Integrity versus despair C. Intimacy versus isolation D. Generativity versus self-absorption Correct Answer: D Section: (none) Explanation Explanation/Reference: Explanation: (A) Identity versus role confusion is experienced by adolescents making the transition from childhood to adulthood as they attempt to develop a sense of identity. (B) Integrity versus despair is experienced by the elderly as they reflect on their life in an attempt to find meaning. (C) Intimacy versus isolation is experienced byyoung adults as they establish intimate bonds of love and friendship. (D) Generativity versus self-absorption is experienced bymiddle-aged adults as they fulfill life goals that involve family, career, and society. The client is experiencing this crisis. QUESTION 552 A female client is anticipating a visit with her parents over the Thanksgiving holidays. She has recently begun experiencing periods of extreme shortness of breath, which her physician has labeled as panic attacks. Which of the following statements by the nurse would enhance therapeutic communication? A. "Why do you feel this way?" B. "Tell me about your dislike for your parents." C. "Don't worry, everything will be all right on your visit with your parents." D. "Perhaps you and I can discover what produces your anxiety." Correct Answer: D Section: (none) Explanation Explanation/Reference: Explanation: (A) Asking the client to provide an explanation for her feelings is often intimidating. (B) This response is probing and may make the client feel used and valued only for the information she can provide. (C) This underrates the client's feelings and belittles her concerns. It may cause the client to stop sharing feelings for fear thatthey will be ridiculed. (D) The emphasis is on working with the client. It shows that there is hope for change through collaboration. QUESTION 553 A female client has experienced varying degrees of depression throughout her life. Now that she is postmenopausal, her depression has increased. She is unable to motivate herself to clean her house or even to get out of bed and get dressed in the morning. The client was begun on fluoxetine (Prozac) therapy. When educating her about fluoxetine, what might the nurse caution her about? A. A daily dose of fluoxetine may be taken in the morning or evening. B. Fluoxetine is not sedating; therefore, restrictions on driving and other hazardous activities are not necessary. C. Rashes or pruritus usually occur early in the therapy and are treatable without discontinuing the medication. D. It is safe to take over-the-counter or other prescription medications with fluoxetine. Correct Answer: C Section: (none) Explanation Explanation/Reference: Explanation:(A) A daily dose of fluoxetine should be taken in the morning. Afternoon doses may cause nervousness and insomnia. (B) Although fluoxetine is less sedating than other antidepressants, it may still cause dizziness or drowsiness in some clients. The nurse should caution clients to avoid driving or hazardous activities until the central nervous system effects of the drug are demonstrated. (C) Rashes or pruritus do commonly occur early in therapy and respond to antihistamines or topical corticosteroids. (D) Advise the client not to take over-the-counter or other prescription drugs without consulting with the physician. Fluoxetine does interact with other common drugs such as monoamine oxidase inhibitors, diazepam, insulin, oral antidiabetic agents, tricyclic antidepressants, and tryptophan. QUESTION 554 A male client seeks counseling after his wife of 19 years threatened to divorce him. For most of their marriage, he has physically and verbally abused her. When asked about his behavior in the process of the nursing assessment, the client states, "I was mean to my wife because she insists on cooking meals and wearing clothes that I do not like." This defense mechanism is an example of: A. Repression B. Regression C. Reaction formation D. Rationalization Correct Answer: D Section: (none) Explanation Explanation/Reference: Explanation: (A) Repression is blocking a desire from conscious expression. The client is conscious of his desires. (B) Regression is returning to an earlier form of expression,which is not demonstrated here. (C) Reaction formation is acting out the opposite of true feelings. The client felt anger concerning his wife's cooking and acted out his feelings. (D) Rationalization is unconsciously falsifying an experience by giving a "rational" explanation. The client is attempting to justify his behavior by giving an explanation. QUESTION 555 A male client is admitted to the psychiatric unit after experiencing severe depression. He states that he intends to kill himself, but he asks the nurse not to repeat his intentions to other staff members. Which response demonstrates understanding and appropriate action on the part of the nurse? A. "I understand you're depressed, but killing yourself is not a reasonable option." B. "We need to discuss this further, but right now let's complete these forms." C. "Don't do that, you have so much to live for. You have a wonderful wife and children. The client in the next room has no one." D. "This is very serious. I do not want any harm to come to you. I will have to report this to the rest of the staff." Correct Answer: D Section: (none)Explanation Explanation/Reference: Explanation: (A) To the client, suicide may be a reasonable action and the only one he can cope with at this time. (B) This response indicates to the client that his intention to commit suicide is not important to the nurse at this time. (C) The client is so depressed that he is not able to see the positive aspects of his life. At no time shouldthe nurse discuss another client's problems in conversation. (D) This statement tells the client that the nurse recognizes his problem is of a serious nature and will take all steps necessary to help him. QUESTION 556 During the admitting mental health assessment, a client demonstrates involuntary muscular activity. He has a marked facial tic around the mouth that is distracting to the nurse during the interview. The nurse recognizes the behavior and documents it as: A. Dyskinesia B. Akathisia C. Echopraxia D. Echolalia Correct Answer: A Section: (none) Explanation Explanation/Reference: Explanation: (A) The client is demonstrating dyskinesia, which is involuntarymuscular activity, such as tic, spasm, or myoclonus. (B) Akathisia is regular rhythmic movements usually of the lower limbs, such as constant motor restlessness. (C) Echopraxia is mimicking the movements of another person. (D) Echolalia is mimicking the speech of another person. QUESTION 557 A female client is seeking counseling for personal problems. She admits to being very unhappy lately at both home and work. During the nursing assessment, she uses many defense mechanisms. Which statement or action made by the client is an example of adaptive suppression? A. "I did not get the raise because my boss does not like me." B. "I felt a lump in my breast 2 weeks ago. I put off getting it checked until after my sister's wedding." C. "My son died 3 years ago. I still cannot bring myself to clean out his room." D. "My husband told me this morning that he wants a divorce. I am upset, but I cannot discuss the matter with him until after my company's board meeting today." Correct Answer: DSection: (none) Explanation Explanation/Reference: Explanation: (A) This statement is an example of adaptive rationalization. She is coping with her disappointment by rationalizing. This is adaptive because no harm is done to self or others. It is used to protect her ego. (B) This is an example of maladaptive suppression. She is suppressing the seriousness of the lump. It is maladaptive because delaying treatment will cause harm to her. (C) The client's actions are an example of maladaptive denial. She is denying her son's death by not facing his possessions. Until she faces his death, she cannot face reality. (D) This is an example of adaptive suppression. She realizes the impact of her husband's statement but delays discussion until she can devote her full attention to the matter. QUESTION 558 When interviewing parents who are suspected of child abuse, the nurse would use which of the following interview techniques? A. Be direct, honest, and attentive. B. Approach them in the emergency room as soon as you suspect abuse to "clear the air" right away. C. Ask the parents what they could have done differently to prevent this from happening to the child. D. After the interview, call child protective services. Correct Answer: A Section: (none) Explanation Explanation/Reference: Explanation: (A) The nurse must be honest, direct, professional, and attentive in her interview to gain the parent's trust. (B) The nurse should approach the parents in private, away from the child. (C) Asking them to relive and evaluate the situation may be looked at as placing blame on the parents for the child's "accident." At this point, the parents may get defensive and stop communicating. (D) Although you may call child protective services, the nurse should inform the parents of their responsibility to do this and explain the process to them. QUESTION 559 In an interview for suspected child abuse, the child's mother openly discusses her feelings. She feels her husband is too aggressive in disciplining their child. The child's father states, "Being a school custodian, I see kids every day that are bad because they did not get enough discipline at home. That will not happen to our child." Based on this remark, the nurse would make the following nursing diagnosis: A. Fear related to retaliation by the father B. Actual injury related to poor impulse control by the father C. Ineffective coping D. Altered family process related to physical abuseCorrect Answer: D Section: (none) Explanation Explanation/Reference: Explanation: (A) There is no evidence of fear as the child is unable to communicate. (B) There is actual injury, but the parents have not yet admitted causing the child's injuries. (C) This diagnosis is incomplete. There is no specific ineffective coping behavior identified in this nursing diagnosis. (D) Altered family process best describes the family dynamics in this situation. The parents have admitted severe disciplinary action. QUESTION 560 A 40-year-old client has lived for 8 years with an abusive spouse. She married her husband in her senior year of high school after becoming pregnant. Shortly after the baby was born, he began to physically abuse her. She has attempted to leave him several times, but she has always returned. She is unable to support herself financially, and her husband threatens to kill her if she leaves him. This time, her husband has beaten her so badly she cannot stop the bleeding from the gash above her eye. She admits her husband caused her injury. In assessing a person after experiencing spousal abuse, which need has the highest priority? A. Assess the level of anxiety, coping responses, and support systems. B. Assess the history of physical abuse. C. Assess suicide potential. D. Assess drug and alcohol use. Correct Answer: C Section: (none) Explanation Explanation/Reference: Explanation: (A) Assessing the level of anxiety, coping responses, and support systems is very important, but not of highest priority at this time. (B) A history of physical abuse is an important part of assessment. The nurses must also always ask if there is abuse of the children. (C) Although all of these answers are very important in assessment, the highest priority is assessment of suicide potential, because this could cause the greatest harm to the client. Feeling there is no other way out, abused spouses may consider suicide. (D) The spouse may be self-medicating herself with alcohol or drugs to escape an awful situation. The abuser may also beabusing drugs or alcohol. If this is so, the nurse should encourage the spouse to seek counseling and not to return to the home. QUESTION 561 As a nurse in the emergency room, you receive an outside call from an elderly woman who states she has just been raped. She states, "I know I must come to the hospital, but what do I do next?" You advise her to call the police, then come to the hospital emergency room. What action by the nurse would indicate an understanding of the examination process once the victim enters the emergency room?A. Inform the victim not to wash, change clothes, douche, brush teeth, or eat or drink anything. B. Inform the victim to bring insurance information with her to the hospital so she can be properly cared for. C. Phone a rape counselor to begin working with the victim as soon as she enters the hospital. D. Do not leave the victim alone to collect her thoughts. Correct Answer: A Section: (none) Explanation Explanation/Reference: Explanation: (A) Providing the victim with these instructions will aid in the determination of physical evidence of rape. Victims frequently feel "dirty" after rape, and their first instinct is to take care of personal hygiene before facing anyone. (B) This action is of lesser importance at this time. (C) Although this is a nursing measure appropriate in this situation, contacting a counselor can be done once the victim enters the hospital. Frequently victims call but do not follow up with the visit. (D) Once the victim enters the emergency room, it is important not to leave her alone. QUESTION 562 A 14-year-old teenager is demonstrating behavior indicative of an obsessive-compulsive disorder. She is obsessed with her appearance. She will not leave her room until her hair, clothes, and makeup are perfect. She always dresses immaculately. Recently, she expressed disgust over her appearance after she gained 5 lb. After observing a marked weight loss over a 2-week period, her mother suspects that she is experiencing bulimia. She eats everything on her plate, then runs to the bathroom. In interviewing the teenager, she discusses in great detail all of the events leading to her bulimia, but not her feelings. What defense mechanism is she using? A. Dissociation B. Intellectualization C. Rationalization D. Displacement Correct Answer: B Section: (none) Explanation Explanation/Reference: Explanation: (A) Dissociation is separating a group of mental processes from consciousness or identity, such as multiple personalities. That is not evident in this situation. (B) Intellectualization is excessive use of reasoning, logic, or words usually without experiencing associated feelings. This is the defense mechanism that this client is using. (C) Rationalization is giving a socially acceptable reason for behavior rather than the actual reason. She is discussing events, not reasons. (D) Displacement is a shift of emotion associated with an anxietyproducing person, object, or situation to a less threatening object.QUESTION 563 A male client is experiencing extreme distress. He begins to pace up and down the corridor. What nursing intervention is appropriate when communicating with the pacing client? A. Ask him to sit down. Speak slowly and use short, simple sentences. B. Help him to recognize his anxiety. C. Walk with him as he paces. D. Increase the level of his supervision. Correct Answer: C Section: (none) Explanation Explanation/Reference: Explanation: (A) The nurse should not ask him to sit down. Pacing is the activity he has chosen to deal with his anxiety. The nurse dealing with this client should speak slowly and with short, simplesentences. (B) The client may already recognize the anxiety and is attempting to deal with it. (C) Walk with the client as he paces. This gives support while he uses anxietygenerated energy. (D) Increasing the level of supervision may be appropriate after he stops pacing. It would minimize self-injury and/ or loss of control. QUESTION 564 A 24-year-old graduate student recognizes that he has a phobia. He suffers severe anxiety when he is in darkness. It has altered his lifestyle because he is unable to go to a movie theater, concert, and other events that may require absence of light. The client is seeking assistance because he is no longer able to socialize with friends due to his phobia. The psychologist working with him is using desensitization. He has asked the nursing staff to assist the client in muscle relaxation techniques. What result would indicate client education has been successful? A. He enters a movie theater, sits in his chair, and replaces anxiety with relaxation as the theater darkens. B. He enters a concert, but as the lights dim, he does not experience anxiety. C. He states that he no longer fears dark places. D. He takes a part-time job as a photographic assistant. His job necessitates his working in a darkroom. Correct Answer: A Section: (none) Explanation Explanation/Reference: Explanation:(A) This situation provides specific evidence that the client is able to integrate muscle relaxation technique into his lifestyle to alleviate anxiety. (B) The client may not experience anxiety at the concert, but there is no evidence regarding the technique that he used to alleviate anxiety. (C) The client may state he no longer experiences anxiety, but there is no evidence demonstrating this. He may be denying anxiety to discontinue therapy prematurely. (D) Does he experience anxiety in the darkroom? He may have taken this job to force himself to deal with the phobia directly. QUESTION 565 A female client has just died. Her family is requesting that all nursing staff leave the room. The family's religious leader has arrived and is ready to conduct a ceremony for the deceased in the room, requesting that only family members be present. The nurse assigned to the client should perform the appropriate nursing action, which might include: A. Inform the family that it is the hospital's policy not to conduct religious ceremonies in client rooms. B. Refuse to leave the room because the client's body is entrusted in the nurse's care until it can be brought to the morgue. C. Tell the family that they may conduct their ceremony in the client's room; however, the nurse must attend. D. Respect the client's family's wishes. Correct Answer: D Section: (none) Explanation Explanation/Reference: Explanation: (A) It is rare that a hospital has a specific policy addressing this particular issue. If the statement is true, the nurse should show evidence of the policy to the family and suggest alternatives, such as the hospital chapel. (B) Refusal to leave the room demonstrates a lack of understanding related to the family's need to grieve in their own manner. (C) The nurse should leave the room and allow the family privacy in their grief. (D) The family's wish to conduct a religious ceremony in the client's room is part of the grief process. The request is based on specific cultural and religious differences dictating social customs. QUESTION 566 A female client has been recently diagnosed as bipolar. She has taken lithium for the past several weeks to control mania. What must be included in client education regarding lithium toxicity? A. Maintain a normal diet; however, limit salt intake to no more than 3 g/day. B. Take lithium between meals to increase absorption. C. Withhold lithium if experiencing diarrhea, vomiting, or diaphoresis. D. For pain or fever, avoid aspirin or acetaminophen (Tylenol). Nonsteroidal anti-inflammatory drugs are preferred. Correct Answer: C Section: (none) ExplanationExplanation/Reference: Explanation: (A) The client should maintain a normal diet including normal salt intake. A low-sodium diet can cause lithium retention, leading to toxicity. (B) Lithium must be taken with meals because it is irritating to the gastric mucosa. (C) Diarrhea, vomiting, or diaphoresis can cause dehydration, which will increase lithium blood levels. If these symptoms occur, the nurse should instruct the client to withhold lithium. (D) Lithium is not to be taken with over-the-counter drugs without specific instruction. Some drugs raise lithium levels, whereas others lower lithium levels. QUESTION 567 For the past several months, an elderly female client with Alzheimer's disease has experienced paranoia; hallucinations; and aggressive, disruptive behavior. The family is utilizing haloperidol as needed to control her behavior. On nursing assessment, you note that the client demonstrates involuntarymovements of the tongue and fingers. This may most likely indicate: A. Tardive dyskinesia, which may be a side effect of antipsychotic medication B. Early symptoms of Parkinson's disease C. A more advanced stage of Alzheimer's disease than previously experienced by the client D. The need to change her medication from haloperidol to another antipsychotic drug to lessen symptoms Correct Answer: A Section: (none) Explanation Explanation/Reference: Explanation: (A) Tardive dyskinesia is a common side effect of antipsychotic medications such as haloperidol. Discontinuing the medication can alleviate symptoms. (B) Although mild tremors are an early sign of Parkinson's disease, haloperidol must be discontinued first and the client further evaluated. (C) These symptoms do not necessarily indicate a more advanced stage of Alzheimer's disease. (D) Most antipsychotic drugs are chemically similar and will produce the same side effects. QUESTION 568 A 32-year-old male client is a marketing representative. His job requires him to have a tremendous amount of energy during the day. He frequently uses cocaine to sustain his energy level. Lately he has increased his use of cocaine and even experimented with crack cocaine. Realizing he can no longer continue this destructive behavior, he is seeking treatment for cocaine addiction. In planning nursing care for the client's inpatient stay, which expected outcome is most appropriate? A. He will attend four consecutive group educational sessions on substance abuse. B. He will name activities that he would most likely be involved in posttreatment. C. He will meet with his family in counseling sessions and discuss his feelings. D. He will be able to deal with his feelingsthrough participation in group therapy sessions. Correct Answer: D Section: (none)Explanation Explanation/Reference: Explanation: (A) This expected outcome is specific as related to attendance, but not specific as related to outcome criteria. (B) Stating activities does not guarantee involvement. (C) This goal may help the recovery process, but postcounseling behavior is not addressed. (D) This statement best describes the expected outcome. The client will be attending group therapy sessions and through them he will deal with his feelings. QUESTION 569 A client has been instructed in how to take her nitroglycerin tablets. The nurse giving her instructions knows the client understands the information when she tells her: A. "I should contact my physician if I have headaches after I take this medicine." B. "I should keep the tablets in the refrigerator." C. "I should call the doctor if three doses of the medicine do not relieve my pain." D. "I should take these with water but not with milk." Correct Answer: C Section: (none) Explanation Explanation/Reference: Explanation: (A) Headaches may occur after taking nitroglycerin because of vasodilation. (B) The tablets do not need to be refrigerated. The client should carry them with her. (C) The client should contact the physician if repeated doses of nitroglycerin do not relieve the discomfort. (D) Nitroglycerin tablets should be dissolved under the tongue, not swallowed. QUESTION 570 A client hasrenal failure. Today's lab values indicate he has an elevated serum potassium. What additional priority information does the nurse need to obtain? A. Evaluation of his level of consciousness B. Evaluation of an electrocardiogram C. Measurement of his urine output for the past 8 hours D. Serum potassium lab values for the last several days Correct Answer: B Section: (none) ExplanationExplanation/Reference: Explanation: (A) The level of consciousness is not affected by elevated potassium levels. (B) An electrocardiogram (EKG) can tell the nurse whether this client is experiencingany cardiac dysfunction or arrhythmiasrelated to the elevated potassium level. (C) Measurement of the urine output is not a priority nursing action at this time. (D) The client's serum potassium values for the past several days may provide information about his renal function, but they are not a priority at this time. QUESTION 571 A client's wife is concerned over his behavior in recent months. He has been diagnosed with Parkinson's disease, and she is telling his nurse that he has been doing "strange things." The nurse reassures the wife that the following behavior is normal with Parkinson's disease: A. "Your husband will experience some periods of muscle flaccidity. Be sure to make him sit down during these periods." B. "Your husband may move his hands in motions that look like he is rolling a pill between his fingers." C. "Twitching of the muscles is to be expected and can occur at any time during the day." D. "Parkinson's disease causes severe pain in the joints. You should give your husband Tylenol at those times." Correct Answer: B Section: (none) Explanation Explanation/Reference: Explanation: (A) Clients with Parkinson's disease generally experience stiffness and rigid movement. (B) Pill- rolling movements are a symptom experienced by the Parkinson client. (C) Twitching of the muscles is not an expected symptom of Parkinson's disease. (D) Parkinson's disease does not cause joint pain. Mild muscular pain may be present. QUESTION 572 A male client tells his nurse that he has had an ulcer in the past and is afraid it is "flaring up again." The nurse begins to ask him specific questions about his symptoms. The nurse knows that a symptom that might indicate a serious complication of an ulcer is: A. Pain in the middle of the night B. A bowel movement every 35 days C. Melena D. Episodes of nausea and vomiting Correct Answer: C Section: (none) ExplanationExplanation/Reference: Explanation: (A) Clients with ulcers generally experience abdominal pain. It is common to have pain in the early morning hours with an ulcer. (B) Constipation is not a symptomassociated with ulcers and would indicate a need to look at other factors. (C) Melena is blood in the stools. This could indicate a slow bleeding ulcer, which could result in significant amounts of blood loss over time.(D) Nausea and vomiting may be present as a result of the ulcer, especially if it is a gastric ulcer. This does not indicate an immediate lifethreatening complication. QUESTION 573 A client has chronic obstructive pulmonary disease. She is slowly losing weight, and her daughter is very concerned about increasing her nutrition. The nurse helps the daughter devise a plan of care for her mother. The plan of care should include which of the following interventions to promote nutrition? A. Offer her oral hygiene before and after meals. B. Encourage her to consume milk products. C. Encourage her to engage in an activity before a meal to stimulate her appetite. D. Restrict her fluid intake to three glasses of water a day. Correct Answer: A Section: (none) Explanation Explanation/Reference: Explanation: (A) Clients with respiratory diseases are generally mouth breathers. Cleaning the oral cavity may improve the client's appetite, increase her feelings of well-being,and remove the taste and odor of sputum. (B) Milk causes thick sputum; therefore, milk products would not be beneficial for this client. (C) Exercise prior to a meal would require increased O2 consumption and most likely would decrease the client's ability to eat. (D) Clients with respiratory diseases need increased fluid to liquefy secretions. QUESTION 574 A female client is concerned that she is in a "high-risk" group for the development of acquired immunodeficiency syndrome (AIDS). She wants to know about the advisability of donating blood. Which of the following responses is correct? A. "Individuals who donate blood are at risk of getting the AIDS virus. You should not donate." B. "It's OK for you to donate because the blood bank has a test that is 100% effective." C. "You should not donate since it takes time to develop antibodies to the AIDS virus. If you donate blood before you develop the antibody, you could pass it on in the blood." D. "It is not a good idea for you to donate. If you have AIDS, the information is made public and could destroy your personal life." Correct Answer: CSection: (none) Explanation Explanation/Reference: Explanation: (A) The AIDS virus cannot be transmitted to the donor through the blood donation procedure. (B) The test for the AIDS virus is not absolutely foolproof; therefore, it is not wise for a person with known risk factors to donate blood. (C) It takes time for antibodies to the AIDS virus to develop. An infected individual could donate contaminated blood without it testing positive for the virus. (D) For reasons of confidentiality, information about individuals infected with AIDS is not made public. QUESTION 575 A 50-year-old male client isto receive chemotherapy. The physician's orders include antiemetics. When planning his care, the nurse should take into consideration that antiemetics are best administered in the following way: A. Give antiemetics when nausea is experienced and continue on a regular schedule for 1224 hours. B. Give antiemetics prior to the client receiving chemotherapy and continue on a regular basis for at least2448 hours after chemotherapy. C. Give antiemetics one at a time because combinations of antiemetics cause overwhelming side effects. D. Give antiemetics intermittently during the entire course of chemotherapy. Correct Answer: B Section: (none) Explanation Explanation/Reference: Explanation: (A) Nausea is more difficult to control if antiemetics are withheld until nausea is experienced. (B) Antiemetics should be given prophylactically at the beginning of chemotherapy and continued on an around-the-clock basis to prevent nausea. (C) Combinations of antiemetics give the best control for nausea by blocking various causes of nausea induced by chemotherapy. (D) Antiemetics should be given around the clock during the course of chemotherapy. This prevents nausea from developing and prevents anticipatory nausea during subsequent chemotherapy administrations. QUESTION 576 A 30-year-old female client isreceiving antineoplastic chemotherapy. Which of the following symptoms should especially concern the nurse when caring for her? A. Respiratory rate of 16 breaths/min B. Pulse rate of 80 bpm C. Complaints of muscle aches D. A sore throat Correct Answer: DSection: (none) Explanation Explanation/Reference: Explanation: (A) A respiratory rate of 16 breaths/min is normal and is not a cause for alarm. (B) A pulse rate of 80 bpm is normal and is not a cause for alarm. (C) Complaints of muscle aches are unrelated to her receiving chemotherapy. There may be other causes related to her hospital stay or the disease process. (D) A sore throat is anindication of a possible infection. A client receiving chemotherapy is at risk of neutropenia. An infection in the presence of neutropenia can result in a life- threatening situation. QUESTION 577 Seven days ago, a 45-year-old female client had an ileostomy. She is self-sufficient and well otherwise. Which of the following long-term objectives would be unrealistic? A. She should be able to control evacuation of her bowels. B. She should be able to return to a regular diet. C. She should be able to resume sexual activity. D. She should be able to manage her own care. Correct Answer: A Section: (none) Explanation Explanation/Reference: Explanation: (A) Because of the location of an ileostomy, the client will not be able to control the evacuation of her bowels. The ileostomy will drain liquid stool continuously. (B) The client should be able to return to a normal, well-balanced diet. She should avoid foods that cause diarrhea or excessive gas production, and she should eat small meals. (C) The client should be able to resume sexual activity. She will be able to wear a pouch. (D) The client has no other health or mental problems andshould be able to manage her own ileostomy. QUESTION 578 A 26-year-old client has no children. She has had an abdominal hysterectomy. In the first 24 hours postoperatively, the nurse would be concerned if the client: A. Cries easily and says she is having abdominal pain B. Develops a temperature of 102_F C. Has no bowel sounds D. Has a urine output of 200 mL for 4 hoursCorrect Answer: B Section: (none) Explanation Explanation/Reference: Explanation: (A) The client may be more tearful than normal due to the stress of the surgery and its implications for her future life. She would be expected to have pain following surgery. (B) A temperature of 102_F indicates an infectious process. This is not a normal sequence to surgery and indicates a need for further assessment. (C) The client is expected to have no bowel soundsfor 2448 hours after surgery because of the trauma to the bowel. (D) Normal urine output is 30 mL/hr. This represents an output of 50 mL/hr, which is greater than normal. QUESTION 579 A 44-year-old female client is receiving external radiation to her scapula for metastasis of breast cancer.Teaching related to skin care for the client would include which of the following? A. Teach her to completely clean the skin to remove all ointments and markings after each treatment. B. Teach her to cover broken skin in the treated area with a medicated ointment. C. Encourage her to wear a tight-fitting vest to support her scapula. D. Encourage her to avoid direct sunlight on the area being treated. Correct Answer: D Section: (none) Explanation Explanation/Reference: Explanation: (A) The skin in a treatment area should be rinsed with water and patted dry. Markings should be left intact, and the skin should not be scrubbed. (B) Clients should avoid putting any creams or lotions on the treated area. This could interfere with treatment. (C) Radiation therapy clients should wear loose-fitting clothes and avoid tight, irritating fabrics. (D) The area of skin being treated is sensitive to sunlight, and the client should take care to prevent sun damage by avoiding direct sunlight and covering the area when she is in the sun. QUESTION 580 A male client is being treated in the burn unit for thirddegree burns on his head, neck, and upper chest received in the last 24 hours. The nurse is evaluating the effectiveness of fluid resuscitation. Which of the following indicates effective fluid balance? A. His weight increases from 165 to 175 lb. B. His urine output is equal to his total fluid intake. C. His urine output has been>35 mL/hr for the past 12 hours.D. His blood pressure is 94/62. Correct Answer: C Section: (none) Explanation Explanation/Reference: Explanation: (A) A weight gain of 10 lb represents a state of overhydration. (B) He is losing fluids through insensible losses; a urine output equal to his intake indicates that he is receiving too little fluids. (C) A urine output greater than his intake indicates that he is receiving adequate fluid resuscitation to account for urinary and insensible losses. (D) A blood pressure of 94/62 indicates a state of underhydration and inadequate circulatory volume. QUESTION 581 A 24-year-old male client is admitted with a diagnosis of sickle cell anemia. The nurse discusses his disease with him and emphasizes the following information: A. He should monitor his sputum, stools, and urine for signs of bleeding. B. His daily diet should include a large amount of fluid. C. He should not be concerned about having to fly on a commuter airplane on a weekly basis. D. He should not worry about having children because this disease is passed on only by female carriers. Correct Answer: B Section: (none) Explanation Explanation/Reference: Explanation: (A) Bleeding is not a symptom of sickle cell anemia or sickle cell crisis. (B) Decreased blood viscosity leads to sickling of red blood cells. Increased fluid intake maintains adequate circulating blood volume and decreases the chance of sickling. (C) Hypoxia leads to sickling of cells. Flying in nonpressurized planes places the client in a situation of low O2 tension, which can lead to sickling. (D) Male and female clients with sickle cell disease can pass the trait on to their offspring. Therefore, this client should receive genetic counseling prior to having children. QUESTION 582 A female client has been diagnosed with chronic renal failure. She is a candidate for either peritoneal dialysis or hemodialysis and must make a choice between the two. Which information should the nurse give her to help her decide? A. Hemodialysis involves less time to filter the blood; but the client must consider travel time, distance, and inconvenience. B. Hemodialysis involves more time to filter the blood than does peritoneal dialysis. C. Peritoneal dialysis has almost no complications and is less time consuming than hemodialysis.Therefore it is preferred. D. Peritoneal dialysis requires that a home health nurse prepare and administer the treatments. Correct Answer: A Section: (none) Explanation Explanation/Reference: Explanation: (A) Hemodialysis is faster in clearing the blood of toxins than peritoneal dialysis. However, clients must consider the time that they spend traveling to the dialysis center and the disruption in their daily lives. (B) Peritoneal dialysis requires several exchanges with dwelling time for the dialysate and therefore takes longer thanhemodialysis. (C) Several serious complications of peritoneal dialysis include peritonitis, catheter displacement and/or plugging, or pain during dialysis. (D) A client can be taught to self-administer peritoneal dialysis without the aid of a professional. QUESTION 583 A female client decides on hemodialysis. She has an internal vascular access device placed. To ensure patency of the device, the nurse must: A. Assess the site for leakage of blood or fluids B. Auscultate the site for a bruit C. Assess the site for bruising or hematoma D. Inspect the site for color, warmth, and sensation Correct Answer: B Section: (none) Explanation Explanation/Reference: Explanation: (A) This is an internal device. Assessment of the site should include assessing for swelling, pain, warmth, and discoloration. This measure does not assess patency. (B) The presence of a bruit indicates good blood flow through the device. (C) The nurse should inspect the site for bruising or hematoma; however, this measure does not assure patency of the device. (D) The nurse should inspect the vascular access site frequently for signs of infection. However, this does not assure patency. QUESTION 584 A client isreceiving peritoneal dialysis. He has been taught to warm the dialyzing fluid prior to instilling it because: A. Warmed solution helps keep the body temperature maintained within a normal range during instillation B. Warmed solution helps dilate the peritoneal blood vesselsC. Warmed solution decreases the risk of peritoneal infection D. Warmed solution promotes a relaxed abdominal muscle Correct Answer: B Section: (none) Explanation Explanation/Reference: Explanation: (A) Instilling a cool solution does not significantly lower the body temperature during peritoneal dialysis. (B) Warmed solution does help dilate the peritoneal blood vessels, facilitating the exchange of fluids. (C) Warming the dialysate does not decrease the risk of peritoneal infection. Sterile technique decreases this risk. (D) Relaxing the abdominal muscles does not facilitate peritoneal dialysis. QUESTION 585 A female client is exhibiting signs of respiratory distress. Which of the following signs indicate a possible pneumothorax? A. Crackles or rales on the affected side B. Bradypnea and bradycardia C. Shortness of breath and sharp pain on the affected side D. Increased breath sounds on the affected side Correct Answer: C Section: (none) Explanation Explanation/Reference: Explanation: (A) With a pneumothorax, air occupies the pleural space. Crackles or rales are heard with increased fluid or secretions and would not be present with air in the space. (B) With a pneumothorax, the client would experience tachypnea and tachycardia to compensate for the decrease in oxygenation. (C) Symptoms of pneumothorax include shortness of breath, sharp pain on the affected side with movement or coughing, asymmetrical chest expansion, and diminished or absent breath sounds on the affected side. (D) With a pneumothorax, breath sounds would be decreased on the affected side (indicates air in the pleural space). QUESTION 586 A female client has a chest tube placed. It is accidentally pulled out of the intrapleural space when she is ambulating. The first action the nurse should take is to: A. Instruct the client to cough deeply to re-expand her lung B. Put on sterile gloves and replace the tubeC. Apply a petrolatum dressing over the site D. Auscultate the lung to determine if she needs the tube replaced Correct Answer: C Section: (none) Explanation Explanation/Reference: Explanation: (A) This action is inappropriate. Coughing will not re-expand the lung and could result in further harm. (B) This action is a medical procedure, not a nursing procedure. (C) An occlusive dressing will prevent further air leak until the physician institutes further treatment. (D) The decision to reinsert the tube is a medical decision, not a nursing one. QUESTION 587 A male client has heart failure. He has been instructed to gradually increase his activities. Which signs and symptoms of worsening heart failure should the nurse tell him to watch for that would indicate a need for him to lower his activity level? A. Pain in his legs when he walks B. Thirst, weight loss, and polyuria C. Drowsiness and lethargy after his activities D. Weight gain, edema in his lower extremities, and shortness of breath Correct Answer: D Section: (none) Explanation Explanation/Reference: Explanation: (A) Pain in the legs could be indicative of doing too much too quickly, but not of worsening heart failure. The client should be cautioned to increase his activities slowly. (B) Thirst, weight loss, and frequent urination are not indicative of heart failure. The client should report these symptoms to his physician. (C) Drowsiness and lethargy are not indicative of worsening heart failure. The client should report these symptoms to his physician. (D) All of these symptoms indicate a worsening cardiac condition possibly associated with too much activity. The client's activity level should be evaluated. QUESTION 588 A male client is undergoing cardiac tests. He has been instructed to wear a Holter monitor. The nurse knows she has included the appropriate information in her teaching when the client tells her: A. "He should remove the electrodes for bathing." B. "Damage to his heart muscle will be recorded by the monitor."C. "He is to keep a record of everything he does during the day." D. "He is to refrain from activities that cause chest pain." Correct Answer: C Section: (none) Explanation Explanation/Reference: Explanation: (A) The client should leave the electrodes in place during the entire time the test is ordered. He should not even remove the electrodes for bathing. (B) The Holter monitor will record cardiac electrical activity but will not record damage to his myocardium. (C) The client should keep a record of all of his activities so the physician can correlate the ECG findings with his activities. (D) The client should continue doing hisregular activities. The purpose of the Holter monitor is to record heart activity during routine activities. QUESTION 589 To prevent thrombophlebitis in a client on complete bed rest, the nursing care plan should include: A. Dangle the client's legs over the edge of the bed every shift. B. Massage the client's calves briskly every shift. C. Keep the client's legs extended and discourage any movement. D. Have the client tighten and relax leg muscles several times daily. Correct Answer: D Section: (none) Explanation Explanation/Reference: Explanation: (A) Dangling the client's legs over the edge of the bed will contribute to stasis and pooling of blood and increases the risk of thrombus formation. (B) Massaging the client's calves could result in dislodging an embolus. (C) Decreased movement will contribute to pooling of blood and increased risk of venous thrombosis. (D) Tightening and relaxing leg muscles increases circulation and decreases the risk of venous thrombosis. QUESTION 590 A 78-year-old female client has a total hip arthroplasty. Her nurse should know that which of the following is contraindicated? A. Encourage exercises in the unaffected extremities. B. Encourage her to cross and uncross her legs.C. Check neurological and circulatory status of the affected leg hourly. D. Place a trochanter roll along the upper thigh of the affected leg. Correct Answer: B Section: (none) Explanation Explanation/Reference: Explanation: (A) Exercising the unaffected extremities will prevent contractures and emboli. (B) Crossing and uncrossing the affected leg after surgery can dislocate the joint. (C) Neurological and circulatory status of the affected leg has been compromised by surgery. Hourly checks are needed to monitor the status of the leg. (D) A trochanter roll will prevent the upper thigh from rolling outward, increasing the chances of dislocation. QUESTION 591 A male client has a history of diverticulosis. He has questions about the foods that he should eat.His nurse gives him the following information: A. He should be on a high-fiber diet. B. He should eat a low-residue diet. C. He should drink minimal amounts of fluids. D. He does not need to make any modifications. Correct Answer: A Section: (none) Explanation Explanation/Reference: Explanation: (A) Clients with diverticulosis should maintain a high-fiber diet and prevent constipation with bran or bulk laxatives. (B) Lowresidue diets lead to constipation and are contraindicated in clients with diverticulosis. (C) Clients with diverticulosis should drink at least eight glasses of water each day to prevent constipation. (D) Clients with diverticulosis should modify their diet to include high-fiber foods and bulk laxatives. QUESTION 592 A term neonate has experienced no distress at birth and has an Apgar score of 9. Her mother has asked to breastfeed her following delivery. Immediately after birth, the neonate was most susceptible to heat loss. The most appropriate intervention to conserve heat loss and promote bonding is to: A. Place her under the radiant warmer B. Dry her with blanketsC. Place her to her mother's breast D. Place her on a heated pad Correct Answer: C Section: (none) Explanation Explanation/Reference: Explanation: (A) A radiant warmer maintains an optimal thermal environment by use of a thermal skin sensor taped to the infant. The warmer limits parental attachment, so, although appropriate, it is not an intervention that promotes infant attachment. (B) Warmed blankets prevent heat loss in the neonate by conduction. In addition, tactile stimuli promote crying and lung expansion. This intervention does not promote attachment, however. (C) Skin-to-skin contact is an effective way to conserve heat after delivery and promotes parental attachment following birth in the healthy term infant. The first period of reactivity lasts approximately 30 minutes following birth. A strong sucking reflex and an active, awake newborn characterize this period. (D) Surfaces of objects warmer than the infant promote overheating by conduction, and neonatal hyperthermia may result. QUESTION 593 A client who is gravida 1 para 1 vaginally delivered a 7- lb girl. She received a midline episiotomy at delivery. When assessing the level of her uterus immediately following delivery, the nurse would expect the fundus to be located: A. At the umbilicus B. At the symphysis pubis C. Midway between the umbilicus and the xiphoid process D. Midway between the umbilicus and the symphysis pubis Correct Answer: D Section: (none) Explanation Explanation/Reference: Explanation: (A) Within 12 hours of delivery, the fundus of the uterus rises to, or slightly above or below, the umbilicus. Fundal height generally decreases 1 fingerbreadth, or 1 cm/day. (B) The uterus descends into the pelvic cavity at approximately 1012 postpartal days and can no longer be palpated abdominally. (C) Within 12 hours ofdelivery, the fundus of the uterus rises to, or slightly above or below, the umbilicus. Fundal height generally decreases 1 fingerbreadth, or 1 cm/day. An enlargeduterus may indicate subinvolution or postpartal hemorrhage. (D) Immediately following delivery, the uterus lies midline, about midway between the umbilicus andthe symphysis pubis. QUESTION 594A 19-year-old primigravida is admitted to the labor and delivery suite of the hospital. Her husband is accompanying her. The couple tells the nurse that this is the first hospital admission for her. The client's vaginal exam indicates she is 3 cm dilated, 80% effaced, and at _0 station. Based on the vaginal exam, she is in: A. Stage 2, latent phase B. Stage 1, active phase C. Stage 3, transition phase D. Stage 1, latent phase Correct Answer: D Section: (none) Explanation Explanation/Reference: Explanation: (A) The second stage of labor is from full cervical dilation through birth of the baby. The three phases of this stage include latency or resting, descent, and final transition. The client is less than fully dilated so she is not in stage 2. (B) The first stage of labor begins with regular uterine contractions and continues until the woman is 10 cm dilated. The three phases of this stage include the early or latent phase (03 cm), the active phase (47 cm), and the transitional phase (710 cm). The client is <4 cm dilated so she is in the latent phase of the first stage of labor. (C) The third stage of labor is from the birth of the baby until the delivery of the placenta. The client is less than fully dilated. (D) The first stage of labor begins with regular uterine contractions and continues until the woman is 10 cm dilated. The three phases of thisstage include the early orlatent phase (03 cm), the active phase (47 cm), and the transitional phase (710 cm). The client is <4 cm dilated so she is in the latent phase of the first stage of labor. QUESTION 595 A client is pregnant for the fourth time and has had three normal vaginal deliveries. She is in active labor and fully dilated. Suddenly she calls, "Nurse, the baby is coming." As the nurse responds to her call, which one of the following observations should the nurse make first? A. Inspect the perineum. B. Time the contractions. C. Prepare a sterile area for delivery. D. Auscultate for fetal heart rate (FHR). Correct Answer: A Section: (none) Explanation Explanation/Reference: Explanation:(A) The nurse must assess the labor status to determine if birth is imminent. The nurse may note perineal bulging, crowning, or birth of the head to ascertain labor status. (B) Assessing uterine contractions is one intervention to ascertain labor status. Based on the client's cry, it is not the intervention of choice. (C) If delivery of the infant is imminent, preparing a clean or sterile area for delivery is appropriate, but labor status must be established, whether delivery is imminent, by perineal assessment. (D) Assessing FHR is one intervention to ascertain fetal well-being. Based on the client's cry, this is not the intervention of choice. QUESTION 596 A client has just received an epidural block. She is laboring on her right side. The nurse notes that her blood pressure has dropped from 132/68 to 78/42 mm Hg. The nurse's first action would be to: A. Call the physician immediately and give dopamine IM B. Turn her on her left side and recheck her blood pressure in 5 minutes C. Administer oxytocin (Pitocin) immediately and increase the rate of IV fluids D. Increase the rate of IV fluids and start O2 by mask Correct Answer: D Section: (none) Explanation Explanation/Reference: Explanation: (A) Nursing measures to support fetal oxygenation and promote maternal blood pressure would precede calling the physician. (B) Systolic pressures below 100 mm Hg or a reduction in the systolic pressure of>30% necessitate treatment. Assessing the blood pressure in 5 minutes may allow for further fetal and/or maternal compromise. Turning the client on her left side will promote uteroplacental perfusion and is appropriate. (C) Oxytocin (Pitocin) increases the strength of uterine contractions and may cause maternal hypotension; thus it is an inappropriate drug for use in this clinical situation. IV fluids would be increased to expand the circulating blood volume and promote increased blood pressure. (D) Turning the mother to her left lateral side promotes uteroplacental perfusion. IV fluids are administered to increase the circulating blood volume, and O2 is administered to promote fetal oxygenation and decrease the nausea accompanying the hypotension. QUESTION 597 A 28-year-old client comes to the clinic for her first prenatal examination. In relating her obstetrical history, she tells the nurse that she has been pregnant twice before. She had a "miscarriage" with the first pregnancy after 6 weeks. With the second pregnancy, she delivered twin girls at 31 weeks' gestation. One of the twins was stillborn and the other twin died at 4 days of age. Using a five-digit system, the nurse records her as being: A. 2-0-2-1-0 B. 2-2-2-1-2 C. 3-0-1-1-0 D. 2-1-1-0-0 Correct Answer: CSection: (none) Explanation Explanation/Reference: Explanation: (A) The first digit represents the total number of pregnancies. This client has been pregnant 3 times including this pregnancy. The twin pregnancy counts as only one pregnancy, and because she delivered prior to 37 weeks' gestation, the third digit is recorded as 1. (B) The first digit represents the total number of pregnancies. This client has been pregnant 3 times including this pregnancy. The second digit represents the total number of fullterm deliveries; she has lost two pregnancies before 37 weeks' gestation. At present, she has no living children, so the fifth digit is noted as 0. (C) The client is pregnant for the third time, and the first digit reflects the total number of pregnancies. She has had no full-term deliveries, because she delivered prior to 37 gestational weeks, so the second digit is recorded as 0. The third digit represents the number of preterm deliveries, and a twin pregnancy counts as only one delivery. She lost an earlier pregnancy prior to 20 gestational weeks, and the fourth digit reflects spontaneous or elective abortions. Lastly, the fifth digit indicates the number of children currently living, and shehas no living children. (D) She is pregnant for the third time, and the first digit reflects the total number of pregnancies. In the previous two pregnancies, she delivered prior to 37 gestational weeks, thus having no full-term deliveries, which is indicated by the second digit. The fourth digit represents the total number of abortions, spontaneous or elective, and she reported a spontaneous abortion with her first pregnancy. QUESTION 598 A 22-year-old client is 16 weeks pregnant. She and her husband are expecting their first baby. The client tells the nurse that her last normal menstrual period was February 16, with 3 days of spotting on February 17, 18, and 19. The nurse calculates her expected date of delivery to be: A. November 23rd B. December 26th C. September 14th D. December 9th Correct Answer: A Section: (none) Explanation Explanation/Reference: Explanation: (A) Naele's rule is as follows: add 7 days to the 1st day of the last menstrual period, subtract 3 months, and then add 1 year. (B) Naele's rule presumes that the woman has a 28- day menstrual cycle, with conception occurring on the 14th day of the cycle. Slight vaginal spotting may occur in early gestation for unknown reasons but is insignificant in the calculation of Naele's rule. (C) Naele's rule presumes that the woman has a 28-daymenstrual cycle, with conception occurringon the 14th day of the cycle. Slight vaginal spotting may occur in early gestation for unknown reasons but is insignificant in the calculation of Naele's rule. (D) Naele's rule presumes that the woman has a 28-day menstrual cycle, with conception occurring on the 14thday of the cycle. Slight vaginal spotting may occur in early gestation for unknown reasons but is insignificant in the calculation of Naele's rule. Topic 6, Questions Set FQUESTION 599 On the third postpartum day, a client complains of extremely tender breasts. On palpation, the nurse notes a very firm, shiny appearance to the breasts and some milk leakage. She is bottle feeding. The nurse should initially recommend to her to: A. Take 2 ibuprofen (Motrin) tablets by mouth now because the baby will be returning for feeding in 20 minutes B. Allow the infant to breast-feed at the next feeding time to empty the breasts C. Apply ice packs to the breasts and wear a supportive, well-fitting bra D. Take a warm shower and express milk from both breasts until empty Correct Answer: C Section: (none) Explanation Explanation/Reference: Explanation: (A) Judicious use of analgesics is appropriate with breast engorgement; however, mechanical suppression would be the initial recommendation. (B) Breast-feeding every 11/23 hours will reduce and/or prevent breast engorgement. Breast-feeding will promote milk production, which will compound the distention and stasis of the venous circulation of engorgement in a bottlefeeding mother. (C) Ice packs reduce milk flow while the snug, supportive bra provides mechanical suppression and decreases pulling on Cooper's ligament. In addition, breast binders or ace bandages may be used for some women. (D) Warmth promotes milk production and may stimulate the let-down reflex. These measures would contribute to the venous congestion of engorgement. QUESTION 600 A registered nurse is trying to determine the appropriate care that she should provide for her obstetrical clients. Which of the following documents is considered the legal standard of practice? A. State nursing practice act B. AWHONN Standards for the Nursing Care of Women and Newborns C. American Nurses' Association Standards of Maternal- Child Health Nursing D. International Council of Nurses' Code Correct Answer: A Section: (none) Explanation Explanation/Reference: Explanation: (A) The state nursing practice act determines the standard of care for the professional nurse. (B) AWHONN Standards are published as recommendations and guidelines for maternal-newborn nursing. (C) American Nurses' Association Standards are published asrecommendations and guidelines for maternalchild healthnursing. (D) The International Council of Nurses' Code emphasizes the nurse's obligations to the client rather than to the physician. It is published as recommendations and guidelines by the international organization for professional nursing. QUESTION 601 A female client admitted to the labor and delivery unit thinks her bag of water "broke" approximately 2 hours ago. She is having mild contractions 5 minutes apart. The most immediate nursing intervention would be to: A. Note the color and amount of fluid on her clothes. B. Assess the FHR. C. Notify the physician. D. Place the nitrazine test paper at the cervical os and note the color change. Correct Answer: B Section: (none) Explanation Explanation/Reference: Explanation: (A) Amniotic fluid is generally pale and straw colored. Meconium- stained amniotic fluid would indicate a previous hypoxic episode. This intervention, though appropriate, is not the immediate priority. (B) With rupture of the membranes, the umbilical cord may prolapse if the presenting part does not fill the pelvis. Assessing FHR ascertains fetal well-being. (C) More information regarding fetal status and assessing for membrane rupture is needed prior to contacting the physician. (D) Nitrazine test paper differentiates amniotic fluid from urine. Amniotic fluid is normally alkaline in contrast to urine, which is acidic. This intervention, though appropriate, is not the immediate priority. QUESTION 602 A new mother experiences strong uterine contractions while breast-feeding her baby. She excitedly rings for the nurse. When the nurse arrives the mother tells her, "Something is wrong. This is like my labor." Which reply by the nurse identifies the physiological response of the client? A. "Your breasts are secreting a hormone that enters your bloodstream and causes your abdominal muscles to contract." B. "Prolactin increases the blood supply to your uterus, and you are feeling the effects of this blood vessel engorgement." C. "The same hormone that is released in response to the baby's sucking, causing milk to flow, also causes the uterus to contract." D. "There is probably a small blood clot or placental fragment in your uterus, and your uterus is contracting to expel it." Correct Answer: C Section: (none) Explanation Explanation/Reference: Explanation:(A) Mammary growth as well as milk production and maintenance in the breast occur in response to hormones produced primarily by the hypothalamus and the pituitary gland. (B) Prolactin stimulates the alveolar cells of the breast to produce milk. It is important in the initiation of breast-feeding. (C) Oxytocin, which is released by the posterior pituitary, stimulates the let-down reflex by contraction of the myoepithelial cells surrounding the alveoli. In addition, it causes contractions of the uterus and uterine involution. (D) Afterpains may occur with retained placental fragments. A boggy uterus and continued bleeding are other symptoms that occur in response to retained placental fragments. QUESTION 603 A 20-year-old female client delivers a stillborn infant. Following the delivery, an appropriate response by the labor nurse to the question, "Why did this happen to my baby?" is: A. "It's God's will. It was probably for the best. There was something probably wrong with your baby." B. "You're young. You can have other children later." C. "I know your other children will be a great comfort to you." D. "I can see you're upset. Would you like to see and hold your baby?" Correct Answer: D Section: (none) Explanation Explanation/Reference: Explanation: (A) The mother and the father require support; the nurse should not minimize their grief in this situation. (B) Attachment to this infant occurs during the pregnancy for both the mother and father. Another child cannot replace this child. (C) Attachment to this infant occurs during the pregnancy for both the mother and father. Siblings will not replace their feelings or minimize their loss of this infant. (D) Holding and viewing the infant decreases denial and may facilitate the grief process. The nurse should prepare family members for how the infant appears ("she is bruised") and provide support. QUESTION 604 A client's prenatal screening indicated that she has no immunity to rubella. She is now 10 weeks pregnant. The best time to immunize her is: A. In the immediate postpartum period B. After the first trimester C. At 28 weeks' gestation D. Within 72 hours postpartum Correct Answer: A Section: (none) ExplanationExplanation/Reference: Explanation: (A) The rubella vaccine is made with attenuated virus and is given in the immediate postpartal period to prevent infection during pregnancy and subsequent adverse fetal and neonatal sequelae. Mothers are advised to prevent pregnancy for 3 months following immunization. (B) Rubella infection during the second trimester may result in permanent hearing loss for the fetus. (C) RhoGam is the drug generally administered at 28 weeks' gestation to Rh-negative women. It is contraindicated to administer rubella vaccine during pregnancy. (D) RhoGam is the drug administered within 72 hours postpartum to Rh-negative women to prevent the development of antibodies to fetal cells. QUESTION 605 A 24-year-old woman who is gravida 1 reports, "I can't take iron pills because they make me sick." She continues, "My bowels aren't moving either." In counseling her based on these complaints, the nurse's most appropriate response would be, "It would be beneficial for you to eat . . . A. prunes." B. green leafy vegetables." C. red meat." D. eggs." Correct Answer: A Section: (none) Explanation Explanation/Reference: Explanation: (A) Prunes provide fiber to decrease constipation and are an excellent source of dietary iron, as the prenatal client is not taking her supplemental iron and iron- deficiency anemia is common during pregnancy. (B) Green leafy vegetables provide a source of fiber and iron; however, prunes are a better source of both. (C) Red meat is a good iron source but will not address the constipation problem. (D) Eggs are a good iron source but do not address the constipation problem. QUESTION 606 A 26-year-old female client presents at 10 weeks' gestation. She currently is a G3 1-0-1-1. Her mother and grandmother have heart disease. Her grandmother also has insulindependent diabetes. The client's previous delivery was a term female infant weighing 9 lb 13 oz. The client is 5 ft 6 inches tall and her current weight is130 lb. Based on her history, she is at risk for developing diabetes in pregnancy. Which of the following factors places her at risk for gestational diabetes? A. Age>25 years B. Maternal weight C. Previous birth of an infant weighing>9 lb D. Family history of heart disease Correct Answer: CSection: (none) Explanation Explanation/Reference: Explanation: (A) Maternal age older than 30 years is an identified risk factor for diabetes. Age younger than 30 years is insignificant for diabetes unless there is a familial historyof diabetes. (B) The client's weight is appropriate for her height. Obesity or pregnancy weight >20% of the ideal weight is a contributing factor to the development of gestational diabetes. (C) The birth of an infant weighing >9 lb (4000 g) is an identified risk factor for gestational diabetes. (D) A familial history of heart disease is insignificant in the development of diabetes. However, a familial history of type II diabetes mellitus is identified as a risk factor in the development of diabetes during pregnancy. QUESTION 607 The nurse assesses a client's monitor strip and finds the following: uterine contractions every 3 4 minutes, lasting 6070 seconds; FHR baseline 134146 bpm, with accelerations to 158 bpm with fetal movement. Which nursing intervention is appropriate? A. Notify physician of nonreassuring FHR pattern. B. Turn the client to her left side. C. Start IV for fetal distress and administer O2 at 68 liters by mask. D. Evaluate to see if the monitor strip is reassuring. Correct Answer: D Section: (none) Explanation Explanation/Reference: Explanation: (A) These indices are within normal parameters; therefore, the nurse does not need to contact the physician. (B) The purpose of turning a client to her left side is to maximize uteroplacental blood flow. Based on the above assessment, there is no indication that blood flow is compromised. (C) These interventions are appropriate nursing interventions for late and prolonged decelerations. Following these interventions, the nurse should notify the physician. These indices are within normal parameters; therefore, the nurse does not need to start an IV and administer O2. (D) Variations of 20 bpm above or below the baseline FHR is considered normal. Normal FHRs range from 120160 bpm. As the fetus moves, the FHR increases, and accelerations often occur in concert with contractions. During the active phase of labor, the frequency of uterine contractions is every 24 minutes, with an appropriate duration of 60 sec. QUESTION 608 Early in her ninth month of pregnancy, a client has been diagnosed as having mild preeclampsia. In counseling her about her diet, the nurse must emphasize the importance of: A. Decreasing her sodium intakeB. Decreasing her fluids C. Increasing her carbohydrate intake D. Eating a moderate to high-protein diet Correct Answer: D Section: (none) Explanation Explanation/Reference: Explanation: (A) Women with pregnancy-induced hypertension have a reduced plasma volume secondary to venous vessel constriction, not hypovolemia; therefore, sodium restriction is not recommended. It is suggested that these women avoid extremely salty foods. (B) Drinking six to eight glasses of water per day facilitates optimalfluid volume and renal perfusion, but it will not decrease the venous vessel constriction of pregnancy-induced hypertension. (C) Carbohydrate needs increase during pregnancy, specifically during the second and third trimesters, but they have not been linked to pregnancy-induced hypertension. (D) Loss of urinary protein (proteinuria) is associated with increased permeability of the large protein molecules with pregnancy-induced hypertension.Additional dietary protein also helps increase the plasma colloidal osmotic pressure. Diets deficient in protein have been linked to pregnancy-induced hypertension. QUESTION 609 A 30-year-old client in the third trimester of her pregnancy asks the nurse for advice about upper respiratory discomforts. She complains of nasal stuffiness and epistaxis, most noticeable on the left side. Which reply by the nurse is correct? A. "It sounds as though you are coming down with a bad cold. I'll ask the doctor to prescribe a decongestant for relief of symptoms." B. "A good vaporizer will help; avoid the cool air kind. Also, try saline nose drops, and spend less time on your left side." C. "These discomforts are all a result of increased blood supply; one of the pregnancy hormones, estrogen, causes them." D. "This is most unusual. I'm sure your obstetrician will want you to see an ENT (ear, nose, throat) specialist." Correct Answer: C Section: (none) Explanation Explanation/Reference: Explanation: (A) Decongestants may exaggerate the nasal stuffiness associated with pregnancy. Judicious use of decongestants and nasal sprays is advocated during pregnancy. (B) Cool air vaporizers and saline drops may help to relieve the nasal stuffiness. Positioning on either lateral side does not decrease nasal stuffiness or prevent epistaxis. (C) Increased estrogen levels result in nasal mucosa edema with subsequent nasal stuffiness. Estrogen also promotes vasodilation, which contributes to epistaxis. The nurse may recommend cool air vaporizers and saline drops to help with the nasal stuffiness. (D) Increased estrogen levels result in nasal mucosa edema with subsequent nasal stuffiness. Estrogen also promotes vasodilation discomforts associated with pregnancy.QUESTION 610 A newborn girl's father expresses concern that the newborn does not have good control of her hands and arms. It is important for the father to realize certain neurological patterns that characterize the newborn: A. Mild hypotonia is expected in the upper extremities. B. Purposeless, uncoordinated movements of the arms are indicative of neurological dysfunction. C. Function progresses in a head-to-toe, proximal-distal fashion. D. Asymmetrical movement of the extremities is not unusual and will disappear with maturation of the central nervous system. Correct Answer: C Section: (none) Explanation Explanation/Reference: Explanation: (A) Term neonates are predominantly in a flexed position with strong active muscle tone that increases. Newborns are slightly hypertonic. (B) Neonatal movements may be jerky and uncoordinated as the neonate works against gravity in contrast to the buoyancy of the amniotic fluid. Jerky movements must be differentiated from the tremors of hypoglycemia, hypocalcemia, and neurological dysfunction. (C) Growth of the newborn progresses in a cephalocaudal, proximal-distal fashion. Knowledge regarding infant development may facilitate parental involvement and infant stimulation. (D) Asymmetrical movements of the extremities are indicative of neurological dysfunction. QUESTION 611 A client delivered a term infant 1 hour ago. Her uterus on assessment is boggy and is U +1 in contrast to the previous assessment of U _2. The immediate nursing response is to: A. Administer methergine IM B. Remove the retained placental fragments C. Assist the client to the bathroom and provide cues to stimulate urination D. Massage the fundus until firm Correct Answer: D Section: (none) Explanation Explanation/Reference: Explanation: (A) Methergine is given following placental delivery to promote uterine contractions and prevent hemorrhage. Methergine may be administered in this clinical situation, but fundal massage would be the first response. (B) Removal of retained placental fragments is done by the physician and is not the first response. (C) If the fundus rises and is deviated, particularly to theright, the nurse should suspect bladder distention secondary to bladder and urethral trauma associated with birthand decreased bladder tone following delivery. Therefore, women have a diminished sensation to void. (D) A boggy fundusrises and is indicative of blood pooling, predisposing the woman to clot formation. Massage the uterus until firm. Too vigorous massage will result in atonia. Clots may be expelled by a kneading motion of the uterus by the nurse. QUESTION 612 A 35-year-old primigravida comes to the clinic for her first prenatal visit. The midwife, on examining the client, suspects that she is approximately 11 weeks pregnant. The pregnancy is positively confirmed by finding: A. Chadwick's sign B. FHR by ultrasound C. Enlargement of the uterus D. Breast tenderness and enlargement Correct Answer: B Section: (none) Explanation Explanation/Reference: Explanation: (A) Chadwick's sign is a presumptive sign of pregnancy. The coloration may not subside from past pregnancy or could be caused by other situations that create vasocongestion. (B) FHR (movement) observed on ultrasound is a positive diagnosis of pregnancy. (C) Enlargement of the uterus may be due to fibroids or infection. It is considered a probable sign. (D) Breast tenderness and enlargement is a presumptive sign because it may be due to other conditions, such as premenstrual changes. QUESTION 613 A female client has been treated since childhood for mitral valve prolapse. The antibiotic of choice for her during pregnancy would be: A. Sulfa B. Tetracycline C. Hydralazine D. Erythromycin Correct Answer: D Section: (none) Explanation Explanation/Reference: Explanation:(A) Sulfa is a teratogen and will cause kernicterus. (B) Tetracycline is a teratogen and will effect tooth development. (C) Hydralazine is not an antibiotic but a calcium channel blocker. (D) Erythromycin is safe during pregnancy and can be used when the client is allergic to penicillin. QUESTION 614 A pregnant client complains of varicosities in the third trimester. Which of the following activities should she be advised to avoid? A. Sitting with legs crossed at ankles B. Wearing thromboembolic disease (TED) stockings C. Wearing support pantyhose D. Wearing knee-high stockings Correct Answer: D Section: (none) Explanation Explanation/Reference: Explanation: (A) Sitting with the legs crossed at the ankles does not interfere with circulation or create pressure points. (B) TED stockings will help to reduce the varicosity by supporting the vein. Stockings must be applied with legs elevated. (C) Support pantyhose help to reduce the varicosity by supporting the vein. They also provide support to the uterus and allow for better return circulation. Hose must be applied like TED stockings. (D) Knee-high stockings create constriction and pressurepoints that interfere with circulation in the lower extremities. QUESTION 615 A client at 9 weeks' gestation comes for an initial prenatal visit. On assessment, the nurse discovers this is her second pregnancy. Her first pregnancy resulted in a spontaneous abortion. She is 28 years old, in good health, and works full-time as an elementary school teacher. This information alerts the nurse to which of the following: A. An increased risk in maternal adaptation to pregnancy B. The need for anticipatory guidance regarding the pregnancy C. The need for teaching regarding family planning D. An increased risk for subsequent abortions Correct Answer: B Section: (none) Explanation Explanation/Reference: Explanation: (A, D) There are no data to support this. (B) Anticipatory guidance and health maintenance is a first-line defense in the promotion of healthy mothers and healthybabies. (C) There are no data to support this at this time. This will be a concern later. QUESTION 616 A client is pleased about being pregnant, yet states, "It is really not the best time, but I guess it will be OK." The nurse's assessment of this response is: A. Initial maternal-infant bonding may be poor. B. Client may have a poor relationship with her husband. C. This response is normal in the first trimester. D. Thisresponse is abnormal, to be re-evaluated at the next visit. Correct Answer: C Section: (none) Explanation Explanation/Reference: Explanation: (A) Ambivalence is normal during the first trimester. Reva Rubin addresses the issue of "not now" in the first trimester. The statement still leaves room forexploration. (B) There are no data to support this. This statement by the mother still leaves room for exploration. (C) Ambivalenceis normal during the first trimester. Reva Rubin addresses the issue of "not now." This fact should be shared with the mother during further exploration of the comment. (D) It is not abnormal. If it were, another month would also be too long to wait. QUESTION 617 A client at 6 months' gestation complains of tiredness and dizziness. Her hemoglobin level is 10 g/dL, and her hematocrit value is 32%. Her nutritional intake is assessed as sufficient. The most likely diagnosis is: A. Iron-deficiency anemia B. Physiological anemia C. Fatigue due to stress D. No problem indicated Correct Answer: A Section: (none) Explanation Explanation/Reference: Explanation: (A) This clinical situation is indicative of iron-deficiency anemia because the client has inadequate nutritional intake. Her blood volume is increasing faster than herred blood cell volume. Anemia is present in the second trimester when the hemoglobin level is <10.5 and the hematocrit value falls below 35%. She needs increased iron supplements with follow-up. (B) The client's values are below levels for physiological anemia. (C) The client is fatigued because of a low hemoglobinlevel. (D) Her hemoglobin level is low and will probably decrease even more when the blood volume peaks at 28 weeks. QUESTION 618 In counseling a client, the nurse emphasizes the danger signals during pregnancy. On the next visit, the client identifies which of the following as a danger signal that should be reported immediately? A. Backache B. Leaking of clear yellow fluid from breasts C. Constipation with hemorrhoids D. Visual changes Correct Answer: D Section: (none) Explanation Explanation/Reference: Explanation: (A) Backache is a common complaint during pregnancy. Proper body mechanics, pelvic rock, back rubs, and other comfort measures should relieve the discomfort. In the presence of uterine contractions, the backache would radiate to the lower abdomen. (B) Colostrum is normal and can be present anytime in the second half of pregnancy. (C) Constipation and hemorrhoids are common and do need attention, but they do not constitute a dangerous situation. (D) Visual changes are possibly related to PIH. The client should be assessed immediately to rule out or prevent worsening of PIH. QUESTION 619 The client will be more comfortable and the results more accurate when the nurse prepares the client for Leopold's maneuvers by having her: A. Empty her bladder B. Lie on her left side C. Place her arms over her head D. Force fluids 1 hour prior to procedure Correct Answer: A Section: (none) Explanation Explanation/Reference: Explanation: (A) A full bladder would cause discomfort and possible urinary incontinence during the exam. (B) The left side-lying position would not accommodate the exam. The head of the exam table or bed can be slightly elevated to prevent supine hypotension. (C) Arms extended over the head would cause the abdomen to be tighter andless easily palpable. (D) Forcing fluids would encourage a full bladder, which is not desired for the exam. QUESTION 620 Before giving methergine postpartum, the nurse should assess the client for: A. Decreased amount of lochial flow B. Elevated blood pressure C. Flushing D. Afterpains Correct Answer: B Section: (none) Explanation Explanation/Reference: Explanation: (A) Methergine is given to contract the uterus and to control postpartal hemorrhage; therefore, lochial flow should decrease. (B) Methergine may elevate the blood pressure. A client with an elevated blood pressure should not receive methergine, but she could be given oxytocin if necessary. (C) Flushing is not a side effect of methergine. (D) Afterpains are increased with methergine usage. The client should be informed that this is a normal response. QUESTION 621 A 24-hours' postpartum client complains of discomfort at the episiotomy site. On assessment, the nurse notes the episiotomy is without signs of infection. To relieve the discomfort, the nurse should first: A. Assist her with a sitz bath B. Administer the prescribed medication for pain C. Teach her Kegel exercises D. Apply an ice pack Correct Answer: A Section: (none) Explanation Explanation/Reference: Explanation: (A) Warm, moist heat will promote circulation and provide comfort. A sitz bath should be tried before medication is given. (B) Pain medication can be given when other comfort measures such as a sitz bath and topical applications are ineffective. (C) Kegel exercises facilitate sitting by decreasing tension on the episiotomy. They will not be effective for pain control or sustained comfort level. (D) Ice packs are appropriate to apply in the first 12 hours postdelivery to producevasoconstriction and to reduce edema to the area. QUESTION 622 The nurse explains perineal hygiene self-care postpartum to the client. She should be instructedto: A. Wear gloves for the procedure B. Place and adjust the pad from back to front C. Cleanse and wipe the perineum from front to back D. Protect the outer surface of the pad from contamination Correct Answer: C Section: (none) Explanation Explanation/Reference: Explanation: (A) Perineal hygiene is a clean procedure and does not require the client to wear gloves. A care provider should wear gloves to adhere to universal precautions. (B) The pad should be applied from front to back to prevent contamination of the birth canal or urinary tract from rectal bacteria. (C) Wiping from front to back and discarding the wipe prevents contamination of the urinary tract and birth canal from rectal bacteria. (D) The inner surface of the pad should not be touched to maintain asepsis. QUESTION 623 In teaching the client about proper umbilical cord care, the nurse recommends that: A. Petrolatum be placed around the cord after the sponge bath B. A belly binder be applied to prevent umbilical hernia C. The area be cleansed at diaper changes with alcohol and inspected for redness or drainage D. The cord clamp be left on until the cord stump separates Correct Answer: C Section: (none) Explanation Explanation/Reference: Explanation: (A) Petrolatum does not allow the cord to dry and will encourage infection. (B) Belly binders do not facilitate drying of the cord and will encourage abdominalrelaxation. (C) Frequent applications of alcohol will facilitate drying and discourage infection. (D) The cord clamp can be removed in 24 hours. Leaving it on iscumbersome and could pull on the cord unnecessarily. QUESTION 624 A baby is circumcised. Immediate postoperative care should include: A. Applying a loose diaper B. Keeping the baby NPO for 4 hours to avoid vomiting C. Changing the dressing frequently using dry, sterile gauze D. Taking the baby to his mother for cuddling Correct Answer: D Section: (none) Explanation Explanation/Reference: Explanation: (A) A pressure diaper should be applied to discourage hemorrhage. (B) The baby can be fed by his mother soon after the procedure, once it is assessed that he is not in any distress and is stable. (C) Dressing changes should not be dry. Dry dressing will stick. (D) Cuddling after the procedure will hopefully quiet the baby. Feeding is also important if his feeding was withheld prior to the procedure or it is time for a feeding. QUESTION 625 A 28-year-old multigravida has class II heart disease. At her prenatal visit at 34 weeks' gestation, all of the following observations are made. Which would require intervention? A. Weight gain of 2 kg in 4 weeks B. Blood pressure of 128/78 C. Subjective data: shortness of breath after showering D. Ankle edema reported present in late afternoon and evenings Correct Answer: C Section: (none) Explanation Explanation/Reference: Explanation: (A) This is not an excessive weight gain indicative of fluid retention. (B) The blood pressure is within normal range. (C) Showering should not cause shortness of breath. This could be a sign ofcardiac decompensation. (D) Dependent ankle edema is normal late in the day among pregnant women. Progressive edema would be a dangerous development.QUESTION 626 A client is admitted to the labor room. She is dilated 4 cm. She is placed on electric fetal monitoring. Which of the following observations necessitates notifying the physician? A. Contractions every 2 minutes, lasting 100 seconds B. Fetal heart decelerations during a contraction C. Beat-to-beat variability between contractions D. Fetal heart decelerations at the beginning of contractions Correct Answer: A Section: (none) Explanation Explanation/Reference: Explanation: (A) These are tetanic in nature and can cause rupture of the uterus. (B) The FHR decreases during contractions owing to vasoconstriction and should recover after the contraction. (C) Beat- to-beat variability is a normal finding and demonstrates fetal well-being. (D) The FHR may decrease at the beginning of a contraction owing to head compression. QUESTION 627 A client has been in labor 10 hours and is becoming very tired. She has dilated to 7 cm and is at 0 station with the fetus in a right occipitoposterior position. She is complaining of severe backache with each contraction. One comfort measure the nurse can employ is to: A. Place her in knee-chest position during the contraction B. Use effleurage during the contraction C. Apply strong sacral pressure during the contraction D. Have her push with each contraction Correct Answer: C Section: (none) Explanation Explanation/Reference: Explanation: (A) This measure is inappropriate. The knee-chest position is employed to take pressure off the cord. (B) Effleurage is a comfort measure but not the one that will contribute most to the relief of backache caused by a posterior position. (C) Sacral pressure will counteract the pressure created by the position of the fetal head. (D) The client is not completely dilated. Pushing is contraindicated until the second stage of labor.QUESTION 628 The nurse in the mental health center is instructing a depressed client about the dietary restrictions necessary in taking her medication, which is a monoamine oxidase (MAO) inhibitor. Which of the following is she restricting from the client's diet? A. Cream cheese B. Fresh fruits C. Aged cheese D. Yeast bread Correct Answer: C Section: (none) Explanation Explanation/Reference: Explanation: (A) Cream cheese does not contain tyramine, which might cause a hypertensive crisis. (B) Fresh fruits do not contain tyramine, which might cause a hypertensive crisis. (C) Aged or matured cheese combined with a monoamine oxidase predisposes the client to a hypertensive crisis. (D) Bread products raised with yeast do not contain tyramine. QUESTION 629 A client suffering from schizophrenia has been taking chlorpromazine (Thorazine) for 6 months. On one of his follow-up visits to the mental health center, the nurse reports to the physician that he has developed tardive dyskinesia. Which of the following symptoms might she have observed in the client to support this conclusion? A. High fever, tachycardia, stupor, renal failure B. Lip smacking, chewing, blinking, lateral jaw movements C. Photosensitivity, orthostatic hypotension, drymouth D. Constipation, blurred vision, drowsiness Correct Answer: B Section: (none) Explanation Explanation/Reference: Explanation: (A) These symptoms are found in clients with neuroleptic malignant syndrome. (B) These symptoms are found in clients with tardive dyskinesia. (C) These are normal side effects found in clients taking antipsychotic medications. (D) These are also normal side effects found in clients taking antipsychotic medications.QUESTION 630 On morning rounds, the nurse found a manic-depressive client who is taking lithium in a confused mental state, vomiting, twitching, and exhibiting a coarse hand tremor. Which one of the following nursing actions is essential at this time? A. Administer her next dosage of lithium, and then call the physician. B. Withhold her lithium, and report her symptoms to the physician. C. Place her on NPO to decrease the excretion of lithium from her body, and call the physician. D. Contact the lab and request a lithium level in 30 minutes, and call the physician. Correct Answer: B Section: (none) Explanation Explanation/Reference: Explanation: (A) The client has lithium toxicity, and the nurse must withhold further dosages. (B) Because of her level of toxicity, further lithium could cause coma and death. The nurse needs further orders from the physician to stabilize the client's lithium level. (C) Ensuring adequate intake of sodium chloride will promote excretion of lithium and will assist in managing the client's lithiumtoxicity. (D) A lithium blood level must be drawn immediately to determine the seriousness of the toxicity and to provide the physician with data for medical orders. QUESTION 631 In acute episodes of mania, lithium is effective in 12 weeks, but it may take up to 4 weeks, or even a few months, to treat symptoms fully. Sometimes an antipsychotic agent is prescribed during the first few days or weeks of an acute episode to manage severe behavioral excitement and acute psychotic symptoms. In addition to the lithium, which one of the following medications might the physician prescribe? A. Diazepam (Valium) B. Haloperidol (Haldol) C. Sertraline (Zoloft) D. Alprazolam (Xanax) Correct Answer: B Section: (none) Explanation Explanation/Reference: Explanation: (A) Diazepam is an antianxietymedication and is not designed to reduce psychotic symptoms. (B) Haloperidol is an antipsychotic medication and may be used until the lithium takes effect. (C) Sertraline is an antidepressant and is used primarily to reduce symptoms of depression. (D) Alprazolam is an antianxiety medication and is not designed to reduce psychotic symptoms.QUESTION 632 The healthcare team determines that an elderly client has had progressive changes in memory over the last 2 years that have interfered with her personal, social, or occupational functioning. Her memory, learning, attention, and judgment have all been affected in some way. These symptoms describe which of the following conditions? A. Dementia B. Parkinsonism C. Delirium D. Mania Correct Answer: A Section: (none) Explanation Explanation/Reference: Explanation: (A) These changes are common characteristics of dementia. (B) Parkinson's disease affects the muscular system. Progressive memory changes are not presenting symptoms. (C) Delirium includes an altered level of consciousness, which is not found in dementia. (D) Mania includes symptoms of hyperactivity, flight of ideas, and delusions of grandeur. QUESTION 633 A husband and wife and their two children, age 9 and age 5, are requesting family therapy. Which of the following strategies is most therapeutic for the nurse to use during the initial interaction with a family? A. Always allow the most vocal person to state the problem first. B. Encourage the mother to speak for the children. C. Interpret immediately what seems to be going on within the family. D. Allow family members to assume the seats as they choose. Correct Answer: D Section: (none) Explanation Explanation/Reference: Explanation: (A) One will always hear what the most vocal person has to say. It is best to start with the quietest family member to encourage that person to express emotions. (B) All family members are encouraged to speak for themselves. (C) In the initial family assessment, only data collection occurs; interpretations are made later. (D)Allowing family members to choose their own seats will assist the nurse in assessing the family system and in determining who feels closer to whom. QUESTION 634 In healthcare settings, nurses must be familiar with primary, secondary, and tertiary levels of care. As a nurse in the community, which of the following interventions might be a primary prevention strategy? A. Crisis intervention with an intoxicated teenager whose mother just committed suicide B. Referring a client who has been on a detoxification unit to a rehabilitation center C. Teaching fifth-grade children the harmful effects of substance abuse D. Counseling a client with post-traumatic stress disorder Correct Answer: C Section: (none) Explanation Explanation/Reference: Explanation: (A) The teenager is already coping ineffectively and requires early detection and treatment, which is secondary prevention. (B) The client must be sent to arehabilitation unit, which requires tertiary prevention. (C) Reducing the incidence of disease through education supports primary prevention. (D) A client with identified symptoms of post-traumatic stress disorder requires intervention by treatment. QUESTION 635 While the nurse is taking a male client's blood pressure, he makes flirtatious remarks to her. The nurse will handle this effectively if she: A. Politely tells the client, "Keep your hands off " B. Ignores the remarks and hopes he will not try it again C. Confronts the remarks but attempts not to reject the client D. Leaves the room in order to compose herself Correct Answer: C Section: (none) Explanation Explanation/Reference: Explanation: (A) This response does not recognize normal feelings of attraction and rejects the client. (B) By ignoring the situation, the nurse has not set limits to discourage other remarks or perhaps more sexually aggressive behavior. (C) By confronting the remarks, she can recognize that his feelings of attraction may be normal but are not appropriate within the context of their nurse-client relationship. (D) Leaving the room does not deal with setting limits for future interactions.QUESTION 636 A client is a victim of domestic violence. She is now receiving assistance at a shelter for battered women. She tells the nurse about the cycle of violence that she has been experiencing in her relationship with her husband of 5 years. In the "tension-building phase," the nurse might expect the client to describe which of the following? A. Promises of gifts that her husband made to her B. Acute battering of the client, characterized by his volatile discharge of tension C. Minor battering incidents, such as the throwing of food or dishes at her D. A period of tenderness between the couple Correct Answer: C Section: (none) Explanation Explanation/Reference: Explanation: (A) This description is characteristic of the "honeymoon" or "respite" phase. (B) This description is characteristic of the "battering" phase. (C) This description is characteristic of the "tension- building" phase prior to the volatile discharge of tension found in the battering phase. (D) This description is characteristic of the "honeymoon" or "respite" phase. QUESTION 637 Which of the following symptoms might the nurse observe in a client with a lithium blood level over 2.0? A. Fine hand tremor, headache, mental dullness B. Vomiting, impaired consciousness, decreased blood pressure C. Polyuria, polydipsia, edema D. Gastric irritation, nausea, diarrhea Correct Answer: B Section: (none) Explanation Explanation/Reference: Explanation: (A) These symptoms are acute, common, and usually harmless central nervous system side effects of lithium. (B) These symptoms of lithium toxicity are usually dose related. (C) These symptoms are acute, common, and usually harmless renal side effects of lithium. (D) These symptoms are acute, common, and usually harmless gastrointestinal side effects of lithium.http://www.gratisexam.com/ QUESTION 638 A psychiatric nurse is providing an orientation to a new staff nurse. She reminds the nurse that psychiatrists often use categories of medications and that it is important that she recall that some categories of medications have synonyms. Another name used to describe minor tranquilizers is which of the following? A. Antipsychotic medications B. Antidepressant medications C. Antianxietymedications D. Antimania medication Correct Answer: C Section: (none) Explanation Explanation/Reference: Explanation: (A) Antipsychotic medications are also known as major tranquilizers. (B) Antidepressants fall into different categories, such as the tricyclics or the MAO inhibitors. (C) Antianxietymedications are also known as minor tranquilizers. (D) Antimania medications are those such as lithium and lithium carbonate (Lithobid). QUESTION 639 The nurse has been caring for a 16-year-old female who recently experienced date rape. After having had crisis intervention and been hospitalized for 2 weeks, the nurse knows that the client is effectively coping with the rape when she tells the nurse: A. "I know it was my fault that it happened, because I shouldn't have been out so late." B. "If I had not worn that sexy dress that night, he wouldn't have raped me." C. "I know my date just had so much passion he couldn't handle me saying `no.' " D. "I know now that it was not my fault, but I want to continue counseling after my discharge." Correct Answer: D Section: (none) ExplanationExplanation/Reference: Explanation: (A) This response does not show any insight; the client falsely assumes that she is responsible for the rape. (B) The client continues to falsely assume responsibility for the rapist's behavior. (C) The client believes falsely that rape is an act of passion, rather than one of violence, control, and domination. (D) The client has insight into the rape; she does not believe it was her fault and shows good judgment in deciding to continue with counseling after discharge. QUESTION 640 A 42-year-old male client has been treated at an alcoholic rehabilitation center for physiological alcohol dependence. The nurse will be able to determine that he is preparing for discharge and is effectively coping with his problem when he shares with her the following information: A. "I know that I will not ever be able to socially drink alcohol again and will need the support of the AA group." B. "I know that I can only drink one or two drinks at social gatherings in the future, but at least I don't have to continue AA." C. "I really wasn't addicted to alcohol when I came here, I just needed some help dealing with my divorce." D. "It really wasn't my fault that I had to come here. If my wife hadn't left, I wouldn't have needed those drinks." Correct Answer: A Section: (none) Explanation Explanation/Reference: Explanation: (A) The client has insight into the severity of his alcohol addiction and has chosen one of the most effective treatment strategies to support him--AlcoholicsAnonymous. (B) The client is still using denial and is not dealing with his alcohol addiction. (C) The client is exhibiting denial about his alcohol addiction and projecting blame on his divorce. (D) The client is projecting blame onto his wife for being in the hospital while still denying his alcohol addiction. QUESTION 641 Degenerative disorders are attributed to many factors. As a nurse assigned to a convalescent home, one must often educate families about how such conditions occur. Which of the following statements might the nurse need to explore when a daughter tries to explain to her mother what caused her degenerative disorder? A. "Some folks believe that aging causes this, Mother." B. "Perhaps, it's the way your parents used those double- bind messages, Mother." C. "I know some people who are having this problem and they were exposed to chemicals at work, Mother." D. "It can be caused by lots of things, toxic agents and even alcohol, Mother." Correct Answer: B Section: (none) ExplanationExplanation/Reference: Explanation: (A) Aging is a factor in the cause of degenerative disorders. (B) Double-bind messages may be found in the histories of families of individuals who develop schizophrenia, but they are not related to degenerative disorders. (C) Chemicals (toxic agents) in work environments are predisposing factors to degenerative disorders. (D) Alcohol causes some degenerative disorders, such as Wernicke's syndrome. QUESTION 642 A family is experiencing changes in their lifestyle in many ways. The invalid grandmother has moved in with them. The couple have a 2-year-old son by their marriage, and the wife has two children by her previous marriage. The older children are in high school. In applying systems theory to this family, it is important for the nurse to remember which of the following principles? A. The parts of a system are only minimally related. B. Dysfunction in one part affects every other part. C. A family system has no boundaries. D. Healthy families are enmeshed. Correct Answer: B Section: (none) Explanation Explanation/Reference: Explanation: (A) The parts of a system are interrelated. (B) Any change in any part of the system affects all other parts. (C) A family system, like any other system, has boundaries. (D) Healthy families are neither enmeshed nor disengaged. QUESTION 643 The nurse is trying to help a mother understand what is happening with her son who has recently been diagnosed with paranoid schizophrenia. At present, he is experiencing hallucinations and delusions of persecution and suffers from poor hygiene. The nurse can best help her understand her son's condition by which of the following statements? A. "Sometimes these symptoms are caused by an overstimulation of a chemical called dopamine in the brain." B. "Has anyone in your family ever had schizophrenia?" C. "If your son has a twin, he probably will eventually develop schizophrenia, too." D. "Some of his symptoms may be a result of his lack of a strong mother-child bonding relationship." Correct Answer: A Section: (none)Explanation Explanation/Reference: Explanation: (A) The most plausible theory to date is that dopamine causes an overstimulation in the brain, which results in the psychotic symptoms. (B) This statement will only create anxiety in the mother, and the genetic theory is only one of the etiological factors. (C) This statement will cause the mother much alarm, and nothing was mentioned about any other child. (D) The motherchild relationship is one of the previous theories examined, but it is not one to be emphasized, thereby causing a lot of anxiety for the mother. QUESTION 644 A male client is experiencing auditory hallucinations. His nurse enters the room and he tells her that his mother is talking to him, and he will take his medicine after she leaves. The nurse looks around the room and sees that she and the client are the only ones in the room. The nurse's most therapeutic response will be: A. "I don't see your mother in the room. Let's talk about how you're feeling." B. "OK, I'll come back later when you're feeling more like taking your medicine." C. "She may be here, but I can't see her." D. "Why don't you finish talking to her, and I'll wait." Correct Answer: A Section: (none) Explanation Explanation/Reference: Explanation: (A) This response uses the principle of reality orientation by the nurse telling the client that he or she does not see anything, but it does recognize his feelings. (B) This response does not make it clear that the nurse does not see anyone else in the room, and the nurse leaves the client alone to continue hallucinating. (C) Thisresponse leaves room for doubt; the nurse is further confusing the client by this statement. (D) Thisresponse reinforces the hallucination and implies that the nurse sees his mother, too. QUESTION 645 A female client with major depression stated that "life is hopeless and not worth living." The nurse should place highest priority on which of the following questions? A. "How has your appetite been recently?" B. "Have you thought about hurting yourself?" C. "How is your relationship with your husband?" D. "How has your depression affected your daily livingactivities?" Correct Answer: B Section: (none)Explanation Explanation/Reference: Explanation: (A) Although eating habits are important to assess, they are less important than suicidal intent. (B) Maintenance of the client's life is the priority; assessment of suicidal intent is imperative. (C) Relationships and support systems are an important part of assessment, but they are less important than suicidal intent. (D) Daily living activities will give additional information about the level of depression, and are less significant than suicidal intent, although this information may give additional information about the actual plan for a suicidal attempt. QUESTION 646 A client presented herself to the mental health center, describing the following symptoms: a weight loss of 20 lb in the past 2 months, difficulty concentrating, repeated absences from work due to "fatigue," and not wanting to get dressed in the morning. She leaves her recorded message on her telephone and has lost interest in answering the phone or doorbell. The nurse's assessment of her behavior would most likely be: A. Deep depression B. Psychotic depression C. Severe anxiety D. Severe depression Correct Answer: D Section: (none) Explanation Explanation/Reference: Explanation: (A) A client in deep depression would have been brought to the mental health center and would not be physically able to seek help for herself. (B) She is not manifesting psychotic symptoms in her behaviors. (C) The client's symptoms are more indicative of depression than anxiety. (D) Although the client was able to bring herself to the mental health center, the extent of her weight loss and the interference of symptoms with activities of daily living indicate that she is severely depressed. QUESTION 647 A 48-year-old male client is hospitalized with mild ascites, bruising, and jaundice. He has a 20- year history of alcohol abuse. The client is diagnosed with cirrhosis. His serum ammonia level is high, indicating hepatic encephalopathy. He has esophageal varices. Which of the following may cause the varices to rupture? A. Lifting heavy objects B. Walking briskly C. Ingestion of barbiturates D. Ingestion of antacidsCorrect Answer: A Section: (none) Explanation Explanation/Reference: Explanation: (A) Lifting heavy objects will increase intrathoracic pressure, thus placing the client at risk for rupturing esophageal varices. (B, C, D) This activity will not cause an increase in intrathoracic pressure. QUESTION 648 Due to his prolonged history of alcohol abuse, an alcoholic client will most likely have deficiencies of which of the following nutrients? A. Vitamin C and zinc B. Folic acid and niacin C. Vitamin A and biotin D. Thiamine and pyroxidine Correct Answer: D Section: (none) Explanation Explanation/Reference: Explanation: (A) Chronic alcoholism can lead to deficiencies of B complex vitamins including thiamine and pyroxidine. (B) Chronic alcoholism can lead to deficiencies of vitamins A, D, K, and B complex. (C) Chronic alcoholism can lead to deficiencies of vitamins A, D, K, and B complex. (D) Vitamins A, D, K, and B require bile salts to be absorbed from the gastrointestinal tract. A damaged liver does not form bile salts. QUESTION 649 The physician of an alcoholic client places him on a low-protein, high-carbohydrate diet. When choosing his menu, the client's best choice from the items below would be: A. Liver and onions, macaroni and cheese, tea with sugar B. Baked chicken, baked potato with bacon bits, milk C. Waffles with butter and honey, orange juice D. Cheese omelette with ham and mushrooms, milk Correct Answer: CSection: (none) Explanation Explanation/Reference: Explanation: (A, B, D) These foods are high in protein, which needs to be restricted. (C) Serum ammonia levels can be decreased by restricting dietary protein intake. Waffles, honey, and orange juice are high in carbohydrate and low or completely lacking in protein. Butter, a concentrated fat, will provide extra calories. QUESTION 650 A chronic alcoholic client's condition deteriorates, and he begins to exhibit signs of hepatic coma. Which of the following is an early sign of impending hepatic coma? A. Hiccups B. Anorexia C. Mental confusion D. Fetor hepaticus Correct Answer: C Section: (none) Explanation Explanation/Reference: Explanation: (A) Hiccups are not a sign of impending hepatic coma. (B) Anorexia is not a sign of impending hepatic coma. (C) One of the earliest symptoms of hepatic coma is mental confusion. Asterixis, a flapping tremor of the hand, may also be seen. (D) This sign is associated with the later stages of hepatic coma. Fetor hepaticus, acharacteristic odor on the breath that smells like acetone, may sometimes be noted when the liver fails. QUESTION 651 The physician of a client diagnosed with alcoholism orders neomycin 0.5 g q6h to prevent hepatic coma. Neomycin decreases serum ammonia levels by: A. Decreasing nitrogen-forming bacteria in the intestines B. Acidifying colon contents by causing ammonia retention in the colon C. Decreasing the uptake of vitamin D, thereby drawing more water into the colon D. Irritating the bowel and promoting evacuation of stool Correct Answer: A Section: (none) ExplanationExplanation/Reference: Explanation: (A) Neomycin interferes with protein synthesis in the bacterial cell, causing bacterial death. Neomycin reduces the growth of the ammonia-producing bacteria in the intestines and is used for the treatment of hepatic coma. (B) This choice describes the action of lactulose, another drug commonly used to decrease systemic ammonia levels. (C) Neomycin's action doesnotdecrease uptake of vitamin D to reduce serum ammonia levels. (D) Bowel irritation with diarrhea is more likely to occur with administration of lactulose rather than of neomycin. Besides, diarrhea is a side effect of a drug, not the action of the drug. QUESTION 652 A 26-year-old male client is brought by his wife to the emergency department (ED) unconscious. Blood is drawn for a stat blood count (CBC), fasting blood sugar level, and electrolytes. An indwelling urinary catheter is inserted. He has a history of type 1 diabetes (insulindependent diabetes mellitus [IDDM]). A diagnosis ofketoacidosis is made. Stat lab values reveal a blood sugar level of 520 mg/dL. Which of the following should the nurse expect to administer in the ER? A. D50W by IV push B. NPH insulin SC C. Regular insulin by IV infusion D. Sweetened grape juice by mouth Correct Answer: C Section: (none) Explanation Explanation/Reference: Explanation: (A) This action would further increase the client's blood sugar. (B) NPH insulin is an intermediate-acting insulin, with an average of 46 hours before onset of action. The client needs insulin that will act immediately. During a ketoacidotic state, the client is dehydrated, so any insulin administered SC will be poorly absorbed. (C) Regular insulin is the fastest acting-insulin; when given IV, it will immediately act to decrease blood sugar. Regular insulin is given to decrease blood glucose levels by promoting metabolism of glucose, inhibiting lipolysis and formation of ketone bodies. (D) This action would further increase the client's blood sugar. QUESTION 653 A client with IDDM is given IV insulin for a blood glucose level of 520 mg/dL. Life-threatening complications may occur initially, so the nurse will monitor him closely for serum: A. Chloride level of 99 mEq/L B. Sodium level of 136 mEq/L C. Potassium level of 3.1 mEq/L D. Potassium level of 6.3 mEq/L Correct Answer: DSection: (none) Explanation Explanation/Reference: Explanation: (A) The chloride level is within acceptable limits. (B) The sodium level is within acceptable limits. (C) This value indicates hypokalemia, rather than the hyperkalemia that occurs during diabetic ketoacidosis. (D) When diabetic ketoacidosis exists, intracellular dehydration occurs and potassium leaves the cells and enters the vascular system, thus increasing the serum level beyond an acceptable range. When insulin and fluids are administered, cell walls are repaired and potassium is transported back into the cells. Normal serum potassium levels range from 3.55.0 mEq/L. QUESTION 654 An IDDM client's condition stabilizes. He begins to receive a daily injection of NPH insulin at 6:30 AM. The nurse can most likely expect a hypoglycemic reaction to occur that same day at: A. 8:30 AM10:30 AM B. 2:30 PM4:30 PM C. 7:30 PM9:30 PM D. 10:30 PM11:30 PM Correct Answer: B Section: (none) Explanation Explanation/Reference: Explanation: (A) This time describes the time of onset of NPH insulin's action, rather than its peak effect. (B) NPH insulin, an intermediateacting insulin, usually begins to lower serum glucose levels about 2 hours after administration. The action of NPH insulin peaks 814 hours after administration. It has a 2030 hour duration. (C) The time stated is not the time of peak action for NPH insulin administered at 6:30 AM. (D) The time stated is not the time of peak action for NPH insulin administered at 6:30 AM. QUESTION 655 After several days, an IDDM client's serum glucose stabilizes, and the registered nurse continues client teaching in preparation for his discharge. The nurse helps him plan an American Diabetes Association diet and explains how foods can be substituted on the exchange list. He can substitute 1 oz of poultry for: A. One frankfurter B. One ounce of ham C. Two slices of bacon D. One-fourth cup dry cottage cheeseCorrect Answer: D Section: (none) Explanation Explanation/Reference: Explanation: (A) A frankfurter is a high-fat meat on the diabetic exchange list. (B) Ham is a medium-fat meat on the diabetic exchange list, unless it is a center-cut slice. (C) One strip of bacon equals onefatexchange rather than ameatexchange. Dietary substitutions should occur within exchange lists and not between exchange lists. (D) Diabetic meat-exchange lists are categorized into leanmeat foods, medium-fat meats, and high-fat meats. Cottage cheese (dry, 2% butterfat), one- fourth cup, cansubstitute for one lean-meat exchange. QUESTION 656 When discussing the relationship between exercise and insulin requirements, a 26-year-old client with IDDM should be instructed that: A. When exercise is increased, insulin needs are increased B. When exercise is increased, insulin needs are decreased C. When exercise is increased, there is no change in insulin needs D. When exercise is decreased, insulin needs are decreased Correct Answer: B Section: (none) Explanation Explanation/Reference: Explanation: (A) If the client's insulin is increased when activity level is increased, hypoglycemia may result. (B) Exercise decreased the blood sugar by promoting uptake of glucose by the muscles. Consequently, less insulin is needed to metabolize ingested carbohydrates. Extra food may be required for extra activity. (C) This statement directly contradicts the correct answer and is inaccurate. (D) When exercise is decreased, the client's insulin dose does not need to be altered unless the blood sugar becomes unstable. QUESTION 657 A 64-year-old client is admitted to the hospital with benign prostatic hypertrophy (BPH). He has a history of adult-onset diabetes and hypertension and is scheduled to undergo a resection of the prostate. When recording his health history, the nurse asks about his chief complaint. The most serious symptom that may accompany BPH is: A. Acute urinary retention B. Hesitancy in starting urinationC. Increased frequency of urination D. Decreased force of the urinary stream Correct Answer: A Section: (none) Explanation Explanation/Reference: Explanation: (A) Acute urinary retention requires urgent medical attention. If measures such as a warm tub bath or warm tea do not occur after 6 hours, the client should go to the ED for catheterization. (B, C, D) This choice is a symptom of BPH, but it is not serious or life threatening. QUESTION 658 A client undergoes a transurethral resection, prostate (TURP). He returns from surgery with a three-way continuous Foley irrigation of normal saline in progress. The purpose of this bladder irrigation is to prevent: A. Bladder spasms B. Clot formation C. Scrotal edema D. Prostatic infection Correct Answer: B Section: (none) Explanation Explanation/Reference: Explanation: (A) The purpose of bladder irrigation is not to prevent bladder spasms, but to drain the bladder and decrease clot formation and obstruction. (B) A three-way system of bladder irrigation will cleanse the bladder and prevent formation of blood clots. A catheter obstructed by clots or other debris will cause prostatic distention and hemorrhage. (C) Scrotal edema seldom occurs after TURP. Bladder irrigation will not prevent this complication. (D) Prostatic infection seldom occurs after TURP. Bladder irrigation will not prevent this complication. QUESTION 659 Following TURP, which of the following instructions would be appropriate to prevent or alleviate anxiety concerning the client's sexual functioning? A. "You may resume sexual intercourse in 2 weeks." B. "Manymen experience impotence following TURP."C. "A transurethral resection does not usually cause impotence." D. "Check with your doctor about resuming sexual activity." Correct Answer: C Section: (none) Explanation Explanation/Reference: Explanation: (A) Sexual activity should be delayed until cleared by the client's physician. (B) Although manymen experience retrograde ejaculation following prostate surgery, potency is seldom affected. (C) Although the client may experience retrograde ejaculation, it will not limit his ability to engage in sexual intercourse. (D) Althoughthe client should obtain clearance from his physician before resuming sexual activity, this statement does not give the client any information or reassurance about future sexual activity or potency that could decrease his anxiety. QUESTION 660 A client is having an amniocentesis. Prior to the procedure, an ultrasound is performed. In preparing the client, the nurse explains the reason for a sonogram in this situation to be: A. Determination of multiple gestations B. Determination of gross anomalies C. Determination of placental location D. Determination of fetal age Correct Answer: C Section: (none) Explanation Explanation/Reference: Explanation: (A) Sonography can be used to determine the presence of multiple gestation. In this question, the sonogram is used as a preparatory step for a specific invasive procedure. (B) Sonography can be used to determine the presence of gross anomalies. In this question, the sonogram is used as a preparatory step for a specific invasive procedure. (C) Prior to amniocentesis, the abdomen is scanned by ultrasound to locate the placenta, thus reducing the possibility of penetrating it with the spinal needle used to obtain amniotic fluid. (D) Sonography can be used to determine fetal age. In this question, the sonogram is used as a preparatory step for aspecific invasive procedure. QUESTION 661 A client is resting comfortably after delivering her first child. When assessing her pulse rate, the nurse would recognize the following finding to be typical:A. Thready pulse B. Irregular pulse C. Tachycardia D. Bradycardia Correct Answer: D Section: (none) Explanation Explanation/Reference: Explanation: (A) A thready pulse is indicative of hypotension and excessive blood loss and is often rapid. (B) Pulse irregularities or dysrhythmias do not occur in the normalpostpartal woman. (C) Tachycardia occurs less frequently than bradycardia and is related to increased blood loss or prolonged difficult labor and/or birth. (D) Puerperal bradycardia with rates of 5070 bpm commonly occurs during the first 610 days of the postpartal period. It may be related to decreased cardiac strain, decreased blood volume, contraction of the uterus, and increased stroke volume. QUESTION 662 A client is being admitted to the labor and delivery unit. She has had previous admissions for "false labor." Which clinical manifestation would be most indicative of true labor? A. Increased bloody show B. Progressive dilatation and effacement of the cervix C. Uterine contractions D. Decreased discomfort with ambulation Correct Answer: B Section: (none) Explanation Explanation/Reference: Explanation: (A) Bloody show is considered a sign of imminent labor, which usually begins in 2448 hours. An increase in bloody show is an indication that the cervix is changing. (B) Contractions of true labor produce progressive cervical effacement and dilatation. (C) Contractions of false labor may mimic those of true labor. However, the contractions of false labor do not produce progressive effacement and dilatation of the cervix. (D) In true labor, the discomfort is not relieved by ambulation; walking may intensify the discomfort. QUESTION 663 In evaluating the effectiveness of magnesium sulfate (MgSO4), which of the following might indicate that the client was developing MgSO4 toxicity?A. A 31 patellar tendon reflex B. Respirations of 12 breaths/min C. Urine output of 40 mL/hr D. A 21 proteinuria value Correct Answer: B Section: (none) Explanation Explanation/Reference: Explanation: (A) Diminished (not accentuated) patellar tendon reflex is a sign of developing MgSO4 toxicity. A value of 21 is considered a normal tendon reflex; 3+ is considered brisker than normal. (B) MgSO4 is a central nervous system (CNS) depressant. It also relaxes smooth muscle. If the respiratory rate is <16 bpm magnesium toxicity may be developing. (C) Urine output of 40mL/hr is enough to allow elimination of toxic levels of magnesium. Urinary output of <100 mL in a 4- hour period may result in toxic levels of magnesium. (D) Presence of protein in the urine is a symptom of pregnancy-induced hypertension (PIH), a clinical syndrome for whichmagnesium sulfate is frequently used in medical management. Protein in the urine is not induced by magnesium sulfate intake. QUESTION 664 A client has had amniocentesis. One of the tests performed on the amniotic fluid is a lecithin/sphingomyelin (L/S) ratio. The results show a ratio of 1:1. This is indicative of: A. Lung immaturity B. Intrauterine growth retardation (IUGR) C. Intrauterine infection D. Neural tube defect Correct Answer: A Section: (none) Explanation Explanation/Reference: Explanation: (A) At about 3032 weeks' gestation, the amounts of the surfactants, lecithin, and sphingomyelin become equal. As the fetal lungs mature, the concentration of lecithin begins to exceed that of sphingomyelin. At 35 weeks, the L/S ratio is 2:1. Respiratory distress syndrome is unlikely if birth occurs at this time. (B) IUGR is associated with compromised uteroplacental perfusion or with viral infections, chromosomal disorders, congenital malformations, and maternal malnutrition. IUGR is not specifically assessed by analysis of the L/S ratio. (C) Analysis of the L/S ratio is not an assessment used to confirm intrauterine infection. (D) Elevated levels of _- fetoprotein in maternal serum or in amniotic fluid have been found to reflect open neural tube defects, such as spina bifida and anencephaly.QUESTION 665 On admission to the postpartal unit, the nurse's assessment identifies the client's fundus to be soft, 2 fingerbreadths above the umbilicus, and deviated to the right. This is most likely an indication of: A. Normal involution B. A full bladder C. An infection pain D. A hemorrhage Correct Answer: B Section: (none) Explanation Explanation/Reference: Explanation: (A) Immediately after expulsion of the placenta, the fundus should be in the midline and remain firm. (B) A boggy displaced uterus in the immediate postpartum period is a sign of urinary distention. Because uterine ligaments are stretched, a full bladder can displace the uterus. (C) Symptoms of infection may include unusual uterine discomfort, temperature elevation, and foul- smelling lochia. The stem of this question does not address any of these factors. (D) While excessive bleeding is associated with a soft, boggy uterus, the stem of this question includes displacement of the uterus, which is more commonly associated with bladder distention. QUESTION 666 A client's membranes have just ruptured spontaneously. Which of the following nursing actions should take priority? A. Assess quantity of fluid. B. Assess color and odor of fluid. C. Document on fetal monitor strip and chart. D. Assess fetal heart rate (FHR). Correct Answer: D Section: (none) Explanation Explanation/Reference: Explanation: (A) Assessing the quantity of amniotic fluid is important as an indication of maternal fetal well- being, but it does not take priority over assessment of FHR. (B) Greenish-brown discoloration of amniotic fluid indicates presence of meconium. Foul odor may indicate presence of infection. Both of these are important assessment data, but they do not take priority over possible lifethreatening compression of the umbilical cord. (C) Documentation is important, but it does not takepriority over the possible life-threatening compression of the umbilical cord. (D) If changes in the FHR are noted, the nurse should check for umbilical cord prolapse. This intervention has priority over the other actions. The danger of a prolapsed cord is increased once membranes have ruptured, especially if the presenting part of the fetus does not fit firmly against the cervix. QUESTION 667 The nurse and prenatal client discuss the effects of cigarette smoking on pregnancy. It would be correct for the nurse to explain that with cigarette smoking there is increased risk that the baby will have: A. A low birth weight B. A birth defect C. Anemia D. Nicotine withdrawal Correct Answer: A Section: (none) Explanation Explanation/Reference: Explanation: (A) Women who smoke during pregnancy are at increased risk for miscarriage, preterm labor, and IUGR in the fetus. (B) Although smoking produces harmful effects on the maternal vascular system and the developing fetus, it has not been directly linked to fetal anomalies. (C) Smoking during pregnancy has not been directly linked to anemia in the fetus. (D) Smoking during pregnancy has not been linked to nicotine withdrawal symptoms in the newborn. QUESTION 668 Which of the following blood values would require further nursing action in a newborn who is 4 hours old? A. Hemoglobin 17.2 g/dL B. Platelets 250,000/mm3 C. Serum glucose 30 mg/dL D. White blood cells 18,000/mm3 Correct Answer: C Section: (none) Explanation Explanation/Reference: Explanation: (A) The normal range for hemoglobin in the newborn is 1719 g/dL; 17.2 g/dL is within normal limits. (B) A normal value range for platelets in the newborn is150,000400,000 mm3; 250,000/mm3 is within normal range. (C) A serum glucose of 30 mg/dL in the first 72 hours of life is indicative of hypoglycemia and warrants further intervention. (D) On the day of birth, a white blood cell count of 18,00040,000/mm3 is normal in the newborn. QUESTION 669 A client is admitted to the psychiatric unit after lavage and stabilization in the emergency room for an overdose of antidepressants. This is her third attempt in 2 years. The highest priority intervention at this time is to: A. Assess level of consciousness B. Assess suicide potential C. Observe for sedation and hypotension D. Orient to her room and unit rules Correct Answer: B Section: (none) Explanation Explanation/Reference: Explanation: (A) The client was stabilized in the ED and consequently would not be sent to the psychiatric unit if comatose. (B) Suicide assessment is always appropriate forclients with a history of previous attempts or depression, because either of these factors places the client at high risk. (C) The admission assessment should include observation for sedation and hypotension, but this is not in priority over suicide assessment. (D) Orientation to room and unit rules is of low priority at this time. QUESTION 670 A client's record from the ED indicates that she overdosed on phenelzine sulfate (Nardil), a monoamine oxidase (MAO) inhibitor. Which diet would be the most appropriate at this time? A. High carbohydrate, low cholesterol B. High protein, high carbohydrate C. 1 g sodium D. Tyramine-free Correct Answer: D Section: (none) Explanation Explanation/Reference: Explanation: (A) There are no data to support the need for increased carbohydrates or decreased cholesterol in the diet. (B) There is no data to support the need for increasedprotein or increased carbohydrates in the diet. (C) There is no assessment or laboratory data indicating that sodium should be restricted in the diet. (D) Tyramine is an amino acid activated by MAO in the liver and intestinal wall. It is released as proteins are hydrolyzed through aging, pickling, smoking, or spoilage of foods. When MAO is inhibited, tyramine levels rise, stimulating the adrenergic system to release large amounts of norepinephrine, which can produce a hypertensive crisis. QUESTION 671 Two weeks after a client's admission for depression, the physician orders a consult for electroconvulsive therapy (ECT). Which of the following conditions, if present, would be a contraindication for ECT? A. Brain tumor or other space-occupying lesion B. History of mitral valve prolapse C. Surgically repaired herniated lumbar disk D. History of frequent urinary tract infections Correct Answer: A Section: (none) Explanation Explanation/Reference: Explanation: (A) A contraindication for ECT is a space-occupying lesion such as a brain tumor. During ECT, intracranial pressure increases. Therefore, ECT would not be prescribed for a client whose intracranial pressure is already elevated. (B) Any cardiac dysrhythmias or complications that arise during ECT are usually attributed to the IV anesthetics used, not to preexisting cardiac structural conditions. (C) Musculoskeletal injuries during ECT are extremely rare because of the IV use of centrally acting muscle relaxers. (D) A history of any kind of infection would not contraindicate the use of ECT. In fact, concurrent treatment of infections with ECT is not uncommon. QUESTION 672 A client is medically cleared for ECT and is tentatively scheduled for six treatments over a 2- week period. Her husband asks, "Isn't that a lot?" The nurse's best response is: A. "Yes, that does seem like a lot." B. "You'll have to talk to the doctor about that. The physician knows what's best for the client." C. "Six to 10 treatments are common. Are you concerned about permanent effects?" D. "Don't worry. Some clients have lots more than that." Correct Answer: C Section: (none) ExplanationExplanation/Reference: Explanation: (A) This response indicates that the nurse is unsure of herself and not knowledgeable about ECT. It also reinforces the husband's fears. (B) This response is "passing the buck" unnecessarily. The information needed to appropriately answer the husband's question is well within the nurse's knowledge base. (C) The most common range for affective disorders is 610 treatments. This response confirms and reinforces the physician's plan for treatment. It also opens communicationwith the husband to identify underlying fears and knowledge deficits. (D) This response offers false reassurance and dismisses the husband's underlying concerns about his wife. QUESTION 673 A husband asks if he can visit with his wife on her ECT treatment days and what to expect after the initial treatment. The nurse's best response is: A. "You'll have to get permission from the physician to visit. Clients are pretty sick after the first treatment." B. "Visitors are not allowed. We will telephone you to inform you of her progress." C. "There'sreally no need to stay with her. She's going to sleep for several hours after the treatment." D. "Yes, you may visit. She may experience temporary drowsiness, confusion, or memory loss after each treatment." Correct Answer: D Section: (none) Explanation Explanation/Reference: Explanation: (A) It is within the nurse's realm of practice to grant visiting privileges according to hospital policy. ECT treatments do not make clients sick. (B) Visitors are allowed and encouraged, particularly family members. (C) Clients are usually awake within 1 hour posttreatment. Drowsiness wanes as the anesthetic wears off. (D) A family member is encouraged to stay with the client after return to the unit. The nurse has used an opportunity to do family teaching and allay fears by explaining temporary side effects of the treatment. QUESTION 674 A client is placed in five-point restraints after exhibiting sudden violence after illegal drug use, and haloperidol (Haldol) 5 mg IM is administered. After 1 hour, his behavior is more subdued, but he tells the nurse, "The devil followed me into this room, I see him standing in the corner with a big knife. When you leave the room, he's going to cut out my heart." The nurse's best response is: A. "I know you're feeling frightened right now, but I want you to know that I don't see anyone in the corner." B. "You'll probably see strange things for a while until the PCP wears off." C. "Try to sleep. When you wake up, the devil will be gone." D. "You're probably feeling guilty because you used illegal drugs tonight." Correct Answer: ASection: (none) Explanation Explanation/Reference: Explanation: (A) The nurse is the client's link to reality. This response validates the authenticity of the client's experience by casting doubt on his belief and reinforcing reality. (B) Although this statement may be literally correct, it is nontherapeutic because it lacks validation. (C) This response encourages the client to attempt to do something that may be impossible at this time, offers false reassurance, and reinforces delusional content. (D) The nurse is making an incorrect assumption about the client's feelings by offering a nontherapeutic interpretation of the motivation for the client's actions. QUESTION 675 A violent client remains in restraints for several hours. Which of the following interventions is most appropriate while he is in restraints? A. Give fluids if the client requests them. B. Assess skin integrity and circulation of extremities before applying restraints and as they are removed. C. Measure vital signs at least every 4 hours. D. Release restraints every 2 hours for client to exercise. Correct Answer: D Section: (none) Explanation Explanation/Reference: Explanation: (A) Fluids (nourishment) should be offered at regular intervals whether the client requests (or refuses) them or not. (B) Skin integrity and circulation of the extremities should be checked regularly while the client is restrained, not only before restraints are applied and after they are removed. (C) Vital signs should be checked at least every 2 hours. If the client remains agitated in restraints, vital signs should be monitored even more closely, perhaps every 12 hours. (D) Restraints should be released every 2 hours for exercise, one extremity at a time, to maintain muscle tone, skin and joint integrity, and circulation. QUESTION 676 After 7 hours in restraints and a total of 30-mg haloperidol in divided doses, a client complains of stiffness in his neck and his tongue "pulling to one side." These extrapyramidal symptoms (EPS) will most likely be relieved by the administration of: A. Lorazepam (Ativan) B. Benztropine (Cogentin) C. Thiothixene (Navane) D. Flurazepan (Dalmane)Correct Answer: B Section: (none) Explanation Explanation/Reference: Explanation: (A) Lorazepam is an antianxiety agent that produces muscle relaxation and inhibits cortical and limbic arousal. It has no action in the basal ganglia of the brain. (B) Benztropine acts to reduce EPS by blocking excess CNS cholinergic activity associated with dopamine deficiency in the basal ganglia by displacing acetylcholine at the receptor site. (C) Thiothixene is an antipsychotic known to block dopamine in the limbic system, thereby causing EPS. (D) Flurazepan is a hypnotic that acts inthe limbic system, thalamus, and hypothalamus of the CNS to produce sleep. It has no known action in the vasal ganglia. QUESTION 677 Medication is administered to a client who has been placed in restraints after a sudden violent episode, and his EPSs subside. Restraints can be removed when: A. The physician orders it B. A therapeutic alliance has been established, and violent behavior subsides C. The violent behavior subsides, and the client agrees to behave D. The nurse deems that removal of restraints is necessary Correct Answer: B Section: (none) Explanation Explanation/Reference: Explanation: (A) The physicianmayorder release of restraints, but prior to that, the client must meet criteria for release. (B) While the client is still restrained, but after violent behavior has subsided, a therapeutic bridge is built. This alliance encourages dialogue between nurse and client, allowing the client to determine causative factors, feelings prior to loss of control, and adaptive alternatives to violence. (C) If the client only "agrees to behave" after violent behavior subsides, he has developed no insight into cause and effect of violence or his response to stress. (D)Removal of restraints occurs only when the client meets the criteria for release, not just because the nurse says it is necessary. QUESTION 678 A 16-year-old female client is admitted to the hospital because she collapsed at home while exercising with videotaped workout instructions. Her mother reports that she has been obsessed with losing weight and staying slim since cheerleader try-outs 6 months ago, when she lost out to two of her best friends. The client is 5'4" and weighs 92 lb, which represents a weight loss of 28 lb over the last 4 months. The most important initial intervention on admission is to: A. Obtain an accurate weight B. Search the client's purse for pillsC. Assess vital signs D. Assign her to a room with someone her own age Correct Answer: C Section: (none) Explanation Explanation/Reference: Explanation: (A) On admission, vital signs are the highest priority. Weight is not a vital sign. (B) Belongings are routinely searched on admission to a psychiatric unit, but this search is not a high priority. (C) Vital signs are a high priority when working with selfdestructive clients. (D) Room assignment is of low priority. QUESTION 679 Assessment of a client reveals a 30% loss of preillness weight, lanugo, and cessation of menses for 3 months. Her vital signs are BP 90/50, P 96 bpm, respirations 30, and temperature 97 F. She admits to the nurse that she has induced vomiting 3 times this morning, but she had to continue exercising to lose "just 5 more lb." Her symptoms are consistent with: A. Pregnancy B. Bulimia C. Gastritis D. Anorexia nervosa Correct Answer: D Section: (none) Explanation Explanation/Reference: Explanation: (A) Presenting behaviors collectively are inconsistent with depression. (B) A preillness weight loss of 30%, lanugo, and cessation of menses are inconsistent with bulimia. (C) Symptoms and vital signs do not indicate the presence of infection. (D) All symptoms and vital signs are consistent with anorexia nervosa. QUESTION 680 Blood work reveals the following lab values for a client who has been diagnosed with anorexia nervosa: hemoglobin 9.6 g/dL, hemocrit 27%, potassium 2.7 mEq/L, sodium 126 mEq/L. The greatest danger to her at this time is: A. Hypoglycemia from low-carbohydrate intake B. Possible cardiac dysrhythmias secondary to hypokalemiaC. Dehydration from vomiting D. Anoxia secondary to anemia Correct Answer: B Section: (none) Explanation Explanation/Reference: Explanation: (A) There is no lab data to support hypoglycemia. (B) Hypokalemia, caused by vomiting and decreased dietary intake of potassium, can result in life-threatening dysrhythmias. (C) Evidence of dehydration is not life threatening at this time, although fluid volume deficit does need to be addressed. (D) The client's hemoglobin does not reflect a life threatening value sufficient to render the client anoxic. QUESTION 681 A client suspected of having anorexia nervosa is placed on bed rest with an IV infusion and a high-carbohydrate liquid diet. Within 72 hours, the results of her lab work show a return to normal limits. She is transferred to the psychiatric service for further treatment. A behavior modification plan is initiated. Three days after her transfer, the client tells the nurse, "I haven't exercised in 6 days. I won't be eating lunch today." This statement by her most likely reflects: A. Her lack of internal awareness about the outcome of the behavior B. Increased knowledge about personal exercise plans C. A manipulative technique to trick the nurse into allowing her to miss a meal D. A true desire to stay fit while in the hospital Correct Answer: A Section: (none) Explanation Explanation/Reference: Explanation: (A) Indirect self-destructive behavior such as that seen in anorexia nervosa is characterized by the client's lack of insight and the awareness that the outcome of the dieting, exercising, and weight loss will ultimately result in death if uninterrupted. (B) Although the client is knowledgeable about exercise, knowledge about the balance between nutrition, exercise, and rest is absent. (C) The client's level of denial and lack of awareness disallow this behavior as a manipulative trick. (D) Theclient's illness-maintaining behaviors are inconsistent with fitness. QUESTION 682 A client who has been diagnosed with anorexia nervosa refuses to eat lunch. The most therapeutic response by the nurse to her refusal is: A. "Okay, missing one meal won't hurt."B. "You'll have to eat lunch, or we'll force-feed you." C. "It's not appropriate for you to try to manipulate the staff into granting your wishes." D. "We will not allow you to starve yourself. You may choose to eat voluntarily or be fed." Correct Answer: D Section: (none) Explanation Explanation/Reference: Explanation: (A) This response reinforces the client's maladaptive behavior, thereby contributing to the client's risk. (B) Ultimatums are not therapeutic. (C) This comment invites an argument because it puts the client on the defensive and stabs at her self-esteem, which is already compromised. (D) Setting limits assures the client that staff has genuine concern for her safety and well-being. Giving her an actual choice will give the client an increased sense of control over her life and avoid an argument or power struggle. QUESTION 683 A client who has been diagnosed with anorexia nervosa reluctantly agrees to eat all prescribed meals. The most important intervention in monitoring her dietary compliance would be to: A. Allow her privacy at mealtimes B. Praise her for eating everything C. Observe behavior for 12 hours after meals to prevent vomiting D. Encourage her to eat in moderation, choose foods that she likes, and avoid foods that she dislikes Correct Answer: C Section: (none) Explanation Explanation/Reference: Explanation: (A) Eating alone is not recommended for anorexic clients because they tend to hoard food instead of eating it. (B) The client should be praised for whatever she eats, which is usually a small portion or percentage of what is served. Praise should not be withheld until she eats everything. (C) The client should be observed eyeto- eye for at least 1 hour following meals to prevent discarding food stashed in her clothing at mealtime or engaging in selfinduced vomiting. (D) If offered these choices, the client would choose lowcaloric foods, not a nutritious diet. QUESTION 684 A 2-year-old boy fell out of bed and has a subdural hematoma. When his mother leaves him for the first time, you will expect the child to:A. Be comforted when he is held B. Cry C. Not notice that his mother has left D. Withdraw and become listless Correct Answer: B Section: (none) Explanation Explanation/Reference: Explanation: (A) It will be difficult to comfort a 2 year old with a headache without his mother. (B) This baby probably will cry, which should be prevented because it will increase his intracranial pressure (ICP). Asking the mother to wait until the baby is asleep may help. (C) An awake 2 year old will notice when his mother leaves. (D) An older child may withdraw when feeling afraid, but a 2 year old will probably show more aggressive behavior. QUESTION 685 The doctor has ordered a restricted fluid intake for a 2- year-old child with a head injury. Normal fluid intake for a child of 2 years is: A. 900 mL/24 hr B. 1300 mL/24 hr C. 1600 mL/24 hr D. 2000 mL/24 hr Correct Answer: C Section: (none) Explanation Explanation/Reference: Explanation: (A, B, D) These values are incorrect. Normal intake for a child of 2 years is about 1600 mL in 24 hours. (C) This value is correct. Normal intake for a child of 2 years is about 1600 mL in 24 hours. QUESTION 686 A 2-year-old child with a scalp laceration and subdural hematoma of the temporal area as a result of falling out of bed should be prevented from: A. Crying B. Falling asleep C. Rolling from his back to his tummyD. Sucking his thumb Correct Answer: A Section: (none) Explanation Explanation/Reference: Explanation: (A) A child with a subdural hematoma has increased ICP. Crying may significantly increase this pressure. (B) Adequate sleep is essential, but it is important that the child can be aroused from sleep after head injury. (C) This child is free to roll from his back to his abdomen. (D) Thumb- sucking serves to reduce anxiety and should not be prevented at this time. QUESTION 687 A seventh grader lost consciousness after being hit in the head with a basketball. In the emergency room his vital signs are stable, and he demonstrates no neurologic deficit. He will not be admitted to the hospital. It is most important that you advise his mother to: A. Encourage him to drink plenty of fluids B. Expect him to have nausea with vomiting C. Keep him awake for the next 12 hours D. Wake him up every 12 hours during the night Correct Answer: D Section: (none) Explanation Explanation/Reference: Explanation: (A) Fluid intake should be normal. Fluid intake may be restricted when there is a risk for increased ICP in a hospitalized client. (B) Nausea is possible, but vomiting without nausea is more likely with increased ICP. Neither one should be expected, but the mother should know to notify the physician or hospital if they occur. (C)The child does not need to be kept awake. It is important that he can be aroused from sleep. (D) If the child cannot be awakened from sleep after head injury, it isan indication of serious increase in ICP. The mother should call an ambulance right away. QUESTION 688 A 14-year-old boy fell off his bike while "popping a wheelie" on the dirt trails. He has sustained a head injury with laceration of his scalp over his temporal lobe. If he were to complain of headache during the first 24 hours of his hospitalization, the nurse would: A. Ask the physician to order a sedative B. Have the client describe his headache every 15 minutesC. Increase his fluid intake to 3000 mL/24 hr D. Offer diversionary activities Correct Answer: D Section: (none) Explanation Explanation/Reference: Explanation: (A) CNS depressants are not given for headache due to head injury because they would mask changes in neurological status and because they could further depress the CNS. (B) The client should not be asked to think about his headache every 15 minutes. (C) Fluid intake should be normal or restricted for a client with a head injury. Normal fluid intake for a 14 year old is about 20002400 mL daily. (D) Diversion may help the child to focus on a pleasant activity instead of on his headache. QUESTION 689 An 18-year-old girl is admitted to the hospital with a depressed skull fracture as a result of a car accident. If the nurse were to observe a rising pulse rate and lowering blood pressure, the nurse would suspect that the client: A. Has a sudden and severe increase in intracranial pressure B. Has sustained an internal injury in addition to the head injury C. Is beginning to experience a dangerously high level of anxiety D. Is having intracranial bleeding Correct Answer: B Section: (none) Explanation Explanation/Reference: Explanation: (A) Widening pulse pressure (high systolic and low diastolic) with compensatory slowing of pulse rate are late signs of increasing ICP. (B) Rising pulse rate and lowering blood pressure are indicative of hypovolemia due to hemorrhage. (C) High anxiety, in the absence of hemorrhage, would result in a high pulse rate and a high blood pressure. (D) Intracranial bleeding results in increased ICP. A change in level of consciousness is an early sign of increasing ICP, and vital sign changes are late signs of increasing ICP. QUESTION 690 The nurse is caring for a 2-year-old girl with a subdural hematoma of the temporal area as a result of falling out of bed and notices that she has a runny nose. The nurse should:A. Call the doctor immediately B. Help her to blow her nose carefully C. Test the discharge for sugar D. Turn her to her side Correct Answer: C Section: (none) Explanation Explanation/Reference: Explanation: (A) The nasal discharge could be due to a cold. It is necessary to gather additional assessment data to identify a possible cerebrospinal fluid leak. (B) If the discharge is cerebrospinal fluid, it would not be safe to encourage the girl to blow her nose. (C) Cerebrospinal fluid is positive for sugar; mucus is not. (D) Turning her to her side will have no effect on her "runny nose." It is necessary to gather further assessment data. QUESTION 691 A 2-year-old boy is in the hospital outpatient department for observation after falling out of his crib and hitting his head. The nurse calls the physician to report: A. Evidence of perineal irritation B. Pulse fell from 102 to 96 C. Pulse increased from 96 to 102 D. Temperature rose to 102_F rectally Correct Answer: D Section: (none) Explanation Explanation/Reference: Explanation: (A) Perineal irritation needs to be addressed, but it is probably not necessary to call the physician. (B) This fall in pulse rate remains within normal limits and is probably insignificant. It is important to monitor for continued change. (C) This rise in pulse rate is probably not significant, but it is important to monitor for continued change. (D) This temperature is above normal limits and needs medical investigation. It may or may not be related to the head injury. QUESTION 692 The nurse is caring for a 3-month-old girl with meningitis. She has a positive Kernig's sign. The nurse expects her to react to discomfort if she: A. Dorsiflexes her ankleB. Flexes her spine C. Plantiflexes her wrist D. Turns her head to the side Correct Answer: B Section: (none) Explanation Explanation/Reference: Explanation: (A) Discomfort with ankle dorsiflexion is not expected with meningitis. (B) Spinal flexion, flexing the neck or the hips with legs extended, causes discomfort if themeninges are irritated. (C) Discomfort with wrist flexion is not expected with meningitis. (D) Rotating the cervical spine may cause discomfort with meningitis, but pain with flexion is more indicative of meningeal irritation. QUESTION 693 The nurse is admitting an infant with bacterial meningitis and is prepared to manage the following possible effects of meningitis: A. Constipation B. Hypothermia C. Seizure D. Sunken fontanelles Correct Answer: C Section: (none) Explanation Explanation/Reference: Explanation: (A) Constipation may occur if the child is dehydrated, but it is not directly associated with meningitis. (B) It is more likely the child will have fever. (C) Seizure is often the initial sign of meningitis in children and could become frequent. (D) It is more likely the child will have bulging fontanelles. QUESTION 694 The nurse is caring for a 6-week-old girl with meningitis. To help her develop a sense of trust, the nurse should: A. Give her a small soft blanket to hold B. Give her good perineal care after each diaper change C. Leave the door open to her room D. Pick her up when she criesCorrect Answer: D Section: (none) Explanation Explanation/Reference: Explanation: (A) A soft blanket may be comforting, but it is not directed toward developing a sense of trust. (B) Good perineal care is important, but it is not directed toward developing a sense of trust. (C) An infant with meningitis needs frequent attention, but leaving the door open does not foster trust. (D) Consistently picking her up when she cries will help the child feel trust in her caregivers. QUESTION 695 A 6-year-old girl is visiting the outpatient clinic because she has a fever and a rash. The doctor diagnoses chickenpox. Her mother asks the nurse how many baby aspirins her daughter can have for fever. The nurse should: A. Advise the mother not to give her aspirin B. Ask if the client is allergic to aspirin before giving further information C. Assess the function of the client's cranial nerve VIII D. Check the aspirin bottle label to determine milligrams per tablet Correct Answer: A Section: (none) Explanation Explanation/Reference: Explanation: (A) Aspirin taken during a viral infection has been implicated as a predisposing factor to Reye's syndrome in children and adolescents. Children and adolescents should not be given aspirin. (B) Allergy to aspirin is not related to Reye's syndrome. (C) Tinnitus, caused by damage to the acoustic nerve, occurs with aspirin toxicity, but this is not related to Reye's syndrome. (D) A 6- year-old child should not be given any baby aspirin. QUESTION 696 A 2-year-old child isrecovering from surgery. Considering growth and development according to Erikson, the nurse identifies which of the following play activities as most appropriate? A. Assembling a puzzle with large pieces B. Being taken for a wheelchair ride C. Listening to a story about the Muppets D. Watching Sesame Street on televisionCorrect Answer: A Section: (none) Explanation Explanation/Reference: Explanation: (A) A 2-year-old child is in the stage of autonomy, according to Erikson. Assembling a puzzle with large pieces enables her to "do it herself." (B) A wheelchair ride would probably be fun, but it is not directed toward helping the child to achieve autonomy. (C) Listening to a story may be fun and educational, but it is not directed toward helping the child to achieve autonomy. (D) Watching television may be a favorite activity, but it does not foster autonomy. QUESTION 697 A 14-year-old boy has a head injury with laceration of his scalp over his ear. The nurse should call the physician to report: A. Blood pressure increase from 100/80 to 115/85 after lunch B. Headache that is unresponsive to acetaminophen (Tylenol) C. Pulse rate ranges between 68 bpm and 76 bpm D. Temperature rise to 102_F rectally Correct Answer: D Section: (none) Explanation Explanation/Reference: Explanation: (A) This change in blood pressure may not be significant and does not indicate a widening pulse pressure, a late sign of increased ICP. It is important to continue to monitor for change in blood pressure. (B) Acetaminophen may be ineffective in relieving headache after head injury. Stronger analgesics are contraindicated because they mask neurological signs and may depress the CNS. (C) Pulse rates between 68 bpm and 76 bpm are within normal limits for a 14-year-old child. It isimportant to monitor for a consistent drop in pulse rate, which is a late sign of increasing ICP. (D) An elevated temperature is abnormal and requires further assessment and medical intervention. The temperature may be unrelated to the head injury, but CNS infection is serious and difficult to control. Topic 7, Questions Set G QUESTION 698 The nurse is teaching a child's parents how to protect the child from lead poisoning. The nurse knows that a common source of lead poisoning in children is: A. Dandelion leaves B. Pencils C. Old paintD. Stuffing from toy animals Correct Answer: C Section: (none) Explanation Explanation/Reference: Explanation: (A) Dandelion leaves are not a source of lead. (B) Pencils are not a source of lead poisoning. (C) Chewing on objects painted before 1960 is a common source of lead poisoning in children. Gasoline is another source. (D) Stuffed animals are not a source of lead. QUESTION 699 A woman diagnosed with multiple sclerosis is disturbed with diplopia. The nurse will teach herto: A. Limit activities which require focusing (close vision) B. Take more frequent naps C. Use artificial tears D. Wear a patch over one eye Correct Answer: D Section: (none) Explanation Explanation/Reference: Explanation: (A) Limiting activities requiring close vision will not alleviate the discomfort of double vision. (B) Frequent naps may be comforting, but they will not prevent double vision. (C) Artificial tears are necessary in the absence of a corneal reflex, but they have no effect on diplopia. (D) An eye patch over either eye will eliminate theeffects of double vision during the time the eye patch is worn. An eye patch is safe for a person with an intact corneal reflex. QUESTION 700 In planning daily care for a client with multiple sclerosis, the nurse would take into consideration that multiple sclerosis: A. Becomes progressively debilitating without remission B. Has unpredictable remissions and exacerbations C. Is rapidly fatal D. Responds quickly to antimicrobial therapyCorrect Answer: B Section: (none) Explanation Explanation/Reference: Explanation: (A) Multiple sclerosis eventually becomes debilitating, but it is characterized by remission of symptoms. (B) Remissions and exacerbations are unpredictable with multiple sclerosis. The client experiences progressive dysfunction after each exacerbation episode. (C) Multiple sclerosis is usually slowly progressive. (D) Multiple sclerosis is an autoimmune disease. Antimicrobial therapy has no effect on its course. QUESTION 701 A client with a head injury asks why he cannot have something for his headache. The nurse's response is based on the understanding that analgesics could: A. Counteract the effects of antibiotics B. Elevate the blood pressure C. Mask symptoms of increasing intracranial pressure D. Stimulate the central nervous system Correct Answer: C Section: (none) Explanation Explanation/Reference: Explanation: (A) Analgesic medication does not counteract the effects of antibiotics. (B) Analgesic medication may lower blood pressure elevated due to anxiety. (C) Analgesic medication, especially CNS depressants, is not given if there is danger of increasing ICP, because neurological changes may not be apparent. Also, further depression of the CNS is contraindicated. (D) Analgesics do not stimulate the CNS. QUESTION 702 To prevent transmission of bacterial meningitis, the nurse would instruct an infected baby's mother to: A. Avoid touching the baby while in the room. B. Stay outside of the baby's room. C. Wear a gown and gloves and wash her hands before and after leaving the room. D. Wear a mask while in the room. Correct Answer: CSection: (none) Explanation Explanation/Reference: Explanation: (A) The mother should be allowed and encouraged to touch her baby. (B) With care, transmission can be prevented. There is no need for the mother to stay outside the room. (C) Everyone entering the baby's room should take appropriate measures to prevent transmission of pathogens. (D) Wearing a mask will not protect against transmission of pathogens. QUESTION 703 A client is scheduled for a magnetic resonance imaging (MRI) to locate a cerebral lesion. It is important for the nurse to find out if he has a(n): A. Allergy to seafood B. History of seizures C. Movable metal implant D. Pin or screw in any bone Correct Answer: C Section: (none) Explanation Explanation/Reference: Explanation: (A) Iodine is not used as a contrast medium for MRI. It is important to inquire about allergy to seafood if the client is to have an arteriogram or enhanced computer tomography. (B) MRI is safe if seizures are under control. It is more important to inquire about movable metal implants. (C) Clients with movable metal implants such as shrapnel or aneurysm clips or clients with permanent pacemakers or implanted pumps can be traumatized during an MRI. (D) Nonmovable metal prostheses or hardware will not cause trauma during an MRI. QUESTION 704 A child is admitted with severe headache, fever, vomiting, photophobia, drowsiness, and stiff neck associated with viral meningitis. She will be more comfortable if the nurse: A. Dims the lights in her room B. Encourages her to breathe slowly and deeply C. Offers sips of warm liquids D. Places a large, soft pillow under her head Correct Answer: ASection: (none) Explanation Explanation/Reference: Explanation: (A) The discomfort of photophobia is alleviated by dimming the lights. (B) Helping the child to breathe slowly and deeply may help to reduce anxiety, but it will not alleviate other discomforts of viral meningitis. (C) It is important to maintain fluid balance, but sips of warm liquids do not alleviate the discomforts of meningitis. (D) A large, soft pillow under her head causing neck flexion is likely to increase her discomfort owing to stretching of the meninges. QUESTION 705 A 30-year-old client has been admitted to the psychiatric service with the diagnosis of schizophrenia. He tells the nurse that when the woman he had been dating broke up with him, the CIA had replaced her with an identical twin. The client is experiencing: A. Grandiose delusions B. Paranoid delusions C. Auditory hallucinations D. Visual hallucinations Correct Answer: B Section: (none) Explanation Explanation/Reference: Explanation: (A) There are no indications that the client's thoughtsreflect special powers or talents characteristic of grandiosity. (B) The client's thought content is fixed, false, persecutory, and suspicious in nature, which is characteristic of paranoid delusions. (C, D) The client is not demonstrating a sensory experience. QUESTION 706 A client tells the nurse that he has been hearing voices that tell him to kill his girlfriend because she is a spy. He further states that he is having difficulty not obeying the voices because, if he does not, his house will be burned down. The highest priority nursing diagnosis for him at this time is: A. Sensory-perceptual alteration: auditory command hallucinations B. Alteration in thought processes: paranoid delusions C. Potential for violence directed at others D. Impaired verbal communication: loose associations Correct Answer: C Section: (none)Explanation Explanation/Reference: Explanation: (A) Although the client is having command hallucinations, this is second in priority to real or potential violence, which can be a threat to life itself. (B) Although the client is experiencing delusions, this is also a lower priority than his potential or actual loss of control. (C) Whether real or potential, violence directed at self or others is always high priority. (D) There is no evidence of loosening of associations. QUESTION 707 A client reports to the nurse that the voices are practically nonstop and that he needs to leave the hospital immediately to find his girlfriend and kill her. The best verbal response to the client by the nurse at this time is: A. "I understand that the voices are real to you, but I want you to know I don't hear them. They are a symptom of your illness." B. "Just don't pay attention to the voices. They'll go away after some medication." C. "You can't leave here. This unit is locked and the doctor has not ordered your discharge." D. "We will have to put you in seclusion and restraints for a while. You could hurt someone with thoughts like that." Correct Answer: A Section: (none) Explanation Explanation/Reference: Explanation: (A) This response validates the client's experience and presents reality to him. (B) This nontherapeutic response minimizes and dismisses the client's verbalizedexperience. (C) This response can be interpreted by a paranoid client as a threat, thereby increasing the client's potential for violence and loss of control. (D) This response is also threatening. The client's behavior does not call for restraints because he has not lost control or hurt anyone. If seclusion or restraints were indicated, the nurse should never confront the client alone. QUESTION 708 The physician orders haloperidol 5 mg IM stat for a client and tells the nurse that the dose can be repeated in 12 hours if needed. The most likely rationale for this order is: A. The client will settle down more quickly if he thinks the staff is medicating him B. The medication will sedate the client until the physician arrives C. Haloperidol is a minor tranquilizer and will not oversedate the client D. Rapid neuroleptization isthe most effective approach to care for the violent or potentially violent client Correct Answer: DSection: (none) Explanation Explanation/Reference: Explanation: (A) If the client could think logically, he would not be paranoid. In fact, he is probably suspicious of the staff, too. Newly admitted clients frequently experience high levels of anxiety, which can contribute to delusions. (B) The goal of pharmacological intervention is to calm the client and assist with reality-based thinking, not tosedate him. (C) Haloperidol is a neuroleptic and antipsychotic drug, not a minor tranquilizer. (D) Haloperidol is a high-potency neuroleptic and first-line choice forrapid neuroleptization, with low potential for sedation. QUESTION 709 Two hours after the second injection of haloperidol, a client complains to the nurse of a stiff neck and inability to sit still. He is experiencing symptoms consistent with: A. Parkinsonism and dystonia B. Dystonia and akathisia C. Akathisia and parkinsonism D. Neuroleptic malignant syndrome Correct Answer: B Section: (none) Explanation Explanation/Reference: Explanation: (A) Stiff neck is consistent with a dystonic reaction, but the client has no symptoms of drooling, shuffling gait, or pill-rolling movements characteristic of parkinsonism. (B) Stiff neck is consistent with a dystonic reaction, and inability to sit still with varying degrees of psychomotor agitation is characteristic of akathisia. (C) The client has symptoms of dystonia but not of parkinsonism. (D) The client has none of the characteristic symptoms of neuroleptic malignant syndrome: hyperpyrexia, generalized muscle rigidity, mutism, obtundation, agitation, sweating, increased blood pressure and pulse. QUESTION 710 The physician orders medication for a client's unpleasant side effects from the haloperidol. The most appropriate drug at this time is: A. Lorazepam B. Triazolam (Halcion) C. Benztropine D. Thiothixene Correct Answer: CSection: (none) Explanation Explanation/Reference: Explanation: (A) Lorazepam is a benzodiazepine, or antianxiety agent, that potentiates the effects of _- aminobutyric acid in the CNS, which is not the CNS neurotransmitter EPS. (B) Triazolam is a benzodiazepine sedative-hypnotic whose action is mediated in the limbic, thalamic, and hypothalamic levels of the CNS by - aminobutyric acid. (C) Benztropine is an anticholinergic agent, and the drug of choice for blocking CNS synaptic response, which causes EPS. (D) Thiothixene is an antipsychotic and neuroleptic drug that blocks dopamine neurotransmission at the CNS synapses, thereby causing EPS. QUESTION 711 A psychiatric client has been stabilized and is to be discharged. The nurse will recognize client insight and behavioral change by which of the following client statements? A. "When I get home, I will need to take my medicines and call my therapist if I have any side effects or begin to hear voices." B. "If I have any side effects from my medicines, I will take an extra dose of Cogentin." C. "When I get home, I should be able to taper myself off the Haldol because the voices are gone now." D. "As soon as I leave here, I'm throwing awaymymedicines. I never thought I needed them anyway." Correct Answer: A Section: (none) Explanation Explanation/Reference: Explanation: (A) The client verbalizes that he is responsible for compliance and keeping the treatment team member informed of progress. This behavior puts him at the lowest risk for relapse. (B) Noncompliance is a major cause of relapse. This statement reflects lack of responsibility for his own health maintenance. (C) This statement reflects lack of insight into the importance of compliance. (D) This statement reflects no insight into his illness or his responsibility in health maintenance. QUESTION 712 The nurse is developing a plan of care for a client with an electrolyte imbalance and identifies a nursing diagnosis of decreased physical mobility. Which alteration is most likely the etiology? A. Hypernatremia B. Hypocalcemia C. Hypokalemia D. HypomagnesemiaCorrect Answer: C Section: (none) Explanation Explanation/Reference: Explanation: (A) A deficit in sodium concentration results in muscular weakness and lethargy. (B) Muscle fatigue and hypotonia are caused by hypercalcemia. (C) Muscle weakness and fatigue are classic signs of hypokalemia. (D) Hypermagnesemia can cause muscle weakness, paralysis, and coma. QUESTION 713 The nurse is assessing breath sounds in a bronchovesicular client. She should expect that: A. Inspiration islonger than expiration B. Breath sounds are high pitched C. Breath sounds are slightly muffled D. Inspiration and expiration are equal Correct Answer: D Section: (none) Explanation Explanation/Reference: Explanation: (A) Inspiration is normally longer in vesicular areas. (B) Highpitched sounds are normal in bronchial area. (C) Muffled sounds are considered abnormal. (D) Inspiration and expiration are equal normally in this area, and sounds are medium pitched. QUESTION 714 Discharge teaching for the client who has a total gastrectomy should include which of the following? A. Need for the client to increase fluid intake to 3000 mL/day B. Follow-up visits every 3 weeks for the first 6 months C. B12 injections needed for the rest of the client's life D. Need to eat three full meals with plenty of fiber per day Correct Answer: C Section: (none) ExplanationExplanation/Reference: Explanation: (A) There will be no need to increase fluid intake excessively, because dumping syndrome could present a problem. (B) Followup visits every 3 weeks are not a standard recommendation. Follow-up visits will be highly individualized. (C) With removal of the stomach, intrinsic factor will no longer be produced. Intrinsic factor is necessary for vitamin B12 absorption. Parenteral injections of B12 will be needed on a monthly basis for the rest of the person's life. (D) Smaller, more frequent meals, rather than large, bulky meals, are recommended to prevent problems with dumping syndrome. QUESTION 715 Which of the following findings would necessitate discontinuing an IV potassium infusion in an adult with ketoacidosis? A. Urine output 22 mL/hr for 2 hours B. Serum potassium level of 3.7 C. Small T wave of ECG D. Serum glucose level of 180 Correct Answer: A Section: (none) Explanation Explanation/Reference: Explanation: (A) Adequate renal flow of 30 mL/hr is a necessity with potassium infusions because potassium is excreted renally. (B) Because potassium level will decrease during correction of diabetic ketoacidosis, potassium will be infused even if plasma levels of potassium are normal. (C) A small T wave is normal and desired on the electrocardiogram. A tall, peaked T-wave could indicate overinfusion of potassium and hyperkalemia. (D) Glucose levels of <200 are desirable. QUESTION 716 A client is receiving IV morphine 2 days after colorectal surgery. Which of the following observations indicate that he may be becoming drug dependent? A. The client requests pain medicine every 4 hours. B. He is asleep 30 minutes after receiving the IV morphine. C. He asks for pain medication although his blood pressure and pulse rate are normal. D. He is euphoric for about an hour after each injection. Correct Answer: D Section: (none) Explanation Explanation/Reference:Explanation: (A) Frequent requests for pain medication do not necessarily indicate drug dependence after complex surgeries such as colorectal surgery. (B) Sleeping after receiving IV morphine is not an unexpected effect because the pain isrelieved. (C) A person may be in pain even with normal vital signs. (D) A subtle sign of drug dependency is the tendency for the person to appear more euphoric than relieved of pain. QUESTION 717 The nurse assesses a postoperative mastectomy client and notes that breath sounds are diminished in both posterior bases. The nurse's action should be to: A. Encourage coughing and deep breathing each hour B. Obtain arterial blood gases C. Increase O2 from 23 L/min D. Remove the postoperative dressing to check for bleeding Correct Answer: A Section: (none) Explanation Explanation/Reference: Explanation: (A) Decreased or absent breath sounds are frequently indicators of postoperative atelectasis. (B) Arterial blood gases are not indicated because there is no other information indicating impendingdanger. (C) Increasing O2 rate is not indicated without additional information. (D) Removing the dressing is not indicated without additional information. QUESTION 718 Which of the following should the nurse anticipate receiving as an as-needed order for a postoperative carotid endarterectomy client? A. Nifedipine 10 mg SL for B/P 140/90 B. Furosemide 20 mg/PO for decreased urine output C. Magnesium salicylate to decrease inflammation D. Nitroglycerin gr 1/150 for chest pain Correct Answer: A Section: (none) Explanation Explanation/Reference: Explanation:(A) It is important to maintain a normal to slightly lower pressure to prevent the graft from blowing and excessive pressure to surgical vascular areas. (B, C, D) None of these drugs isrelated to managing the problem at hand. Also, none of the problems for which these drugs would be indicated is expected with this type of surgery, except if there is a prior history. QUESTION 719 Three hours postoperatively, a 27-year-old client complains of right leg pain after knee reduction. The first action by the nurse will be to: A. Assess vital signs B. Elevate the extremity C. Perform a lower extremity neurovascular check D. Remind the client that he has a client-controlled analgesic pump, and reinstruct him on its use Correct Answer: C Section: (none) Explanation Explanation/Reference: Explanation: (A) Vital signs may be altered if there is acute pain or complications related to bleeding or swelling, but they should not be assessed before checking the affected extremity. (B) The extremity will be elevated if ordered by the doctor. (C) Assessment of the postoperative area is important to determine if bleeding, swelling, ordecreased circulation is occurring. (D) Reinforcement of teaching on use of the client-controlled analgesic pump is important, but not the first action. QUESTION 720 Goal setting for a client with Meniere's disease should include which of the following? A. Frequent ambulation B. Prevention of a fall injury C. Consumption of three meals per day D. Prevention of infection Correct Answer: B Section: (none) Explanation Explanation/Reference: Explanation: (A) Although not contraindicated, initially ambulation may be difficult because of vertigo and is recommended only with assistance. (B) Vertigo resulting in balance problems is one of the most common manifestations of Meniere's disease. (C) Adequate nutrition is important, but the emphasis in Meniere's disease is not thenumber of meals per day but a decrease in intake of sodium. (D) Infection is not an anticipated problem. QUESTION 721 Which of the following physician's orders would the nurse question on a client with chronic arterial insufficiency? A. Neurovascular checks every 2 hours B. Elevate legs on pillows C. Arteriogram in the morning D. No smoking Correct Answer: B Section: (none) Explanation Explanation/Reference: Explanation: (A) Neurovascular checks are a routine part of assessment with clients having this diagnosis. (B) Elevation of the legs is contraindicated because it reduces blood flow to areas already compromised. (C) Arteriogram is a routine diagnostic order. (D) Smoking is highly correlated with this disorder. QUESTION 722 A client is admitted to the hospital with a diagnosis of aplastic anemia and placed on isolation. The nurse notices a family member entering the room without applying the appropriate apparel. The nurse will approach the family member using the following information as a basis for discussion: A. The risks of exposure of the visitor to infectious organisms is great. B. Hospital regulations mandate that everyone in the facility adhere to appropriate codes. C. The client is at extreme risk of acquiring infections. D. Adherence to the guidelines are the latest Centers for Disease Control and Prevention recommendations on use of protective apparel. Correct Answer: C Section: (none) Explanation Explanation/Reference: Explanation: (A) Although clients with a compromised immune system may acquire infections, the primary emphasis is on protecting the client. (B, D) Most people are aware ofthe guidelines once they see posted signs, so quoting regulations is not likely to result in consistent adherence to regulations. (C) Clients with aplastic anemia have white cell counts of 2000 or lower, making them more vulnerable to infections from others.QUESTION 723 The nurse enters the room of a client on which a "do not resuscitate" order has been written and discovers that she is not breathing. Once the husband realizes what has occurred he yells, "please save her!" The nurse's action would be: A. Call the physician and inform him that the client has expired. B. Remind the husband that the physician wrote an order not to resuscitate. C. Discuss with the husband that these orders are written only on clients who are not likely to recover with resuscitative efforts. D. Call a code and proceed with cardiopulmonary resuscitation. Correct Answer: D Section: (none) Explanation Explanation/Reference: Explanation: (A, B, C) The last request from the husband overrides the decision not to initiate resuscitation efforts. (D) The nurse should begin cardiopulmonary resuscitation unless a living will and durable power of attorney are in force. In the meantime, the nurse should talk with the husband and notify the doctor. QUESTION 724 The nurse is in the hallway and one of the visitors faints. The nurse should: A. Sit the victim up and lightly slap his face B. Elevate the victim's legs C. Apply a cool cloth to the victim's neck and forehead until he recovers D. Sit the victim up and place the head between the knees Correct Answer: B Section: (none) Explanation Explanation/Reference: Explanation: (A) Sitting the client up defeats the goal of re-establishing cerebral blood flow. (B) Elevating the legs anatomically redirects blood flow to the cerebral area. (C) Thisstrategy is a nice general comfort measure after the victim has regained consciousness. (D) This strategy is not as effective a strategy in helping the client to regain consciousness as elevating the legs. QUESTION 725 The nurse is teaching a client how to perform monthly testicular self-examination (TSE) and states that it is best to perform the procedure right after showering. This statement is made by the nurse based on the knowledge that:A. The client is more likely to remember to perform the TSE when in the nude B. When the scrotum is exposed to cool temperatures, the testicles become large and bulky C. The scrotum will be softer and more relaxed after a warm shower, making the testicles easier to palpate D. The examination will be less painful at this time Correct Answer: C Section: (none) Explanation Explanation/Reference: Explanation: (A) Nudity is not a trigger for reminding males to perform TSE. (B) Testicles become more firm when exposed to cool temperatures, but not large and bulky. (C) The testicles will be lower and more easily palpated with warmer temperatures. A protective mechanism of the body to protect sperm production is for the scrotum to pull closer to the body when exposed to cooler temperatures. (D) The examination should not be painful. QUESTION 726 The nurse discovers that a 78-year-old client who received hydralazine (Apresoline) 20 mg 45 minutes ago has a blood pressure of 70/40 mm Hg. The client has been on this dose of the medication for 3 years. Which of the following data is most likely significant in relation to the cause of the low blood pressure? A. Pedal pulses 11 (weak) B. Twenty-four-hour intake 1000 mL/day for past 2 days C. Serum potassium 3.3 D. Pulse rate 150 bpm Correct Answer: B Section: (none) Explanation Explanation/Reference: Explanation: (A, D) Decreased pulse volume and increased pulse rate are signs of an acute hypotensive episode. (B) Inadequate fluid volume when taking vasodilators canresult in a drop in blood pressure when vasodilation starts to physiologically occur as an action of the drug. (C) A potassium level of 3.3 would not be associated with a significant drop in blood pressure. QUESTION 727 A client admitted with a diagnosis of possible myocardial infarction is admitted to the unit from the emergency room. The nurse's first action when admitting the client will be to:A. Obtain vital signs B. Connect the client to the cardiac monitor C. Ask the client if he is still having chest pain D. Complete the history profile Correct Answer: B Section: (none) Explanation Explanation/Reference: Explanation: (A) Obtaining vital signs is important after connecting the client to the monitor because vital signs should be stable before the client is discharged from the emergency room. (B) All are important, but the first priority is to monitor the client'srhythm. (C) If the client is in severe pain, pain medication should be given after connecting him to the monitor and obtaining vital signs. (D) Completion of the history profile is the least important of the nursing actions. QUESTION 728 The nurse is interviewing a client with a diagnosis of possible abdominal aortic aneurysm. Which of the following statements will be reflected in the client's chief complaint? A. "I've been having a dull pain at the upper left shoulder." B. "My legs have been numb for three months." C. "I've only been urinating three times a day lately." D. "I don't remember anything in particular, I just haven't felt well." Correct Answer: D Section: (none) Explanation Explanation/Reference: Explanation: (A, B, C) These complaints are not specific signs and symptoms associated with abdominal aortic aneurysm. If symptoms are present, the aneurysm is expanding or rupture is imminent. (D) Many clients may experience no symptoms. The only symptom may be a pulsation noted in the abdomen in the reclining position. QUESTION 729 The nurse is collecting a nutritional history on a 28- year-old female client with iron-deficiency anemia and learns that the client likes to eat white chalk. When implementing a teaching plan, the nurse should explain that this practice:A. Will bind calcium and therefore interfere with its metabolism B. Will cause more premenstrual cramping C. Interferes with iron absorption because the iron precipitates as an insoluble substance D. Causes competition at iron-receptor sites between iron and vitamin B1 Correct Answer: C Section: (none) Explanation Explanation/Reference: Explanation: (A) Eating chalk is not related to calcium and its absorption. (B) Poor nutritional habits may result in increased discomfort during premenstrual days, but this is not a primary reason for the client to stop eating chalk. Premenstrual discomfort has not been mentioned. (C) Iron is rendered insoluble and is excreted through the gastrointestinal tract. (D) There is no competition between the two nutrients. QUESTION 730 Which of the following lab data is representative of a client with aplastic anemia? A. Hemoglobin 9.2, hematocrit 27, red blood cells 3.2 million B. White blood cells 4000, erythrocytes 2.5 million, thrombocytes 100,000 C. White blood cells 3000, hematocrit 27, red blood cells 2.8 million D. Red blood cells 1 million, white blood cells 1500, thrombocytes 16,000 Correct Answer: D Section: (none) Explanation Explanation/Reference: Explanation: (A, B, C) Although all of the lab data are abnormal and although these values are decreased in aplastic anemia, the disorder is defined by severe deficits in red cell, white cell, and platelet counts. (D) Aplastic anemia is typically defined in terms of abnormalities of red blood cell count, usually <1 million, white cell count <2,000, and thrombocytes <20,000. QUESTION 731 A 20-year-old male client is being treated for protein deficiency. If he likes all of the following foods, which one would the nurse recommend to increase in the diet? A. CantaloupeB. Rice C. Chicken D. Green beans Correct Answer: C Section: (none) Explanation Explanation/Reference: Explanation: (A) Cantaloupe is a good source of carbohydrates, vitamin C, and vitamin A. (B) Rice contains about 4 g of protein per 200 g. (C) Chicken contains 35 g protein per breast. Chicken is a rich source of vitamin B6 (pyridoxine), which is needed for adequate protein synthesis. As protein intake increases, vitamin B6 intake must also be increased. Vitamin B6 is a coenzyme in amino acid metabolism. (D) Green beans only contain 2 g of protein per cup. QUESTION 732 A client states to his nurse that "I was told by the doctor not to take one of my drugs because it seems to have caused decreasing blood cells." Based on this information, which drug might the nurse expect to be discontinued? A. Prednisone B. Timolol maleate (Blocadren) C. Garamycin (Gentamicin) D. Phenytoin (Dilantin) Correct Answer: D Section: (none) Explanation Explanation/Reference: Explanation: (A) Prednisone is not linked with hematological side effects. (B) Timolol, a -adrenergic blocker is metabolized by the liver. It has not been linked to blood dyscrasia. (C) Gentamicin is ototoxic and nephrotoxic. (D) Phenytoin usage has been linked to blood dyscrasias such as aplastic anemia. The drug most commonly linked to aplastic anemia is chloramphenicol (Chlormycetin). QUESTION 733 Forty-eight hours after a thyroidectomy, a female client complains of numbness and tingling of the toes and fingers. The nurse notes upper arm and facial twitching. The nurse needs to: A. Report the findings to the physicianB. Assist the client to do range of motion exercises C. Check the client's potassium level D. Administer the as-needed dose of phenytoin (Dilantin) Correct Answer: A Section: (none) Explanation Explanation/Reference: Explanation: (A) Muscular hyperactivity and parasthesias may indicate hypocalcemic tetany and require immediate administration of calcium gluconate. Tetany can occur if the parathyroid glands were erroneously excised during surgery. (B) Range of motion exercises are not appropriate topresenting symptoms. (C) These characteristics are not usual signs of potassium imbalance, but of calcium imbalance. (D) Phenytoin is indicated for seizure activity mainly of neurological origin. QUESTION 734 The nurse is admitting a client with folic acid deficiency anemia. Which of the following questions is most important for the nurse to ask the client? A. "Do you take aspirin on a regular basis?" B. "Do you drink alcohol on a regular basis?" C. "Do you eat red meat?" D. "Have your stools been normal?" Correct Answer: B Section: (none) Explanation Explanation/Reference: Explanation: (A) Aspirin does not affect folic acid absorption. (B) Folic acid deficiency is strongly associated with alcohol abuse. (C) Because folic acid is a coenzyme for single carbon transfer purines, calves liver or other purines are the meat sources. (D) Folic acid does not affect stool character. QUESTION 735 An 18-month-old child has been playing in the garage. His mother brings him to a nurse's home complaining of his mouth being sore. His lips and mouth are soapy and white, with small ulcerated areas beginning to form. The child begins to vomit. His pulse is rapid and weak. The nurse suspects that the child has: A. Inhaled gasoline fumes B. Ingested a caustic alkaliC. Eaten construction chalk D. Lead poisoning Correct Answer: B Section: (none) Explanation Explanation/Reference: Explanation: (A, C, D) These agents would not cause ulcerations on mouthand lips. (B) Strong alkali or acids will cause burns and ulcerationson the mucous membranes. QUESTION 736 In discussing the plan of care for a child with chronic nephrosis with the mother, the nurse identifies that the purpose of weighing the child is to: A. Measure adequacy of nutritional management B. Check the accuracy of the fluid intake record C. Impress the child with the importance of eating well D. Determine changes in the amount of edema Correct Answer: D Section: (none) Explanation Explanation/Reference: Explanation: (A) Weighing a child with nephrosis is to assess for edema, not nutrition. (B, C) This is not the purpose for weighing the child. (D) Weight and measurement are the primary ways of evaluating edema and fluid shifts. QUESTION 737 The parents of a 9-year-old child with acute lymphocytic leukemia expressed concern about his alopecia from cranial irradiation. The nurse explains that: A. Alopecia is an unavoidable side effect. B. There are several wig makers for children. C. Most children select a favorite hat to protect their heads. D. His hair will grow back in a few months. Correct Answer: D Section: (none)Explanation Explanation/Reference: Explanation: (A) Alopecia has occurred, and knowing it is a side effect does not address their concern. (B) Although true, it does not give them hope for the future. (C) Although true, it does not provide them with information of the temporary nature of the situation. (D) Knowing the hair will grow back provides comfort that the alopecia is temporary. QUESTION 738 Loss of appetite for a child with leukemia is a major recurrent problem. The plan of care should be designed to: A. Reinforce attempts to eat B. Help the child gain weight C. Increase his appetite D. Make mealtimes pleasant Correct Answer: A Section: (none) Explanation Explanation/Reference: Explanation: (A) Ignoring refusals to eat and rewarding eating attempts are the most successful means of increasing intake. (B) This goal is not specific enough or related to the loss of appetite. (C) This goal is not possible at this time based on his illness. (D) This goal is helpful, but alone will not address his loss of appetite. QUESTION 739 The mother of a child taking phenytoin will need to plan appropriate mouth care and gingival stimulation. When tooth-brushing is contraindicated, the next most effective cleansing and gingival stimulation technique would be: A. Using a water pik B. Rinsing with water C. Rinsing with hydrogen peroxide D. Rinsing with baking soda Correct Answer: A Section: (none) ExplanationExplanation/Reference: Explanation: (A) This technique provides effective rinsing and gingival stimulation. (B) This technique does not provide gingival stimulation. (C) This technique provides effective rinsing but not gingival stimulation. Using peroxide is not pleasant for the child. (D) This technique provides effective rinsing but not gingival stimulation. QUESTION 740 When planning care for a 9-year-old client, the nurse uses which of the most effective means of helping siblings cope with their feelings about a brother who is terminally ill? A. Open discussion and understanding B. Play-acting out feelings in different roles C. Storytelling D. Drawing pictures Correct Answer: B Section: (none) Explanation Explanation/Reference: Explanation: (A) When dealing with grief, siblings are usuallymost comfortable initially with open discussion. (B) Assuming different roles allows children to act out their feelings without fear of reprisals and to gain insight and control. (C) This method may be helpful, but having the child take an active part through role playing is more effective. (D) This technique may be helpful, but being an active participant through role playing is more effective. QUESTION 741 During the active phase of rheumatic fever, the nurse teaches parents of a child with acute rheumatic fever to assist in minimizing joint pain and promoting healing by: A. Putting all joints through full range-of-motion twice daily B. Massaging the joints briskly with lotion or liniment after bath C. Immobilizing the jointsin functional position using splints, rolls, and pillows D. Applying warm water bottle or heating pads over involved joints Correct Answer: C Section: (none) Explanation Explanation/Reference: Explanation:(A) Any movement of the joint causes severe pain. (B) Touching or moving the joint causes severe pain. (C) Immobilization in a functional position allows the joint to rest and heal. (D) Pressure from the warm water bottle or pads can cause severe pain or burning of the skin. QUESTION 742 The initial focus when providing nursing care for a child with rheumatic fever during the acute phase of the illness should be to: A. Maintain contact with her parents B. Provide for physical and psychological rest C. Provide a nutritious diet D. Maintain her interest in school Correct Answer: B Section: (none) Explanation Explanation/Reference: Explanation: (A) This goal is helpful, but rest is essential during the acute phase. (B) Rest is essential for healing to occur and for pain to be relieved. (C) This goal is important, but rest is essential. (D) This goal should be part of the plan of care, but it is not the priority during the acute phase. QUESTION 743 During discharge planning, parents of a child with rheumatic fever should be able to identify which of the following as toxic symptoms of sodium salicylate? A. Tinnitus and nausea B. Dermatitis and blurred vision C. Unconsciousness and acetone odor of the breath D. Chills and an elevation of temperature Correct Answer: A Section: (none) Explanation Explanation/Reference: Explanation: (A) These are toxic symptoms of sodium salicylate. (B, C, D) These are not symptoms associated with sodium salicylate. QUESTION 744Parents of a child with rheumatic fever express concern that she will always be arthritic. The nurse discusses their concerns and tells them the joint pain usually: A. Subsides in<3 weeks B. Is relieved by aspirin C. Is responsive to ibuprofen (Motrin) D. Subsides in 36 days Correct Answer: A Section: (none) Explanation Explanation/Reference: Explanation: (A) Joints usually remain inflamed and tender until the disease runs its course in<3 weeks. (B) Thisresponse does not answer the question of whether she will always be arthritic. (C) This response does not answer the question asked. (D) The disease takes<3 weeks to run its course. QUESTION 745 In caring at home for a child who just ingested a caustic alkali, the nurse would immediately tell the mother to: A. Give vinegar, lemon juice, or orange juice B. Phone the doctor C. Take the child to the emergency room D. Induce vomiting Correct Answer: A Section: (none) Explanation Explanation/Reference: Explanation: (A) The immediate action is to neutralize the action of the chemical before further damage takes place. (B) This action should be done after neutralizing the chemical. (C) This action should be done after neutralizing the chemical. (D) Never induce vomiting with a strong alkali or acid. Additional damage will be done when the child vomits the chemical. QUESTION 746 The most important goal in the care plan for a child who was hospitalized with an accidental overdose would be to: A. Determine child's activity patternB. Reduce mother's sense of guilt C. Instruct parents in use of ipecac D. Teach parents appropriate safety precautions Correct Answer: D Section: (none) Explanation Explanation/Reference: Explanation: (A) This goal is not the most important. (B) There is always some guilt when an accident occurs; however, the priority is to be sure future accidents are prevented. (C) Ipecac is not used for caustic alkali and acid ingestions. (D) Determining the parent's knowledge about safety hazards and teaching appropriate preventive measures are likely to prevent recurrence of accidents. QUESTION 747 A 4 year old has an imaginary playmate, which concerns the mother. The nurse's best response would be: A. "I understand your concern and will assist you with a referral." B. "Try not to worry because you will just upset your child." C. "Just ignore the behavior and it should disappear by age 8." D. "This is appropriate behavior for a preschooler and should not be a concern." Correct Answer: D Section: (none) Explanation Explanation/Reference: Explanation: (A) This is normal for a preschooler, and a referral is not appropriate. (B) Telling a parent not to worry is unhelpful. This response does not address the mother's concern. (C) This response is incorrect. The behavior is normal and will usually disappear by the time the child enters school. (D) This behavior is normal development for a preschooler. QUESTION 748 Parents should be taught not to prop the bottle when feeding their infants. In addition to the risk of choking, it puts the infant at risk for: A. Otitis media B. AsthmaC. Conjunctivitis D. Tonsillitis Correct Answer: A Section: (none) Explanation Explanation/Reference: Explanation: (A) Because the eustachian tube is short and straight in the infant, formula that pools in the back of the throat attacks bacteria which can enter the middle ear and cause an infection. (B) Asthma is not associated with propping the bottle. (C) Conjunctivitis is an eye infection and not associated with propping the bottle. (D) Tonsillitis is usually a result of pharyngitis and not propping the bottle. QUESTION 749 When assessing a female child for Turner's syndrome, the nurse observes for which of the following symptoms? A. Tall stature B. Amenorrhea C. Secondary sex characteristics D. Gynecomastia Correct Answer: B Section: (none) Explanation Explanation/Reference: Explanation: (A) This syndrome is caused by absence of one of the X chromosomes. These children are short in stature. (B) Amenorrhea is a symptom of Turner's syndrome, which appears at puberty. (C) Sexual infantilism is characteristic of this syndrome. (D) Gynecomastia is a symptom in Klinefelter's syndrome. QUESTION 750 The mother of a client is apprehensive about taking home her 2 year old who was diagnosed with asthma after being admitted to the emergency room with difficulty breathing and cyanosis. She asks the nurse what symptoms she should look for so that this problem will not happen again. The nurse instructs her to watch for the following early symptoms: A. Fever, runnynose, and hyperactivity B. Changes in breathing pattern, moodiness, fatigue, and edema of eyesC. Fatigue, dark circles under the eyes, changes in breathing pattern, glassy eyes, and moodiness D. Fever, cough, paleness, and wheezing Correct Answer: C Section: (none) Explanation Explanation/Reference: Explanation: (A) The child with asthma may not have fever unless there is an underlying infection. (B) Edema of the eyes will not be present because the child with asthma is more likely to have dehydration related to excessive water loss during the work of breathing. (C) All of these symptoms indicate decreased oxygenation and are early symptoms of asthma. (D) Coughing and wheezing are not early signs of difficulty. QUESTION 751 A 3-year-old female client is brought into the pediatric clinic because she limps. She has not been to the clinic since she was 9 months old. The nurse practitioner describes the limp as a "Trendelenburg gait." This gait is characteristic of: A. Scoliosis B. Dislocated hip C. Fractured femur D. Fractured pelvis Correct Answer: B Section: (none) Explanation Explanation/Reference: Explanation: (A, C, D) A Trendelenburg gait is not characteristic of any of these disorders. (B) The downward slant of one hip is a positive sign of dislocation in the weight- bearing hip. If one hip is dislocated, the child walks with a characteristic limp known as the Trendelenburg gait. QUESTION 752 A child has a nursing diagnosis of fluid volume excess related to compromised regulatory mechanisms. Which of the following nursing interventions is the most accurate measure to include in his care? A. Weigh the child twice daily on the same scale. B. Monitor intake and output.C. Check urine specific gravity of each voiding. D. Observe for edema. Correct Answer: A Section: (none) Explanation Explanation/Reference: Explanation: (A) Although all of these interventions are important aspects of care, weight is the most sensitive indicator of fluid balance. (B) Although monitoring intake and output is important, weight is a more accurate indicator of fluid status. (C) Urine specific gravity does not necessarily indicatefluid volume excess. (D) Edema may not be apparent, yet the client may have fluid volume excess. QUESTION 753 The pediatric nurse charts that the parents of a 4-yearold child are very anxious. Which observation would indicate to the nurse unhealthy coping by these parents: http://www.gratisexam.com/ A. Discussing their needs with the nursing staff B. Discussing their needs with other family members C. Seeking support from their minister D. Refusing to participate in the child's care Correct Answer: D Section: (none) Explanation Explanation/Reference: Explanation: (A, B, C) These methods are healthy ways of dealing with anxiety. (D) Participation minimizes feelings of helplessness and powerlessness. It is important that parents have accurate information and that they seek support from sources available to them.QUESTION 754 A 9-week-old female infant has a diagnosis of bilateral cleft lip and cleft palate. She has been admitted to the pediatric unit after surgical repair of the cleft lip. Which of the following nursing interventions would be appropriate during the first 24 hours? A. Position on side or abdomen. B. Maintain elbow restraints in place unless she is being directly supervised. C. Clean suture line every shift. D. Offer pacifier when she cries. Correct Answer: B Section: (none) Explanation Explanation/Reference: Explanation: (A) Placing the infant on her abdomen may allow for injury to the suture line. (B) Elbow restraints prevent the infant from touching the suture line and yet leaves hands free. (C) The suture line is cleaned as often as every hour to prevent crusting and scarring. (D) Sucking of a bottle or pacifier places pressure on the suture line and may delay healing and cause scarring. QUESTION 755 A 6-month-old infant who was diagnosed at 4 weeks of age with a ventricular septal defect, was admitted today with a diagnosis of failure to thrive. His mother stated that he had not been eating well for the past month. A cardiac catheterization reveals congestive heart failure. All of the following nursing diagnoses are appropriate. Which nursing diagnosis should have priority? A. Altered nutrition: less than body requirementsrelated to inability to take in adequate calories B. Altered growth and development related to decreased intake of food C. Activity intolerance related to imbalance between oxygen supply and demand D. Decreased cardiac output related to ineffective pumping action of the heart Correct Answer: D Section: (none) Explanation Explanation/Reference: Explanation: (A) Altered nutrition occurs owing to the fatigue from decreased cardiac output associated with congestive heart failure. (B) The decreased intake occurs due to fatigue from the altered cardiac output. (C) Fatigue occurs due to the decreased cardiac output. (D) The ineffective action of the myocardium leads to inadequate O2 to the tissues, which produces activity intolerance, altered nutrition, and altered growth and development.QUESTION 756 A 9-year-old child was in the garage with his father, who was repairing a lawnmower. Some gasoline ignited and caused an explosion. His father was killed, and the child has splitthickness and full-thickness burns over 40% of his upper body, face, neck, and arms. All of the following nursing diagnoses are included on his care plan. Which of these nursing diagnoses should have top priority during the first 2448 hours postburn? A. Pain related to tissue damage from burns B. Potential for infection related to contamination of wounds C. Fluid volume deficit related to increased capillary permeability D. Potential for impaired gas exchange related to edema of respiratory tract Correct Answer: D Section: (none) Explanation Explanation/Reference: Explanation: (A, B, C) These answers are all correct; however, maintenance of airway is the top priority. (D) Persons burned about the face and neck during an explosion are also likely to suffer burns of the respiratory tract, which can lead to edema and respiratory arrest. QUESTION 757 A female baby was born with talipes equinovarus. Her mother hasrequested that the nurse assigned to the baby come to her room to discuss the baby's condition. The nurse knows that the pediatrician has discussed the baby's condition with her mother and that an orthopedist has been consulted but has not yet seen the baby. What should the nurse do first? A. Call the orthopedist and request that he come to see the baby now. B. Question the mother and find out what the pediatrician has told her about the baby's condition. C. Tell the mother that this is not a serious condition. D. Tell the mother that this condition has been successfully treated with exercises, casts, and/or braces. Correct Answer: B Section: (none) Explanation Explanation/Reference: Explanation: (A) The nurse should call the orthopedist after assessing the mother's knowledge. (B) The nurse must first assess the knowledge of the parent before attempting any explanation. (C) The nurse should assess the mother's knowledge of the baby's condition as the first priority. (D) This answer is correct, but the priority is B.QUESTION 758 Cystic fibrosis is transmitted as an autosomal recessive trait. This means that: A. Mothers carry the gene and pass it to their sons B. Fathers carry the gene and pass it to their daughters C. Both parents must have the disease for a child to have the disease D. Both parents must be carriers for a child to have the disease Correct Answer: D Section: (none) Explanation Explanation/Reference: Explanation: (A) Cystic fibrosis is not an X-linked or sex-linked disease. (B) The only characteristic on the Y chromosome is the trait for hairy ears. (C) Both parents do not need to have the disease but must be carriers. (D) If a trait is recessive, two genes (one from each parent) are necessary to produce an affected child. QUESTION 759 Diabetes mellitus is a disorder that affects 3.1 out of every 1000 children younger than 20 years old. It is characterized by an absence of, or marked decrease in, circulating insulin. When teaching a newly diagnosed diabetes client, the nurse includes information on the functions of insulin: A. Transport of glucose into body cells and storage of glycogen in the liver B. Glycogenolysis and facilitation of glucose use for energy C. Glycogenolysis and catabolism D. Catabolism and hyperglycemia Correct Answer: A Section: (none) Explanation Explanation/Reference: Explanation: (A) Lack of insulin causes glycogenolysis, catabolism, and hyperglycemia. (B) Insulin promotes the conversion of glucose to glycogen for storage and regulates the rate at which carbohydrates are used by cells for energy. (C) Insulin is anabolic in nature. (D) Glucose stimulates protein synthesis within the tissue and inhibits the breakdown of protein into amino acids. QUESTION 760 A 14-year-old boy has had diabetes for 7 years. He takes 30 U of NPH insulin and 10 U of regular insulin every morning at 7 AM. He eats breakfast at 7:30 AM andlunch at noon. What time should he expect the greatest risk for hypoglycemia? A. 9 AM B. 1 PM C. 11 AM D. 3 PM Correct Answer: C Section: (none) Explanation Explanation/Reference: Explanation: (A) This time is incorrect because regular insulin would peak after the teenager has eaten breakfast. (B) This time is incorrect because it is after lunch when the NPH peaks. (C) Regular insulin peaks in 23 hours and has a duration of 46 hours. NPH insulin's onset is 46 hours and peaks in 816 hours. Blood sugar would peak after meals and be lowest before meals and during the night. (D) This time is incorrect because it is before the NPH and after the regular insulin peak times. QUESTION 761 A 16-year-old diabetic girl has been selected as a cheerleader at her school. She asks the nurse whether she should increase her insulin when she is planning to attend cheerleading practice sessions lasting from 8 to 11 AM. The most appropriate answer would be: A. "You should ask your doctor about this." B. "Yes, increase your insulin by 1 U for each hour of practice because exercise causes the body to need more insulin." C. "No, do not increase your insulin. Exercise will not affect your insulin needs." D. "No, do not increase your insulin, but eating a snack prior to practice exercise will make insulin more effective and move more glucose into the cells." Correct Answer: D Section: (none) Explanation Explanation/Reference: Explanation: (A) A nurse can give this information to a client. (B) Exercise makes insulin more efficient in moving more glucose into the cells. No more insulin is needed. (C) Exercise makes insulin more efficient unless the diabetes is poorly controlled. (D) Exercise makes insulin more efficient in moving more glucose into the cells. QUESTION 762 The physician decides to prescribe both a short-acting insulin and an intermediate-acting insulin for a newly diagnosed 8-year-old diabetic client. An example of a short-acting insulin is:A. Novolin Regular B. Humulin NPH C. Lente Beef D. Protamine zinc insulin Correct Answer: A Section: (none) Explanation Explanation/Reference: Explanation: (A) Novolin is a short-acting insulin. (B, C) NPH and Lente are intermediate-acting insulins. (D) Protamine zinc insulin is a long-acting insulin preparation. QUESTION 763 When preparing insulin for IV administration, the nurse identifies which kind of insulin to use? A. NPH B. Human or pork C. Regular D. Long acting Correct Answer: C Section: (none) Explanation Explanation/Reference: Explanation: (A, B, D) Intermediate-acting and long-acting preparations contain materials that increase length of absorption time from the subcutaneous tissues but cause the preparation to be cloudy and unsuitable for IV use. Human insulin must be given SC. (C) Only regular insulin can be given IV. QUESTION 764 A 33-year-old client is diagnosed with bipolar disorder, acute phase. This is her first psychiatric hospitalization, and she is being evaluated for treatment with lithium. Which of the following diagnostic tests are essential prior to the initiation of lithium therapy with this client? A. Hematocrit, hemoglobin, and white blood cell (WBC) count B. Blood urea nitrogen, electrolytes, and creatinineC. Glucose, glucose tolerance test, and random blood sugar D. X-rays, electroencephalogram, and electrocardiogram(ECG) Correct Answer: B Section: (none) Explanation Explanation/Reference: Explanation: (A) These are general diagnostic blood studies (usually done on admission), but they are not reliable indicators of lithium therapy clearance. (B) These are the primary diagnostic tests to determine kidney functioning. Because lithium is excreted through the kidneys and because it can be very toxic, adequate renal function must be ascertained before therapy begins. (C) These are diagnostic blood tests used to determine the presence of endocrine (not renal) dysfunction. (D) These are other types of diagnostic procedures used to determine musculoskeletal, neural, and cardiac (rather than renal) functioning. QUESTION 765 A client has been taking lithium 300 mg po bid for the past two weeks. This morning her lithium level was 1 mEq/L. The nurse should: A. Notify the physician immediately B. Hold the morning lithium dose and continue to observe the client C. Administer the morning lithium dose as scheduled D. Obtain an order for benztropine (Cogentin) Correct Answer: C Section: (none) Explanation Explanation/Reference: Explanation: (A) There is no need to phone the physician because the lithium level is within therapeutic range and because there are no indications of toxicity present. (B) There is no reason to withhold the lithium because the blood level is within therapeutic range. Also, it is necessary to give the medication as scheduled to maintain adequate blood levels. (C) The lab results indicate that the client's lithium level is within therapeutic range (0.21.4 mEq/L), so the medication should be given as ordered. (D) Benztropine is an antiparkinsonism drug frequently given to counteractextrapyramidal symptoms associated with the administration of antipsychotic drugs (not lithium). QUESTION 766 A 23-year-old male client is admitted to the chemical dependency unit with a medical diagnosis of alcoholism. He reports that the last time he drank was 3 days ago, and that now he is starting to "feel kind of shaky." Based on the information given above, nursing care goals for this client will initially focus on:A. Self-concept problems B. Interpersonal issues C. Ineffective coping skills D. Physiological stabilization Correct Answer: D Section: (none) Explanation Explanation/Reference: Explanation: (A) Self-concept and self-esteem problems may emerge during the client's treatment, but these are not immediate concerns. (B) Interpersonal issues may become evident during the course of the client's treatment, but these are also not immediate areas of concern. (C) Improving individual coping skills is generally a primary focus in the treatment and nursing care of persons with substance abuse problems. However, this is still not the immediate concern in this client situation. (D) Correction of fluid and electrolyte status and vitamin deficiencies, as well as prevention of delirium, is the immediate concern in the care of this client. QUESTION 767 One afternoon 3 weeks into his alcohol treatment program, a client says to the nurse, "It's really not all my fault that I have a drinking problem. Alcoholism runs in my family. Both my grandfather and father were heavy drinkers." The nurse's best response would be: A. "That might be a problem. Tell me more about them." B. "Risk factors can often be controlled by self-responsibility." C. "It sounds like you're intellectualizing your drinking problem." D. "Your grandfather and father were both alcoholics?" Correct Answer: B Section: (none) Explanation Explanation/Reference: Explanation: (A) Focusing is an effective therapeutic strategy. Thisresponse, however, allows the client to "defocus" off the topic of learning how to accept responsibility for his behavior and future growth. (B) The nurse can educate the client about both the "genetic risk" for the development of alcoholism and ways to make long-term healthy lifestyle changes. (C) This response is inappropriately confrontational and condescending to the client. (D) Reflection of content can be an effective verbal therapeutic technique. It is used inappropriately here. QUESTION 768 A 14-year-old teenager is hospitalized for anorexia nervosa. She is admitted to the adolescent mental health unit and placed on a behavior modification program.Nursing interventions for the teenager will most likely include: A. Establishing routine tasks and activities around mealtimes B. Administering medications such aslithium C. Requiring the client to eat more during meals D. Checking the client'sroom frequently Correct Answer: A Section: (none) Explanation Explanation/Reference: Explanation: (A) Providing a more structured, supportive environment addresses safety and comfort needs, thereby helping the anorexic client develop more internal control. (B) Medications (commonly antidepressants) are frequently ordered for the anorexic client. However, lithium (used primarily with bipolar disorder) is not commonly used to treat the anorexic client. (C) Requiring and/or demanding that the anorexic client "eat more" at mealtimes increases the client's feelings of powerlessness. (D) Like the previous strategy, checking the client's room frequently contributes to the client's feelings of powerlessness. QUESTION 769 A measurable outcome criterion in the nursing care of an adolescent with anorexia nervosa would be: A. Accepting her present body image B. Verbalizing realistic feelings about her body C. Having an improved perception of her body image D. Exhibiting increased self-esteem Correct Answer: B Section: (none) Explanation Explanation/Reference: Explanation: (A) This outcome criterion is inadequate because the term "accepts" is not directly measurable. (B) This outcome criterion is directly measurable because specific goal-related verbalizations can be heard and verified by the nurse. (C) "Improved perception of body image" is not directly measurable and is therefore open to many interpretations. (D) Although long-term goals for the anorexic client should focus on increased self-esteem, this outcome criterion (as stated) does not include specific indicators or behaviors for which to observe. QUESTION 770A 23-year-old female client is brought to the emergency room by her roommate for repeatedly making superficial cuts on her wrists and experiencing wide mood swings. She is very angry and hostile. Her medical diagnosis is adjustment disorder versus borderline personality disorder. The client comments to the nurse, "Nobody in here seems to really care about the clients. I thought nurses cared about people!" The client is exhibiting the ego defense mechanism: A. Reaction formation B. Rationalization C. Splitting D. Sublimation Correct Answer: C Section: (none) Explanation Explanation/Reference: Explanation: (A) Reaction formation is the development and demonstration of attitudes and/or behaviors opposite to what an individual actually feels. The client's comment does reveal her anger and hostility. (B) Rationalization, another ego defense mechanism, is offering a socially acceptable or seemingly logical explanation to justify one's feelings, behaviors, or motives. The client's comment does not reflect rationalization. (C) Splitting, the viewing of people or situations as either all good or all bad, is frequently used by persons experiencing a disruption in self-concept. This ego defense mechanism is reflective of the individual's inability to integrate the positive and negative aspects of self. (D) Sublimation, the channeling of socially unacceptable impulses and behaviors into more acceptable patterns of behavior, is another ego defense mechanism. The client's comment reveals that she is not engaging in sublimation. QUESTION 771 A client hospitalized with a medical diagnosis of adjustment disorder versus personality disorder states, "Nobody cares about the clients." The nurse's most effective response would be: A. "How can you say that I don't care? We just met." B. "What makes you think the nurses don't care?" C. "You will feel differently about us in a few days." D. "You seem angry. Tell me more about how you feel." Correct Answer: D Section: (none) Explanation Explanation/Reference: Explanation: (A) This statement is a defensive response that places the nurse in a vulnerable countertransference position, and at the same time, fails to challenge the client's"splitting" behavior. (B) This statement is a defensive response by the nurse. In addition, this type of nontherapeutic statement requests that the client explain the reasons for her behavior, a difficult task for an individual with limited insight. (C) This statement is a nontherapeutic response that both ignores the intensity of the client's emotions and the dynamics underlying "splitting" behavior. (D) By simultaneously acknowledging the client's emotional intensity and gently challenging her "splitting" behavior, the nurse addresses the client's current distortions and prepares for further interventions with angry or ambivalent feelings. QUESTION 772 A 45-year-old client diagnosed with major depression is scheduled for electroconvulsive therapy (ECT) in the morning. Which of the following medications are routinely administered either before or during ECT? A. Thioridazine (Mellaril), lithium, and benztropine B. Atropine, sodium brevitol, and succinylcholine chloride (Anectine) C. Sodium, potassium, and magnesium D. Carbamazepine (Tegretol), haloperidol, and trihexyphenidyl (Artane) Correct Answer: B Section: (none) Explanation Explanation/Reference: Explanation: (A) Thioridazine (an antipsychotic drug), lithium (an antimanic drug), and benztropine (an antiparkinsonism agent) are generally administered to treat schizophrenic and bipolar disorders. (B) Atropine (a cholinergic blocker), sodium brevitol (a shortacting anesthetic), and succinylcholine (a neuromuscular blocker) are administered either before or during ECT to coun teract bradycardia and to provide anesthesia and total muscle relaxation. (C) These are electrolyte substances administered to correct fluid and electrolyte imbalances in the body. (D) Carbamazepine (an anticonvulsant), haldoperidol (an antipsychotic), and trihexyphenydyl (an antiparkinsonism agent) are usually administered in psychiatric settings to control problems associated with psychotic behavior. QUESTION 773 A 35-year-old client isreceiving psychopharmacological treatment of his major depression with tranylcypromine sulfate (Parnate), a monoamine oxidase (MAO) inhibitor. The nurse teaches the client that while he is taking this type of antidepressant, he needs to restrict his dietary intake of: A. Potassium-rich foods B. Tryptophan C. Tyramine D. Saturated fats Correct Answer: C Section: (none) ExplanationExplanation/Reference: Explanation: (A) The client may need to avoid some potassium-rich foods (such as bananas, raisins, etc.). However, this is not because of the potassium content of these foods. (B) Tryptophan is an essential amino acid that is present in high concentrations in animal and fish protein. (C) The client will need to watch his dietary intake of tyramine. Tyramine is a by-product of the conversion of tyrosine to epinephrine. Tyramine is found in a variety of foods and beverages, ranging from aged cheese to caffeine drinks. Ingestion of tyramine-rich foods while taking a MAO inhibitor may lead to an increase in blood pressure and/or a life-threatening hypertensive crisis. (D) To maintain a healthy lifestyle, restriction of dietary saturated fats is advisable. QUESTION 774 The nurse will be alert to the most potentially lifethreatening side effect associated with the administration of monoamine oxidase (MAO) inhibitor. This is: A. Oculogyric crisis B. Hypertensive crisis C. Orthostatic hypotension D. Tardive dyskinesia Correct Answer: B Section: (none) Explanation Explanation/Reference: Explanation: (A) Oculogyric crisis, involuntary upward deviation and fixation of the eyeballs, is usually associated with either postencephalitic parkinsonian or drug-induced extrapyramidal symptoms (EPS). (B) Hypertensive crisis is a potentially life-threatening side effect. This may occur if the client ingests foods, beverages, or medications containing tyramine. (C) Orthostatic hypotension, a drop in blood pressure resulting from a rapid change of body position, can occur with the administration of antidepressants. (D) Tardive dyskinesia, characterized by slow, rhythmical, automatic or stereotyped muscular movements, usually is associated with the administration of certain antipsychotic medications. QUESTION 775 A 38-year-old female client with a history of chronic schizophrenia, paranoid type, is currently an outpatient at the local mental health and mental retardation clinic. The client comes in once a week for medication evaluation and/or refills. She self-administers haloperidol 5 mg twice a day and benztropine 1 mg once a day. During a recent clinic visit, she says to the nurse, "I can't stay still at night. I toss and turn and can't fall asleep." The nurse suspects that she may be experiencing: A. Akathisia B. Akinesia C. Dystonia D. OpisthotonosCorrect Answer: A Section: (none) Explanation Explanation/Reference: Explanation: (A) Akathisia, or motor restlessness, is a reversible EPS frequently associated with the administration of antipsychotic drugs such as haloperidol. (B) Akinesia, or muscular or motor retardation, is an example of reversible EPS frequently associated with the administration of major tranquilizers such as haloperidol. (C) Acute dystonic reactions, bizarre and severe muscle contractions usually of the tongue, face, neck or extraocular muscles, are examples of EPS. (D) Opisthotonos, a severe type of whole-body dystonic reaction in which the head and heels are bent backward while the body is bowed forward, is an example of EPS. QUESTION 776 On assessment, the nurse learns that a chronic paranoid schizophrenic has been taking "the blue pill" (haloperidol) in the morning and evening, and "the white pill" (benztropine) right before bedtime. The nurse might suggest to the client that she try: A. Doubling the daily dose of benztropine B. Decreasing the haloperidol dosage for a few days C. Taking the benztropine in the morning D. Taking her medication with food or milk Correct Answer: C Section: (none) Explanation Explanation/Reference: Explanation: (A) Suggesting that a client increase a medication dosage is an inappropriate (and illegal) nursing action. This action requires a physician's order. (B) To suggestthat a client decrease a medication dosage is an inappropriate (and illegal) nursing action. This action requires a physician's order. (C) This response is an appropriate independent nursing action. Because motorrestlessness can also be a side effect of cogentin, the nurse may suggest that the client try taking the drug early in the day rather than at bedtime. (D) Certain medications can cause gastric irritation and may be taken with food or milk to prevent this side effect. QUESTION 777 A 27-year-old male client is admitted to the acute care mental health unit for observation. He has recently lost his job, and his wife told him yesterday that she wants a divorce. The client is placed on suicide precautions. In assessing suicide potential, the nurse should pay close attention to the client's: A. Level of insight B. Thought processes C. Mood and affectD. Abstracting abilities Correct Answer: C Section: (none) Explanation Explanation/Reference: Explanation: (A) Assessing the client's level of insight is an important part of the mental status exam (MSE), but it does not reflect suicide potential. (B) Assessing the client's thought processes is an important part of the MSE, but it does not reflect suicide potential. (C) Assessing the client's mood and affect is an important part of the MSE, and it can be a very valuable indicator of suicide potential. Frequently a client who has decided to proceed with suicide plans will exhibit a suddenly improved mood and affect. (D) Assessing a client's abstracting abilities is an important part of the MSE, but it does not reflect suicide potential. QUESTION 778 The nurse knows that children are more susceptible to respiratory tract infections owing to physiological differences. These childhood differences, when compared to an adult, include: A. Fewer alveoli, slower respiratory rate B. Diaphragmatic breathing, larger volume of air C. Larger number of alveoli, diaphragmatic breathing D. Rounded shape of chest, smaller volume of air Correct Answer: D Section: (none) Explanation Explanation/Reference: Explanation: (A) Although a child has fewer alveoli than an adult, the child's respiratory rate is faster. (B) Although a child may use diaphragmatic breathing, the adult exchanges a larger volume of air. (C) The adult has a larger number of alveoli than a child. (D) The child's chest is rounded whereas the adult chest is more of an oval shape,and the child does exchange a smaller volume of air than an adult. QUESTION 779 A 2-year-old toddler is hospitalized with epiglottitis. In assessing the toddler, the nurse would expect to find: A. A productive cough B. Expiratory stridor C. DroolingD. Crackles in the lower lobes Correct Answer: C Section: (none) Explanation Explanation/Reference: Explanation: (A) A productive cough is not associated with epiglottitis. (B) Children with epiglottitis seldom have expiratory stridor. Inspiratory stridor is more common due to edema of the supraglottic tissues. (C) Because of difficulty with swallowing, drooling often accompanies epiglottitis. (D) Crackles are not heard in the lower lobes with epiglottitis because the infection is usually confined to the supraglottic structures. QUESTION 780 Which of the following nursing care goals has the highest priority for a child with epiglottitis? A. Sleep or lie quietly 10 hr/day. B. Consume foods from all four food groups. C. Be afebrile throughout her hospital stay. D. Participate in play activities 4 hr/day. Correct Answer: A Section: (none) Explanation Explanation/Reference: Explanation: (A) Of these four goals, maintenance of a calm, quiet atmosphere to reduce anxiety and to allow for rest is the most important. (B) Although nutrition is important, the child needs fluids to maintain fluid and electrolyte balance more than solid foods. In addition, the child may not be able to swallow solid foods owing to epiglottic swelling. (C) This goal is unrealistic because fever is a common symptom of the infection associated with epiglottitis. (D) If overexerted, the child will need more O2 and energy than available, and these requirements may exacerbate the condition. QUESTION 781 Which of the following nursing orders has the highest priority for a child with epiglottitis? A. Vital signs every shift B. Tracheostomy set at bedside C. Intake and outputD. Specific gravity every shift Correct Answer: B Section: (none) Explanation Explanation/Reference: Explanation: (A) Because of the possibility of fever or respiratory failure, vital signs should be done more often than every eight hours. (B) If the epiglottitis worsens, the edema and laryngospasm may close the airway and an emergency tracheostomy may be necessary. (C) Although intake and output are a part of the nursing care of a child with epiglottitis, it is not as important as the safety measure of keeping the tracheostomy set at the bedside. (D) Specific gravity will indicate hydration status, but it is not as important as keeping the tracheostomy set at the bedside. QUESTION 782 Often children are monitored with pulse oximeter. The pulse oximeter measures the: A. O2 content of the blood B. Oxygen saturation of arterial blood C. PO2 D. Affinity of hemoglobin for O2 Correct Answer: B Section: (none) Explanation Explanation/Reference: Explanation: (A) The O2 content of whole blood is determined by the partial pressure of oxygen (PO2) and the oxygen saturation. The pulse oximeter does not measure the PO2. (B) The pulse oximeter is a noninvasive method of measuring the arterial oxygen saturation. (C) The PO2 is the amount of O2 dissolved in plasma, which the pulse oximeter does not measure. (D) The affinity of hemoglobin for O2 is the relationship between oxygen saturation and PO2 and is not measured by the pulse oximeter. QUESTION 783 A 4-year-old boy is brought to the emergency room with bruises on his head, face, arms, and legs. His mother states that he fell down some steps. The nurse suspects that he may have been physically abused. In accordance with the law, the nurse must: A. Tell the physician her concerns B. Report her suspicions to the authorities C. Talk to the child's fatherD. Confront the child's mother Correct Answer: B Section: (none) Explanation Explanation/Reference: Explanation: (A) Although the nurse probably would talk to the physician about these concerns, the nurse is not required by law to do so. (B) All healthcare workers are required by the Federal Child Abuse Prevention and Treatment Act of 1974 to report suspected and actual cases of child abuse and/or neglect. (C) Talking to the child's father may or may not help the child, and the nurse is not required by law to do so. (D) Confrontation may not be indicated; the nurse is not required by law to confront the child's mother with these suspicions. QUESTION 784 The mother of a preschooler reports to the nurse that he frequently tells lies. The admission assessment of the child indicates possible child abuse. The nurse knows that his: A. Behavior is not normal, and a child psychiatrist should be consulted. B. Mother is lying to protect herself. C. Lying is normal behavior for a preschool child who is learning to separate fantasy from reality. D. Behavior indicates a developmental delay, because preschoolers should be able to tell right from wrong. Correct Answer: C Section: (none) Explanation Explanation/Reference: Explanation: (A) Because preschoolers often tell "stories" as they learn to differentiate fantasy from reality, the child's behavior is normal. (B) The nurse has no reason to believe the child's mother is lying, because children of his age often tell lies. (C) The child's lying is actually "storytelling" as he learns to separate fantasy from reality, a normal developmental task for his age group. (D) The child's behavior is consistent with his age and does not indicate a developmental delay. QUESTION 785 A family by court order undergoes treatment by a family therapist for child abuse. The nurse, who is the child's case manager knows that treatment has been effective when: A. The child is removed from the home and placed in foster care B. The child's parents identify the ways in which he is different from the rest of the familyC. The child's father is arrested for child abuse D. The child's parents can identify appropriate behaviors for children in his age group Correct Answer: D Section: (none) Explanation Explanation/Reference: Explanation: (A) Removing an abused child from the home and placement in a foster home are not the desired outcome of treatment. (B) Children who are perceived as"different" from the rest of the family are more likely to be abused. (C) Although legal action may be taken against abusive parents, it is not an indicator of an effective treatment program. (D) Identification of age-appropriate behaviors is essential to the role of parents, because misunderstanding children's normal developmental needs often contributes to abuse or neglect. QUESTION 786 Nursing assessment of early evidence of septic shock in children at risk includes: A. Fever, tachycardia, and tachypnea B. Respiratory distress, cold skin, and pale extremities C. Elevated blood pressure, hyperventilation, and thready pulses D. Normal pulses, hypotension, and oliguria Correct Answer: A Section: (none) Explanation Explanation/Reference: Explanation: (A) Fever, tachycardia, and tachypnea are the classic early signs of septic shock in children. (B) Respiratory distress, cold skin, and pale extremities are later signs of septic shock. (C) Elevated blood pressure, hyperventilation, and thready pulses are later signs of septic shock. (D) Normal pulses, hypotension, and oliguria are not early signs of septic shock. QUESTION 787 A 3-year-old child is in the burn unit following a home accident. The first sign of sepsis in burned children is: A. Disorientation B. Low-grade feverC. Diarrhea D. Hypertension Correct Answer: A Section: (none) Explanation Explanation/Reference: Explanation: (A) Disorientation is the first sign of sepsis in burn children. (B) Low-grade fever is not indicative of sepsis. (C) Diarrhea is not indicative of sepsis. (D) Hypertension is not indicative of sepsis. QUESTION 788 One of the most reliable assessment tools for adequacy of fluid resuscitation in burned childrenis: A. Blood pressure B. Level of consciousness C. Skin turgor D. Fluid intake Correct Answer: B Section: (none) Explanation Explanation/Reference: Explanation: (A) Blood pressure can remain normotensive in a state of hypovolemia. (B) Capillary refill, alterations in sensorium, and urine output are the most reliable indicators for assessing hydration. (C) Skin turgor is not a reliable indicator for assessing hydration in a burn client. (D) Fluid intake does not indicate adequacy of fluid resuscitation in a burn client. QUESTION 789 A child with celiac disease is being discharged from the hospital. The mother demonstrates knowledge of nutritional needs of her child when she is able to state the foods which are included in a: A. Lactose-restricted diet B. Gluten-restricted diet C. Phenylalanine-restricted dietD. Fat-restricted diet Correct Answer: B Section: (none) Explanation Explanation/Reference: Explanation: (A) A lactose-restricted diet is prescribed for children with lactose intolerance or diarrhea. (B) A gluten-restricted diet is the diet for children with celiac disease. (C) A phenylalaninerestricted diet is prescribed for children with phenylketonuria. (D) A fat-restricted diet is prescribed for children with disorders of the liver, gallbladder, or pancreas. QUESTION 790 An 80-year-old male client with a history of arteriosclerosis is experiencing severe pain in his left leg that started approximately 20 minutes ago. When performing the admission assessment, the nurse would expect to observe which of the following: http://www.gratisexam.com/ A. Both lower extremities warm to touch with 2_pedal pulses B. Both lower extremities cyanotic when placed in a dependent position C. Decreased or absent pedal pulse in the left leg D. The left leg warmer to touch than the right leg Correct Answer: C Section: (none) Explanation Explanation/Reference: Explanation: (A) This statement describes a normal assessment finding of the lower extremities. (B) This assessment finding reflects problems caused by venous insufficiency. (C) Decreased or absentpedal pulses reflect a problem caused by arterial insufficiency. (D) The leg that is experiencing arterial insufficiency would be cool to touch due to the decreased circulation.QUESTION 791 A male client is scheduled to have angiography of his left leg. The nurse needs to include which of the following when preparing the client for this procedure? A. Validate that he is not allergic to iodine or shellfish. B. Instruct him to start active range of motion of his left leg immediately following the procedure. C. Inform him that he will not be able to eat or drink anything for 4 hours after the procedure. D. Inform him that vital signs will be taken every hour for 4 hours after the procedure. Correct Answer: A Section: (none) Explanation Explanation/Reference: Explanation: (A) Angiography, an invasive radiographic examination, involves the injection of a contrast solution (iodine) through a catheter that has been inserted into an artery. (B) The client is kept on complete bed rest for 612 hours after the procedure. The extremity in which the catheter was inserted must be immobilized and kept straight during this time. (C) The contrast dye, iodine, is nephrotoxic. The client must be instructed to drink a large quantity of fluids to assist the kidneys in excreting this contrast media. (D) The major complication of this procedure is hemorrhage. Vital signs are assessed every 15 minutes initially for signs of bleeding. QUESTION 792 A female client is started on warfarin (Coumadin) 5 mg po bid. To adequately evaluate the effectiveness of the warfarin therapy, the nurse must know that this medication: A. Dissolves any clots already formed in the arteries B. Prevents the conversion of prothrombin to thrombin C. Interferes with the synthesis of vitamin K-dependent clotting factors D. Stimulates the manufacturing of platelets Correct Answer: C Section: (none) Explanation Explanation/Reference: Explanation: (A) Thrombolytic agents (e.g., streptokinase) directly activate plasminogen, dissolving fibrin deposits, which in turn dissolves clots that have already formed. (B)Heparin prevents the formation of clots by potentiating the effects of antithrombin III and the conversion of prothrombin to thrombin. (C) Warfarin prevents the formation of clots by interfering with the hepatic synthesis of the vitamin K-dependent clotting factors. (D) Platelets initiate the coagulation of blood by adhering toeach other and the site of injury to form platelet plugs. QUESTION 793 A client is to be discharged from the hospital and is to continue taking warfarin 2.5 mg po bid. Which of the following should be included in her discharge teaching concerning the warfarin therapy? A. "If you forget to take your morning dose, double the night time dose." B. "You should take aspirin instead of acetaminophen (Tylenol) for headaches." C. "Carry a medications alert card with you at all times." D. "You should use a straight-edge razor when shaving your arms and legs." Correct Answer: C Section: (none) Explanation Explanation/Reference: Explanation: (A) Warfarin must always be taken exactly as directed. Clients should be instructed never to skip or double up on their dosage. (B) Aspirin decreases platelet aggregation, which would potentiate the effects of the coumadin. (C) Healthcare providers need to be aware of persons on warfarin therapy prior to the initiation of any diagnostic tests and/or surgery to help prevent bleeding complications. (D) An electric razor should be used to prevent accidental cutting, which can lead to bleeding. QUESTION 794 A 40-year-old client has been admitted to the hospital with severe substernal chest pain radiating down his left arm. The nurse caring for the client establishes the following priority nursing diagnosis--Alteration in comfort, pain related to: A. Increased excretion of lactic acid due to myocardial hypoxia B. Increased blood flow through the coronary arteries C. Decreased stimulation of the sympathetic nervous system D. Decreased secretion of catecholamines secondary to anxiety Correct Answer: A Section: (none) Explanation Explanation/Reference: Explanation: (A) Anaerobic metabolism results because the decreased blood supply to the myocardium causes a release of lactic acid. Lactic acid is an irritant to the myocardialneural receptors, producing chest pain. (B) Chest pain is caused by a decrease in the O2 supply to the myocardial cells. Treatment modalities for chest pain are aimed toward increasing the blood flow through coronary arteries. (C) Chest pain causes an increase in the stimulation of the sympathetic nervous system. This stimulation increases the heart rate and blood pressure, causing an increase in myocardial workload aggravating the chest pain. (D) Chest pain and anxiety cause increased secretion of catecholamines by stimulating the sympathetic nervous system. This stimulation increases chest pain by increasing the workload of the heart. QUESTION 795 Morphine sulfate 4 mg IV push q2h prn for chest pain was ordered for a client in the emergency room with severe chest pain. The nurse administering the morphine sulfate knows which of the following therapeutic actions is related to the morphine sulfate? A. Increased level of consciousness B. Increased rate and depth of respirations C. Increased peripheral vasodilation D. Increased perception of pain Correct Answer: C Section: (none) Explanation Explanation/Reference: Explanation: (A) Morphine sulfate, a narcotic analgesic, causes sedation and a decrease in level of consciousness. (B) The side effects of morphine sulfate include respiratory depression. (C) Morphine sulfate causes peripheral vasodilation, which decreases afterload, producing a decrease in the myocardial workload. (D) Morphine sulfate alters the perception of pain through an unclear mechanism. This alteration promotes pain relief. QUESTION 796 A client had a cardiac catheterization with angiography and thrombolytic therapy with streptokinase. The nurse should initiate which of the following interventions immediately after he returns to his room? A. Place him on NPO restriction for 4 hours. B. Monitor the catheterization site every 15 minutes. C. Place him in a high Fowler position. D. Ambulate him to the bathroom to void. Correct Answer: B Section: (none) Explanation Explanation/Reference:Explanation: (A) A contrast dye, iodine, is used in this procedure. This dye is nephrotoxic. The client must be encouraged to drink plenty of liquids to assist the kidneys in eliminating the dye. (B) Streptokinase activates plasminogen, dissolving fibrin deposits. To prevent bleeding, pressure is applied at the insertion site. The client is assessed for both internal and external bleeding. (C) The extremity used for the insertion site must be kept straight and be immobilized because of the potential for bleeding. (D) The client is kept on bed rest for 812 hours following the procedure because of the potential for bleeding. QUESTION 797 The nurse notes scattered crackles in both lungs and 1+ pitting edema when assessing a cardiac client. The physician is notified and orders furosemide (Lasix) 80 mg IV push stat. Which of the following diagnostic studies is monitored to assess for a major complication of this therapy? A. Serum electrolytes B. Arterial blood gases C. Complete blood count D. 12-Lead ECG Correct Answer: A Section: (none) Explanation Explanation/Reference: Explanation: (A) Furosemide, a potassium-depleting diuretic, inhibits the reabsorption of sodium and chloride from the loop of Henle and the distal renal tubules. Serum electrolytes are monitored for hypokalemia. (B) Severe acid-base imbalances influence the movement of potassium into and out of the cells, but arterial blood gases to not measure the serum potassium level. (C) Furosemide is a potassium-depleting diuretic. A complete blood count does not reflect potassium levels. (D) Abnormalities in potassium (both hyperkalemia and hypokalemia) are reflected in ECG changes, but these changes do not occur until the abnormality is severe. QUESTION 798 Prior to his discharge from the hospital, a cardiac client is started on digoxin (Lanoxin) 25 mg po qd. The nurse initiates discharge teaching. Which of the following statements by the client would validate an understanding of his medication? A. "I would notifymy physician immediately if I experience nausea, vomiting, and double vision." B. "I could stop taking this medication when I begin to feel better." C. "I should only take the medication if my heart rate is greater than 100 bpm." D. "I should always take this medication with an antacid." Correct Answer: A Section: (none) ExplanationExplanation/Reference: Explanation: (A) The first signs of digoxin toxicity include abdominal pain, anorexia, nausea, vomiting, and visual disturbances. The physician should be notified if any of these symptoms are experienced. (B) The positive inotropic effects of digoxin increase cardiac output and result in an enhanced activity tolerance. "Feeling better" indicates the drug is working and medication therapy must be continued. (C) Clients should be taught to take their pulse prior to taking the digoxin. If their pulse rate becomes irregular, slows significantly, or is >100 bpm the physician should be notified. (D) Antacids decrease the effectiveness of digoxin. QUESTION 799 When assessing a client, the nurse notes the typical skin rash seen with systemic lupus erythematosus. Which of the following descriptions correctly describes this rash? A. Small round or oval reddish brown macules scattered over the entire body B. Scattered clusters of macules, papules, and vesicles over the body C. Bright red appearance of the palmar surface of the hands D. Reddened butterfly shaped rash over the cheeks and nose Correct Answer: D Section: (none) Explanation Explanation/Reference: Explanation: (A) The appearance of small, round or oval reddish brown macules scattered over the entire body is characteristic of rubeola. (B) The appearance of scattered clusters of macules, papules, and vesicles throughout the body is characteristic of chickenpox. (C) Palmar redness is seen in clients with cirrhosis of the liver. (D) The characteristic butterfly rash over the cheek and nose and into the scalp is seen with systemic lupus erythematosus. QUESTION 800 The nurse notes multiple bruises on the arms and legs of a newly admitted client with lupus. The client states, "I get them whenever I bump into anything." The nurse would expect to note a decrease in which of the following laboratory tests? A. Number of platelets B. WBC count C. Hemoglobin level D. Number of lymphocytes Correct Answer: A Section: (none) ExplanationExplanation/Reference: Explanation: (A) Thrombocytopenia, a decrease in platelets, occurs in lupus and causes a decrease in blood coagulation and thrombus formation. (B) Clients with lupus will have a decrease in the WBC count decreasing their resistance to infection. (C) Clients with lupus may have a decrease in the hemoglobin level causing anemia. (D) Leukopenia, a decrease in white blood cells, is seen in lupus and decreases resistance to infection. QUESTION 801 A client is started on prednisone 2.5 mg po bid. Which of the following instructions should be included in her discharge teaching specific to this medication? A. Increase your oral intake of fluids to at least 4000 mL every day. B. Avoid contact with people who have contagious illnesses. C. Brush your teeth at least 4 times a day with a firm toothbrush. D. Immediately stop taking the prednisone if you feel depressed. Correct Answer: B Section: (none) Explanation Explanation/Reference: Explanation: (A) Fluid retention is a side effect of prednisone. The nurse should teach clients to weigh themselves daily and to observe for signs of edema. If these signs of fluid retention occur, they should notify the physician. (B) Prednisone, a glucocorticoid, suppresses the normal immune response making the client more susceptible toinfections. (C) An increase in bleeding tendencies is a side effect of prednisone therapy. The nurse should teach clients to use preventive measures (i.e., electric razors and soft toothbrushes). (D) Depression and personality changes are side effects of prednisone therapy. Prednisone should never be discontinued abruptly. QUESTION 802 During the assessment, the nurse observes a client scratching his skin. He has been admitted to rule out Laennec's cirrhosis of the liver. The nurse knows the pruritus is directly related to: A. A loss of phagocytic activity B. Faulty processing of bilirubin C. Enhanced detoxification of drugs D. The formation of collateral circulation Correct Answer: B Section: (none) ExplanationExplanation/Reference: Explanation: (A) A loss in the phagocytic activity of the Kupffer cells occurs with cirrhosis of the liver, which increases the susceptibility to infections. (B) The faulty processing of bilirubin produces bilesalts, which are irritating to the skin. (C) The detoxification of drugs is impaired with cirrhosis of the liver. (D) Collateral circulation develops due to portal hypertension. This is manifest through the development of esophageal varices, hemorrhoids, and caput medusae. QUESTION 803 Four days after admission for cirrhosis of the liver, the nurse observes the following when assessing a male client: increased irritability, asterixis, and changes in his speech pattern. Which of the following foods would be appropriate for his bedtime snack? A. Fresh fruit B. A milkshake C. Saltine crackers and peanut butter D. A ham and cheese sandwich Correct Answer: A Section: (none) Explanation Explanation/Reference: Explanation: (A) High levels of ammonia, a by-product of protein metabolism, can precipitate metabolic encephalopathy. These clients need a diet high in carbohydrates and bulk. (B) Metabolic encephalopathy of the brain associated with liver failure is precipitated by elevated ammonia levels. Ammonia is a by-product of protein metabolism. (C, D) Metabolic encephalopathy in liver failure is precipitated by elevated ammonia levels. Ammonia is a by-product of protein metabolism. QUESTION 804 Four days after admission for cirrhosis of the liver, the nurse observes the following when assessing a male client: increased irritability, asterixis, and changes in his speech pattern. Which of the following foods would be appropriate for his bedtime snack? A. Fresh fruit B. A milkshake C. Saltine crackers and peanut butter D. A ham and cheese sandwich Correct Answer: B Section: (none) ExplanationExplanation/Reference: Explanation: (A) Albumin, a blood volume expander, increases the circulating blood volume by exerting an osmotic pull on tissue fluids, pulling them into the vascular system. This fluid shift causes an increase in the heart rate and blood pressure. (B) Albumin, a blood volume expander, exerts an osmotic pull on fluids in the interstitial spaces, pulling the fluid back into the circulatory system. This fluid shift causes an increase in the urinary output. (C) Adventitious breath sounds and dyspnea can occur due to circulatory overload if the albumin is infused too rapidly. (D) Chills, fever, itching, and rashes are signs of a hypersensitivity reaction to albumin. QUESTION 805 A client with cirrhosis of the liver becomes comatose and is started on neomycin 300 mg q6h via nasogastric tube. The rationale for this therapy is to: A. Prevent systemic infection B. Promote diuresis C. Decrease ammonia formation D. Acidify the small bowel Correct Answer: C Section: (none) Explanation Explanation/Reference: Explanation: (A) Neomycin is an antibiotic, but this is not the Rationale for administering it to a client in hepatic coma. (B) Diuretics and salt-free albumin are used to promote diuresis in clients with cirrhosis of the liver. (C) Neomycin destroys the bacteria in the intestines. It is the bacteria in the bowel that break down protein into ammonia. (D) Lactulose is administered to create an acid environment in the bowel. Ammonia leaves the blood and migrates to this acidic environment where it is trapped and excreted. QUESTION 806 The following nursing diagnosis is written for a comatose client with cirrhosis of the liver and secondary splenomegaly--High risk for injury: Increased susceptibility to bleeding related to: A. Increased absorption of vitamin K B. Thrombocytopenia due to hypersplenism C. Diminished function of the Kupffer cells D. Increased synthesis of the clotting factors Correct Answer: B Section: (none)Explanation Explanation/Reference: Explanation: (A) There is a decreased absorption of vitamin K with cirrhosis of the liver. This decrease impairs blood coagulation and the formation of prothrombin. (B) Thrombocytopenia, an increased destruction of platelets, occurs secondary to hypersplenism. (C) A diminished function of the Kupffer cells occurs with cirrhosis of the liver, causing the client to become more susceptible to infections. (D) A decrease in the synthesis of fibrinogen and clotting factors VII, IX, and X occurs with cirrhosis of the liver and increases the susceptibility to bleeding. QUESTION 807 A 52-year-old female client is admitted to the hospital in acute renal failure. She has been on hemodialysis for the past 2 years. Stat arterial blood gases are drawn on the client yielding the following results: pH 7.30, PCO2 51 mm Hg, HCO3, 18 mEq/L, PaO2, 84 mm Hg. The nurse would interpret these results as: A. Compensated metabolic alkalosis B. Respiratory acidosis C. Partially compensated metabolic alkalosis D. Combined respiratory and metabolic acidosis Correct Answer: D Section: (none) Explanation Explanation/Reference: Explanation: (A) Compensated metabolic alkalosis would be reflected by the following: pH within normal limit (7.357.45), PCO2 > 45 mm Hg, HCO3 >26 mEq/L. (B) Respiratoryacidosis would be reflected by the following: pH < 7.35, PCO2 > 45 mm Hg, HCO3 within normal limits (2226 mEq/L). (C) Partially compensated metabolic alkalosis would be reflected by the following: pH > 7.45, PCO2 > 45 mm Hg, HCO3 > 26 mEq/L. (D) Combined respiratory and metabolicacidosis would be reflected by the following: pH < 7.35, PCO2 > 45 mm Hg, HCO3 < 22 mEq/L. QUESTION 808 Stat serum electrolytes ordered for a client in acute renal failure revealed a serum potassium level of 6.4. The physician is immediately notified and orders 50 mL of dextrose and 10 U of regular insulin IV push. The nurse administering these drugs knows the Rationale for this therapy is to: A. Remove the potassium from the body by renin exchange B. Protect the myocardium from the effects of hypokalemia C. Promote rapid protein catabolism D. Drive potassium from the serum back into the cellsCorrect Answer: D Section: (none) Explanation Explanation/Reference: Explanation: (A) Sodium polystyrene sulfonate (Kayexalate), a cation exchange resin, exchanges sodium ions for potassium ions in the large intestine reducing the serum potassium. (B) Calcium is administered to protect the myocardium from the adverse effects of hyperkalemia. Serum levels reflect hyperkalemia. (C) Rapid catabolism releases potassium from the body tissue into the bloodstream. Infection and hyperthermia increase the process of catabolism. (D) The administration of dextrose and regular insulin IV forces potassium back into the cells decreasing the potassium in the serum. QUESTION 809 The nurse writes the following nursing diagnosis for a client in acute renal failure--Impaired gas exchange related to: A. Decreased red blood cell production B. Increased levels of vitamin D C. Increased red blood cell production D. Decreased production of renin Correct Answer: A Section: (none) Explanation Explanation/Reference: Explanation: (A) Red blood cell production is impaired in renal failure owing to impaired erythropoietin production. This causes a decrease in the delivery of oxygen to the tissue and impairs gas exchange. (B) The conversion of vitamin D to its physiologically active form is impaired in renal failure. (C) In renal failure, a decrease in red blood cell production occurs owing to an impaired production of erythropoietin, leading to impaired gas exchange at the cellular level. (D) The decreased production of renin in renal failure causes an increased production of aldosterone causing sodium and water retention. QUESTION 810 A 6-year-old child returned to the surgical floor 20 hours ago after an appendectomy for a gangrenous appendix. His mother tells the nurse that he is becoming more restless and is anxious. Assessment findings indicate that the child has atelectasis. Appropriate nursing actions would include: A. Allowing the child to remain in the position of comfort, preferably semi-or high-Fowler position B. Administering analgesics as ordered C. Having the child turn, cough, and deep breathe every 12 hours D. Remaining with the child and keeping as calm and quiet as possibleCorrect Answer: C Section: (none) Explanation Explanation/Reference: Explanation: (A) Allowing the client to remain in the position of comfort will not resolve the atelectasis. This position, if left unchanged, over time may actually increase the atelectasis. (B) Analgesics will not resolve the atelectasis and may contribute to it if proper nursing actions are not taken to help resolve the atelectasis. (C) Having the client turn, cough, and deep breathe every 12 hours will aid in resolving the atelectasis. Surgery clients are at risk for postoperative respiratory complications because pulmonary function is reduced as a result of anesthesia and surgery. (D) Remaining with the client and keeping him calm and quiet will not affect the client's anxiety, restlessness, or help to resolve the atelectasis. The cause (atelectasis) needs to be treated, not the symptoms (anxiety and restlessness). QUESTION 811 A client develops an intestinal obstruction postoperatively. A nasogastric tube is attached to low, intermittent suction with orders to "Irrigate NG tube with sterile saline q1h and prn." The rationale for using sterile saline, as opposed to using sterile water to irrigate the NG tube is: A. Water will deplete electrolytes resulting in metabolic acidosis. B. Saline will reduce the risk of severe, colicky abdominal pain during NG irrigation. C. Water is not isotonic and will increase restlessness and insomnia in the immediate postoperative period. D. Saline will increase peristalsis in the bowel. Correct Answer: A Section: (none) Explanation Explanation/Reference: Explanation: (A) Water is a hypotonic solution and will deplete electrolytes and cause metabolic acidosis when used for nasogastric irrigation. (B) Irrigating with saline does notcause abdominal discomfort. Severe, colicky abdominal pain is a symptom of intestinal obstruction. (C) Irrigating with water will not cause restlessness or insomnia in the postoperative client. Restlessness and insomnia can be emotional complications of surgery. (D) A nasogastric tube placed in the stomach is used to decompress the bowel. Irrigating with saline ensures a patent, well- functioning tube. Irrigating with saline will not increase peristalsis. QUESTION 812 A 35-year-old client has returned to her room following surgery on her right femur. She has an IV of D5 in onehalf normal saline infusing at 125 mL/hr and is receiving morphine sulfate 1015 mg IM q4h prn for pain. She last voided 51/2 hours ago when she was given her preoperative medication. In monitoring and promoting return of urinary function after surgery, the nurse would:A. Provide food and fluids at the client'srequest B. Maintain IV, increasing the rate hourly until the client voids C. Report to the surgeon if the client is unable to void within 8 hours of surgery D. Hold morphine sulfate injections for pain until the client voids, explaining to her that morphine sulfate can cause urinary retention Correct Answer: C Section: (none) Explanation Explanation/Reference: Explanation: (A) Provision of food and fluid promotes bowel elimination. Nutritional needs postoperatively are determined by the physician, not the client. (B) Increasing IV fluids postoperatively will not cause a client to void. Any change in rate of administration of IV fluids should be determined by the physician. (C) The postoperative client with normal kidney function who cannot void 8 hours after surgery is retaining urine. The client may need catheterization or medication. The physician must provide orders for both as necessary. (D) Although morphine sulfate can cause urinary retention, withholding pain medication will not ensure that the client will void. The client with uncontrolled pain will probably not be able to void. QUESTION 813 A 47-year-old male client is admitted for colon surgery. Intravenous antibiotics are begun 2 hours prior to surgery. He has no known infection. The rationale for giving antibiotics prior to surgery is to: A. Provide cathartic action within the colon B. Reduce the risk of wound infection from anaerobic bacteria C. Relieve the client's concern regarding possible infection D. Reduce the risk of intraoperative fever Correct Answer: B Section: (none) Explanation Explanation/Reference: Explanation: (A) Cathartic drugs promote evacuation of intestinal contents. (B) The client undergoing intestinal surgery is at increased risk for infection from large numbers ofanaerobic bacteria that inhabit the intestines. Administering antibiotics prophylactically can reduce the client'srisk for infection. (C) Antibiotics are indicated in the treatment of infections and have no effect on emotions. (D) Antipyretics are useful in the treatment of elevated temperatures. Antibiotics would have an effect on infection, which causes temperature elevation, but would not directly affect such an elevation. QUESTION 814A 19-month-old child is admitted to the hospital for surgical repair of patent ductus arteriosus. The child is being given digoxin. Prior to administering the medication, the nurse should: A. Not give the digoxin if the pulse is_60 B. Not give the digoxin if the pulse is_100 C. Take the apical pulse for a full minute D. Monitor for visual disturbances, a side effect of digoxin Correct Answer: C Section: (none) Explanation Explanation/Reference: Explanation: (A) Digoxin should not be given to adults with an apical pulse < 60 bpm. (B) Digoxin should be given to children with an apical pulse > 100 bpm. With a pulse < 100 bpm, the medication should be withheld and the physician notified. (C) Prior to digoxin administration in both children and adults, an apical pulse should be taken for 1 full minute. Aside from the rate per minute, the nurse should note any sudden increase or decrease in heart rate, irregular rhythm, or regularization of a chronic irregular heart rhythm. (D) Early indications of digoxin toxicity, such as visual disturbances, occur rarely as initial signs in children. QUESTION 815 The family member of a child scheduled for heart surgery states, "I just don't understand this open-heart or closed-heart business. I'm so confused! Can you help me understand it?" The nurse explains that patent ductus arteriosus repair is: A. Open-heart surgery. The child will be placed on a heart-lung machine while the surgery is being performed. B. Closed-heart surgery. It does not require that the child be placed on the heart-lung machine while the surgery is being performed. C. A pediatric version of the coronary artery bypass graft surgery performed on adults. It is an open-heart surgery. D. A pediatric version of percutaneous transluminal coronary angioplasty performed on adults. It is a closed-heart surgery. Correct Answer: B Section: (none) Explanation Explanation/Reference: Explanation: (A) Patent ductus arteriosus repair is a closed-heart procedure. The client is not placed on a heart-lung machine. (B) Patent ductus arteriosus is a ductus arteriosus that does not close shortlyafter birth but remains patent. Repair is a closed-heart procedure involving ligation of the patent ductus arteriosus. (C) Coronary artery bypass graft surgery is an open-heart surgical procedure in which blocked coronary arteries are bypassed using vessel grafts. (D) Percutaneous transluminal coronary angioplasty is a closedheart procedure that improves coronary blood flow by increasing the lumen size of narrowed vessels.QUESTION 816 A child is to receive atropine 0.15 mg (1/400 g) as part of his preoperative medication. A vial containing atropine 0.4 mg (1/150 g)/mL is on hand. How much atropine should be given? A. 0.06 mL B. 0.38 mL C. 2.7 mL D. Information given insufficient to determine the amount of atropine to be administered Correct Answer: B Section: (none) Explanation Explanation/Reference: Explanation: (A, C) Information was incorrectly placed in the formula, resulting in an incorrect answer. (B) The answer is correct. 0.4 mg = 1 mL:0.15 mg 5 = mL0.4 x = 0.15 x = 0.15/0.4 x = 0.375 or 0.38 mL (D) Sufficient information is provided to determine the amount of atropine to administer. The amount of atropine available and the amount of atropine ordered is required to determine the amount of atropine to be given. QUESTION 817 Following a gastric resection, a 70-year-old client is admitted to the postanesthesia care unit. He was extubated prior to leaving the suite. On arrival at the postanesthesia care unit, the nurse should: A. Check airway, feeling for amount of air exchange noting rate, depth, and quality of respirations B. Obtain pulse and blood pressure readings noting rate and quality of pulse C. Reassure the client that his surgery is over and that he is in the recovery room D. Review physician's orders, administering medications as ordered Correct Answer: A Section: (none) Explanation Explanation/Reference: Explanation:(A) Adequate air exchange and tissue oxygenation depend on competent respiratory function. Checking the airway is the nurse's priority action. (B) Obtaining the vital signs is an important action, but it is secondary to airway management. (C) Reorienting a client to time, place, and person after surgery is important, but it is secondary to airway and vital signs. (D) Airway management takes precedence over physician's orders unless they specifically relate to airway management. QUESTION 818 A 25-year-old client is admitted for a tonsillectomy. She tellsthe nurse that she has had episodes of muscle cramps, weakness, and unexplained temperature elevation. Many years ago her father died shortly after surgery after developing a high fever. She further tells the nurse that her surgeon is having her take dantrolene sodium (Dantrium) prophylactically prior to her tonsillectomy. Dantrolene sodium is ordered preoperatively to reduce the risk or prevent: A. Infection postoperatively B. Malignant hyperthermia C. Neuroleptic malignant syndrome D. Fever postoperatively Correct Answer: B Section: (none) Explanation Explanation/Reference: Explanation: (A, D) Dantrolene sodium is a peripheral skeletal muscle relaxant and would have no effect on a postoperative infection. (B) Dantrolene sodium is indicated prophylactically for clients with malignant hyperthermia or with a family history of the disorder. The mortality rate for malignant hyperthermia is high. (C) Neuroleptic malignant syndrome is an exercise-induced muscle pain and spasm and is unrelated to malignant hyperthermia. QUESTION 819 A 44-year-old client had an emergency cholecystectomy 3 days ago for a ruptured gallbladder. She complains of severe abdominal pain. Assessment reveals abdominal rigidity and distention, increased temperature, and tachycardia. Diagnostic testing reveals an elevated WBC count. The nurse suspects that the client has developed: A. Gastritis B. Evisceration C. Peritonitis D. Pulmonary embolism Correct Answer: C Section: (none) ExplanationExplanation/Reference: Explanation: (A) Assessment findings for gastritis would reveal anorexia, nausea and vomiting, epigastric fullness and tenderness, and discomfort. (B) Evisceration is the extrusion of abdominal viscera as a result of trauma or sutures failing in a surgical incision. (C) Peritonitis, inflammation of the peritoneum, can occur when an abdominal organ, such as the gallbladder, perforates and leaks blood and fluid into the abdominal cavity. This causes infection and irritation. (D) Assessmentfindings of pulmonary embolism would reveal severe substernal chest pain, tachycardia, tachypnea, shortness of breath, anxiety or panic, and wheezing andcoughing often accompanied by blood-tinged sputum. QUESTION 820 A 35-year-old client is admitted to the hospital for elective tubal ligation. While the nurse is doing preoperative teaching, the client says, "The anesthesiologist said she was going to give me balanced anesthesia. What exactly is that?" The best explanation for the nurse to give the client would be that balanced anesthesia: A. Is a type of regional anesthesia B. Uses equal amounts of inhalation agents and liquid agents C. Does not depress the central nervous system D. Is a combination of several anesthetic agents or drugs producing a smooth induction and minimal complications Correct Answer: D Section: (none) Explanation Explanation/Reference: Explanation: (A) Regional anesthesia does not produce loss of consciousness and is indicated for excision of moles, cysts, and endoscopic surgeries. (B) Varying amounts of anesthetic agents are used when employing balanced anesthesia. Amounts depend on age, weight, condition of the client, and surgical procedure. (C) General anesthesia is a drug-induced depression of the central nervous system that produces loss of consciousness and decreased muscle activity. (D) Balanced anesthesia is a combination of a number of anesthetic agents that produce a smooth induction, appropriate depth of anesthesia, and appropriate muscle relaxation with minimal complications. QUESTION 821 A 29-year-old client is admitted for a hysterectomy. She has repeatedly told the nurses that she is worried about having this surgery, has not slept well lately, and is afraid that her husband will not find her desirable after the surgery. Shortly into the preoperative teaching, she complains of a tightness in her chest, a feeling of suffocation, lightheadedness, and tingling in her hands. Her respirations are rapid and deep. Assessment reveals that the client is: A. Having a heart attack B. Wanting attention from the nurses C. Suffering from complete upper airway obstruction D. HyperventilatingCorrect Answer: D Section: (none) Explanation Explanation/Reference: Explanation: (A) Classic symptoms of a heart attack include heaviness or squeezing pain in the chest, pain spreading to the jaw, neck, and arm. Nausea and vomiting, sweating, and shortness of breath may be present. The client does not exhibit these symptoms. (B) Clients suffering from anxiety or fear prior to surgical procedures may develop hyperventilation. This client is not seeking attention. (C) Symptoms of complete airway obstruction include not being able to speak, and no airflow between the nose and mouth. Breath sounds are absent. (D) Tightness in the chest; a feeling of suffocation; lightheadedness; tingling in the hands; and rapid, deep respirations are signs and symptoms of hyperventilation. This is almost always a manifestation of anxiety. QUESTION 822 A client develops complications following a hysterectomy. Blood cultures reveal Pseudomonas aeruginosa. The nurse expects that the physician would order an appropriate antibiotic to treat P. aeruginosa such as: A. Cefoperazone (Cefobid) B. Clindamycin (Cleocin) C. Dicloxacillin (Dycill) D. Erythromycin (Erythrocin) Correct Answer: A Section: (none) Explanation Explanation/Reference: Explanation: (A) Cefoperazone is indicated in the treatment of infection withPseudomonas aeruginosa.(B) Clindamycin is not indicated in the treatment of infection withP. aeruginosa.(C) Dicloxacillin is not indicated in the treatment of infection withP. aeruginosa.(D) Erythromycin is not indicated in the treatment of infection withP. aeruginosa. QUESTION 823 A couple is experiencing difficulties conceiving a baby. The nurse explains basal body temperature (BBT) by instructing the female client to take her temperature: A. Orally in the morning and at bedtime B. Only one time during the day as long as it is always at the same time of dayC. Rectally at bedtime D. As soon as she awakens, prior to any activity Correct Answer: D Section: (none) Explanation Explanation/Reference: Explanation: (A) Monitoring temperature twice a day predicts the biphasic pattern of ovulation. (B) Prediction of ovulation relies on consistency in taking temperature. (C) Nightly rectal temperatures are more accurate in predicting ovulation. (D) Activity changes the accuracy of basal body temperature and ability to detect the luteinizing hormone surge. QUESTION 824 A client is having episodes of hyperventilation related to her surgery that is scheduled tomorrow. Appropriate nursing actionsto help control hyperventilating include: A. Administering diazepam (Valium) 1015 mg po q4h and q1h prn for hyperventilating episode B. Keeping the temperature in the client's room at a high level to reduce respiratory stimulation C. Having the client hold her breath or breathe into a paper bag when hyperventilation episodes occur D. Using distraction to help control the client's hyperventilation episodes Correct Answer: C Section: (none) Explanation Explanation/Reference: Explanation: (A) An adult diazepam dosage for treatment of anxiety is 210 mg PO 24 times daily. The order as written would place a client at risk for overdose. (B) A high room temperature could increase hyperventilating episodes by stimulating the respiratory system. (C) Breath holding and breathing into a paper bag may be useful in controlling hyperventilation. Both measures increase CO2 retention. (D) Distraction will not prevent or control hyperventilation caused by anxiety or fear. QUESTION 825 A client delivered a stillborn male at term. An appropriate action of the nurse would be to: A. State, "You have an angel in heaven." B. Discourage the parents from seeing the baby.C. Provide an opportunity for the parents to see and hold the baby for an undetermined amount of time. D. Reassure the parents that they can have other children. Correct Answer: C Section: (none) Explanation Explanation/Reference: Explanation: (A) This is not a supportive statement. There are also no data to indicate the family's religious beliefs. (B) Seeing their baby assists the parents in the grieving process. This gives them the opportunity to say "good-bye." (C) Parents need time to get to know their baby. (D) This is not a comforting statement when a baby has died. There are also no guarantees that the couple will be able to have another child. QUESTION 826 A pregnant client is having a nonstress test (NST). It is noted that the fetal heart beat rises 20 bpm, lasting 20 seconds, every time the fetus moves. The nurse explains that: A. The test is inconclusive and should be repeated B. Further testing is needed C. The test is normal and the fetus is reacting appropriately D. The fetus is distressed Correct Answer: C Section: (none) Explanation Explanation/Reference: Explanation: (A) The test results were normal, so there would be no need to repeat to determine results. (B) There are no data to indicate further tests are needed, because the result of the NST was normal. (C) An NST is reported as reactive if there are two to three increases in the fetal heart rate of 15 bpm, lasting at least 15 seconds during a 15-minute period. (D) The NST results were normal, so there was no fetal distress. QUESTION 827 Which stage of labor lasts from delivery of the baby to delivery of the placenta? A. Second B. ThirdC. Fourth D. Fifth Correct Answer: B Section: (none) Explanation Explanation/Reference: Explanation: (A) This stage is from complete dilatation of the cervix to delivery of the fetus. (B) This is the correct stage for the definition. (C) This stage lasts for about 2 hours after the delivery of the placenta. (D) There is no fifth stage of labor. QUESTION 828 On the third postpartum day, the nurse would expect the lochia to be: A. Rubra B. Serosa C. Alba D. Scant Correct Answer: A Section: (none) Explanation Explanation/Reference: Explanation: (A) This discharge occurs from delivery through the 3rd day. There is dark red blood, placental debris, and clots. (B) This discharge occurs from days 410. The lochia is brownish, serous, and thin. (C) This discharge occurs from day 10 through the 6thweek. The lochia is yellowish white. (D) This is not a classification of lochia but relates to the amount of discharge. http://www.gratisexam.com [Show More]

Last updated: 1 year ago

Preview 1 out of 424 pages

Reviews( 0 )

$25.00

Add to cart

Instant download

Can't find what you want? Try our AI powered Search

OR

GET ASSIGNMENT HELP
64
1

Document information


Connected school, study & course


About the document


Uploaded On

Jul 06, 2021

Number of pages

424

Written in

Seller


seller-icon
QuizMaster

Member since 3 years

11 Documents Sold


Additional information

This document has been written for:

Uploaded

Jul 06, 2021

Downloads

 1

Views

 64

Document Keyword Tags

Recommended For You


$25.00
What is Browsegrades

In Browsegrades, a student can earn by offering help to other student. Students can help other students with materials by upploading their notes and earn money.

We are here to help

We're available through e-mail, Twitter, Facebook, and live chat.
 FAQ
 Questions? Leave a message!

Follow us on
 Twitter

Copyright © Browsegrades · High quality services·